<<

8/9/2016 MyPastest

Back to Filters (/Secure/TestMe/Filter/429893/QA) Question 1 of 153

At what CD4 count should highly active anti-retroviral treatment (HAART) commence in asymptomatic HIV patients?

A Below 600/mm3

B Below 400/mm3

C Below 350/mm3

D Below 100/mm3

E Below 50/mm3

Explanation

Timing of treatment in human immunodeficiency virus

A number of cohorts exist, providing important data on the natural history and progression of HIV infection Multiple logistic regression can and has been used to determine the optimal point at which to start HAART, and it appears that the point where the benefit of HAART outweighs the risk is around 350 mm3

5490

Next Question

Previous Question Tag Question Feedback End Review

Difficulty: Average

Peer Responses

https://mypastest.pastest.com/Secure/TestMe/Browser/429893 1/2 8/9/2016 MyPastest

Session Progress

Responses Correct: 1

Responses Incorrect: 152

Responses Total: 153

Responses - % Correct: 1%

Blog (https://www.pastest.com/blog) About Pastest (https://www.pastest.com/about-us) Contact Us (https://www.pastest.com/contact-us) Help (https://www.pastest.com/help)

© Pastest 2016

https://mypastest.pastest.com/Secure/TestMe/Browser/429893 2/2 8/9/2016 MyPastest

Back to Filters (/Secure/TestMe/Filter/429893/QA) Question 2 of 153

A 22-year-old woman returns from a holiday on the Kenyan coast. She develops a , deteriorates over the next 48 h and becomes unconscious and unrousable. She has acute renal failure. Which one of the following options is the most appropriate investigation?

A Computed tomography (CT) scan, head

B Electroencephalograph (EEG)

C Erythrocyte sedimentation rate (ESR)

D Repeated thick and thin blood smear

E C-reactive protein (CRP)

Explanation

Complications in malaria treatment

The patient in the present case has extremely severe falciparum malaria, with cerebral malaria (coma) and renal failure (usually pre-renal) needing renal replacement therapy Patients with full-blown cerebral malaria are at an increased risk of fitting, which may be treated with diazepam Administration of prophylactic anticonvulsants may be associated with an increased mortality Exchange transfusion is recommended for a parasitaemia > 10% with complications (or > 30% if no other complications) Treatment of the malaria is with IV quinine, which increases insulin secretion and the sensitivity of cells to insulin and can cause hypoglycaemia Malaria itself can cause hypoglycaemia too, so blood glucose should be monitored every 2 h

3678

Next Question

https://mypastest.pastest.com/Secure/TestMe/Browser/429893 1/2 8/9/2016 MyPastest

Previous Question Tag Question Feedback End Review

Difficulty: Easy

Peer Responses

Session Progress

Responses Correct: 1

Responses Incorrect: 152

Responses Total: 153

Responses - % Correct: 1%

Blog (https://www.pastest.com/blog) About Pastest (https://www.pastest.com/about-us) Contact Us (https://www.pastest.com/contact-us) Help (https://www.pastest.com/help)

© Pastest 2016

https://mypastest.pastest.com/Secure/TestMe/Browser/429893 2/2 8/9/2016 MyPastest

Back to Filters (/Secure/TestMe/Filter/429893/QA) Question 3 of 153

Which one of the following organisms is the most frequent cause of hospital-acquired and is also developing increasing resistance to antimicrobial agents?

A Staphylococcus aureus

B Streptococcus pneumoniae

C Toxoplasma gondii

D Pneumocystis jirovecii (formerly called Pneumocystis carinii)

E Listeria monocytogenes

Explanation

Staphylococcus aureus infection

Epidemiological studies of Staphylococcus aureus infection, and increasingly these concern meticillin-resistant Staph. aureus (MRSA) strains, require typing methods to distinguish between epidemic and endemic strains Staph. aureus is part of the normal flora in some individuals; about 25% of people carry the organism permanently, a similar proportion never do, and the rest do so intermittently Common carriage sites are the nose, axillae, perineum and toe webs Nasal carriage rates vary from 10% to 40% in normal adults outside a hospital environment, but higher rates are often found in hospital patients, particularly those who have been in hospital for several weeks High carriage rates are also found in those with skin diseases such as eczema, those with insulin-dependent diabetes, patients on chronic haemodialysis or chronic ambulatory peritoneal dialysis, intravenous drug users and human immunodeficiency virus (HIV)-positive patients Some carriers disperse large numbers of staphylococci into the environment on skin squamae https://mypastest.pastest.com/Secure/TestMe/Browser/429893 1/2 8/9/2016 MyPastest The carrier state is highly relevant to the epidemiology of Staph. aureus infection as to whether or not this complicates surgery or trauma; the source of Staph. aureus in most patients who develop a staphylococcal infection is endogenous

1308

Next Question

Previous Question Tag Question Feedback End Review

Difficulty: Easy

Peer Responses

Session Progress

Responses Correct: 1

Responses Incorrect: 152

Responses Total: 153

Responses - % Correct: 1%

Blog (https://www.pastest.com/blog) About Pastest (https://www.pastest.com/about-us) Contact Us (https://www.pastest.com/contact-us) Help (https://www.pastest.com/help) © Pastest 2016

https://mypastest.pastest.com/Secure/TestMe/Browser/429893 2/2 8/9/2016 MyPastest

Back to Filters (/Secure/TestMe/Filter/429893/QA) Question 4 of 153

A 49-year-old woman is referred to you by her GP for suspected chronic fatigue syndrome. Which one of the following features would suggest that this was an incorrect diagnosis?

A Dysphagia

B Frequent

C Memory impairment

D Recurrent sore throats

E Severe myalgia

Explanation

Chronic fatigue syndrome

A diagnosis of chronic fatigue syndrome (CFS) requires the presence of unexplained chronic fatigue for more than six months Although several formal definitions exist, cardinal features of CFS (besides fatigue) include impaired memory or concentration, sore throats, myalgia, arthralgia, headaches, unrefreshing sleep and post-exertion malaise CFS is a diagnosis of exclusion, which requires the absence of any other underlying organic or psychiatric problem

Dysphagia

Dysphagia might reflect an underlying oesophageal cancer, and should be investigated urgently

1643

Next Question

https://mypastest.pastest.com/Secure/TestMe/Browser/429893 1/2 8/9/2016 MyPastest Previous Question Tag Question Feedback End Review

Difficulty: Average

Peer Responses

Session Progress

Responses Correct: 1

Responses Incorrect: 152

Responses Total: 153

Responses - % Correct: 1%

Blog (https://www.pastest.com/blog) About Pastest (https://www.pastest.com/about-us) Contact Us (https://www.pastest.com/contact-us) Help (https://www.pastest.com/help) © Pastest 2016

https://mypastest.pastest.com/Secure/TestMe/Browser/429893 2/2 8/9/2016 MyPastest

Back to Filters (/Secure/TestMe/Filter/429893/QA) Question 5 of 153

A 25-year-old soldier presents to the Emergency Department with a high fever, diarrhoea and vomiting. He returned from his recent posting to rural Sierra Leone 10 days ago and has become unwell over the last 24 h. On admission he looks unwell and has a temperature of 39 °C. He has a pulse rate of 110 bpm. Examination is otherwise unremarkable. Which one of the following options is the most appropriate next step?

A Send samples for FBC, clotting, U&Es, LFTs and a malaria film to the lab

B Send the patient direct to an isolation unit

C Send samples for a malaria film to the lab

D Send samples for FBC, clotting, U&Es, LFTs, a malaria film and blood cultures to the lab

E Send the patient home

Explanation

Malaria/Viral Haemorrhagic Fever Differential

The most likely diagnosis in the present case is malaria, in the current climate however the major concern is for viral haemorrhagic fever, in particular a differential of Ebola, and as such guidance has changed from that previously recommended In the past, in someone returning from rural Sierra Leone guidance was send only one sample for malaria (if positive for malaria, can relax and treat for malaria) New guidance dictates that if suspicion of viral haemorrhagic fever, then don't even take a single sample to avoid exposure risk for lab staff, send directly to an isolation unit (Royal Free, Newcastle, Liverpool, Sheffield) - so this would encompass anyone returning from rural West Africa and with a differential of viral haemorrhagic fever and within the 21 day maximum incubation period

624

Next Question https://mypastest.pastest.com/Secure/TestMe/Browser/429893 1/2 8/9/2016 MyPastest

Previous Question Tag Question Feedback End Review

Difficulty: Difficult

Peer Responses

Session Progress

Responses Correct: 1

Responses Incorrect: 152

Responses Total: 153

Responses - % Correct: 1%

Blog (https://www.pastest.com/blog) About Pastest (https://www.pastest.com/about-us) Contact Us (https://www.pastest.com/contact-us) Help (https://www.pastest.com/help)

© Pastest 2016

https://mypastest.pastest.com/Secure/TestMe/Browser/429893 2/2 8/9/2016 MyPastest

Back to Filters (/Secure/TestMe/Filter/429893/QA) Question 6 of 153

A 12-year-old boy has had a gradually progressive plaque on his buttock for the past 3 years. The plaque is 15 cm in diameter, irregular in shape with crusting and induration at the periphery and scarring at the centre. Which one of the following options is the most likely diagnosis?

A Tinea corporis

B Granuloma annulare

C Lupus vulgaris

D Borderline leprosy

E Cutaneous leishmaniasis

Explanation Differential diagnosis of plaque-forming infections Lupus vulgaris

An irregular plaque like lesion with central scarring is suggestive of lupus vulgaris This is the commonest manifestation of cutaneous tuberculosis

Ringworm infection

Ringworm (Tinea corporis) infection usually presents with slightly itchy, asymmetrical scaly patches that show central clearing and an advanced scaly raised edge Occasionally vesicles or pustules may be seen in the edge

Granuloma annulare

This is a dermatosis predominantly of children and young adults It is characterised by clusters of small dermal papules that often form into rings or part of a ring As they heal, the centre becomes dusky and altered in texture

https://mypastest.pastest.com/Secure/TestMe/Browser/429893 1/2 8/9/2016 MyPastest Borderline leprosy (BB)

This presents with numerous skin lesions that may form macules, papules and plaques The annular-rimmed lesion with a punched-out hypopigmented anaesthetic centre is characteristic There is widespread nerve involvement and limb deformity

Cutaneous leishmaniasis

This presents as a single or multiple painless nodules that enlarge and ulcerate with a characteristic erythematous raised border An overlying crust may develop

3592

Next Question

Previous Question Tag Question Feedback End Review

Difficulty: Difficult

Peer Responses

Session Progress

Responses Correct: 1

Responses Incorrect: 152

Responses Total: 153

Responses - % Correct: 1%

Blog (https://www.pastest.com/blog) About Pastest (https://www.pastest.com/about-us) Contact Us (https://www.pastest.com/contact-us) Help (https://www.pastest.com/help) © Pastest 2016

https://mypastest.pastest.com/Secure/TestMe/Browser/429893 2/2 8/9/2016 MyPastest

Back to Filters (/Secure/TestMe/Filter/429893/QA) Question 7 of 153

A 24-year-old asylum seeker from Zimbabwe is currently on anti-tuberculous therapy with rifampicin and isoniazid for pulmonary tuberculosis. He is HIV-seropositive with a CD4 count of 300 cells/μl and 2 weeks ago he was commenced on high active anti-retroviral therapy with zidovudine, lamivudine and nevirapine. He presents with a widespread maculopapular rash. Which one of the following options is the most likely cause of the rash?

A Rifampicin

B Immune reconstitution disease

C Kaposi’s sarcoma

D Non-nucleoside reverse-transcriptase inhibitors

E Nucleoside reverse-transcriptase inhibitors

Explanation

Rashes in anti-retroviral therapy

Maculopapular rash an important and frequently encountered problem in anti-retroviral therapy Nevirapine is a non-nucleoside reverse-transcriptase inhibitor, a class of drugs well recognised to cause maculopapular rashes This is the rationale for starting low-dose therapy with nevirapine in the first 2 weeks Zidovudine and lamivudine are nucleoside reverse-transcriptase inhibitors, which are less frequently associated with rashes

Further investigations

In addition to the temporal clues, the presence of an eosinophilia should be sought, a finding seen in drug-associated rashes

759

https://mypastest.pastest.com/Secure/TestMe/Browser/429893 1/2 8/9/2016 MyPastest Next Question

Previous Question Tag Question Feedback End Review

Difficulty: Average

Peer Responses

Session Progress

Responses Correct: 1

Responses Incorrect: 152

Responses Total: 153

Responses - % Correct: 1%

Blog (https://www.pastest.com/blog) About Pastest (https://www.pastest.com/about-us) Contact Us (https://www.pastest.com/contact-us) Help (https://www.pastest.com/help)

© Pastest 2016

https://mypastest.pastest.com/Secure/TestMe/Browser/429893 2/2 8/9/2016 MyPastest

Back to Filters (/Secure/TestMe/Filter/429893/QA) Question 8 of 153

A 21-year-old student taking the contraceptive pill develops pain and soreness around the genitals. She has just completed an elective year in the USA. On examination there are multiple, shallow and tender ulcers at the skin and mucous membrane of the vagina. The most probable diagnosis is:

A Genital herpes

B Chancroid

C Granuloma inguinale

D Primary

E Lymphogranuloma venereum

Explanation

Genital herpes Underlying causes

Infectious aetiologies of genital ulcers include herpes simplex virus (HSV), chancroid (Haemophilus ducreyi), granuloma inguinale (Calymmatobacterium granulomatis), syphilis (Treponema pallidum), HIV-specific ulcers (acute HIV infection or late HIV), and lymphogranuloma venereum (LGV, Chlamydia trachomatis serovars L1–3) Non-infectious aetiologies include fixed drug reactions, Behçet’s disease, neoplasms, and trauma It is particularly important to consider these alternative causes if evaluations for the infectious aetiologies do not lead to a diagnosis HIV infection should always be considered and tested for

Epidemiology

Genital ulcers occur in sexually active individuals throughout the world The relative frequency of each of the infectious aetiologies differs depending on

https://mypastest.pastest.com/Secure/TestMe/Browser/429893 1/3 8/9/2016 MyPastest geographic location The most common causes of genital ulcers in sexually active young adults in the USA are herpes simplex virus (HSV), syphilis, and chancroid Of these three infections, ulcers due to HSV are the most prevalent, followed by primary syphilis and then chancroid Infection with each of these organisms is not mutually exclusive, and coinfection with multiple organisms occurs

Clinical presentation

Genital ulcers caused by HSV are frequently multiple, shallow and tender, while chancroid often presents with deep, undermined and purulent ulcers When a painless, indurated, clean-based ulcer is present, the diagnosis of syphilis is more likely

Laboratory investigations

A number of laboratory tests can be used to make an accurate diagnosis:

Gram stain and culture on selective media (for H. ducreyi) Tzanck preparation, direct fluorescence antibody (DFA), and viral culture (for HSV) serological tests (for syphilis and LGV) darkfield microscopy (for syphilis) tissue biopsy (syphilis, granuloma inguinale)

1634

Next Question

Previous Question Tag Question Feedback End Review

Difficulty: Easy

Peer Responses

Session Progress

Responses Correct: 1 https://mypastest.pastest.com/Secure/TestMe/Browser/429893 2/3 8/9/2016 MyPastest

Responses Incorrect: 152

Responses Total: 153

Responses - % Correct: 1%

Blog (https://www.pastest.com/blog) About Pastest (https://www.pastest.com/about-us) Contact Us (https://www.pastest.com/contact-us) Help (https://www.pastest.com/help) © Pastest 2016

https://mypastest.pastest.com/Secure/TestMe/Browser/429893 3/3 8/9/2016 MyPastest

Back to Filters (/Secure/TestMe/Filter/429893/QA) Question 9 of 153

A backpacker who has recently returned from Indonesia is diagnosed as having as a nocardial infection. What would be the best technique for isolating and culturing the organism?

A Paraffin bait

B Cell culture

C Anaerobic culture

D Blood culture

E Footpads of mice

Explanation

Culture techniques for pathogenic organisms

Nocardia spp use paraffin as a source of carbon for growth

In this technique, a paraffin wax-coated glass rod is placed in the inoculated carbon-free broth Nocardia grow on the rod at the air–liquid interface They are relatively slow to grow in blood cultures

Footpads of mice are used to grow of Mycobacterium leprae Cell culture techniques are used to isolate Chlamydia trachomatis

3578

Next Question

Previous Question Tag Question Feedback End Review

Difficulty: Difficult

https://mypastest.pastest.com/Secure/TestMe/Browser/429893 1/2 8/9/2016 MyPastest Peer Responses

Session Progress

Responses Correct: 1

Responses Incorrect: 152

Responses Total: 153

Responses - % Correct: 1%

Blog (https://www.pastest.com/blog) About Pastest (https://www.pastest.com/about-us) Contact Us (https://www.pastest.com/contact-us) Help (https://www.pastest.com/help) © Pastest 2016

https://mypastest.pastest.com/Secure/TestMe/Browser/429893 2/2 8/9/2016 MyPastest

Back to Filters (/Secure/TestMe/Filter/429893/QA) Question 10 of 153

A 42-year-old single man who has been living in Thailand returns to the UK for a medical consultation as he has deteriorating health. He admits to engaging in casual sexual relationships during his time in the country. Over the past few weeks he has begun suffering increasing night sweats, weight loss and diarrhoea. On examination his BP is 105/65 mmHg, pulse is 85/min and regular. There is peripheral lymphadenopathy and right upper quadrant pain on abdominal examination. His BMI is 21. Investigations;

Hb 9.1 g/dl

WCC 5.1 x109 /l

PLT 181 x109 /l

Na+ 137 mmol/l

K+ 4.9 mmol/l

Creatinine 138 micromol/l

ALT 249 U/l

Stool culture and microscopy negative for cysts and ovae

Which of the following is the most likely diagnosis?

Your answer was incorrect

A Blastomycosis

B Cryptosporidium

C Cytomegalovirus

D Histoplasmosis

E Mycobacterium avium complex

https://mypastest.pastest.com/Secure/TestMe/Browser/429893 1/2 8/9/2016 MyPastest Explanation The answer is Mycobacterium avium complex (MAC) - The implication of this man’s gradually deteriorating health during his stay in Thailand is that he has developed AIDS and the GI upset seen here is related to MAC. Serological testing to confirm the diagnosis of MAC is under development, currently culture of AAFB is required to confirm its presence. This patient should of course also have an HIV test. Typically patients are treated with combination antimicrobials, (a macrolide, Ethambutol and Rifampicin). Blastomycosis is manifest by predominant respiratory symptoms, as is histoplasmosis. Cryptosporidium is diagnosed by the presence of oocysts on stool culture, and CMV is associated with a more acute presentation than that seen here. 36458

Next Question

Previous Question Tag Question Feedback End Review

Difficulty: Difficult

Peer Responses

Session Progress

Responses Correct: 1

Responses Incorrect: 152

Responses Total: 153

Responses - % Correct: 1%

Blog (https://www.pastest.com/blog) About Pastest (https://www.pastest.com/about-us) Contact Us (https://www.pastest.com/contact-us) Help (https://www.pastest.com/help)

© Pastest 2016

https://mypastest.pastest.com/Secure/TestMe/Browser/429893 2/2 8/9/2016 MyPastest

Back to Filters (/Secure/TestMe/Filter/429893/QA) Question 11 of 153

A patient presents with diarrhoea and vomiting. He is jaundiced. Hepatitis A infection is diagnosed. Which one of the following modes of transmission is most likely?

A Blood transfusion

B Contaminated food

C Ticks

D Sexually

E Mosquitoes

Explanation

Hepatitis A infection

Hepatitis A virus causes a self-limiting hepatitis This RNA virus is acquired orally The incubation period is between 2 and 6 weeks

3679

Next Question

Previous Question Tag Question Feedback End Review

Difficulty: Easy

Peer Responses

https://mypastest.pastest.com/Secure/TestMe/Browser/429893 1/2 8/9/2016 MyPastest

Session Progress

Responses Correct: 1

Responses Incorrect: 152

Responses Total: 153

Responses - % Correct: 1%

Blog (https://www.pastest.com/blog) About Pastest (https://www.pastest.com/about-us) Contact Us (https://www.pastest.com/contact-us) Help (https://www.pastest.com/help)

© Pastest 2016

https://mypastest.pastest.com/Secure/TestMe/Browser/429893 2/2 8/9/2016 MyPastest

Back to Filters (/Secure/TestMe/Filter/429893/QA) Question 12 of 153

A 32-year-old man from Uganda is referred to hospital with a high eosinophil count by his GP following routine blood tests. He is entirely asymptomatic and has no past medical history of note. Which of the following organisms is LEAST likely to be responsible?

A Strongyloides stercoralis

B Wuchereria bancrofti

C Schistosoma mansoni

D Schistosoma haematobium

E Entamoeba histolytica

Explanation

Eosinophilia Aetiology

Eosinophilia is associated with tissue-invasive helminths: strongyloidiasis, Wucheria infection, schistosomiasis could all present in this way and should be screened for here

Other notes

Entamoeba histolytica is associated with a spectrum of illness, including colitis, liver abscess, extra-gastrointestinal disease (central nervous system (CNS), pulmonary involvement) and asymptomatic infection. Leukocytosis may occur but eosinophilia is NOT a feature

597

Next Question

Previous Question Tag Question Feedback End Review https://mypastest.pastest.com/Secure/TestMe/Browser/429893 1/2 8/9/2016 MyPastest

Difficulty: Average

Peer Responses

Session Progress

Responses Correct: 1

Responses Incorrect: 152

Responses Total: 153

Responses - % Correct: 1%

Blog (https://www.pastest.com/blog) About Pastest (https://www.pastest.com/about-us) Contact Us (https://www.pastest.com/contact-us) Help (https://www.pastest.com/help) © Pastest 2016

https://mypastest.pastest.com/Secure/TestMe/Browser/429893 2/2 8/9/2016 MyPastest

Back to Filters (/Secure/TestMe/Filter/429893/QA) Question 13 of 153

A 55-year-old man goes to the Kruger National Park in South Africa on a 3-week safari. He was fully vaccinated before his trip and took regular mefloquine malaria prophylaxis. He drank bottled water and ate only cooked food. He reported having seen lions and gazelles and being bitten by mosquitoes and tsetse flies. Two days following his return to the UK he has developed a fever and notices a black spot on his thigh. He has a faint macular rash. On arrival to hospital he has a fever of 37.7 °C. Which one of the following options is the most likely diagnosis?

A Malaria

B

C African tick typhus

D Leptospirosis

E Anthrax

Explanation

African tick typhus Diagnostic considerations in the present case

Both the eschar and travel history are very suggestive of African tick typhus However, malaria should be excluded

Causative organism

The organism most commonly involved is Rickettsia conorii

Treatment

Treatment is with doxycycline, which often results in a very quick resolution of symptoms and fever

606 https://mypastest.pastest.com/Secure/TestMe/Browser/429893 1/2 8/9/2016 MyPastest

Next Question

Previous Question Tag Question Feedback End Review

Difficulty: Average

Peer Responses

Session Progress

Responses Correct: 1

Responses Incorrect: 152

Responses Total: 153

Responses - % Correct: 1%

Blog (https://www.pastest.com/blog) About Pastest (https://www.pastest.com/about-us) Contact Us (https://www.pastest.com/contact-us) Help (https://www.pastest.com/help)

© Pastest 2016

https://mypastest.pastest.com/Secure/TestMe/Browser/429893 2/2 8/9/2016 MyPastest

Back to Filters (/Secure/TestMe/Filter/429893/QA) Question 14 of 153

A 20-year-old woman presents to casualty with fever, diarrhoea, myalgia and a diffuse rash that started 6 h ago. She recovered from a similar episode 6 months ago. She is currently menstruating and is using tampons. On examination her temperature is 40.1 °C, blood pressure 80/50 mmHg, pulse 140/min. Which one of the following options is the most likely diagnosis?

A Infectious mononucleosis

B Aspirin overdose

C Toxic shock syndrome

D Salmonella infection

E HIV infection

Explanation

Toxic shock syndrome Epidemiology

In the late 1970s there was an epidemic of toxic shock syndrome associated with menstruation and tampon use Toxic shock syndrome has also been described in non-menstruating women and in men, in association with a wide variety of conditions and operations Additionally, it has been reported in association with burns in children

Prognosis

Toxic shock syndrome may be fatal: a mortality rate of around 5% was reported during the ‘tampon epidemic’

Management

Since the syndrome is mediated by a toxin (toxic shock-syndrome toxin), the mainstay https://mypastest.pastest.com/Secure/TestMe/Browser/429893 1/2 8/9/2016 MyPastest of treatment is supportive Anti-staphylococcal antibiotics should be given to eradicate Staphylococcus aureus from the local site; bacteraemia has rarely been reported in toxic shock syndrome The staphylococci isolated are usually resistant only to penicillin

1318

Next Question

Previous Question Tag Question Feedback End Review

Difficulty: Easy

Peer Responses

Session Progress

Responses Correct: 1

Responses Incorrect: 152

Responses Total: 153

Responses - % Correct: 1%

Blog (https://www.pastest.com/blog) About Pastest (https://www.pastest.com/about-us) Contact Us (https://www.pastest.com/contact-us) Help (https://www.pastest.com/help)

© Pastest 2016

https://mypastest.pastest.com/Secure/TestMe/Browser/429893 2/2 8/9/2016 MyPastest

Back to Filters (/Secure/TestMe/Filter/429893/QA) Question 15 of 153

A 20-year-old backpacker from Australia returns from a 3-month journey to Guatemala. She was fully vaccinated prior to travel and took malaria prophylaxis. She is vegetarian and ate mainly salads and fruit while on holiday. On her return, she has severe diarrhoea that has not responded to a course of ciprofloxacin and metronidazole. On examination she is dehydrated but relatively well and apyrexial. Which treatment is most likely to be effective?

A Repeat ciprofloxacin and metronidazole

B Co-trimoxazole

C Paromomycin

D Amoxicillin

E Mebendazole

Explanation

Cyclospora cayetanensis infection Epidemiology

Cyclospora cayetanensis infection is a protozoan infection associated with eating Guatemalan fruit (especially raspberries) and presents with a similar history to infection with Giardia spp

Treatment considerations

This infection does not respond to metronidazole treatment and is best treated with co-trimoxazole if necessary

622

Next Question

Previous Question Tag Question Feedback End Review https://mypastest.pastest.com/Secure/TestMe/Browser/429893 1/2 8/9/2016 MyPastest

Difficulty: Difficult

Peer Responses

Session Progress

Responses Correct: 1

Responses Incorrect: 152

Responses Total: 153

Responses - % Correct: 1%

Blog (https://www.pastest.com/blog) About Pastest (https://www.pastest.com/about-us) Contact Us (https://www.pastest.com/contact-us) Help (https://www.pastest.com/help) © Pastest 2016

https://mypastest.pastest.com/Secure/TestMe/Browser/429893 2/2 8/9/2016 MyPastest

Back to Filters (/Secure/TestMe/Filter/429893/QA) Question 16 of 153

A 25-year-old lawyer from Edinburgh was brought into hospital with a 3-day history of increasing shortness of breath and . She recently received a diagnosis of Cushing’s disease and is awaiting treatment. Initial examination revealed a respiratory rate of 20 breaths per minute and bilateral sparse crackles, with a characteristic buffalo hump and centripetal obesity. Chest X-ray revealed sparse perihilar shadowing only. She has been on the admission’s ward for 2 days and is being treated with intravenous cefotaxime and oral

clarithromycin. Overnight, she has deteriorated and arterial blood gases reveal a p (O2 ) of 6.5 on 24% oxygen. Which one of the following management options is most appropriate?

A Increase the oxygen to 100% and arrange admission to medical HDU for closer monitoring, and change the antibiotics to iv Tazocin

B Increase the oxygen to 100% and arrange admission to medical HDU, with the addition of iv co-trimoxazole

C Increase the oxygen to 100% and arrange admission to medical HDU, with the addition of amphotericin iv

D Increase the oxygen to 100% and arrange admission to medical HDU, with no change in therapy

E Increase the oxygen to 100% and arrange admission to medical HDU, adding co- trimoxazole, amphotericin and switching to iv Tazocin

Explanation

Pneumocystis infection

The patient in the present case is at an increased risk of contracting Pneumocystis pneumonia

Aetiology

Although classically associated with HIV infection (and a CD4 count < 200 cells/mm3 ), Pneumocystis infection also occurs in patients who are immunosuppressed for other reasons, eg post bone-marrow transplantation and those on high-dose steroids; https://mypastest.pastest.com/Secure/TestMe/Browser/429893 1/2 8/9/2016 MyPastest effectively this woman has endogenous high dose steroids as a result of Cushing’s disease

Treatment considerations

In addition to increasing the oxygen to 100% and arrange admission to medical high- dependency unit (HDU) and iv co-trimoxazole, amphotericin should also be considered, as she is also likely to be at risk of a fungal infection However, this is less likely than Pneumocystis jirovecii (PCJ; formerly Pneumocystis carinii) pneumonia Alternatives to co-trimoxazole for the treatment of PCJ pneumonia include nebulised pentamidine and dapsone

586

Next Question

Previous Question Tag Question Feedback End Review

Difficulty: Difficult

Peer Responses

Session Progress

Responses Correct: 1

Responses Incorrect: 152

Responses Total: 153

Responses - % Correct: 1%

Blog (https://www.pastest.com/blog) About Pastest (https://www.pastest.com/about-us) Contact Us (https://www.pastest.com/contact-us) Help (https://www.pastest.com/help) © Pastest 2016

https://mypastest.pastest.com/Secure/TestMe/Browser/429893 2/2 8/9/2016 MyPastest

Back to Filters (/Secure/TestMe/Filter/429893/QA) Question 17 of 153

A 45-year-old man of Sudanese origin is admitted with a history of low-grade fever for over 7 days. He migrated to the UK 1 year ago and has a past history of well-controlled asthma. His temperature chart shows that on some days there is a doubled rise in his temperature during 24 h. Examination shows a massively enlarged spleen and mild hepatomegaly. His full blood count shows a mild microcytic and hypochromic anaemia along with granulocytopenia and thrombocytopenia. Which one of the following investigations will establish a diagnosis?

A Bone marrow aspirate

B Widal test

C Xenodiagnosis

D Examination of a wet blood film taken at night

E Blood culture

Explanation

Visceral leishmaniasis

The diagnosis in the present case is visceral leishmaniasis

Causative organism

It is caused by infection with Leishmania donovani L. donovani is found in the Mediterranean and Red Sea area, Sudan, India, China and South America The organism multiplies in the monocytes and macrophages in various organs, especially in the liver and spleen (which become enlarged), the bone marrow, lymphoid tissue and the small intestinal mucosa

Incubation period

The incubation period may be up to 10 years with an insidious onset and low-grade https://mypastest.pastest.com/Secure/TestMe/Browser/429893 1/2 8/9/2016 MyPastest fever

Clinical features and prognosis

The temperature typically rises twice in 24 h The spleen and liver are enlarged and if not treated the patient becomes wasted

Diagnosis

Diagnosis is by bone marrow, spleen, lymph node or liver aspiration

1364

Next Question

Previous Question Tag Question Feedback End Review

Difficulty: Difficult

Peer Responses

Session Progress

Responses Correct: 1

Responses Incorrect: 152

Responses Total: 153

Responses - % Correct: 1%

Blog (https://www.pastest.com/blog) About Pastest (https://www.pastest.com/about-us) Contact Us (https://www.pastest.com/contact-us) Help (https://www.pastest.com/help)

© Pastest 2016

https://mypastest.pastest.com/Secure/TestMe/Browser/429893 2/2 8/9/2016 MyPastest

Back to Filters (/Secure/TestMe/Filter/429893/QA) Question 18 of 153

A 25-year-old man presents to the GP with a history of severe diarrhoea which contained occasional flecks of blood. This was accompanied by unpleasant gripping abdominal pain. It came on some 6–7 h after eating a rice-based dish. He had eaten the food from a local Chinese takeaway. His flatmate had had similar symptoms after visiting the same takeaway a few days earlier and these resolved within a day.

Which one of the following is the most likely causative organism?

A Salmonella

B Shigella

C Staphylococcus aureus

D Bacillus cereus

E Campylobacter

Explanation

Food poisoning

Bacillus cereus has an incubation period of 1 16 h and produces symptoms over 1–2 days in total It produces a toxin that may be associated with either profuse vomiting or severe diarrhoea It appears that infection in the European setting is more commonly associated with diarrhoea Fried rice is frequently the causative foodstuff when it is reheated from the previous evening The infection is self-limiting and best managed with appropriate oral rehydration advice

18601

Next Question https://mypastest.pastest.com/Secure/TestMe/Browser/429893 1/2 8/9/2016 MyPastest

Previous Question Tag Question Feedback End Review

Difficulty: Average

Peer Responses

Session Progress

Responses Correct: 1

Responses Incorrect: 152

Responses Total: 153

Responses - % Correct: 1%

Blog (https://www.pastest.com/blog) About Pastest (https://www.pastest.com/about-us) Contact Us (https://www.pastest.com/contact-us) Help (https://www.pastest.com/help)

© Pastest 2016

https://mypastest.pastest.com/Secure/TestMe/Browser/429893 2/2 8/9/2016 MyPastest

Back to Filters (/Secure/TestMe/Filter/429893/QA) Question 19 of 153

A 17-year-old girl who started medical school 2 weeks ago presents with fever, confusion, hypotension (75/50 mmHg) and a rapidly spreading purpuric rash over her whole body. Which one of the following options is the investigation most likely to deliver a rapid diagnosis? (Ceftriaxone has already been commenced).

A CT scan

B

C EEG

D CRP

E Take an EDTA blood sample for PCR

Explanation

Meningococcal septicaemia

The diagnosis of meningococcal septicaemia (with or without – it does not make any difference to management) is clear in the present case

Immediate management

The patient needs immediate intensive care unit (ITU) care A CT scan will delay this

Other considerations

A lumbar puncture (LP) will probably be contraindicated as she will undoubtedly be thrombocytopenic and/or have a disseminated intravascular coagulation (DIC) High-dose steroids probably worsen the outcome She should be in respiratory isolation for 24 h Meningococcal PCR is now a standard investigation

3704 https://mypastest.pastest.com/Secure/TestMe/Browser/429893 1/2 8/9/2016 MyPastest 3704

Next Question

Previous Question Tag Question Feedback End Review

Difficulty: Difficult

Peer Responses

Session Progress

Responses Correct: 1

Responses Incorrect: 152

Responses Total: 153

Responses - % Correct: 1%

Blog (https://www.pastest.com/blog) About Pastest (https://www.pastest.com/about-us) Contact Us (https://www.pastest.com/contact-us) Help (https://www.pastest.com/help) © Pastest 2016

https://mypastest.pastest.com/Secure/TestMe/Browser/429893 2/2 8/9/2016 MyPastest

Back to Filters (/Secure/TestMe/Filter/429893/QA) Question 20 of 153

Trachoma is a chronic keratoconjunctivitis that affects more than 500 million people worldwide, mainly in the developing countries. Some seven million people are blind as a result of it. Which one of the following organisms is most likely to be responsible for this infection?

A Herpes simplex virus

B C. trachomatis

C Human immunodeficiency virus

D M. leprae

E M. tuberculosis

Explanation

Trachoma

Trachoma is a chronic keratoconjunctivitis caused by Chlamydia trachomatis, believed to affect some 500 million people, of whom 7 million are blind and 10 million have some visual impairment After cataracts, it is the most common cause of blindness world-wide, but is now confined largely to developing countries

2712

Next Question

Previous Question Tag Question Feedback End Review

Difficulty: Easy

Peer Responses

https://mypastest.pastest.com/Secure/TestMe/Browser/429893 1/2 8/9/2016 MyPastest

Session Progress

Responses Correct: 1

Responses Incorrect: 152

Responses Total: 153

Responses - % Correct: 1%

Blog (https://www.pastest.com/blog) About Pastest (https://www.pastest.com/about-us) Contact Us (https://www.pastest.com/contact-us) Help (https://www.pastest.com/help)

© Pastest 2016

https://mypastest.pastest.com/Secure/TestMe/Browser/429893 2/2 8/9/2016 MyPastest

Back to Filters (/Secure/TestMe/Filter/429893/QA) Question 21 of 153

A 72-year-old woman has recently returned from her 3-month winter holiday to the Spanish Riviera. Over the past few days she has suffered from increasing cough and breathlessness, other symptoms include a and diarrhoea. By the time she presents to the Emergency Department with her daughter she is confused and incontinent of urine. On examination in the Emergency Department she is pyrexial 38.4 °C with a BP of 100/60 mmHg and a pulse of 105/min. She has bilateral wheeze on auscultation of the chest.

Investigations:

Hb 13.1 g/dl

WCC 13.2 × 109 /l

PLT 130 × 109 /l

Na+ 131 mmol/l

K+ 4.5 mmol/l

Creatinine 145 μmol/l

Urine protein +, blood +

pa (O2 ) 7.1 kPa

pa (CO2 ) 3.8 kPa

Which one of the following options is the most likely diagnosis?

Your answer was correct

A Urinary sepsis

B Listeria monocytogenes

C Legionnaire’s disease

D Pneumococcal pneumonia

E Meningococcal meningitis https://mypastest.pastest.com/Secure/TestMe/Browser/429893 1/3 8/9/2016 MyPastest

Explanation

Legionnaire’s disease Epidemiology

Outbreaks of Legionnaire’s occur because of carriage in air conditioning systems and tend to affect workers in large air-conditioned offices, or hotel clients

Clinical presentation

Central nervous system symptoms and diarrhoea are also frequently seen

Laboratory findings

Urinary antigen testing is a very rapid and sensitive way to confirm the diagnosis Syndrome of inappropriate antidiuretic hormone secretion (SIADH) occurs commonly in association with legionnaire’s, hence the low sodium seen here

Treatment

Macrolides, namely erythromycin, with or without rifampicin, are the antibiotics of choice for treatment of legionnaire’s disease

21089

Next Question

Previous Question Tag Question Feedback End Review

Difficulty: Easy

Peer Responses

Session Progress

Responses Correct: 1

Responses Incorrect: 152

Responses Total: 153 https://mypastest.pastest.com/Secure/TestMe/Browser/429893 2/3 8/9/2016 MyPastest

Responses - % Correct: 1%

Blog (https://www.pastest.com/blog) About Pastest (https://www.pastest.com/about-us) Contact Us (https://www.pastest.com/contact-us) Help (https://www.pastest.com/help) © Pastest 2016

https://mypastest.pastest.com/Secure/TestMe/Browser/429893 3/3 8/9/2016 MyPastest

Back to Filters (/Secure/TestMe/Filter/429893/QA) Question 22 of 153

A 35-year-old woman returns from a journey to the Gambia with fevers of 39 °C, rigors, vomiting and diarrhoea. She took chloroquine and proguanil prophylaxis. Her oxygen saturations are 90% on air, pulse 120 bpm, blood pressure 80/60 mmHg. Which one of the following options is the most likely diagnosis?

A Malaria

B Typhoid

C Gastroenteritis

D Atypical pneumonia

E Pneumococcal pneumonia

Explanation

Malaria

Malaria is the most likely diagnosis in the present patient

Epidemiology

Although she could also have the other infections, malaria is the most common in travellers returning from West Africa

Diagnostic considerations

Malaria always presents with a history of fever but it may also present with diarrhoea, which is commonly underestimated in the Emergency Department as being due to gastroenteritis Patients with high fevers and diarrhoea should be considered to have sepsis until proven otherwise

Complications https://mypastest.pastest.com/Secure/TestMe/Browser/429893 1/2 8/9/2016 MyPastest She is likely to have developed acute respiratory distress syndrome (ARDS), so caution with intravascular filling is important in order to avoid exacerbating this

Treatment

The patient may well require inotropic support As the other infections are also possible and are more common in patients with malaria – due to the production of an antigenic smokescreen, resulting in immunosuppression secondary to diversion of the immune response (non-protective antibodies produced against a large variety of antigens) – antibiotics should be commenced as well as quinine

587

Next Question

Previous Question Tag Question Feedback End Review

Difficulty: Average

Peer Responses

Session Progress

Responses Correct: 1

Responses Incorrect: 152

Responses Total: 153

Responses - % Correct: 1%

Blog (https://www.pastest.com/blog) About Pastest (https://www.pastest.com/about-us) Contact Us (https://www.pastest.com/contact-us) Help (https://www.pastest.com/help)

© Pastest 2016

https://mypastest.pastest.com/Secure/TestMe/Browser/429893 2/2 8/9/2016 MyPastest

Back to Filters (/Secure/TestMe/Filter/429893/QA) Question 23 of 153

A 34-year-old walker returns from a journey to the USA with fevers and a rash affecting his hands and feet. He says he was admitted to hospital and has a medical note from the hospital stating he was admitted for several days with high fevers and a tachycardia of 105 bpm but that his blood pressure did not drop at any time. After several days of illness, he developed desquamation of his hands and feet. He received some treatment in the USA and is now better. His GP sent some tests that revealed a normal FBC and U&Es. Syphilis serology is negative. What treatment did he receive?

A Intravenous cefuroxime

B Intravenous benzylpenicillin

C Oral doxycycline

D Oral erythromycin

E None

Explanation

Rocky Mountain spotted fever Epidemiology

Rocky Mountain spotted fever is a rickettsial infection caused by the organism Rickettsia rickettsii, which is spread by ticks It occurs in the East, South and West United States and also in Central and South America

Treatment

Treatment is with doxycycline Chloramphenicol is also effective

588

Next Question https://mypastest.pastest.com/Secure/TestMe/Browser/429893 1/2 8/9/2016 MyPastest

Previous Question Tag Question Feedback End Review

Difficulty: Average

Peer Responses

Session Progress

Responses Correct: 1

Responses Incorrect: 152

Responses Total: 153

Responses - % Correct: 1%

Blog (https://www.pastest.com/blog) About Pastest (https://www.pastest.com/about-us) Contact Us (https://www.pastest.com/contact-us) Help (https://www.pastest.com/help)

© Pastest 2016

https://mypastest.pastest.com/Secure/TestMe/Browser/429893 2/2 8/9/2016 MyPastest

Back to Filters (/Secure/TestMe/Filter/429893/QA) Question 24 of 153

A 44-year-old woman returned from India 4 days ago, and has been complaining of fever since her return. She also has generalised body aches and diarrhoea. Examination shows that, despite a temperature of 40 °C, she has a bradycardia of 60 bpm. She also has a non-itching rash, which blanches on pressure on her upper abdomen. The spleen is palpable and is not tender. A full blood count shows a leukopenia. Which one of the following options is the most likely diagnosis?

A Staphylococcal poisoning

B Amoebiasis

C Typhoid fever

D Viral fever

E Clostridium difficile infection

Explanation

Typhoid

The woman in the present case has typhoid fever

Causative organism

Typhoid is caused by Salmonella typhi

Clinical features

It is characterised by stepladder pyrexia and this is accompanied by headache and body aches Diarrhoea and vomiting may occur Rose spots on the trunk may be seen towards the end of the first week Splenomegaly is also a common feature

Laboratory investigations and findings https://mypastest.pastest.com/Secure/TestMe/Browser/429893 1/2 8/9/2016 MyPastest

Typically, patients have a leukopenia, and blood culture can establish the diagnosis The Widal test detects antibodies to the organism

Complications

Complications are bowel perforation, haemorrhage, bone and joint infection, meningitis, myocarditis and cholecystitis

1363

Next Question

Previous Question Tag Question Feedback End Review

Difficulty: Easy

Peer Responses

Session Progress

Responses Correct: 1

Responses Incorrect: 152

Responses Total: 153

Responses - % Correct: 1%

Blog (https://www.pastest.com/blog) About Pastest (https://www.pastest.com/about-us) Contact Us (https://www.pastest.com/contact-us) Help (https://www.pastest.com/help) © Pastest 2016

https://mypastest.pastest.com/Secure/TestMe/Browser/429893 2/2 8/9/2016 MyPastest

Back to Filters (/Secure/TestMe/Filter/429893/QA) Question 25 of 153

A 29-year-old homosexual man has been complaining of anal warts for the last 6 months. They have gradually increased in size and he has also noticed some fresh blood when opening his bowels. On examination there are grey lesions, approximately 5 mm in size, around his anus. Which one of the following options is the most likely cause for these lesions?

A Human papillomavirus

B Neisseria gonorrhoea

C Candida albicans

D Human immunodeficiency virus

E Chlamydia trachomatis

Explanation

Human papillomavirus infection

Condyloma is the most common clinical presentation of human papillomavirus (HPV) infection, a lesion that is almost always associated with HPV type 6/11 In homosexual men, condylomas are found on the genitalia and in the perianal region and within the anal canal, where they may cause pruritus ani and bleeding during defecation

2688

Next Question

Previous Question Tag Question Feedback End Review

Difficulty: Easy

Peer Responses https://mypastest.pastest.com/Secure/TestMe/Browser/429893 1/2 8/9/2016 MyPastest

Session Progress

Responses Correct: 1

Responses Incorrect: 152

Responses Total: 153

Responses - % Correct: 1%

Blog (https://www.pastest.com/blog) About Pastest (https://www.pastest.com/about-us) Contact Us (https://www.pastest.com/contact-us) Help (https://www.pastest.com/help)

© Pastest 2016

https://mypastest.pastest.com/Secure/TestMe/Browser/429893 2/2 8/9/2016 MyPastest

Back to Filters (/Secure/TestMe/Filter/429893/QA) Question 26 of 153

A man presents with a high fever that started 2 days after his return from the tropics. He has reduced consciousness and suffers from constipation. His heart rate is 72 bpm, temperature 40 °C. He also has a generalised rash. The haematology laboratory report is normal except for thrombocytopenia. His chest X-ray is normal. Which one of the following options is the most likely diagnosis?

A Typhoid

B Acute tuberculosis

C Acute HIV infection

D Infectious mononucleosis

E Giardia lamblia infection

Explanation

Clinical features of typhoid Symptoms

Most patients complain of headache and malaise Constipation is a frequent early symptom Most patients will experience diarrhoea and typhoid can present as an acute gastroenteritis Severe diarrhoea or colitis has been reported in human immunodeficiency virus (HIV)- infected patients Bloody diarrhoea may be seen

Signs

Physical examination is often unremarkable apart from fever Careful examination may reveal splenomegaly, hepatomegaly or rose spots Tachycardia is common although temperature pulse-dissociation (relative bradycardia) is considered characteristic https://mypastest.pastest.com/Secure/TestMe/Browser/429893 1/2 8/9/2016 MyPastest Lenticular rose spots appear at the end of the first week; they form a sparse collection of maculopapular lesions on the abdominal skin, which blanch with pressure and fade after 2–3 days Rose spots have been noted in 90% of white and 20% of black patients The rash may extend on to the trunk and arms Abdominal examination may reveal the typhoid rash, distension, or a diffuse tenderness, occasionally localized to the area of the terminal ileum Intra-abdominal sometimes provokes retention of urine A moderate, soft, tender hepatosplenomegaly eventually develops in most patients but it less likely to be found early

1302

Next Question

Previous Question Tag Question Feedback End Review

Difficulty: Average

Peer Responses

Session Progress

Responses Correct: 1

Responses Incorrect: 152

Responses Total: 153

Responses - % Correct: 1%

Blog (https://www.pastest.com/blog) About Pastest (https://www.pastest.com/about-us) Contact Us (https://www.pastest.com/contact-us) Help (https://www.pastest.com/help) © Pastest 2016

https://mypastest.pastest.com/Secure/TestMe/Browser/429893 2/2 8/9/2016 MyPastest

Back to Filters (/Secure/TestMe/Filter/429893/QA) Question 27 of 153

A patient with HIV complains of visual impairment. On examination he has patchy visual field loss affecting both eyes. Which one of the following options is the most likely diagnosis?

A Cytomegalovirus retinitis

B Toxoplasmosis

C Aspergillosis

D Tuberculosis

E Herpes keratitis

Explanation

Cytomegalovirus retinitis Pathogenesis

Visual loss due to cytomegalovirus (CMV) retinitis may occur in several ways Direct infection of the retinal cells by CMV causes retinal necrosis, which may result in a visual field defect or scotoma, depending on where in the retina the lesion occurs This permanent, irreversible loss of vision is not amenable to therapy and depends on the location and extent of the retinal necrosis Normal central vision may be preserved if the macula is uninvolved Retinal involvement of the area near the macula may produce oedema in the macula and result in loss of central visual acuity

Prognosis

The macular oedema and loss of visual acuity are potentially reversible if recognised and treated promptly before the retinal cells are infected

5634

Next Question https://mypastest.pastest.com/Secure/TestMe/Browser/429893 1/2 8/9/2016 MyPastest

Previous Question Tag Question Feedback End Review

Difficulty: Easy

Peer Responses

Session Progress

Responses Correct: 1

Responses Incorrect: 152

Responses Total: 153

Responses - % Correct: 1%

Blog (https://www.pastest.com/blog) About Pastest (https://www.pastest.com/about-us) Contact Us (https://www.pastest.com/contact-us) Help (https://www.pastest.com/help) © Pastest 2016

https://mypastest.pastest.com/Secure/TestMe/Browser/429893 2/2 8/9/2016 MyPastest

Back to Filters (/Secure/TestMe/Filter/429893/QA) Question 28 of 153

A 50-year-old homeless alcoholic presents with fever, rash and progressive swelling of the right side of his face which began with an itchy scab on his right cheek 24 h ago. His temperature is 39 °C, and he is unable to open his right eye because of the swelling. Which one of the following is the most likely causal organism?

A Streptococcus pneumoniae

B Haemophilus influenzae

C Herpes simplex virus

D Neisseria meningitidis

E Group A streptococcus

Explanation

Streptococcal infections

Although Streptococcus pyogenes (β-haemolytic group A) is an invasive organism, it lives on epithelial surfaces (asymptomatic carriage) usually in the nose and throat; carriage can also be anal, vaginal and on the scalp

Clinical presentation

Infection causes an acute spreading inflammation of the skin and subcutaneous tissues with local pain, swelling and erythema The appearance of the skin lesion and associated lymphangitis and tender lymphadenopathy may be preceded, by a few hours, by fever, rigors and malaise Streptococcal cellulitis differs from erysipelas in that the lesion is not raised and the demarcation between affected and unaffected skin is indistinct It may result from infection of burns, mild trauma, or surgical wounds

Treatment

https://mypastest.pastest.com/Secure/TestMe/Browser/429893 1/2 8/9/2016 MyPastest

Penicillin is the treatment of choice

1317

Next Question

Previous Question Tag Question Feedback End Review

Difficulty: Average

Peer Responses

Session Progress

Responses Correct: 1

Responses Incorrect: 152

Responses Total: 153

Responses - % Correct: 1%

Blog (https://www.pastest.com/blog) About Pastest (https://www.pastest.com/about-us) Contact Us (https://www.pastest.com/contact-us) Help (https://www.pastest.com/help)

© Pastest 2016

https://mypastest.pastest.com/Secure/TestMe/Browser/429893 2/2 8/9/2016 MyPastest

Back to Filters (/Secure/TestMe/Filter/429893/QA) Question 29 of 153

A 44-year-old patient is referred to the hospital because of a 3-day history of general malaise associated with nausea, diarrhoea and headache. On examination the patient looks drowsy and dehydrated, and red/bluish petechiae can be seen on the extensor surfaces of both legs. Which one of the following options is the most likely diagnosis?

A Hepatitis C infection

B Legionella infection

C Neisseria meningitidis infection

D Pneumocystis jirovecii infection

E Active tuberculosis

Explanation

Haemorrhagic skin lesions Underlying diagnosis

Haemorrhagic skin lesions are the hallmark of systemic meningococcal disease, occurring in 70–80% of all cases in industrialised countries The petechial rash indicates meningococcaemia, not necessarily severe sepsis Many patients are initially diagnosed as having gastric flu, gastroenteritis or an upper respiratory tract infection

Clinical presentation

Haemorrhagic skin lesions appear as red or bluish petechiae These lesions are larger and more irregular in size than the petechiae of thrombocytopenic purpura

Histological features

Each lesion represents a local nidus of meningococci within the endothelial cells, https://mypastest.pastest.com/Secure/TestMe/Browser/429893 1/2 8/9/2016 MyPastest thrombus formation and extravasation of erythrocytes Pneumocystis jirovecii used to be called Pneumocystis carinii

2704

Next Question

Previous Question Tag Question Feedback End Review

Difficulty: Average

Peer Responses

Session Progress

Responses Correct: 1

Responses Incorrect: 152

Responses Total: 153

Responses - % Correct: 1%

Blog (https://www.pastest.com/blog) About Pastest (https://www.pastest.com/about-us) Contact Us (https://www.pastest.com/contact-us) Help (https://www.pastest.com/help)

© Pastest 2016

https://mypastest.pastest.com/Secure/TestMe/Browser/429893 2/2 8/9/2016 MyPastest

Back to Filters (/Secure/TestMe/Filter/429893/QA) Question 30 of 153

You are taking part in the clinical trial of a new agent for the treatment of HIV which is thought to block the entry of HIV into host cells. Which of the following HIV peptides is thought to play a role in the initial step for HIV entry into cells?

A CCR5

B CXCR4

C GP41

D P55

E P160

Explanation The answer is GP41 - Gp120 fuses to the CD4 receptor, this then allows GP41 to penetrate the cell membrane. Shortly after the viral capsid enters the cell, reverse transcriptase liberates the single- stranded RNA genome from the attached viral proteins and copies it into a complementary DNA (cDNA) molecule. CCR5 and CXCR4 are host receptors which allow binding of HIV. P55 and p160 associate with the inner surface of the cell membrane. 39063

Next Question

Previous Question Tag Question Feedback End Review

Difficulty: Average

Peer Responses

https://mypastest.pastest.com/Secure/TestMe/Browser/429893 1/2 8/9/2016 MyPastest

Session Progress

Responses Correct: 1

Responses Incorrect: 152

Responses Total: 153

Responses - % Correct: 1%

Blog (https://www.pastest.com/blog) About Pastest (https://www.pastest.com/about-us) Contact Us (https://www.pastest.com/contact-us) Help (https://www.pastest.com/help)

© Pastest 2016

https://mypastest.pastest.com/Secure/TestMe/Browser/429893 2/2 8/9/2016 MyPastest

Back to Filters (/Secure/TestMe/Filter/429893/QA) Question 31 of 153

A 26-year-old traveller has just returned from South America. He notices several erythematous nodules all over his body. Some have a golden crust. Which one of the following options is the most likely diagnosis?

A Leishmaniasis

B Tuberculosis

C Malaria

D Loiasis

E Infectious mononucleosis

Explanation

Leishmaniasis Causative organism

Leishmaniasis is caused by parasites of the genus Leishmania, which are transmitted by phlebotomine sandflies

Clinical course

After an incubation period of a few days to several months an erythematous nodule develops at the site of the infected sandfly bite A golden crust forms The sore reaches its final size, usually 1–5 cm in diameter, over weeks or months The crust may fall away leaving an ulcer with a raised edge Satellite papules are common After months or years the lesion starts to heal, leaving a depressed, mottled scar

3710

Next Question https://mypastest.pastest.com/Secure/TestMe/Browser/429893 1/2 8/9/2016 MyPastest

Previous Question Tag Question Feedback End Review

Difficulty: Easy

Peer Responses

Session Progress

Responses Correct: 1

Responses Incorrect: 152

Responses Total: 153

Responses - % Correct: 1%

Blog (https://www.pastest.com/blog) About Pastest (https://www.pastest.com/about-us) Contact Us (https://www.pastest.com/contact-us) Help (https://www.pastest.com/help)

© Pastest 2016

https://mypastest.pastest.com/Secure/TestMe/Browser/429893 2/2 8/9/2016 MyPastest

Back to Filters (/Secure/TestMe/Filter/429893/QA) Question 32 of 153

A 24-year-old man has just returned to the UK from backpacking in the Far East. He feels tired and listless and has had diarrhoea since his last 2 weeks of travelling. He is pyrexial at 37.8 °C and on examination there is abdominal tenderness, more marked in the right upper quadrant. Examination of the distal bowel reveals evidence of colitis. Which one of the following diagnoses fits best with this clinical picture?

A Ulcerative colitis

B Amoebiasis

C Shigella infection

D Campylobacter infection

E Tropical sprue

Explanation

Amoebiasis Causative organism

Amoebiasis is caused by the protozoal parasite Entamoeba histolytica

History

The man in the present case has been travelling in the Far East and is likely to have been exposed to poor faecal–oral hygiene and possible infection with amoeba

Clinical features

Although it is primarily a disease of the colon, distal infection (particularly liver abscess) may occur Symptoms are often non-specific, but may include bloody diarrhoea, abdominal and back pain; and, in the case of liver abscess, may include hepatomegaly, right upper quadrant tenderness and fever https://mypastest.pastest.com/Secure/TestMe/Browser/429893 1/2 8/9/2016 MyPastest Laboratory investigations

Diagnosis is via stool examination, mucosal biopsy is occasionally necessary and serum antibody testing may also be useful in confirming the diagnosis

Treatment

Antibiotic therapy with a 10-day course of metronidazole is usually sufficient In the case of liver abscess, surgical intervention is only rarely required

2643

Next Question

Previous Question Tag Question Feedback End Review

Difficulty: Average

Peer Responses

Session Progress

Responses Correct: 1

Responses Incorrect: 152

Responses Total: 153

Responses - % Correct: 1%

Blog (https://www.pastest.com/blog) About Pastest (https://www.pastest.com/about-us) Contact Us (https://www.pastest.com/contact-us) Help (https://www.pastest.com/help)

© Pastest 2016

https://mypastest.pastest.com/Secure/TestMe/Browser/429893 2/2 8/9/2016 MyPastest

Back to Filters (/Secure/TestMe/Filter/429893/QA) Question 33 of 153

A 34-year-old with AIDS on a combination anti-retroviral therapy was brought into casualty because he had a fit lasting approximately 15 minutes at home. The day before he had complained of headaches and fever. On examination he is confused but has no localising neurological signs. A CT scan of his brain shows ring-enhancing masses with surrounding oedema. Which one of the following options is the most likely diagnosis?

A Tuberculosis

B Toxoplasma gondii cysts

C Cerebrovascular accident

D Pneumocystis jirovecii infection

E Cryptococcus infection

Explanation

CNS infections in acquired immune deficiency syndrome Principal causative organism

Cerebral infection with the intracellular protozoan Toxoplasma gondii is the most frequent infection of the central nervous system in patients with AIDS when their CD4 lymphocyte count is below 200/mm3

Pathogenesis

It usually results from the reactivation of Toxoplasma cysts in the brain, leading to the formation of focal lesions that are typically multiple but may be single

Clinical features

Symptoms develop subacutely and include focal neurological disturbance, headache, confusion, fever and convulsions

Further investigations https://mypastest.pastest.com/Secure/TestMe/Browser/429893 1/2 8/9/2016 MyPastest

On CT scanning the lesions appear as ring-enhancing masses with surrounding oedema Magnetic resonance imaging (MRI) is more sensitive and frequently detects lesions not visible on the computed tomography (CT) scan Serum antibodies to Toxoplasma spp are usually detectable; their absence makes the diagnosis unlikely but does not exclude it

2700

Next Question

Previous Question Tag Question Feedback End Review

Difficulty: Easy

Peer Responses

Session Progress

Responses Correct: 1

Responses Incorrect: 152

Responses Total: 153

Responses - % Correct: 1%

Blog (https://www.pastest.com/blog) About Pastest (https://www.pastest.com/about-us) Contact Us (https://www.pastest.com/contact-us) Help (https://www.pastest.com/help) © Pastest 2016

https://mypastest.pastest.com/Secure/TestMe/Browser/429893 2/2 8/9/2016 MyPastest

Back to Filters (/Secure/TestMe/Filter/429893/QA) Question 34 of 153

A 32-year-old farmer’s wife presents with fever and malaise, feeling generally ‘washed-out’ and off her food. She has recently been helping out with lambing on the farm. On examination there is generalised lymph node swelling and a palpable liver edge. Her white blood cell count is just below the normal range. Which one of the following diagnoses fits best with this clinical picture?

A Tuberculosis

B Subacute bacterial endocarditis

C Brucellosis

D Amoebic liver abscess

E Mixed connective tissue disease

Explanation

Brucellosis Definition

Brucellosis is a zoonotic infection that results from contact with farm animals (commonly sheep, goats, pigs, cattle or dogs)

Transmission of infection and causative organisms

Most cases occur due to occupational exposure, eg in farmers, vets or abattoir workers The most common infecting agents are Brucella melitensis and Brucella abortus It is acquired via inhalation or ingestion of organisms or via a break in the skin

Clinical presentation

The incubation period is between 1 week and 3 months, with symptoms of fever, sweating, weight loss and mild depression Hepatosplenomegaly is often present https://mypastest.pastest.com/Secure/TestMe/Browser/429893 1/3 8/9/2016 MyPastest Laboratory diagnosis

The white cell count may be normal or low, and the diagnosis may be confirmed via antibody testing

Complications

Infection may rarely seed to heart valves (causing endocarditis), or to bone, resulting in osteomyelitis

Treatment and prognosis

Combination treatment with doxycycline and rifampicin, or using Septrin may be considered, for 6 weeks or more Courses of less than 6 weeks are associated with a significant relapse rate

2659

Next Question

Previous Question Tag Question Feedback End Review

Difficulty: Easy

Peer Responses

Session Progress

Responses Correct: 1

Responses Incorrect: 152

Responses Total: 153

Responses - % Correct: 1%

Blog (https://www.pastest.com/blog) About Pastest (https://www.pastest.com/about-us) Contact Us (https://www.pastest.com/contact-us) Help (https://www.pastest.com/help) © Pastest 2016 https://mypastest.pastest.com/Secure/TestMe/Browser/429893 2/3 8/9/2016 MyPastest

https://mypastest.pastest.com/Secure/TestMe/Browser/429893 3/3 8/9/2016 MyPastest

Back to Filters (/Secure/TestMe/Filter/429893/QA) Question 35 of 153

A 22-year-old cocktail waitress presents for review. She complains of excess vaginal discharge, which stains green-yellow on a pad, and pelvic pain on sexual intercourse. She currently has no regular partner. There is occasional bleeding after sexual intercourse. On examination the cervix is tender on bimanual palpation. Which one of the following diagnoses fits best with this clinical picture?

A Cervicitis

B Cervical erosion

C Cervical carcinoma

D Cervical dysplasia

E Uterine carcinoma

Explanation

Cervicitis Epidemiology

Cervicitis accounts for 20–25% of women presenting with an abnormal vaginal discharge It occurs most commonly in adolescents, but may be found in any sexually active woman

Risk factors

Unsafe sex with multiple sexual partners increases the risk of developing cervicitis

Clinical features

Cervicitis may be asymptomatic or associated with vaginal discharge, dyspareunia and a friable mucosa, which leads to bleeding on smear examination or after intercourse

Laboratory investigations https://mypastest.pastest.com/Secure/TestMe/Browser/429893 1/3 8/9/2016 MyPastest

Microscopic examination of a cervical smear will reveal 10 or more polymorphs per microscopic field with a positive Gram stain Cultures should be taken for chlamydia and Neisseria gonorrhoeae, and a wet mount to look for trichomonas A cervical smear is also advisable to look for cervical dysplasia, as co-exposure to the human papillomavirus is likely

Treatment

A suspected chlamydia or gonorrhoea infection may be treated with a single injection of ceftriaxone im, followed by doxycycline, or with quinolone antibiotics Trichomonas is best treated with metronidazole

Complications

Long-term complications may include pelvic inflammatory disease or infertility (in up to 5–10% of patients)

2644

Next Question

Previous Question Tag Question Feedback End Review

Difficulty: Easy

Peer Responses

Session Progress

Responses Correct: 1

Responses Incorrect: 152

Responses Total: 153

Responses - % Correct: 1%

https://mypastest.pastest.com/Secure/TestMe/Browser/429893 2/3 8/9/2016 MyPastest Blog (https://www.pastest.com/blog) About Pastest (https://www.pastest.com/about-us) Contact Us (https://www.pastest.com/contact-us) Help (https://www.pastest.com/help) © Pastest 2016

https://mypastest.pastest.com/Secure/TestMe/Browser/429893 3/3 8/9/2016 MyPastest

Back to Filters (/Secure/TestMe/Filter/429893/QA) Question 36 of 153

A 38-year-old patient has fever, neck swelling, sore throat and mouth ulcers. His brother, who lives with him, has similar symptoms but has refused to come into hospital. On questioning he admits eating raw meat once a week, which is imported by a local African specialist delicatessen. Which one of the following options is the most likely diagnosis?

A Anthrax

B Infectious mononucleosis

C Candidiasis

D Helicobacter pylori infection

E Legionellosis

Explanation

Anthrax Aetiology

Gastrointestinal anthrax develops following consumption of contaminated meat It can occur as familial clusters Oropharyngeal anthrax should be considered in patients who present with fever, neck swelling, sore throat, and oropharyngeal ulcer and who give a history of eating raw or undercooked meat

Clinical course

The incubation period is 2 to 5 days Oropharyngeal anthrax follows deposition of the bacteria in the oropharynx Patients present with fever, neck swelling, sore throat, and dysphagia The neck swelling is caused by enlargement of the lymph nodes together with subcutaneous oedema as in diphtheria The lymph node enlargement commonly involves the upper group of the jugular chain

https://mypastest.pastest.com/Secure/TestMe/Browser/429893 1/2 8/9/2016 MyPastest There is an inflammatory lesion in the oral cavity or oropharynx The lesion starts as an inflamed mucosa, progressing through necrosis and ulceration to the formation of a pseudomembrane covering the ulcer In severe cases, the subcutaneous oedema extends to the anterior chest wall and axilla, with the overlying skin showing signs of inflammation Toxaemia and death may follow

3718

Next Question

Previous Question Tag Question Feedback End Review

Difficulty: Average

Peer Responses

Session Progress

Responses Correct: 1

Responses Incorrect: 152

Responses Total: 153

Responses - % Correct: 1%

Blog (https://www.pastest.com/blog) About Pastest (https://www.pastest.com/about-us) Contact Us (https://www.pastest.com/contact-us) Help (https://www.pastest.com/help) © Pastest 2016

https://mypastest.pastest.com/Secure/TestMe/Browser/429893 2/2 8/9/2016 MyPastest

Back to Filters (/Secure/TestMe/Filter/429893/QA) Question 37 of 153

A 62-year-old diabetic patient with peripheral arterial disease underwent a left foot amputation. She developed a foot infection with osteomyelitis, which has been treated with several different antibiotics for the last 3 weeks. Suddenly she developed watery diarrhoea and stomach cramps, now, 2 days later, she has fever associated with diarrhoea (frequency 20 times per day). Which one of the following options is the most likely diagnosis?

A Salmonella gastroenteritis

B Clostridium difficile enterocolitis

C Enteritis due to enterohaemorrhagic Escherichia coli (EHEC)

D Shigella infection

E Acute mesenterial ischaemia

Explanation

Pseudomembranous colitis Characteristic history

Patients with community-acquired infection tend to have a week or more of diarrhoea before seeking medical attention

Clinical presentation

Initial symptoms vary from mild, self-limiting diarrhoea to acute fulminating toxic megacolon Illness can begin surreptitiously where persistent diarrhoea resists all efforts at symptomatic relief Stools are described as watery or porridge-like, or patients may be constipated Other initial symptoms are sudden chills, fever and signs of an abdominal catastrophe Elderly patients may have diarrhoea that resolves and then recurs at intervals of one to several days

1298 https://mypastest.pastest.com/Secure/TestMe/Browser/429893 1/2 8/9/2016 MyPastest 1298

Next Question

Previous Question Tag Question Feedback End Review

Difficulty: Easy

Peer Responses

Session Progress

Responses Correct: 1

Responses Incorrect: 152

Responses Total: 153

Responses - % Correct: 1%

Blog (https://www.pastest.com/blog) About Pastest (https://www.pastest.com/about-us) Contact Us (https://www.pastest.com/contact-us) Help (https://www.pastest.com/help) © Pastest 2016

https://mypastest.pastest.com/Secure/TestMe/Browser/429893 2/2 8/9/2016 MyPastest

Back to Filters (/Secure/TestMe/Filter/429893/QA) Question 38 of 153

A 24-year-old man who went trekking in the South American jungle comes to the clinic. He was previously bitten by a tick on the back of his hand during the course of his recent travels. He removed the tick at the time, but now complains of intermittent fevers which come on every few days, and that the area on the back of his hand has not healed properly. When he gets these fevers he says that he feels like he has the flu, with severe headache, myalgias and vague abdominal tenderness. On examination he has a pyrexia of 38.1 °C. His BP is 105/60 mmHg, and his pulse is 90/min regular. There is a blackened area on the back of his hand where the tick had attached, and he has a maculopapular rash surrounding it. Examination of the abdomen reveals both left and right upper quadrant tenderness; you can feel a liver edge and are able to tip his spleen. Investigations:

Hb 10.9 g/dl

WCC 9.2 × 109 /l

PLT 167 × 109 /l

Na+ 140 mmol/l

K+ 4.5 mmol/l

Creatinine 120 μmol/l

ALT 165 U/l

Bilirubin 55 μmol/l

Which one of the following options is the most likely diagnosis?

A Borrelia burgdorferi infection

B Borrelia recurrentis infection

C Borrelia hermsii infection

D Pediculus humanus infection

https://mypastest.pastest.com/Secure/TestMe/Browser/429893 1/3 8/9/2016 MyPastest

E Tick typhus infection

Explanation

Relapsing fever following a tick bite

This question is difficult

Aetiology

Borrelia recurrentis infection is identified with the human louse, Pediculus humanus, not with tick infection, whereas Borrelia hermsii is identified with the tick-borne variety Both are causes of relapsing fever, the pattern of which is seen here with fever and myalgias seen every few days, hepatosplenomegaly, mild anaemia, raised transaminases and hepatosplenomegaly The disease is transmitted quickly and the tick may only have been on the patient’s hand for a very short period of time

Treatment

Tetracyclines are the drug of choice for the treatment of relapsing fever Erythromycin a reasonable alternative in patients who cannot tolerate tetracyclines

22529

Next Question

Previous Question Tag Question Feedback End Review

Difficulty: Difficult

Peer Responses

Session Progress

Responses Correct: 1

Responses Incorrect: 152

Responses Total: 153 https://mypastest.pastest.com/Secure/TestMe/Browser/429893 2/3 8/9/2016 MyPastest

Responses - % Correct: 1%

Blog (https://www.pastest.com/blog) About Pastest (https://www.pastest.com/about-us) Contact Us (https://www.pastest.com/contact-us) Help (https://www.pastest.com/help) © Pastest 2016

https://mypastest.pastest.com/Secure/TestMe/Browser/429893 3/3 8/9/2016 MyPastest

Back to Filters (/Secure/TestMe/Filter/429893/QA) Question 39 of 153

A 35-year-old man who had a splenectomy 7 years earlier for idiopathic thrombocytopenia (ITP) presents with sudden onset high fever and severe hypotension. Which one of the following options is the most likely organism to be causing this?

A Staphylococcus aureus

B Streptococcus pneumoniae

C Plasmodium falciparum

D Herpes simplex virus

E Epstein–Barr virus

Explanation

Post-splenectomy infection

Overwhelming post-splenectomy infection (OPSI) can occur any time after splenectomy, but is more common within the first 5 years It is less common if the spleen is removed for trauma Typically the causative organisms are encapsulated Capnocytophaga canimorsus (DF-2, a dog-bite organism) has also been identified as a causative agent and can give a clinical picture similar to meningococcal septicaemia

3697

Next Question

Previous Question Tag Question Feedback End Review

Difficulty: Easy

Peer Responses https://mypastest.pastest.com/Secure/TestMe/Browser/429893 1/2 8/9/2016 MyPastest

Session Progress

Responses Correct: 1

Responses Incorrect: 152

Responses Total: 153

Responses - % Correct: 1%

Blog (https://www.pastest.com/blog) About Pastest (https://www.pastest.com/about-us) Contact Us (https://www.pastest.com/contact-us) Help (https://www.pastest.com/help)

© Pastest 2016

https://mypastest.pastest.com/Secure/TestMe/Browser/429893 2/2 8/9/2016 MyPastest

Back to Filters (/Secure/TestMe/Filter/429893/QA) Question 40 of 153

A young man, just back from a trip to South America, presents with fever, productive cough, headache, conjunctivitis, orbital pain, purpura and generalised myalgia. On examination, there is splenomegaly and crepitations in the bases of both lungs. Which one of the following options is the most likely diagnosis?

A Cerebral malaria

B Q fever

C Trypanosomiasis

D Epidemic typhus

E Scrub typhus

Explanation

Epidemic typhus

The clinical features in the present case are suggestive of epidemic typhus

Clinical presentation and course

A -like eruption appears on the fifth day after the onset of symptoms The macules increase in size and eventually become purpuric in character At the end of the first week, signs of appear that may progress to stupor or coma, sometimes with extrapyramidal symptoms The most common initial symptoms in cerebral malaria are fever and malaise, which may be followed by a rapid deterioration in consciousness and convulsions

Differential diagnosis

Q fever presents with flu-like symptoms

The acute illness usually resolves spontaneously

https://mypastest.pastest.com/Secure/TestMe/Browser/429893 1/2 8/9/2016 MyPastest Scrub typhus is common in Asia and the Western Pacific

Symptoms range from mild illness to a fulminant fatal disease

Trypanosomiasis caused by Trypanosoma gambiense is a chronic, slowly progressive disease and is found on the African Sub-continent

Episodes of fever and lymphadenopathy occur over months or years and hepatosplenomegaly may develop

3580

Next Question

Previous Question Tag Question Feedback End Review

Difficulty: Difficult

Peer Responses

Session Progress

Responses Correct: 1

Responses Incorrect: 152

Responses Total: 153

Responses - % Correct: 1%

Blog (https://www.pastest.com/blog) About Pastest (https://www.pastest.com/about-us) Contact Us (https://www.pastest.com/contact-us) Help (https://www.pastest.com/help) © Pastest 2016

https://mypastest.pastest.com/Secure/TestMe/Browser/429893 2/2 8/9/2016 MyPastest

Back to Filters (/Secure/TestMe/Filter/429893/QA) Question 41 of 153

A 40-year-old patient noticed an abscess on the floor of his mouth 6 month ago, which gradually developed into cellulitis with a hard, painless swelling. The abscess was drained and sulphur granules were found microscopically. Which one of the following options is the most likely diagnosis?

A Leprosy

B Tuberculosis

C AIDS

D Syphilis

E Actinomycosis

Explanation

Actinomycosis

Site of infection

In the vast majority of cases, actinomycotic lesions primarily involve the face or neck

Predisposing conditions

Conditions predisposing to these cervicofacial infections include tooth extractions, fractures of the jaw, periodontal abscesses, foreign bodies penetrating the mucosal barrier (bone splinters, fish bones) or suppurating tonsillar crypts

Clinical course

Initially, the cervicofacial actinomycoses present either as an acute, usually odontogenic, abscess or cellulitis of the floor of the mouth, or as a slowly developing hard, painless, reddish or livid swelling Small, acute actinomycotic abscesses may heal after surgical drainage alone More often, however, the acute initial stage is followed by a subacute to chronic course https://mypastest.pastest.com/Secure/TestMe/Browser/429893 1/2 8/9/2016 MyPastest if no specific antimicrobial treatment is given, thereby imitating the primarily chronic form, which is characterised by regression of central suppurative foci while the infection progresses peripherally

Confirmation of diagnosis

A quick and comparatively reliable diagnosis is possible microscopically, when sulphur granules are present

2707

Next Question

Previous Question Tag Question Feedback End Review

Difficulty: Average

Peer Responses

Session Progress

Responses Correct: 1

Responses Incorrect: 152

Responses Total: 153

Responses - % Correct: 1%

Blog (https://www.pastest.com/blog) About Pastest (https://www.pastest.com/about-us) Contact Us (https://www.pastest.com/contact-us) Help (https://www.pastest.com/help) © Pastest 2016

https://mypastest.pastest.com/Secure/TestMe/Browser/429893 2/2 8/9/2016 MyPastest

Back to Filters (/Secure/TestMe/Filter/429893/QA) Question 42 of 153

A 31-year-old man has just returned from a holiday to recuperate after the death of his partner. He has been suffering night sweats, a chronic cough and shortness of breath on exercise. Over the past 6 months he has lost a few kilograms in weight and suffered from intermittent diarrhoea. On auscultation the lung fields appear relatively clear. Laboratory testing reveals a relative lymphopenia, with the CD4 lymphocyte subfraction reduced at only 85/mm3 (normal 200–800). There is desaturation on blood gas monitoring associated with exercise. Other blood tests reveal a raised lactate dehydrogenase level. Chest X-ray reveals diffuse pulmonary infiltrates. Which one of the following diagnoses fits best with this clinical picture?

A Tuberculosis

B Pneumocystis jirovecii pneumonia

C Histoplasmosis

D

E Mycoplasma pneumonia

Explanation

Pneumocystis jirovecii pneumonia

Pneumocystis jirovecii used to be called Pneumocystis carinii

Risk factors

The man in the present case has HIV infection with a reduced CD4 count One association when the CD4 count falls below 100 cells/mm3 is Pneumocystis pneumonia The peak incidence is in the 20–40 year age group, in parallel with the incidence of HIV infection

Clinical presentation

https://mypastest.pastest.com/Secure/TestMe/Browser/429893 1/3 8/9/2016 MyPastest

Symptoms include fever, cough and shortness of breath The lung fields are frequently clear to auscultation Cyanosis and pronounced shortness of breath may appear in severe cases Haemoptysis is unusual

Diagnosis

Diagnosis is via desaturation on exercise and sputum examination, which reveals the cysts associated with Pneumocystis infection Bronchoscopy with bronchoalveolar lavage may be required if sputum is equivocal

Treatment

Treatment is with oral or iv co-trimoxazole, or pentamidine Patients should take co-trimoxazole as long-term prophylaxis and be considered for combination antiviral therapy for their HIV infection

2650

Next Question

Previous Question Tag Question Feedback End Review

Difficulty: Easy

Peer Responses

Session Progress

Responses Correct: 1

Responses Incorrect: 152

Responses Total: 153

Responses - % Correct: 1%

Blog (https://www.pastest.com/blog) About Pastest (https://www.pastest.com/about-us) Contact Us (https://www.pastest.com/contact-us) Help (https://www.pastest.com/help) https://mypastest.pastest.com/Secure/TestMe/Browser/429893 2/3 8/9/2016 MyPastest © Pastest 2016

https://mypastest.pastest.com/Secure/TestMe/Browser/429893 3/3 8/9/2016 MyPastest

Back to Filters (/Secure/TestMe/Filter/429893/QA) Question 43 of 153

A 22-year-old man who is known to be HIV-positive presents with diarrhoea for the past 10 days, which he put down to food poisoning from chicken. He appears to be unresponsive to ciprofloxacin and is currently opening his bowels 5–10 times per day with profuse watery diarrhoea.

Investigations:

Hb 11.0 g/dl

WCC 10.1 × 109 /l

CD4 75 cells/mm3

PLT 120 × 109 /l

Na+ 141 mmol/l

K+ 3.9 mmol/l

Urea 12.1 mmol/l

Creatinine 180 μmol/l

Acid fast staining of stool Red oocysts against a blue-green background

Which one of the following options is the most likely diagnosis?

A Mycobacterium avium-intracellulare

B Cryptosporidiosis

C Giardiasis

D Shigellosis

E Isosporiasis

Explanation https://mypastest.pastest.com/Secure/TestMe/Browser/429893 1/3 8/9/2016 MyPastest Cryptosporidiosis Diagnosis

The acid-fast appearance in the present case and the voluminous watery stool is typical of Cryptosporidium infection in human immunodeficiency virus (HIV)-positive patients

Management

Azithromycin or Nitazoxanide are partially effective in resolving the infection, but aggressive anti-retroviral therapy also reduces the duration and severity of diarrhoea Apart from this, rehydration and use of therapies to reduce the volume of diarrhoea, such as codeine and loperamide, are also helpful

Complications and prevention

Prolonged diarrhoea lasting for greater than 1 month is associated with a poor outcome Continuation of therapy is recommended for all patients after infection to reduce the chances of relapse All HIV-infected patients are recommended to drink filtered water

21092

Next Question

Previous Question Tag Question Feedback End Review

Difficulty: Average

Peer Responses

Session Progress

Responses Correct: 1

Responses Incorrect: 152

Responses Total: 153

Responses - % Correct: 1%

https://mypastest.pastest.com/Secure/TestMe/Browser/429893 2/3 8/9/2016 MyPastest

Blog (https://www.pastest.com/blog) About Pastest (https://www.pastest.com/about-us) Contact Us (https://www.pastest.com/contact-us) Help (https://www.pastest.com/help) © Pastest 2016

https://mypastest.pastest.com/Secure/TestMe/Browser/429893 3/3 8/9/2016 MyPastest

Back to Filters (/Secure/TestMe/Filter/429893/QA) Question 44 of 153

Which organism produces the toxin that leads to impetigo?

A Legionella

B Herpes virus

C Cytomegalovirus

D Staphylococcus aureus

E E.coli

Explanation

Impetigo

Epidermolytic toxins cause intraepidermal splitting and are responsible for the blistering of impetigo The toxin is produced by Staphylococcus aureus

5633

Next Question

Previous Question Tag Question Feedback End Review

Difficulty: Easy

Peer Responses

https://mypastest.pastest.com/Secure/TestMe/Browser/429893 1/2 8/9/2016 MyPastest

Session Progress

Responses Correct: 1

Responses Incorrect: 152

Responses Total: 153

Responses - % Correct: 1%

Blog (https://www.pastest.com/blog) About Pastest (https://www.pastest.com/about-us) Contact Us (https://www.pastest.com/contact-us) Help (https://www.pastest.com/help)

© Pastest 2016

https://mypastest.pastest.com/Secure/TestMe/Browser/429893 2/2 8/9/2016 MyPastest

Back to Filters (/Secure/TestMe/Filter/429893/QA) Question 45 of 153

An elderly woman is admitted with a 4-week history of fevers. She says that she feels generally unwell, tired, and has been prone to headaches. She has had pain around the shoulders and hips, and has had jaw ache occassionally when she eats a large meal. On examination she has a pansystolic murmur of mitral regurgitation but no signs of cardiac failure or peripheral stigmata of endocarditis. She has raised inflammatory markers with a white cell count of 15 × 109 /ml (neutrophilia), a CRP of 80 mg/l and an ESR of 110 mm/1st hour; three sets of blood cultures are negative and a transthoracic echocardiogram shows moderate mitral regurgitation, but no vegetations are seen. Given the likeliest differential diagnosis, which one of the following tests is most important?

A An indium-labelled white cell scan

B Bone scintigram

C Isolator blood cultures

D Temporal artery biopsy

E Transoesophageal echocardiogram

Explanation

Pyrexia of unknown origin Diagnostic considerations

The investigation of pyrexia of unknown origin (PUO) remains a challenge It is essential that multiple blood cultures are performed for such a patient and empirical antibiotic use avoided, which might cloud the picture In elderly people, temporal arteritis is a frequently missed or delayed diagnosis with potentially disastrous consequences of loss of vision

Differential diagnosis

Microscopic haematuria, splinter haemorrhages, a Roth’s spot and a normocytic anaemia would all be strong pointers to subacute bacterial endocarditis, and in those https://mypastest.pastest.com/Secure/TestMe/Browser/429893 1/2 8/9/2016 MyPastest circumstances a transoesophageal echocardiogram would be the next investigation Isolator blood cultures may be of value if fungal endocarditis was suspected

764

Next Question

Previous Question Tag Question Feedback End Review

Difficulty: Average

Peer Responses

Session Progress

Responses Correct: 1

Responses Incorrect: 152

Responses Total: 153

Responses - % Correct: 1%

Blog (https://www.pastest.com/blog) About Pastest (https://www.pastest.com/about-us) Contact Us (https://www.pastest.com/contact-us) Help (https://www.pastest.com/help)

© Pastest 2016

https://mypastest.pastest.com/Secure/TestMe/Browser/429893 2/2 8/9/2016 MyPastest

Back to Filters (/Secure/TestMe/Filter/429893/QA) Question 46 of 153

A 21-year-old man returns from travelling in India. Investigations show he is IgG HAV positive, HBs antibody negative, HBeAg positive and has a raised ALT. Which one of the following is the most likely diagnosis?

A Hepatitis A

B Hepatitis B

C Hepatitis C

D Hepatitis D

E Hepatitis E

Explanation

Viral hepatitis Interpreting serological tests

In hepatitis A, serum transaminases are raised, IgM HAV implies recent infection, IgG HAV remains detectable for life and therefore implies infection with hepatitis A at some time in the past HBsAg becomes positive 1–6 months after infection; if it remains positive for longer than 6 months then the patient is a chronic carrier, this occurs in 5–10% of patients infected with hepatitis B HBeAg is present for 1–3 months after the infection and implies high infectivity The presence of the HBeAg makes hepatitis B the most likely diagnosis

Treatment of hepatitis B

Treatment of hepatitis B is with antivirals such as interferon alfa Liver transplantation may be required

Complications https://mypastest.pastest.com/Secure/TestMe/Browser/429893 1/2 8/9/2016 MyPastest

Complications of hepatitis B are fulminant liver failure, persistent cholestasis, chronic hepatitis, cirrhosis, liver cell carcinoma, glomerulonephritis and cryoglobulinaemia

13861

Next Question

Previous Question Tag Question Feedback End Review

Difficulty: Average

Peer Responses

Session Progress

Responses Correct: 1

Responses Incorrect: 152

Responses Total: 153

Responses - % Correct: 1%

Blog (https://www.pastest.com/blog) About Pastest (https://www.pastest.com/about-us) Contact Us (https://www.pastest.com/contact-us) Help (https://www.pastest.com/help) © Pastest 2016

https://mypastest.pastest.com/Secure/TestMe/Browser/429893 2/2 8/9/2016 MyPastest

Back to Filters (/Secure/TestMe/Filter/429893/QA) Question 47 of 153

A 36-year-old woman presents complaining of a yellowish-green vaginal discharge that started 1 week ago. On examination her vagina is swollen and erythematous. Which one of the following options is the most sensitive diagnostic test?

A Colposcopy

B Blood cultures

C Serology

D Vaginal fluid microscopy and culture

E Vaginal pH test

Explanation

Diagnostic considerations in vaginal diseases

Culture is the current diagnostic ‘gold standard’ but requires 2–5 days before a result can be obtained Trichomonas (the likely infecting organism in the present case) are motile by means of flagella that can be seen beating even when the organism is at rest; the organism is about the size of a white blood cell (10–20μm wide). The vaginal fluid contains numerous polymorphonuclear neutrophils because this disease is a true vaginitis, causing a local inflammatory response The vaginal pH is usually elevated to above 4.5 and can be as high as 6.5–7.5 The background bacterial flora is often abnormal as bacterial vaginosis may also be present The sensitivity of microscopical examination of the vaginal fluid for the diagnosis of trichomoniasis ranges from 40 to 80%

2684

Next Question

https://mypastest.pastest.com/Secure/TestMe/Browser/429893 1/2 8/9/2016 MyPastest

Previous Question Tag Question Feedback End Review

Difficulty: Average

Peer Responses

Session Progress

Responses Correct: 1

Responses Incorrect: 152

Responses Total: 153

Responses - % Correct: 1%

Blog (https://www.pastest.com/blog) About Pastest (https://www.pastest.com/about-us) Contact Us (https://www.pastest.com/contact-us) Help (https://www.pastest.com/help)

© Pastest 2016

https://mypastest.pastest.com/Secure/TestMe/Browser/429893 2/2 8/9/2016 MyPastest

Back to Filters (/Secure/TestMe/Filter/429893/QA) Question 48 of 153

A 19-year-old student is admitted directly to emergency after being taken ill on a return flight from Central America. It is understood that he was on the last leg of a round-the-world ticket. His vaccination history is unavailable. He had a flu-like illness around 10 days ago, from which he had recovered. On examination in emergency he is pyrexial at 39.0 °C, has extensive bruising, with bleeding around the gum line, and deep jaundice. Which one of the following diagnoses fits best with this clinical picture?

A Malaria

B Influenza

C Weil’s disease

D Yellow fever

E Dengue fever

Explanation

Yellow fever Definition

Yellow fever is caused by a flavivirus, and can vary in severity from a mild illness to the severe classical form seen here

Epidemiology and transmission of infection

It is confined to Africa and South America between the latitudes of 15°N and 15°S It is transmitted in South and Central America by the Haemagogus mosquito species

Clinical features

There is an incubation period of 3–6 days In the classical illness the patients then present with a severe flu-like illness with pyrexia up to 40 °C; there may be associated epigastric pain and vomiting https://mypastest.pastest.com/Secure/TestMe/Browser/429893 1/3 8/9/2016 MyPastest Relative bradycardia (Faget’s sign) is present from the second day of the illness There is then a recovery phase and the patient feels well for several days After this time, severe fever develops once again, the patients become jaundiced, with hepatomegaly, evidence of severe bruising, bleeding from the gums, haematemesis and melaena

Histological features

Liver biopsy at late presentation shows mid-zone necrosis, with eosinophilic degranulation of hepatocytes

Management and prognosis

Supportive therapy only is possible Mortality is unfortunately up to 40%

2661

Next Question

Previous Question Tag Question Feedback End Review

Difficulty: Difficult

Peer Responses

Session Progress

Responses Correct: 1

Responses Incorrect: 152

Responses Total: 153

Responses - % Correct: 1%

Blog (https://www.pastest.com/blog) About Pastest (https://www.pastest.com/about-us) Contact Us (https://www.pastest.com/contact-us) Help (https://www.pastest.com/help) © Pastest 2016 https://mypastest.pastest.com/Secure/TestMe/Browser/429893 2/3 8/9/2016 MyPastest

https://mypastest.pastest.com/Secure/TestMe/Browser/429893 3/3 8/9/2016 MyPastest

Back to Filters (/Secure/TestMe/Filter/429893/QA) Question 49 of 153

A 16-year-old boy develops a severe tonsillitis that prevents him swallowing even his own saliva. On examination he has symmetrically enlarged, inflamed, mildly exudative tonsils that are almost meeting in the midline. His white blood cell (WBC) is 12 × 109 /l with 60% lymphocytes. You are concerned that his airway may become compromised if his tonsils enlarge further. Which one of the following options is the initial management of choice whilst awaiting ENT/anaesthetic review?

A Amoxicillin and metronidazole

B Helium–oxygen mixture

C Elective tracheostomy

D Intravenous hydrocortisone

E Immediate respiratory isolation

Explanation

Tonsillitis Underlying infection

The patient has acute Epstein–Barr virus (EBV) infection

Treatment considerations

The tonsillar swelling responds to intravenous hydrocortisone (up to 200 mg qds) and can prevent respiratory obstruction, a recognised cause of death Amoxicillin will cause a rash, and is unlikely to help the tonsillitis unless there is a concomitant streptococcal infection (in which case benzylpenicillin should be used) Helium–oxygen (Heliox) is useful acutely in respiratory obstruction as it has a lower viscosity than air or oxygen, but is only an adjunct to management while the anaesthetist is on the way! The only slightly similar condition that may require respiratory isolation is diphtheria

3691 https://mypastest.pastest.com/Secure/TestMe/Browser/429893 1/2 8/9/2016 MyPastest 3691

Next Question

Previous Question Tag Question Feedback End Review

Difficulty: Average

Peer Responses

Session Progress

Responses Correct: 1

Responses Incorrect: 152

Responses Total: 153

Responses - % Correct: 1%

Blog (https://www.pastest.com/blog) About Pastest (https://www.pastest.com/about-us) Contact Us (https://www.pastest.com/contact-us) Help (https://www.pastest.com/help)

© Pastest 2016

https://mypastest.pastest.com/Secure/TestMe/Browser/429893 2/2 8/9/2016 MyPastest

Back to Filters (/Secure/TestMe/Filter/429893/QA) Question 50 of 153

For how long after a splenectomy is a patient at most increased risk of pneumococcal infection?

A 6 months

B 1 year

C 5 years

D More than 10 years

E 5-10 years

Explanation

Post-splenectomy infection Risk factors

All patients, whatever the reason for splenectomy, are at risk of overwhelming post- splenectomy infection (OPSI) The risk of OPSI is thought to be greatest during the first few years after splenectomy Children are at the greatest risk, followed by adults splenectomised for an underlying disorder that itself is immunosuppressive, or who require immunosuppressive treatment Adults splenectomised for trauma are at least risk, but they still carry a lifelong susceptibility OPSI has been recorded more than 40 years after splenectomy although the incidence is diminished vs the first 5-10 years The relative risk of severe infection compared with the non-splenectomised population is about 10-fold for traumatic splenectomy, and as much as 100-fold for small children and patients with Hodgkin’s disease

Aetiology

By far the most important causative organism is the pneumococcus (Streptococcus https://mypastest.pastest.com/Secure/TestMe/Browser/429893 1/3 8/9/2016 MyPastest pneumoniae), but Haemophilus influenzae, Neisseria meningitidis, Escherichia coli and Pseudomonas spp. have all been implicated In endemic areas, Plasmodium and Babesia infections are of increased severity in non- immune individuals Special warnings should be given to splenectomised patients travelling to malarial areas Viral illnesses may also be of increased severity posts-splenectomy.

Clinical features

Classically, OPSI presents with a vague general prodrome, followed by prostration, bacteraemic shock, and frequently with disseminated intravascular coagulation

Prognosis

Death may occur within 6 h of the onset The mortality rate in patients reaching hospital alive is in excess of 30%

5504

Next Question

Previous Question Tag Question Feedback End Review

Difficulty: Average

Peer Responses

Session Progress

Responses Correct: 1

Responses Incorrect: 152

Responses Total: 153

Responses - % Correct: 1%

Blog (https://www.pastest.com/blog) About Pastest (https://www.pastest.com/about-us) Contact Us (https://www.pastest.com/contact-us) Help (https://www.pastest.com/help) https://mypastest.pastest.com/Secure/TestMe/Browser/429893 2/3 8/9/2016 MyPastest © Pastest 2016

https://mypastest.pastest.com/Secure/TestMe/Browser/429893 3/3 8/9/2016 MyPastest

Back to Filters (/Secure/TestMe/Filter/429893/QA) Question 51 of 153

A 79-year-old woman in the respiratory unit has profuse watery, greenish diarrhoea. A diagnosis of pseudomembranous colitis is suspected, related to her antimicrobial therapy. Which one of the following antibiotics is most likely to have caused this?

A Cefuroxime

B Doxycycline

C Co-trimoxazole

D Erythromycin

E Flucloxacillin

Explanation

Pseudomembranous colitis Epidemiology

Pseudomembranous colitis was identified as a pathological entity before any clinical use of antimicrobials Community-acquired cases occur sporadically, but case clustering in hospitals or nursing homes is not uncommon The disease is more common in older patients but the typical syndrome has been described in people of all ages including infants

Diagnosis

The single most pertinent detail of the medical history is previous antimicrobial treatment Direct questioning may be needed to elicit this history; antimicrobials may have been self-administered, taken for trivial complaints, or used as long as 3–4 weeks before the start of diarrhoea

Risk factors https://mypastest.pastest.com/Secure/TestMe/Browser/429893 1/2 8/9/2016 MyPastest

Pseudomembranous colitis has been reported to follow the use of every antimicrobial in common medical practice, and is reported to occur in association with lincomycin, clindamycin, ciprofloxacin,ampicillin, amoxicillin and cephalosporins It occasionally occurs in individuals with no history of antimicrobial treatment or as a complication of chronic colonic obstruction, carcinoma, leukaemia, or uraemia

5499

Next Question

Previous Question Tag Question Feedback End Review

Difficulty: Average

Peer Responses

Session Progress

Responses Correct: 1

Responses Incorrect: 152

Responses Total: 153

Responses - % Correct: 1%

Blog (https://www.pastest.com/blog) About Pastest (https://www.pastest.com/about-us) Contact Us (https://www.pastest.com/contact-us) Help (https://www.pastest.com/help) © Pastest 2016

https://mypastest.pastest.com/Secure/TestMe/Browser/429893 2/2 8/9/2016 MyPastest

Back to Filters (/Secure/TestMe/Filter/429893/QA) Question 52 of 153

A 19-year-old student presented to his university GP complaining of a severe sore throat, headache and malaise. On examination there was a severe exudative pharyngitis with grossly enlarged inflamed tonsils. There was some evidence of cervical lymphadenopathy. He was diagnosed with a streptococcal infection and received a course of ampicillin. Unfortunately he re-presents to the GP with a maculopapular rash, still feeling unwell. Blood testing reveals a relative lymphocytosis with atypical lymphocytes. Which one of the following options is the most likely cause of this clinical picture?

A Stevens–Johnson syndrome

B Epstein–Barr virus (EBV)

C Toxoplasmosis

D Cytomegalovirus

E Streptococcal pharyngitis

Explanation

Epstein–Barr virus infection History and symptoms

Epstein–Barr virus infection is either asymptomatic or characterised by a prodrome of malaise, headache and chills, followed by a triad of pharyngitis, fever and lymphadenopathy Pharyngitis is the most common symptom and is usually severe

Clinical features

Lymphadenopathy is commonest in the cervical region Splenomegaly may also occur, and is commonest in the second week of the illness A maculopapular rash is uncommon, but occurs in 90% of patients who receive ampicillin

https://mypastest.pastest.com/Secure/TestMe/Browser/429893 1/3 8/9/2016 MyPastest Laboratory findings

Relative lymphocytosis may occur, with atypical lymphocytes being seen on the blood film There may also be mild thrombocytopenia Elevated AST or ALT is seen in most cases Monospot antibody test may be positive at presentation, or may become positive later

Complications

Complications are uncommon, but rarely haematological, pulmonary, cardiac or nervous system involvement may occur Splenic rupture is rare Symptoms of malaise and fatigue may persist for some months after infection (a postviral syndrome)

Treatment

Treatment is generally supportive, but sufferers should avoid contact sports during the first month of illness due to the risk of splenic rupture

2655

Next Question

Previous Question Tag Question Feedback End Review

Difficulty: Easy

Peer Responses

Session Progress

Responses Correct: 1

Responses Incorrect: 152

Responses Total: 153

Responses - % Correct: 1%

https://mypastest.pastest.com/Secure/TestMe/Browser/429893 2/3 8/9/2016 MyPastest

Blog (https://www.pastest.com/blog) About Pastest (https://www.pastest.com/about-us) Contact Us (https://www.pastest.com/contact-us) Help (https://www.pastest.com/help) © Pastest 2016

https://mypastest.pastest.com/Secure/TestMe/Browser/429893 3/3 8/9/2016 MyPastest

Back to Filters (/Secure/TestMe/Filter/429893/QA) Question 53 of 153

A 21-year-old man presents with fever, headache, myalgia and increasing breathlessness. A chest X-ray shows bilateral alveolar shadowing and he is commenced on a broad-spectrum antibiotic, cefotaxime. He remains pyrexial and develops a rash with erythematous papules and central pallor. He becomes anaemic and thrombocytopenic but his white cell count is normal. Cold agglutinins are present. Which one of the following options is the most likely causative agent?

A Borrelia burgdorferi

B Legionella pneumophila

C Mycoplasma

D Q fever

E Viral meningitis

Explanation

Mycoplasma Diagnostic considerations

The combination of a clinical diagnosis of pneumonia apparently resistant to cephalosporins with extrapulmonary features of an immune complex disease (the rash of erythema multiforme, anaemia, thrombocytopenia and presence of cold agglutins) in a young adult is most likely to be due to Mycoplasma The diagnosis can be confirmed by a rising antibody titre to Mycoplasma

Treatment

Treatment is with macrolide antibiotics or tetracycline

779

Next Question

https://mypastest.pastest.com/Secure/TestMe/Browser/429893 1/2 8/9/2016 MyPastest

Previous Question Tag Question Feedback End Review

Difficulty: Easy

Peer Responses

Session Progress

Responses Correct: 1

Responses Incorrect: 152

Responses Total: 153

Responses - % Correct: 1%

Blog (https://www.pastest.com/blog) About Pastest (https://www.pastest.com/about-us) Contact Us (https://www.pastest.com/contact-us) Help (https://www.pastest.com/help) © Pastest 2016

https://mypastest.pastest.com/Secure/TestMe/Browser/429893 2/2 8/9/2016 MyPastest

Back to Filters (/Secure/TestMe/Filter/429893/QA) Question 54 of 153

A 24-year-old woman is admitted with difficulty in breathing and unable to give a history. There is evidence of intravenous drug use with needle marks over the cutaneous fossas but no evidence of localised skin infection. The patient is apyrexial. Sensation is intact. She is admitted to ITU for ventilation. Which one of the following options is the most likely diagnosis?

A Anthrax

B Botulism

C Guillain–Barré syndrome

D Rabies

E Tetanus

Explanation

Botulism Aetiology

Botulism is caused by an exotoxin, produced by Clostridium botulinum, which leads to neuromuscular blockade The clue here is the presence of iv drug use, as wound botulism is commoner in this group, especially those using black tar heroin

Clinical features

Diplopia and laryngeal paralysis are early signs, consciousness is unaltered and sensation not affected

Differential diagnosis

Anthrax (bioterrorism) usually manifests as a cutaneous ulcer or a pneumonic illness with fever and cough https://mypastest.pastest.com/Secure/TestMe/Browser/429893 1/2 8/9/2016 MyPastest Guillain–Barré syndrome is an ascending demyelinating polyneuropathy affecting motor and sensory nerves In rabies and tetanus a wound is usually evident

rabies classically leads to hyperexcitability with hydrophobia, although an ascending paralysis akin to Guillain–Barré syndrome may be found spasms are characteristic of tetanus

783

Next Question

Previous Question Tag Question Feedback End Review

Difficulty: Average

Peer Responses

Session Progress

Responses Correct: 1

Responses Incorrect: 152

Responses Total: 153

Responses - % Correct: 1%

Blog (https://www.pastest.com/blog) About Pastest (https://www.pastest.com/about-us) Contact Us (https://www.pastest.com/contact-us) Help (https://www.pastest.com/help)

© Pastest 2016

https://mypastest.pastest.com/Secure/TestMe/Browser/429893 2/2 8/9/2016 MyPastest

Back to Filters (/Secure/TestMe/Filter/429893/QA) Question 55 of 153

A 32-year-old Thai woman attends the Emergency Department with a rash. Although she does not speak good English, she says she has had the rash for 2 months and that she has been taking the medication prescribed in Thailand for the last 4 weeks but has now run out of tablets. She says she has had several investigations including a skin biopsy and ‘needle tests’ on her ears and elbows, which she says were ‘positive’. On examination she has several erythematous, raised, plaque-like lesions on her arms and legs around 1 cm in diameter. They have not changed in appearance recently. She is apyrexial. Routine blood tests are normal. Which one of the following treatment options is most likely to be appropriate?

A Intravenous benzylpenicillin and flucloxacillin

B Isoniazid, rifampicin, pyrazinamide and ethambutol

C Prednisolone

D Clindamycin and ciprofloxacin

E Dapsone, clofazimine and rifampicin

Explanation

Leprosy – diagnosis

The diagnosis of leprosy may be made by sampling areas of the body that have a lower temperature, such as the ear lobes and elbows Patients with the lepromatous form of the disease are commonly acid-fast bacilli (AFB)-positive from these sites (modified stain) The bacteria may be cultured in the cold-blooded, nine-banded armadillo: this animal has been proposed to be a reservoir of infection in certain parts of the world

600

Next Question

https://mypastest.pastest.com/Secure/TestMe/Browser/429893 1/2 8/9/2016 MyPastest Previous Question Tag Question Feedback End Review

Difficulty: Average

Peer Responses

Session Progress

Responses Correct: 1

Responses Incorrect: 152

Responses Total: 153

Responses - % Correct: 1%

Blog (https://www.pastest.com/blog) About Pastest (https://www.pastest.com/about-us) Contact Us (https://www.pastest.com/contact-us) Help (https://www.pastest.com/help) © Pastest 2016

https://mypastest.pastest.com/Secure/TestMe/Browser/429893 2/2 8/9/2016 MyPastest

Back to Filters (/Secure/TestMe/Filter/429893/QA) Question 56 of 153

A 20-year-old teacher presents with a 4-day history of general malaise, conjunctivitis and a cough. He is starting to develop a maculopapular rash on his face and upper trunk. Which one of the following options is the most likely diagnosis?

A Parvovirus B19

B Measles

C Rubella

D EBV

E Primary HIV

Explanation

Differential diagnosis of rashes

The four-day prodrome with cough and conjunctivitis are typical of measles None of the other conditions in the list of options (parvovirus B19, rubella, Epstein–Barr virus (EBV), primary human immunodeficiency virus (HIV)) is associated with cough and conjunctivitis The rubella prodrome is 3 days or less The rash of parvovirus B19 appears in the convalescent phase of the illness, a week or more after the acute febrile illness

The rash of EBV is usually truncal

Primary HIV rashes are associated with painful oral ulceration and lymphadenopathy

3686

Next Question

Previous Question Tag Question Feedback End Review https://mypastest.pastest.com/Secure/TestMe/Browser/429893 1/2 8/9/2016 MyPastest

Difficulty: Average

Peer Responses

Session Progress

Responses Correct: 1

Responses Incorrect: 152

Responses Total: 153

Responses - % Correct: 1%

Blog (https://www.pastest.com/blog) About Pastest (https://www.pastest.com/about-us) Contact Us (https://www.pastest.com/contact-us) Help (https://www.pastest.com/help) © Pastest 2016

https://mypastest.pastest.com/Secure/TestMe/Browser/429893 2/2 8/9/2016 MyPastest

Back to Filters (/Secure/TestMe/Filter/429893/QA) Question 57 of 153

A 30-year-old man has a painless, large, spreading and exuberant ulcer with bright-red granulation tissue over his glans penis. There is no inguinal lymphadenopathy. Which one of the following options is the most likely causative organism?

A Treponema pallidum

B Chlamydia trachomatis

C Neisseria gonorrhoeae

D Haemophilus ducreyi

E Klebsiella granulomatis

Explanation

STIs

The clinical features in the present case are highly suggestive of granuloma inguinale ( donovanosis), which is caused by the Gram-negative pleomorphic bacillus Klebsiella granulomatis (previously known as Calymmatobacterium granulomatis or Donovania granulomatis) Extension of the primary infection from the external genitalia to the inguinal regions produces the characteristic lesion – pseudobubo N. gonorrhoeae and C. trachomatis present mainly with anterior urethritis, dysuria and/or urethral discharge

Ascending infection leads to epididymitis and prostatitis C. trachomatis types LGV 1, 2 and 3 cause lymphogranuloma venereum The primary lesion is a painless ulcerating papule on the genitalia A few days after this heals, regional lymphadenopathy develops The lymph nodes are painful and fixed These nodes – called ‘buboes’ – become fluctuant and can rupture

Chancroid, caused by Haemophilus ducreyi, presents as an erythematous papular lesion https://mypastest.pastest.com/Secure/TestMe/Browser/429893 1/2 8/9/2016 MyPastest that breaks down into a painful ulcer with a necrotic base with ragged edges Unilateral lymphadenopathy may be seen although it is often either absent or mild during the early stages of the disease A primary syphilitic lesion caused by T. pallidum would be expected to be associated with more marked lymphadenopathy

3590

Next Question

Previous Question Tag Question Feedback End Review

Difficulty: Difficult

Peer Responses

Session Progress

Responses Correct: 1

Responses Incorrect: 152

Responses Total: 153

Responses - % Correct: 1%

Blog (https://www.pastest.com/blog) About Pastest (https://www.pastest.com/about-us) Contact Us (https://www.pastest.com/contact-us) Help (https://www.pastest.com/help) © Pastest 2016

https://mypastest.pastest.com/Secure/TestMe/Browser/429893 2/2 8/9/2016 MyPastest

Back to Filters (/Secure/TestMe/Filter/429893/QA) Question 58 of 153

A 38-year-old man presents some 3 weeks after a stag party weekend in Prague. He has developed a painless ulcer on his penis. You suspect that he may have syphilis. Which one of the following percentages is the best estimate of how many untreated syphilis patients go on to develop late-stage CNS or cardiovascular complications?

A 80%

B 90%

C 10%

D 30%

E 0%

Explanation

Syphilis Definition

Syphilis is a sexually transmitted treponemal disease

Clinical features

The characteristic lesion is a painless chancre on the genitalia, mouth or anus and usually appears 3 weeks after first exposure Secondary syphilis is characterised by localised or diffuse mucocutaneous lesions and generalised lymphadenopathy Accompanying flu-like symptoms are common Secondary syphilis may begin around 4–6 weeks after the primary lesion and resolve between 1 and 52 weeks Between 60% and 80% of people with secondary syphilis have maculopapular lesions on their palms and soles, while condyloma lata papules form at areas of friction and moisture such as the vulva in women

https://mypastest.pastest.com/Secure/TestMe/Browser/429893 1/3 8/9/2016 MyPastest Some 21–58% of people with secondary syphilis develop mucosal lesions (usually pharyngitis or tonsillitis) Tertiary or late latent syphilis is characterised by gummas (nodular ulcerative lesions) affecting the skin, mucous membranes, skeletal system or viscera Cardiovascular manifestations of syphilis may include aortitis, aneurysms or aortic regurgitation Neurosyphilis may manifest as tabes dorsalis, general paralysis of the insane, iritis or choroidoretinitis

Diagnosis

Diagnosis is via dark-field microscopy of lesion fluid to identify bacteria, serological testing and by lumbar puncture if latent syphilis rather than primary infection is suspected

Treatment

Treatment is with penicillin, or with doxycycline in penicillin-allergic patients

2651

Next Question

Previous Question Tag Question Feedback End Review

Difficulty: Average

Peer Responses

Session Progress

Responses Correct: 1

Responses Incorrect: 152

Responses Total: 153

Responses - % Correct: 1%

https://mypastest.pastest.com/Secure/TestMe/Browser/429893 2/3 8/9/2016 MyPastest Blog (https://www.pastest.com/blog) About Pastest (https://www.pastest.com/about-us) Contact Us (https://www.pastest.com/contact-us) Help (https://www.pastest.com/help) © Pastest 2016

https://mypastest.pastest.com/Secure/TestMe/Browser/429893 3/3 8/9/2016 MyPastest

Back to Filters (/Secure/TestMe/Filter/429893/QA) Question 59 of 153

Which infection is now classified as an AIDS-defining illness in patients with HIV?

A Aspergillus

B Pseudomonas aeruginosa

C Staphylococcus aureus

D Mycobacterium tuberculosis

E Burkholderia cepacia

Explanation Staging in acquired immune deficiency syndrome

A clinical staging classification for HIV/AIDS has been developed by the World Health Organization for use in clinics without access to CD4 counts or viral load measurement Patients with asymptomatic disease or minor clinical problems are classified as stage 1 and 2, respectively. Examples of minor clinical problems include recurrent respiratory tract infections, angular chelitis and fungal nail infections The importance of high-grade virulent pathogens early on in the disease process is encompassed; pulmonary tuberculosis (now classified as an AIDS-defining disease in the United States) and severe bacterial infections are stage 3 problems Opportunistic infections characteristic of AIDS are classified as stage 4 events The staging system has recently been validated against CD4 counts in Uganda and is useful for predicting survival

http://aidsetc.org/resources#t-2 (http://aidsetc.org/resources#t-2) 5636

Next Question

https://mypastest.pastest.com/Secure/TestMe/Browser/429893 1/2 8/9/2016 MyPastest Previous Question Tag Question Feedback End Review

Difficulty: Average

Peer Responses

Session Progress

Responses Correct: 1

Responses Incorrect: 152

Responses Total: 153

Responses - % Correct: 1%

Blog (https://www.pastest.com/blog) About Pastest (https://www.pastest.com/about-us) Contact Us (https://www.pastest.com/contact-us) Help (https://www.pastest.com/help) © Pastest 2016

https://mypastest.pastest.com/Secure/TestMe/Browser/429893 2/2 8/9/2016 MyPastest

Back to Filters (/Secure/TestMe/Filter/429893/QA) Question 60 of 153

A gardener is admitted with difficulty moving his jaw after receiving a hand wound 1 week earlier. He starts to develop respiratory failure and is admitted to ITU. Which one of the following treatments should be avoided?

A Early physiotherapy

B Early ventilation and sedation

C Metronidazole

D Tetanus toxoid

E Wound debridement

Explanation

Tetanus

The clinical diagnosis in the present case is tetanus

Aetiology

Tetanus is caused by the effects on peripheral nerves of an exotoxin produced by Clostridium tetani

High-risk population

Neonates and the elderly are at risk of severe disease if they have not received tetanus toxoid

Management

Management includes wound debridement, administration of metronidazole and tetanus immunoglobulin, prompt ITU care with ventilation and sedation and the use of diazepam to control spasms https://mypastest.pastest.com/Secure/TestMe/Browser/429893 1/2 8/9/2016 MyPastest Patients should be rested in a quiet environment

774

Next Question

Previous Question Tag Question Feedback End Review

Difficulty: Difficult

Peer Responses

Session Progress

Responses Correct: 1

Responses Incorrect: 152

Responses Total: 153

Responses - % Correct: 1%

Blog (https://www.pastest.com/blog) About Pastest (https://www.pastest.com/about-us) Contact Us (https://www.pastest.com/contact-us) Help (https://www.pastest.com/help) © Pastest 2016

https://mypastest.pastest.com/Secure/TestMe/Browser/429893 2/2 8/9/2016 MyPastest

Back to Filters (/Secure/TestMe/Filter/429893/QA) Question 61 of 153

Which one of the following options is the best marker to monitor treatment response after subacute bacterial endocarditis treatment?

A Blood cultures

B White cell count

C Echocardiography

D Erythrocyte sedimentation rate

E C-reactive protein

Explanation

Serum markers for monitoring treatment response

The most useful laboratory test for monitoring the response to treatment (which is usually obvious clinically) is serial C-reactive protein estimation This is of much more use than the erythrocyte sedimentation rate, which is much slower to fall

5638

Next Question

Previous Question Tag Question Feedback End Review

Difficulty: Average

Peer Responses

https://mypastest.pastest.com/Secure/TestMe/Browser/429893 1/2 8/9/2016 MyPastest

Session Progress

Responses Correct: 1

Responses Incorrect: 152

Responses Total: 153

Responses - % Correct: 1%

Blog (https://www.pastest.com/blog) About Pastest (https://www.pastest.com/about-us) Contact Us (https://www.pastest.com/contact-us) Help (https://www.pastest.com/help)

© Pastest 2016

https://mypastest.pastest.com/Secure/TestMe/Browser/429893 2/2 8/9/2016 MyPastest

Back to Filters (/Secure/TestMe/Filter/429893/QA) Question 62 of 153

A 29-year-old missionary is admitted to the Emergency Department with a rash, fever and diarrhoea. She has been working in Bangladesh and has returned to the UK to visit relatives with her 8-week-old baby. You make a diagnosis of typhoid fever and wish to commence antibiotic therapy. Which one of the following antibiotics is the best choice, bearing in mind that she wishes to continue breast feeding?

A Ofloxacin

B Co-trimoxazole

C Ceftriaxone

D Ciprofloxacin

E Chloramphenicol

Explanation

Antibiotics and breast feeding

Quinolones are not recommended due to effects seen in animal studies on cartilage formation in growing mammals Nalidixic acid has not been associated with cartilage disruption, use of quinolones in breast feeding is at present contraindicated Chloramphenicol and co-trimoxazole may both lead to blood dyscrasias and as such should be avoided Ceftriaxone has excellent in vitro activity against Salmonella typhi and would be recommended in this case

No reports of significantly increased adverse events in pregnancy or breast feeding have been reported, but as in any prescribing decision, use of ceftriaxone is only considered when benefits outweigh the risks

14963 https://mypastest.pastest.com/Secure/TestMe/Browser/429893 1/2 8/9/2016 MyPastest

Next Question

Previous Question Tag Question Feedback End Review

Difficulty: Average

Peer Responses

Session Progress

Responses Correct: 1

Responses Incorrect: 152

Responses Total: 153

Responses - % Correct: 1%

Blog (https://www.pastest.com/blog) About Pastest (https://www.pastest.com/about-us) Contact Us (https://www.pastest.com/contact-us) Help (https://www.pastest.com/help) © Pastest 2016

https://mypastest.pastest.com/Secure/TestMe/Browser/429893 2/2 8/9/2016 MyPastest

Back to Filters (/Secure/TestMe/Filter/429893/QA) Question 63 of 153

An 18-year-old student is brought to the Emergency Department by ambulance. According to her house mate she has been complaining of a sore throat and flu-like symptoms over the past few days and has steadily deteriorated. When they called the ambulance she was confined to bed and mumbling incomprehensibly. On examination she is pyrexial 38.9o C, her BP is 85/40 mmHg, pulse is 105/min and regular. She has a generalised petechial rash, and evidence of a severe bacterial pharyngitis. When you move her neck she becomes increasingly agitated. There is obvious .

Which of the following is optimal immediate management?

A IV colloid

B IV crystalloid

C IV Benzylpenicillin

D IV Ceftriaxone

E IV corticosteroids

Explanation The answer is IV Ceftriaxone

Time is of the essence in managing this patient with suspected meningococcal septicaemia, with later administration of Ceftriaxone being strongly associated with a worse prognosis. Of course she also needs circulatory support with IV crystalloid coming a close second with regards to next steps. IV Benzylpenicillin is not recommended because of the possibility of meningococcal resistance to penicillin. IV corticosteroids may reduce hearing loss and neurological sequelae, but there is no evidence that these affect mortality. 36429

Next Question

Previous Question Tag Question Feedback End Review

https://mypastest.pastest.com/Secure/TestMe/Browser/429893 1/2 8/9/2016 MyPastest

Difficulty: Average

Peer Responses

Session Progress

Responses Correct: 1

Responses Incorrect: 152

Responses Total: 153

Responses - % Correct: 1%

Blog (https://www.pastest.com/blog) About Pastest (https://www.pastest.com/about-us) Contact Us (https://www.pastest.com/contact-us) Help (https://www.pastest.com/help) © Pastest 2016

https://mypastest.pastest.com/Secure/TestMe/Browser/429893 2/2 8/9/2016 MyPastest

Back to Filters (/Secure/TestMe/Filter/429893/QA) Question 64 of 153

A 23-year-old man from the Gambia presents with a 4-day history of fevers, headache and loose stools. He has not noticed any blood in his stools. His temperature is 38.5 °C, there are no signs of meningism and his abdomen is diffusely tender. Blood tests reveal leukocytosis and thrombocytopenia with a CRP of 85 mg/l. Which one of the following options is the most important investigation to perform at this stage?

A Blood cultures

B Chest X-ray

C Lumbar puncture

D Stool microscopy for ova, cysts and parasites

E Thick blood film

Explanation

Fever in travellers Investigating for malaria

A febrile illness in any traveller from a region where malaria is endemic must have a thick blood film to look for evidence of malaria parasites Platelet clumping with thrombocytopenia is a classical effect of the malarial parasite, and leukocytosis and a raised C-reactive protein (CRP) are commonly seen Being of Gambian origin the patient in the present case is likely to have partial immunity, which wanes on residing in a non-endemic region Malaria may present with a variety of signs, with headache and diarrhoea not infrequent features While making every effort to exclude malaria with three thick blood films, other diagnoses should be sought

Other diagnostic considerations

Enteric organisms such as salmonella and Entamoeba histolytica may lead to systemic https://mypastest.pastest.com/Secure/TestMe/Browser/429893 1/2 8/9/2016 MyPastest illness, hence the rationale for performing blood cultures and stool microscopy The atypical pneumonias such as Mycoplasma may present with extrapulmonary symptoms – a chest X-ray may reveal patchy consolidation Meningitis should always be held in the differential diagnosis: though with no rash (typical of meningococcal disease the commonest cause of meningitis in this age group) or meningism, it seems less likely

756

Next Question

Previous Question Tag Question Feedback End Review

Difficulty: Average

Peer Responses

Session Progress

Responses Correct: 1

Responses Incorrect: 152

Responses Total: 153

Responses - % Correct: 1%

Blog (https://www.pastest.com/blog) About Pastest (https://www.pastest.com/about-us) Contact Us (https://www.pastest.com/contact-us) Help (https://www.pastest.com/help) © Pastest 2016

https://mypastest.pastest.com/Secure/TestMe/Browser/429893 2/2 8/9/2016 MyPastest

Back to Filters (/Secure/TestMe/Filter/429893/QA) Question 65 of 153

A 35-year-old woman returns from a 2-year work placement in West Africa. She has had numerous insect bites. She has oedematous, red, itchy lesions on her forearms. Which one of the following options is the most appropriate investigation?

A Blood film

B Ultrasound

C Muscle biopsy

D Blood cultures

E CRP

Explanation

Diagnosis of loiasis

Traditionally the finding of microfilariae in a daytime blood sample or typical history and clinical findings confirms the diagnosis Loa Loa is also known as the African eye worm, as the worms occasionally make a slow passage across the eye, and this can actually be sensed by the patient Loiasis differs from other filarial diseases in that the skin lesions are confined to the extremities (ie arms and legs) It is thought that the skin lesions represent overlying areas of urticaria where muscle cysts occur around tendon sheaths Systemic microfilariasis is normally asymptomatic Diethylcarbamazine is the drug of choice

3709

Next Question

https://mypastest.pastest.com/Secure/TestMe/Browser/429893 1/2 8/9/2016 MyPastest Previous Question Tag Question Feedback End Review

Difficulty: Easy

Peer Responses

Session Progress

Responses Correct: 1

Responses Incorrect: 152

Responses Total: 153

Responses - % Correct: 1%

Blog (https://www.pastest.com/blog) About Pastest (https://www.pastest.com/about-us) Contact Us (https://www.pastest.com/contact-us) Help (https://www.pastest.com/help) © Pastest 2016

https://mypastest.pastest.com/Secure/TestMe/Browser/429893 2/2 8/9/2016 MyPastest

Back to Filters (/Secure/TestMe/Filter/429893/QA) Question 66 of 153

A 29-year-old farmer noticed several 2–3-cm long, whitish proglottids in his faeces. On closer examination they were motile, elongating and contracting. Given the likely clinical diagnosis, which one of the following options is the most appropriate therapy?

A Co-trimoxazole

B Fluconazole

C Niclosamide

D Methotrexate

E Thalidomide

Explanation

Tapeworm infection Clinicopathological features

Gravid proglottids are passed at defecation, often in short chains; free eggs also occur in faeces The whitish proglottids, approximately 2–3 cm long, are actively motile, elongating and contracting The presentation is similar to that in Taenia solium (pork tapeworm) and Taenia saginata (beef tapeworm) infection

Treatment

A single morning dose of 2 g niclosamide is given on an empty stomach to adults and older children; the tablets should be chewed Children of 2–6 years should receive 1 g, and those below 2 years, 500 mg The alternative is praziquantel given in a single dose of 10–20 mg/kg after a light breakfast After either drug the proximal part of the worm disintegrates in the gut and the scolex https://mypastest.pastest.com/Secure/TestMe/Browser/429893 1/2 8/9/2016 MyPastest cannot be found Failure of proglottids to reappear within 3–4 months indicates cure

2709

Next Question

Previous Question Tag Question Feedback End Review

Difficulty: Average

Peer Responses

Session Progress

Responses Correct: 1

Responses Incorrect: 152

Responses Total: 153

Responses - % Correct: 1%

Blog (https://www.pastest.com/blog) About Pastest (https://www.pastest.com/about-us) Contact Us (https://www.pastest.com/contact-us) Help (https://www.pastest.com/help)

© Pastest 2016

https://mypastest.pastest.com/Secure/TestMe/Browser/429893 2/2 8/9/2016 MyPastest

Back to Filters (/Secure/TestMe/Filter/429893/QA) Question 67 of 153

A 36-year-old woman presents complaining of a yellowish-green vaginal discharge that started 1 week ago. On examination her vagina is swollen and erythematous. Given the likely diagnosis, which one of the following options is the most appropriate treatment?

A Ampicillin

B Nystatin

C Metronidazole

D Fluconazole

E Erythromycin

Explanation

Trichomoniasis

The likely diagnosis in the present case is trichomoniasis

Treatment

Treatment is with nitroimidazoles; metronidazole is the most frequently used Metronidazole can be given either 400 mg orally twice daily for 5 days or as a single dose protocol

2685

Next Question

Previous Question Tag Question Feedback End Review

Difficulty: Average

https://mypastest.pastest.com/Secure/TestMe/Browser/429893 1/2 8/9/2016 MyPastest Peer Responses

Session Progress

Responses Correct: 1

Responses Incorrect: 152

Responses Total: 153

Responses - % Correct: 1%

Blog (https://www.pastest.com/blog) About Pastest (https://www.pastest.com/about-us) Contact Us (https://www.pastest.com/contact-us) Help (https://www.pastest.com/help) © Pastest 2016

https://mypastest.pastest.com/Secure/TestMe/Browser/429893 2/2 8/9/2016 MyPastest

Back to Filters (/Secure/TestMe/Filter/429893/QA) Question 68 of 153

A 17-year-old, non-pregnant, asymptomatic girl with no past medical history is found to have 106 colony-forming units of Escherichia coli/ml urine on a routine health check. Which one of the following options is the most appropriate management?

A Treat with oral co-trimoxazole for 10 days

B Treat with a single dose of oral trimethoprim

C Investigate her renal tract

D Treat with an intravenous antibiotic

E No antibiotics are indicated

Explanation

Asymptomatic bacteriuria Considerations in non-pregnant females

The girl in the present case has asymptomatic but significant bacteriuria This is common (approximately 3% of non-pregnant females) and does not require treatment as there is no risk of morbidity or mortality In fact, treatment of asymptomatic bacteriuria in a young person may increase the frequency of symptomatic infections However, treatment is required if there are co-morbid factors, eg diabetes, renal transplantation, invasive genitourinary investigations or a renal stone; however, single finding of asymptomatic bacteriuria is not an indication for renal tract investigation

Considerations in pregnant females

Pregnancy is an absolute indication to treat asymptomatic bacteriuria because of the increased risk of pyelonephritis (30%), pre-eclampsia, prematurity and perinatal death In pregnant females, a single dose of trimethoprim is as effective as the other antibiotic options mentioned in the question

297 https://mypastest.pastest.com/Secure/TestMe/Browser/429893 1/2 8/9/2016 MyPastest 297

Next Question

Previous Question Tag Question Feedback End Review

Difficulty: Easy

Peer Responses

Session Progress

Responses Correct: 1

Responses Incorrect: 152

Responses Total: 153

Responses - % Correct: 1%

Blog (https://www.pastest.com/blog) About Pastest (https://www.pastest.com/about-us) Contact Us (https://www.pastest.com/contact-us) Help (https://www.pastest.com/help) © Pastest 2016

https://mypastest.pastest.com/Secure/TestMe/Browser/429893 2/2 8/9/2016 MyPastest

Back to Filters (/Secure/TestMe/Filter/429893/QA) Question 69 of 153

Which one of the following serological markers shows vaccination success after hepatitis B immunisation?

A HBs antigen

B Anti-HBs antibodies

C Anti-HBe antibodies

D Anti-HBc antibodies

E HBe antigen

Explanation

Hepatitis B Use of serological markers

Prophylaxis, the development and level of the protective antibody (anti-HBs), is used to monitor the response to vaccination The antibody profile in the serum, together with the result of HBsAg and HBeAg, is used to define the phase of HBV infection

Vaccination considerations

Active immunisation for the prevention of HBV infection initially involved the use of a vaccine derived from viral proteins in infected blood, but now recombinant HBsAg proteins are used Vaccination strategies range from universal vaccination in infancy to the vaccination of only high-risk individuals In areas of high carriage in the Far East, universal vaccine programmes have already reduced the national incidence of infection, carriage and hepatocellular cancer Conventional three-dose immunisation in adults leads to protective immunity, as judged by anti-HBsAg, in 90% of individuals https://mypastest.pastest.com/Secure/TestMe/Browser/429893 1/2 8/9/2016 MyPastest Passive immunisation with anti-HBsAg hyperimmune globulin provides rapid protection after exposure (eg after a needlestick injury) A combination of passive and active immunisation is recommended for children born to infected mothers In some infants, chronic infection with a mutant escape virus has subsequently occurred

1300

Next Question

Previous Question Tag Question Feedback End Review

Difficulty: Easy

Peer Responses

Session Progress

Responses Correct: 1

Responses Incorrect: 152

Responses Total: 153

Responses - % Correct: 1%

Blog (https://www.pastest.com/blog) About Pastest (https://www.pastest.com/about-us) Contact Us (https://www.pastest.com/contact-us) Help (https://www.pastest.com/help) © Pastest 2016

https://mypastest.pastest.com/Secure/TestMe/Browser/429893 2/2 8/9/2016 MyPastest

Back to Filters (/Secure/TestMe/Filter/429893/QA) Question 70 of 153

A patient presents with hydrophobia, increased salivation, generalised spasms and increasing respiratory muscle paralysis. Which one of the following options is the most likely cause?

A Tetanus

B Rabies

C

D Actinomycosis

E Herpes encephalitis

Explanation

Rabies Clinical presentation

Most patients with rabies have the diagnostic symptom of hydrophobia: a combination of inspiratory muscle spasm, with or without painful laryngopharyngeal spasm, associated with terror Initially provoked by attempts to drink water, this reflex can be excited by a variety of stimuli including a draught of air (aerophobia), water splashed on the skin, irritation of the respiratory tract or, ultimately, the sight, sound or even mention of water The inspiratory spasm is violent and jerky The neck and back are extended, the arms thrown up and the episode may end in generalised convulsions with cardiac or respiratory arrest Patients experience hyperaesthesia and generalised arousal, during which they become wild, hallucinated, fugitive and aggressive, alternating with lucid intervals Despite these symptoms, attributable to brainstem encephalitis, neurological examination may prove surprisingly normal Abnormalities include meningism, cranial nerve lesions (especially III, VI, VII, IX, X, XI and XII), upper motor neurone lesions, fasciculation and involuntary movements

https://mypastest.pastest.com/Secure/TestMe/Browser/429893 1/2 8/9/2016 MyPastest Disturbances of the hypothalamus or autonomic nervous system cause hypersalivation, lacrimation, sweating, hypertension or hypotension, hyperthermia or hypothermia, inappropriate secretion of antidiuretic hormone, or diabetes insipidus and, rarely, priapism with spontaneous orgasms

Prognosis

Without supportive treatment, about one-third of the patients will die during a hydrophobic spasm in the first few days The rest lapse into coma and generalised flaccid paralysis, and rarely survive for more than a week without intensive care

1327

Next Question

Previous Question Tag Question Feedback End Review

Difficulty: Easy

Peer Responses

Session Progress

Responses Correct: 1

Responses Incorrect: 152

Responses Total: 153

Responses - % Correct: 1%

Blog (https://www.pastest.com/blog) About Pastest (https://www.pastest.com/about-us) Contact Us (https://www.pastest.com/contact-us) Help (https://www.pastest.com/help)

© Pastest 2016

https://mypastest.pastest.com/Secure/TestMe/Browser/429893 2/2 8/9/2016 MyPastest

Back to Filters (/Secure/TestMe/Filter/429893/QA) Question 71 of 153

A 17-year-old woman presents with mild drowsiness, headache and neck stiffness. She has also had bilateral painful parotid swellings over the past few days. On examination she is pyrexial, 38.2 °C, with signs of mild meningism. There are bilateral parotid swellings. GCS is 15 at the time of examination with no focal neurological signs.

Investigations:

Hb 13.1 g/dl

WCC 8.2 × 109 /l, slight lymphocytosis

PLT 230 × 109 /l

Amylase 340 U/l

Na+ 141 mmol/l

K+ 4.8 mmol/l

Creatinine 110 μmol/l

CSF lymphocytes 16

CSF protein 0.50 g/dl

CSF glucose 2.8 mmol/l

Blood glucose 3.5 mmol/l

Which one of the following represents the most appropriate medication for the patient in the present case?

A Cefotaxime iv

B Aciclovir IV

C Benzylpenicillin iv

D Paracetamol and codeine

E Ciprofloxacin iv https://mypastest.pastest.com/Secure/TestMe/Browser/429893 1/3 8/9/2016 MyPastest

Explanation

Mumps Diagnosis

The clinical picture in the present case is consistent with mumps (paramyxovirus infection) complicated by mild central nervous system (CNS) involvement Mumps virus can be isolated from nasopharyngeal swabs, urine, blood and buccal fluid for 7–9 days after the onset of parotitis PCR is available for virus detection, the alternative for confirming the diagnosis being viral serology

Management and prognosis

Management is conservative with appropriate hydration and pain relief Cerebellar ataxia may occur post-CNS involvement Mortality for mumps with CNS involvement is around 1.4%

20475

Next Question

Previous Question Tag Question Feedback End Review

Difficulty: Difficult

Peer Responses

Session Progress

Responses Correct: 1

Responses Incorrect: 152

Responses Total: 153

Responses - % Correct: 1%

https://mypastest.pastest.com/Secure/TestMe/Browser/429893 2/3 8/9/2016 MyPastest Blog (https://www.pastest.com/blog) About Pastest (https://www.pastest.com/about-us) Contact Us (https://www.pastest.com/contact-us) Help (https://www.pastest.com/help) © Pastest 2016

https://mypastest.pastest.com/Secure/TestMe/Browser/429893 3/3 8/9/2016 MyPastest

Back to Filters (/Secure/TestMe/Filter/429893/QA) Question 72 of 153

There are some important differences between the life-cycle of Plasmodium vivax and that of Plasmodium falciparum. From the list below, which one life-cycle stage occurs in P. vivax but not in P. falciparum infection?

A Gametocytes

B Hypnozoites

C Schizonts

D Sporozoites

E Trophozoites

Explanation

Malaria life-cycle

It is important to know the malaria life-cycle Hypnozoites represent the ‘dormant’ liver stage of the life-cycle of Plasmodium vivax and P. ovale infection (ie benign malarias), and can cause late relapses after treatment They are not part of the life-cycle of P. falciparum To eradicate hypnozoites and prevent such relapses, a course of primaquine must be given following chloroquine therapy for P. vivax and P. ovale infection Primaquine is not required after quinine therapy for P. falciparum, but Fansidar (pyrimethamine + sulfadoxine) or tetracycline is given to cover the possibility of low- grade quinine resistance

1639

Next Question

Previous Question Tag Question Feedback End Review

https://mypastest.pastest.com/Secure/TestMe/Browser/429893 1/2 8/9/2016 MyPastest

Difficulty: Average

Peer Responses

Session Progress

Responses Correct: 1

Responses Incorrect: 152

Responses Total: 153

Responses - % Correct: 1%

Blog (https://www.pastest.com/blog) About Pastest (https://www.pastest.com/about-us) Contact Us (https://www.pastest.com/contact-us) Help (https://www.pastest.com/help) © Pastest 2016

https://mypastest.pastest.com/Secure/TestMe/Browser/429893 2/2 8/9/2016 MyPastest

Back to Filters (/Secure/TestMe/Filter/429893/QA) Question 73 of 153

A 19-year-old man, born and brought up in Nepal and now studying accountancy, attended the outpatients’ clinic with a swelling in his neck. He had a history of fevers, night sweats and weight loss. An AFB stain from a fine-needle aspiration proved positive and he was commenced on quadruple antituberculous therapy. Now, 2 weeks later, his LFTs have become deranged with an ALT of 400 U/l and bilirubin of 50 μmol/l. Which one of the following options is the most important management step?

A Liver ultrasound

B Blood cultures

C Stop the TB medication

D HIV test

E Give prednisolone

Explanation

Treatment of tuberculosis Monitoring treatment

The British Thoracic Society (BTS) guidelines recommend stopping TB medication in the presence of a transaminitis > 5 times normal and/or a rising bilirubin level The medications can be gradually re-introduced at increasing doses once the LFTs have normalised Isoniazid, rifampicin and pyrazinamide can all cause hepatotoxicity, which may be fatal

620

Next Question

Previous Question Tag Question Feedback End Review

https://mypastest.pastest.com/Secure/TestMe/Browser/429893 1/2 8/9/2016 MyPastest

Difficulty: Average

Peer Responses

Session Progress

Responses Correct: 1

Responses Incorrect: 152

Responses Total: 153

Responses - % Correct: 1%

Blog (https://www.pastest.com/blog) About Pastest (https://www.pastest.com/about-us) Contact Us (https://www.pastest.com/contact-us) Help (https://www.pastest.com/help) © Pastest 2016

https://mypastest.pastest.com/Secure/TestMe/Browser/429893 2/2 8/9/2016 MyPastest

Back to Filters (/Secure/TestMe/Filter/429893/QA) Question 74 of 153

A 36-year-old woman presents complaining of a yellowish-green vaginal discharge that started 1 week ago. On examination her vagina is swollen and erythematous. Which one of the following options is the most likely diagnosis?

A Candidiasis

B Trichomoniasis

C AIDS

D Papillomavirus infection

E Lactobacilli infection

Explanation

Trichomoniasis History

Symptomatic patients with trichomoniasis most frequently complain of a discharge and vaginal pruritus Intermenstrual or postcoital spotting may occur because the ectocervix is involved Occasionally the urethra and Skene’s glands may be infected, resulting in dysuria

Clinical features

Signs of trichomoniasis are vaginal discharge (42%), odour (50%), oedema or erythema (22–37%) The often copious discharge is described as frothy and yellowish-green, but varies in consistency and colour and is actually frothy in only 8–12% of women Colpitis macularis (strawberry cervix), detected by colposcopy, is reported in almost half the patients and is the most specific clinical sign for trichomoniasis, but is rarely seen during routine examination Some 50% of all infected women are probably asymptomatic

2683 https://mypastest.pastest.com/Secure/TestMe/Browser/429893 1/2 8/9/2016 MyPastest 2683

Next Question

Previous Question Tag Question Feedback End Review

Difficulty: Easy

Peer Responses

Session Progress

Responses Correct: 1

Responses Incorrect: 152

Responses Total: 153

Responses - % Correct: 1%

Blog (https://www.pastest.com/blog) About Pastest (https://www.pastest.com/about-us) Contact Us (https://www.pastest.com/contact-us) Help (https://www.pastest.com/help) © Pastest 2016

https://mypastest.pastest.com/Secure/TestMe/Browser/429893 2/2 8/9/2016 MyPastest

Back to Filters (/Secure/TestMe/Filter/429893/QA) Question 75 of 153

A 32-year-old woman has just returned from a holiday in the Middle-East. She had to spend much of the flight in the toilet and has been brought by ambulance from the airport. On admission she is severely dehydrated and gives a history of passing voluminous watery stools that look like rice water, which were initially mixed with mucus and blood. Blood testing reveals a raised haemoglobin, markedly raised urea and raised creatinine. Blood glucose is measured at only 3.1 mmol/l (normal 3.0–6.0). Which one of the following diagnoses fits best with this clinic picture?

A Cholera

B Typhoid fever

C Shigella

D Salmonella

E Amoebic dysentery

Explanation

Cholera History

Cholera often presents with an abrupt onset of diarrhoea with classically described rice-water stools

Complications

If cholera is not treated, severe dehydration, hypoglycaemia, acidosis and death may result

Transmission of infection

Infection may occur after the ingestion of shellfish

Diagnosis https://mypastest.pastest.com/Secure/TestMe/Browser/429893 1/3 8/9/2016 MyPastest

The diagnosis is made via a wet mount of stool, visualised under dark-ground microscopy to demonstrate the characteristic organisms with darting motility

Treatment

Adequate fluid replacement is the key to treatment, usually with oral rehydration fluids, but iv support may be required Antibiotic therapy may shorten the course of the illness and reduce the volume of diarrhoeal fluid output: doxycycline is an appropriate choice for antibiotic

Complications

If cholera is not treated, severe dehydration, hypoglycaemia, acidosis and death may result Mortality among adequately treated patients is less than 1%, although this is appreciably higher in the developing world

2657

Next Question

Previous Question Tag Question Feedback End Review

Difficulty: Easy

Peer Responses

Session Progress

Responses Correct: 1

Responses Incorrect: 152

Responses Total: 153

Responses - % Correct: 1%

Blog (https://www.pastest.com/blog) About Pastest (https://www.pastest.com/about-us) Contact Us (https://www.pastest.com/contact-us) Help (https://www.pastest.com/help) https://mypastest.pastest.com/Secure/TestMe/Browser/429893 2/3 8/9/2016 MyPastest © Pastest 2016

https://mypastest.pastest.com/Secure/TestMe/Browser/429893 3/3 8/9/2016 MyPastest

Back to Filters (/Secure/TestMe/Filter/429893/QA) Question 76 of 153

A 45-year-old woman returns from South Africa with confusion and headache, but with no neck stiffness. She has a purpuric rash. Malaria prophylaxis included mefloquine. Which one of the following investigations is most important in this case?

A Lumbar puncture

B Computed tomography (CT) scan

C Multiple blood cultures

D Malaria films

E Clotting

Explanation

Cerebral malaria History

Although the patient in the present case has used malaria prophylaxis, she is still at risk of malaria infection There are symptoms of possible cerebral malaria here and if this goes unrecognised, it is universally fatal

Causative organism

The most likely malarial parasite with these symptoms would be Plasmodium falciparum

Prognosis

Cerebral malaria is associated with diminished consciousness and confusion, eventually patients progress to coma and death

Differential diagnosis https://mypastest.pastest.com/Secure/TestMe/Browser/429893 1/2 8/9/2016 MyPastest

Blackwater fever may also occur in association with P. falciparum infection and is associated with intravascular haemolysis, causing the characteristic dark urine

Treatment

Widespread P. falciparum resistance to chloroquine now exists, so that patients are now managed with quinine for acute infection

This is given either orally, or intravenously in severe cases

5235

Next Question

Previous Question Tag Question Feedback End Review

Difficulty: Average

Peer Responses

Session Progress

Responses Correct: 1

Responses Incorrect: 152

Responses Total: 153

Responses - % Correct: 1%

Blog (https://www.pastest.com/blog) About Pastest (https://www.pastest.com/about-us) Contact Us (https://www.pastest.com/contact-us) Help (https://www.pastest.com/help) © Pastest 2016

https://mypastest.pastest.com/Secure/TestMe/Browser/429893 2/2 8/9/2016 MyPastest

Back to Filters (/Secure/TestMe/Filter/429893/QA) Question 77 of 153

A 54-year-old man being treated in the Infectious Diseases Unit becomes acutely unwell. A diagnosis of the Jarisch–Herxheimer reaction is made. Which one of the following is most likely to be the cause?

A Brucellosis

B Dengue fever

C Leishmaniasis

D Borreliosis

E Typhoid

Explanation

Jarisch–Herxheimer reaction

The Jarisch–Herxheimer reaction can occur several hours after the administration of the first dose of an antibiotic due to the sudden release of endotoxin The systemic reaction that ensues can be fatal It is a feature of all spirochaete illnesses including syphilis, leptospirosis and borreliosis (relapsing fever and ) infections It may also occur in brucellosis but this condition is rarer

775

Next Question

Previous Question Tag Question Feedback End Review

Difficulty: Difficult

Peer Responses https://mypastest.pastest.com/Secure/TestMe/Browser/429893 1/2 8/9/2016 MyPastest

Session Progress

Responses Correct: 1

Responses Incorrect: 152

Responses Total: 153

Responses - % Correct: 1%

Blog (https://www.pastest.com/blog) About Pastest (https://www.pastest.com/about-us) Contact Us (https://www.pastest.com/contact-us) Help (https://www.pastest.com/help)

© Pastest 2016

https://mypastest.pastest.com/Secure/TestMe/Browser/429893 2/2 8/9/2016 MyPastest

Back to Filters (/Secure/TestMe/Filter/429893/QA) Question 78 of 153

A 30-year-old man is admitted with a 1-week history of fevers and breathlessness. He is an intravenous drug user. On examination he is hypotensive, his JVP is raised with giant cv waves and there is a pansystolic murmur. He had received 2 days of oral amoxicillin for a chest infection prior to admission. Blood cultures are taken and empirical antibiotics started. Which one of the following options is the most likely causative organism?

A Candida spp

B Enterococcus spp

C HACEK group

D Staphylococcus aureus

E Viridans group streptococci

Explanation

Infective endocarditis Right-sided endocarditis

The man in the present case is septic with signs of acute tricuspid regurgitation, likely to be secondary to infective endocarditis Intravenous drug users are prone to right-sided endocarditis, and by far the most likely organism is Staphylococcus aureus (50–70% of cases) acquired from the skin The presentation is usually rapid with a high risk of valve prolapse and cardiac failure

Left-sided endocarditis

Left-sided native valve endocarditis with a longer prodrome is typical of viridans group streptococci acquired from the oral cavity

HACEK

HACEK is the acronym for: Haemophilus, Actinobacillus, Cardiobacter, Eikenella, and https://mypastest.pastest.com/Secure/TestMe/Browser/429893 1/2 8/9/2016 MyPastest Kingella spp

767

Next Question

Previous Question Tag Question Feedback End Review

Difficulty: Average

Peer Responses

Session Progress

Responses Correct: 1

Responses Incorrect: 152

Responses Total: 153

Responses - % Correct: 1%

Blog (https://www.pastest.com/blog) About Pastest (https://www.pastest.com/about-us) Contact Us (https://www.pastest.com/contact-us) Help (https://www.pastest.com/help) © Pastest 2016

https://mypastest.pastest.com/Secure/TestMe/Browser/429893 2/2 8/9/2016 MyPastest

Back to Filters (/Secure/TestMe/Filter/429893/QA) Question 79 of 153

A 40-year-old patient presents with a sudden collapse. By the time you see her in the Emergency Department she feels as if she is back to normal. She recalls having had a tick bite about 3 months previously when in Thetford Forest in Norfolk. The bite was followed by a prolonged local rash. What is the likely cause of the collapse?

A Meningoencephalitis

B Acute Bell’s palsy

C Atrioventricular heart block

D Bannwarth syndrome

E Acute large joint arthritis

Explanation

Lyme disease

Lyme disease can cause all of the problems in the list of options but the likely cause of the present patient’s collapse is an atrioventricular (AV) block This usually responds to treatment with ceftriaxone

3703

Next Question

Previous Question Tag Question Feedback End Review

Difficulty: Easy

Peer Responses

https://mypastest.pastest.com/Secure/TestMe/Browser/429893 1/2 8/9/2016 MyPastest

Session Progress

Responses Correct: 1

Responses Incorrect: 152

Responses Total: 153

Responses - % Correct: 1%

Blog (https://www.pastest.com/blog) About Pastest (https://www.pastest.com/about-us) Contact Us (https://www.pastest.com/contact-us) Help (https://www.pastest.com/help)

© Pastest 2016

https://mypastest.pastest.com/Secure/TestMe/Browser/429893 2/2 8/9/2016 MyPastest

Back to Filters (/Secure/TestMe/Filter/429893/QA) Question 80 of 153

A 33-year-old woman has returned from a holiday in Africa. She has been diagnosed as having malaria due to Plasmodium falciparum. Which one of the following statements is FALSE?

A Following successful treatment, fever recurs due to persistence of the parasite in the liver

B Cough and mild diarrhoea are a common presentation

C Splenectomy increases the risk of infection

D Jaundice is usually due to haemolysis and hepatitis

E The fever has no particular pattern

Explanation

Malaria

Plasmodium falciparum malaria is more dangerous than other types of malaria It does not have a persistent exoerythrocytic phase, but recurrence of fever is due to multiplication in the red blood cells of parasites that have not been eliminated by treatment Splenectomy increases the risk of acquiring malaria, and jaundice is commonly due to hepatitis and haemolysis Where a pattern occurs, the fever in P. falciparum malaria follows a 48-h periodicity, although a pattern is not invariably seen

1361

Next Question

Previous Question Tag Question Feedback End Review

https://mypastest.pastest.com/Secure/TestMe/Browser/429893 1/2 8/9/2016 MyPastest

Difficulty: Average

Peer Responses

Session Progress

Responses Correct: 1

Responses Incorrect: 152

Responses Total: 153

Responses - % Correct: 1%

Blog (https://www.pastest.com/blog) About Pastest (https://www.pastest.com/about-us) Contact Us (https://www.pastest.com/contact-us) Help (https://www.pastest.com/help)

© Pastest 2016

https://mypastest.pastest.com/Secure/TestMe/Browser/429893 2/2 8/9/2016 MyPastest

Back to Filters (/Secure/TestMe/Filter/429893/QA) Question 81 of 153

A keratitis with dendritic ulceration of the cornea is diagnosed in a 32-year-old patient. Which one of the following options is the most likely cause?

A Adenovirus

B Chlamydia

C Gram-positive bacteria

D Herpes simplex virus

E Reduced tear formation

Explanation

Herpes simplex virus keratitis

Herpes simplex virus (HSV) keratitis is characterised by the acute onset of pain, blurred vision, conjunctival injection and dendritic ulceration of the cornea HSV keratitis can cause corneal blindness and thus requires urgent treatment Topical aciclovir is the drug of choice, as topical steroids may make the infection worse HSV can also cause an acute necrotising retinitis, usually seen in immunosuppressed patients such as those with HIV infection, but rarely in immunocompetent people

1299

Next Question

Previous Question Tag Question Feedback End Review

Difficulty: Easy

Peer Responses

https://mypastest.pastest.com/Secure/TestMe/Browser/429893 1/2 8/9/2016 MyPastest

Session Progress

Responses Correct: 1

Responses Incorrect: 152

Responses Total: 153

Responses - % Correct: 1%

Blog (https://www.pastest.com/blog) About Pastest (https://www.pastest.com/about-us) Contact Us (https://www.pastest.com/contact-us) Help (https://www.pastest.com/help)

© Pastest 2016

https://mypastest.pastest.com/Secure/TestMe/Browser/429893 2/2 8/9/2016 MyPastest

Back to Filters (/Secure/TestMe/Filter/429893/QA) Question 82 of 153

A 23-year-old woman presents to the Sexual Health Clinic. She had unprotected sex after an office party 4 days ago. She is currently taking antibiotics for a respiratory tract infection. There is intense difficulty passing urine, accompanied by burning, itching and pain over her labia. On examination there is a crop of vesicles with ulceration. Which one of the following diagnoses fits best with this clinical picture?

A Syphilis

B Herpes zoster infection

C Herpes simplex infection (HSV-1)

D Herpes simplex infection (HSV-2)

E Stevens–Johnson syndrome

Explanation

Herpes simplex infection Causative organisms

Herpes simplex infection is associated with two viral subtypes, HSV-1 and 2 HSV-1 infection is primarily associated with oral infections, whereas HSV-2 is mainly responsible for genital infections Following primary infection, the virus enters nerve endings in the skin and ascends to the dorsal root ganglia where it remains latent until reactivated

Symptoms

Symptoms occur 3–7 days after contact These may include a low-grade fever and general malaise Symptoms at the site of infection include pain, burning, itching and tingling and the presence of groups of vesicles with surrounding erythema (these usually ulcerate and then crust over within 48 h) Lesions generally last in total between 2 and 6 weeks and heal without scarring https://mypastest.pastest.com/Secure/TestMe/Browser/429893 1/2 8/9/2016 MyPastest Urinary retention due to associated pain may occur in women Recurrent infection may occur due to fatigue, stress, in relation to the menstrual cycle, local skin trauma or exposure to sunlight

Management

Treatment is with aciclovir or a similar antiviral agent Studies have shown that the chronic use of oral antiviral therapy may reduce the frequency and severity of recurrent infection

2649

Next Question

Previous Question Tag Question Feedback End Review

Difficulty: Average

Peer Responses

Session Progress

Responses Correct: 1

Responses Incorrect: 152

Responses Total: 153

Responses - % Correct: 1%

Blog (https://www.pastest.com/blog) About Pastest (https://www.pastest.com/about-us) Contact Us (https://www.pastest.com/contact-us) Help (https://www.pastest.com/help)

© Pastest 2016

https://mypastest.pastest.com/Secure/TestMe/Browser/429893 2/2 8/9/2016 MyPastest

Back to Filters (/Secure/TestMe/Filter/429893/QA) Question 83 of 153

A 45-year-old man presents with fever, headache, and progressive confusion over the past 72 hours. His wife also describes a short period of vacant staring which may have been an absence seizure. He has no past medical history of note apart from slight symptoms of a cold over the past few days. On examination his BP is 125/72 mmHg, pulse is 72/min and regular. His temperature is 37.9o C. He is drowsy with signs of meningism and repeats the same question three times during the examination. CSF shows an opening pressure of 28 cm, protein 0.8 g/l, glucose 4.5 mmol/l (blood glucose 6.2 mmol/l), 560 lymphocytes/mm3 . Which is the most likely causative agent?

A Herpes simplex type 2

B HIV

C Listeria monocytogenes

D Tuberculosis

E Streptococcus pneumoniae

Explanation

Meningitis

HSV-2 meningitis is a common cause of recurrent aseptic meningitis (formerly Mollaret’s meningitis) Factors associated with its onset are uncertain although a subacute onset may be seen in patients who have immunosuppression The history of headache, progressive confusion and a possible seizure, coupled with the CSF picture support the diagnosis Tuberculosis is associated with a longer onset of symptoms, and we are not told of any respiratory symptoms or risk factors associated with infection Listeriosis is associated with immunocompromise/exposure to high risk foodstuffs such as unpasteurised cheeses etc Secondary syphilis, Lyme disease and acute HIV seroconversion can all present with

https://mypastest.pastest.com/Secure/TestMe/Browser/429893 1/2 8/9/2016 MyPastest lymphocytic meningitis with ‘viral’ levels of protein and glucose

3705

Next Question

Previous Question Tag Question Feedback End Review

Difficulty: Average

Peer Responses

Session Progress

Responses Correct: 1

Responses Incorrect: 152

Responses Total: 153

Responses - % Correct: 1%

Blog (https://www.pastest.com/blog) About Pastest (https://www.pastest.com/about-us) Contact Us (https://www.pastest.com/contact-us) Help (https://www.pastest.com/help) © Pastest 2016

https://mypastest.pastest.com/Secure/TestMe/Browser/429893 2/2 8/9/2016 MyPastest

Back to Filters (/Secure/TestMe/Filter/429893/QA) Question 84 of 153

An 18-year-old woman has been diagnosed with human papillomavirus infection. Which one of the following options is the most significant long-term risk following this infection?

A Coronary artery disease

B Endometriosis

C Infertility

D Cervical cancer

E Carcinoma of the endometrium

Explanation

Aetiology of cervical cancer

There is now overwhelming evidence that the vast majority of cervical cancers are caused by specific types of the human papillomavirus DNA from human papillomavirus types 16, 18, 31, 33 and 35 (mostly type 16) has been found in as many as 99% of cervical cancers

2689

Next Question

Previous Question Tag Question Feedback End Review

Difficulty: Easy

Peer Responses

https://mypastest.pastest.com/Secure/TestMe/Browser/429893 1/2 8/9/2016 MyPastest

Session Progress

Responses Correct: 1

Responses Incorrect: 152

Responses Total: 153

Responses - % Correct: 1%

Blog (https://www.pastest.com/blog) About Pastest (https://www.pastest.com/about-us) Contact Us (https://www.pastest.com/contact-us) Help (https://www.pastest.com/help)

© Pastest 2016

https://mypastest.pastest.com/Secure/TestMe/Browser/429893 2/2 8/9/2016 MyPastest

Back to Filters (/Secure/TestMe/Filter/429893/QA) Question 85 of 153

A 50-year-old man is admitted following a 1-week history of myalgia, rash, headache and conjunctivitis. He feels increasingly unwell with neck stiffness. On examination Kernig’s sign is positive and hepatosplenomegaly is noted. He has a creatinine concentration of 180 μmol/l, ALT 250 IU/l, bilirubin 90 μmol/l and raised inflammatory markers with a neutrophilia. A lumbar puncture confirms meningitis with a CSF lymphocytosis. Which one of the following options is the most likely causative agent?

A Borrelia burgdorferi

B Leptospira ictohaemorrhagica

C Mycoplasma spp

D Treponema pallidum

E Typhus

Explanation

Weil’s disease

Clinical course

Leptospira ictohaemorrhagica (Weil’s disease) classically has two phases: a leptospiraemic phase, lasting a week with generalised symptoms of myalgia, rash and headache; and an immune phase, often with a lymphocytic meningitis Some patients go on to develop hepatitis, hepatosplenomegaly and renal failure

Diagnosis

Diagnosis can be confirmed by culture from blood, urine or CSF in the first week or by serology after 7–10 days

Differential diagnosis

Borrelia burgdorferi (Lyme disease) and Treponema pallidum (syphilis) can both cause https://mypastest.pastest.com/Secure/TestMe/Browser/429893 1/2 8/9/2016 MyPastest lymphocytic meningitis, but renal failure and hepatitis are not features and a longer prodrome is characteristic of these infections Mycoplasma usually causes pneumonia but may present with fever, headache, myalgia and diarrhoea Typhus is a rickettsial infection, which may cause a meningoencephalitis but is no longer endemic to the UK

777

Next Question

Previous Question Tag Question Feedback End Review

Difficulty: Average

Peer Responses

Session Progress

Responses Correct: 1

Responses Incorrect: 152

Responses Total: 153

Responses - % Correct: 1%

Blog (https://www.pastest.com/blog) About Pastest (https://www.pastest.com/about-us) Contact Us (https://www.pastest.com/contact-us) Help (https://www.pastest.com/help) © Pastest 2016

https://mypastest.pastest.com/Secure/TestMe/Browser/429893 2/2 8/9/2016 MyPastest

Back to Filters (/Secure/TestMe/Filter/429893/QA) Question 86 of 153

A 47-year-old woman undergoing chemotherapy for breast cancer has a neutrophil count of 0.2 × 109 /l. She presents with an acute deterioration in conscious level associated with headache, photophobia and neck stiffness. Following CT and CSF examination, a provisional diagnosis of bacterial meningitis is made. Which one of the following organisms is most likely to be responsible for this patient’s condition?

A Pseudomonas aeruginosa

B Staphylococcus aureus

C Haemophilus influenzae

D Listeria monocytogenes

E Group B streptococcus

Explanation

Bacterial meningitis High-risk population

In immunosuppressed patients, the commonest pathogens causing bacterial meningitis are the following: Streptococcus pneumoniae, Neisseria meningitidis, Listeria monocytogenes and aerobic gram-negative bacilli. Pseudomonas aeruginosa may cause meningitis following intracranial manipulation, including neurosurgery Meningitis caused by Listeria monocytogenes occurs mainly in adult patients with malignancies, in transplant recipients and in immunosuppressed and elderly people

Disease development

Meningoencephalitis may start abruptly In adults, it can also develop insidiously, with progressive focal neurological signs even in the absence of a brain abscess https://mypastest.pastest.com/Secure/TestMe/Browser/429893 1/3 8/9/2016 MyPastest Most patients have meningism, but fever may not be marked, particularly in elderly or immunosuppressed people

Clinical presentation

Bacterial meningitis should be considered in any patient with an acute brainstem disorder associated with fever, particularly if there are no risk factors for cerebrovascular disease Cerebritis is increasingly recognised, particularly in the immunosuppressed patient Headache, fever and varying degrees of paralysis can resemble a cerebrovascular accident Rhomboencephalitis begins with a headache, fever, nausea and vomiting followed in several days’ time by symmetrical, progressive cranial nerve palsies, decreased consciousness and cerebellar signs

Further investigations

Areas of uptake without ring enhancement may be shown by magnetic resonance imaging (MRI) or computed tomography (CT) The cerebrospinal fluid shows few, if any, cells and normal protein and sugar concentrations

Treatment considerations

Since meningitis due to L. monocytogenes is not distinguishable clinically from other types of bacterial meningitis, it is recommended to cover Listeria spp in the initial empirical therapy of bacterial meningitis in immunosuppressed patients

1289

Next Question

Previous Question Tag Question Feedback End Review

Difficulty: Average

Peer Responses

https://mypastest.pastest.com/Secure/TestMe/Browser/429893 2/3 8/9/2016 MyPastest

Session Progress

Responses Correct: 1

Responses Incorrect: 152

Responses Total: 153

Responses - % Correct: 1%

Blog (https://www.pastest.com/blog) About Pastest (https://www.pastest.com/about-us) Contact Us (https://www.pastest.com/contact-us) Help (https://www.pastest.com/help)

© Pastest 2016

https://mypastest.pastest.com/Secure/TestMe/Browser/429893 3/3 8/9/2016 MyPastest

Back to Filters (/Secure/TestMe/Filter/429893/QA) Question 87 of 153

Infection with Campylobacter spp usually spreads by which one of the following modes of transmission?

A Contaminated food

B Contaminated needles

C Droplet transmission

D Direct contact with the patient

E Vectors (ticks)

Explanation

Gastroenteritis Epidemiology

Campylobacter enteritis is the most common bacterial infection of the gut in industrialised countries Some 55,000 laboratory isolations per annum are currently reported in the UK, representing an annual incidence of 100/100,000, but the true incidence is likely to be at least ten times this figure In developed countries campylobacter enteritis affects people of all ages, especially young adults However, in developing countries it is almost entirely confined to children below the age of 2–3 years, after which they are immune through repeated exposure to infection

Risk factors

Normal cooking destroys Campylobacter, but the consumption of raw or barbecued meats, especially poultry, carries a distinct risk of infection Broiler chickens are the most prolific source of Campylobacter Retailed chickens are almost universally contaminated (frozen ones less so than fresh https://mypastest.pastest.com/Secure/TestMe/Browser/429893 1/2 8/9/2016 MyPastest ones), so self-infection when handling them in the kitchen, or cross-contamination to other foods, readily occurs if good hygiene is not observed

1307

Next Question

Previous Question Tag Question Feedback End Review

Difficulty: Easy

Peer Responses

Session Progress

Responses Correct: 1

Responses Incorrect: 152

Responses Total: 153

Responses - % Correct: 1%

Blog (https://www.pastest.com/blog) About Pastest (https://www.pastest.com/about-us) Contact Us (https://www.pastest.com/contact-us) Help (https://www.pastest.com/help)

© Pastest 2016

https://mypastest.pastest.com/Secure/TestMe/Browser/429893 2/2 8/9/2016 MyPastest

Back to Filters (/Secure/TestMe/Filter/429893/QA) Question 88 of 153

A 17-year-old girl, who has been doing work experience at a local farm, presents to the Emergency Department with fever, headache and diarrhoea. She is usually well although she has mild asthma for which she uses a Seretide 50 inhaler and also takes the oral contraceptive. On examination her BP is 155/84 mmHg, and her temperature is 37.8 °C. She looks unwell with a widespread purpuric rash and petechial haemorrhages.

Investigations:

Hb 9.8 g/dl (schistocytes seen on film)

WCC 11.1 × 109 /l

PLT 84 × 109 /l

Na+ 141 mmol/l

K+ 5.8 mmol/l

Creatinine 210 μmol/l

PT 13.1 s

APTT 24.9 s

blood ++, protein ++ Urine (only able to pass 50 ml in total)

Which one of the following options is the most likely causative organism?

A Salmonella

B Shigella

C E.coli

D Brucella

E Listeria

https://mypastest.pastest.com/Secure/TestMe/Browser/429893 1/3 8/9/2016 MyPastest Explanation

Escherichia coli infection Diagnosis

The clinical picture in the present case is one of thrombotic thrombocytopenic purpura/haemolytic uraemic syndrome, which is associated with outbreaks of Escherichia coli 157

Pathogenesis

Endothelial cell injury due to E. coli toxin production is thought to be responsible for the clinical picture seen, with microthrombi widespread in the circulation

Management

Plasmapheresis, coupled with the use of corticosteroids is the mainstay of therapy

Prognosis and complications

Mortality may approach 15%, with up to 70% requiring dialysis support during the acute period A number of patients have long-term chronic renal impairment and hypertension

21090

Next Question

Previous Question Tag Question Feedback End Review

Difficulty: Average

Peer Responses

Session Progress

Responses Correct: 1

Responses Incorrect: 152

Responses Total: 153

Responses - % Correct: 1% https://mypastest.pastest.com/Secure/TestMe/Browser/429893 2/3 8/9/2016 MyPastest

Blog (https://www.pastest.com/blog) About Pastest (https://www.pastest.com/about-us) Contact Us (https://www.pastest.com/contact-us) Help (https://www.pastest.com/help)

© Pastest 2016

https://mypastest.pastest.com/Secure/TestMe/Browser/429893 3/3 8/9/2016 MyPastest

Back to Filters (/Secure/TestMe/Filter/429893/QA) Question 89 of 153

A 32-year-old school teacher presents to the clinic with diarrhoea and weight loss. He returned from a spell teaching in the Thai jungle around a year or so ago, but claims that he has never been right since, suffering from vague abdominal pains and a dislike of food. He also says that he has had a recurrent maculopapular rash over the past few months. On examination his BMI is 19. Cardiovascular and respiratory examination is unremarkable; abdominal examination reveals mild epigastric tenderness. Investigations:

Hb 11.1 g/dl

WCC 8.1 × 109 /l (raised eosinophil count)

PLT 167 × 109 /l

Na+ 139 mmol/l

K+ 4.9 mmol/l

Creatinine 120 μmol/l

Stool samples Ova identified on one out of three stool samples

Given the suspected diagnosis, which one of the following options is the most appropriate therapy for him?

A Tinidazole

B Metronidazole

C Artemesin

D Mebendazole

E Tiabendazole

Explanation

https://mypastest.pastest.com/Secure/TestMe/Browser/429893 1/3 8/9/2016 MyPastest Strongyloides infection Diagnosis

This man has Strongyloides infection, the history of intermittent diarrhoea, coupled with episodic vague abdominal pain being typical of the condition The presence of eosinophilia as well as ova identified on one of the stool samples also supports the diagnosis

Treatment

Ivermectin is an option in chronic Strongyloides infection, but the first choice in the UK is mebendazole (albendazole is not widely available in the UK) These treatments are contraindicated in the first trimester of pregnancy

Mechanism of drug action

The azoles block adenosine triphosphate (ATP) production in the worm leading to decreased energy production, stasis and eventual death

22440

Next Question

Previous Question Tag Question Feedback End Review

Difficulty: Average

Peer Responses

Session Progress

Responses Correct: 1

Responses Incorrect: 152

Responses Total: 153

Responses - % Correct: 1%

https://mypastest.pastest.com/Secure/TestMe/Browser/429893 2/3 8/9/2016 MyPastest Blog (https://www.pastest.com/blog) About Pastest (https://www.pastest.com/about-us) Contact Us (https://www.pastest.com/contact-us) Help (https://www.pastest.com/help) © Pastest 2016

https://mypastest.pastest.com/Secure/TestMe/Browser/429893 3/3 8/9/2016 MyPastest

Back to Filters (/Secure/TestMe/Filter/429893/QA) Question 90 of 153

A 30-year-old man presents with fever, malaise, fatigue and sore throat. On examination there is pharyngitis, cervical adenopathy, hepatomegaly and a rash. AST and ALT are raised to twice the upper limit of normal. On direct questioning he admits having unprotected receptive anal intercourse with a new male sex partner 8 weeks ago. Which one of the following options is the most appropriate test to confirm your suspicion?

A Dark-field examination

B ELISA and Western blot

C Fluorescent antibody

D Complement fixation

E CD4+count

Explanation

HIV seroconversion illness

The patient in the present case has a typical human immunodeficiency virus (HIV) seroconversion illness

Differential diagnosis

HIV seroconversion illness resembles infection with cytomegalovirus, toxoplasmosis and the Epstein–Barr virus The association with anal intercourse with a new partner increases the suspicion of a sexually transmitted infection

Appropriate laboratory investigations

The diagnostic test for HIV uses an enzyme-linked immunosorbent antibody (ELISA), confirmed by Western blotting to see if the patient has a characteristic pattern of antibodies to important HIV antigens https://mypastest.pastest.com/Secure/TestMe/Browser/429893 1/2 8/9/2016 MyPastest The dark-field examination is used in the diagnosis of syphilis Fluorescent antibody staining and complement fixation assays are used in a variety of tests, but not for HIV The CD4+ count is not used for diagnostic purposes

1323

Next Question

Previous Question Tag Question Feedback End Review

Difficulty: Average

Peer Responses

Session Progress

Responses Correct: 1

Responses Incorrect: 152

Responses Total: 153

Responses - % Correct: 1%

Blog (https://www.pastest.com/blog) About Pastest (https://www.pastest.com/about-us) Contact Us (https://www.pastest.com/contact-us) Help (https://www.pastest.com/help)

© Pastest 2016

https://mypastest.pastest.com/Secure/TestMe/Browser/429893 2/2 8/9/2016 MyPastest

Back to Filters (/Secure/TestMe/Filter/429893/QA) Question 91 of 153

A 35-year-old homosexual, known to be HIV-seropositive, presents with right-sided weakness and a 2-week history of difficulty with his speech. The CD4 T-lymphocyte count is 50 cells/μl. An MRI scan of the brain demonstrates a large ring-enhancing lesion in the parietofrontal region of the left hemisphere and several small lesions in the right hemisphere. Which one of the following options is the most likely diagnosis?

A HIV encephalopathy

B Lymphoma

C Progressive multifocal leucoencephalopathy (PML)

D Toxoplasmosis

E Tuberculosis

Explanation

Toxoplasmosis Diagnostic features

The rapid development of focal neurological signs, in the context of severe immunosuppression with HIV and the presence of multiple ring-enhancing lesions in the brain, make toxoplasmosis the most likely aetiology in the present patient. CSF should be obtained, if there is no risk of herniation, and may be diagnostic

Differential diagnosis

Tuberculosis, progressive multifocal leucoencephalopathy (PML), lymphoma and human immunodeficiency virus (HIV) encephalopathy are also possible diagnoses in the present case. Lymphoma typically presents with a solitary lesion

PML demonstrates a hyperintense signal on T2 -weighted images, classically of the cerebellar hemispheres

758 https://mypastest.pastest.com/Secure/TestMe/Browser/429893 1/2 8/9/2016 MyPastest 758

Next Question

Previous Question Tag Question Feedback End Review

Difficulty: Easy

Peer Responses

Session Progress

Responses Correct: 1

Responses Incorrect: 152

Responses Total: 153

Responses - % Correct: 1%

Blog (https://www.pastest.com/blog) About Pastest (https://www.pastest.com/about-us) Contact Us (https://www.pastest.com/contact-us) Help (https://www.pastest.com/help) © Pastest 2016

https://mypastest.pastest.com/Secure/TestMe/Browser/429893 2/2 8/9/2016 MyPastest

Back to Filters (/Secure/TestMe/Filter/429893/QA) Question 92 of 153

A patient is admitted for urgent assessment on the infectious diseases ward. The nurses suggest that you should inform the consultant in communicable disease control (CCDC). Which one of the following is best recognised as a notifiable disease?

A HIV

B Malaria

C Mycoplasma pneumonia

D Necrotising fasciitis

E Rheumatic fever

Explanation

Notifiable diseases

The list of notifiable diseases mostly refers to those which are communicable and hence of public health importance In some of these, early enhanced surveillance may help to identify unusual situations such as nosocomial or ‘airport’ malaria All newly diagnosed cases of HIV should be entered on the national register but there is no requirement to inform the CCDC.

784

Next Question

Previous Question Tag Question Feedback End Review

Difficulty: Average

Peer Responses https://mypastest.pastest.com/Secure/TestMe/Browser/429893 1/2 8/9/2016 MyPastest

Session Progress

Responses Correct: 1

Responses Incorrect: 152

Responses Total: 153

Responses - % Correct: 1%

Blog (https://www.pastest.com/blog) About Pastest (https://www.pastest.com/about-us) Contact Us (https://www.pastest.com/contact-us) Help (https://www.pastest.com/help)

© Pastest 2016

https://mypastest.pastest.com/Secure/TestMe/Browser/429893 2/2 8/9/2016 MyPastest

Back to Filters (/Secure/TestMe/Filter/429893/QA) Question 93 of 153

A patient with acute myeloblastic leukaemia received 3 weeks of chemotherapy. He develops pyrexia and jaundice. Blood cultures are taken and intravenous antibiotics are started; however, he does not respond to therapy after 72 hours. Which one of the following options is the most likely diagnosis?

A Fungal infection

B Cytomegalovirus infection

C Hepatic leukaemic deposits

D Miliary tuberculosis

E Toxoplasmosis

Explanation

Complications of cytomegalovirus infection

Immunocompromised patients with cytomegalovirus (CMV) infection develop spiking pyrexia which resolves within a few days Some may develop a viraemia with a septicaemia-like syndrome in the presence or absence of hepatitis Pneumonitis may develop which carries a grave prognosis, with 80–90% mortality The virus may disseminate to involve the retina, causing a CMV retinitis CMV may disseminate to the gut, where it may cause an asymptomatic infection, ulceration or haemorrhage by the erosion of nearby blood vessels It is likely that defective viral surveillance has occurred 3 weeks into the chemotherapy and accounted for the symptoms. Fungal infection is less likely than viral reactivation in percentage terms

5511

Next Question

https://mypastest.pastest.com/Secure/TestMe/Browser/429893 1/2 8/9/2016 MyPastest

Previous Question Tag Question Feedback End Review

Difficulty: Average

Peer Responses

Session Progress

Responses Correct: 1

Responses Incorrect: 152

Responses Total: 153

Responses - % Correct: 1%

Blog (https://www.pastest.com/blog) About Pastest (https://www.pastest.com/about-us) Contact Us (https://www.pastest.com/contact-us) Help (https://www.pastest.com/help) © Pastest 2016

https://mypastest.pastest.com/Secure/TestMe/Browser/429893 2/2 8/9/2016 MyPastest

Back to Filters (/Secure/TestMe/Filter/429893/QA) Question 94 of 153

There is an outbreak of diarrhoea and vomiting on an acute surgical ward, initially affecting patients, but then rapidly also staff. Which one of the following is the most likely causal agent?

A Salmonella enteritidis

B Clostridium difficile

C Norovirus

D Enterovirus

E E.coli 0157:H7

Explanation

Norovirus outbreaks

There have been dramatic Norovirus outbreaks (also known as Norwalk-like virus, small round-structured virus, SRSV) in UK hospitals in 2002/03 necessitating closure of wards Transmission is by direct contact, but also possibly by droplet spread The prodromal illness can be minutes! Enteroviruses are so called because they are replicate in the gastrointestinal tract, but they only very rarely give rise to gastrointestinal symptoms

3719

Next Question

Previous Question Tag Question Feedback End Review

Difficulty: Easy https://mypastest.pastest.com/Secure/TestMe/Browser/429893 1/2 8/9/2016 MyPastest Peer Responses

Session Progress

Responses Correct: 1

Responses Incorrect: 152

Responses Total: 153

Responses - % Correct: 1%

Blog (https://www.pastest.com/blog) About Pastest (https://www.pastest.com/about-us) Contact Us (https://www.pastest.com/contact-us) Help (https://www.pastest.com/help) © Pastest 2016

https://mypastest.pastest.com/Secure/TestMe/Browser/429893 2/2 8/9/2016 MyPastest

Back to Filters (/Secure/TestMe/Filter/429893/QA) Question 95 of 153

A 44-year-old woman who is taking oral prednisolone for a flare-up of her rheumatoid arthritis is planning a 6-week holiday to a remote jungle region of Latin America. She has completed her childhood vaccination programme, and received a polio booster 8 years ago. However, she has heard that she requires further travel vaccinations. Her travel agent has suggested the items below, but she is a bit concerned about the safety of these given her medical history. Which one of the following vaccines do you feel poses the greatest difficulty?

A Polio

B Hepatitis A

C Tetanus

D Typhoid Vi

E Yellow fever

Explanation

Safety of travel vaccines

Live vaccines, including rubella, measles, mumps, BCG, yellow fever and oral polio vaccine are contraindicated in patients taking prednisolone The hepatitis (formaldehyde inactivated virus), typhoid Vi (polysaccharide antigen) and tetanus (adsorbed toxoid) vaccines pose no risk, although their efficacy may be reduced in the immunocompromised Inactivated polio vaccine is available – if needed (not the case here, as less than 10 years since her last booster Furthermore, polio has been eradicated in the Americas since 1991) Yellow fever vaccine is therefore the main problem

1642

Next Question

https://mypastest.pastest.com/Secure/TestMe/Browser/429893 1/2 8/9/2016 MyPastest

Previous Question Tag Question Feedback End Review

Difficulty: Average

Peer Responses

Session Progress

Responses Correct: 1

Responses Incorrect: 152

Responses Total: 153

Responses - % Correct: 1%

Blog (https://www.pastest.com/blog) About Pastest (https://www.pastest.com/about-us) Contact Us (https://www.pastest.com/contact-us) Help (https://www.pastest.com/help) © Pastest 2016

https://mypastest.pastest.com/Secure/TestMe/Browser/429893 2/2 8/9/2016 MyPastest

Back to Filters (/Secure/TestMe/Filter/429893/QA) Question 96 of 153

A 25-year-old woman is admitted to hospital having become acutely unwell with malaise, fever, profuse vomiting and mild diarrhoea over a 36-hour period. There is no history of foreign travel and her food history is unremarkable. On admission her pulse is 126/min, blood pressure 84/62 mmHg and temperature 38.9°C. She is confused, but has no focal neurology. There are no signs of meningism. She has a faint, erythematous rash, particularly noticeable on her extremities. There are no purpuric lesions. Her tongue and buccal mucosa are noted to be somewhat red and hyperaemic. Which one of the following options is the most likely diagnosis?

A E.coli 0157 infection

B Meningococcal septicaemia

C Salmonella gastroenteritis

D Toxic shock syndrome

E Typhoid fever

Explanation

Toxic shock syndrome

Never assume that every patient who presents with vomiting and diarrhoea has got gastroenteritis! The history in the present case is a classic one for staphylococcal toxic shock syndrome (TSS)

Diagnostic criteria required for a diagnosis of TSS

Temperature > 38.5°C Hypotension, systolic blood pressure (BP) < 90 mmHg Rash with subsequent desquamation, particularly on palms and soles Involvement of at least three of the following organ systems: gastrointestinal (GI) (diarrhoea, vomiting), muscular (severe myalgia or raised creatine phosphokinase), https://mypastest.pastest.com/Secure/TestMe/Browser/429893 1/2 8/9/2016 MyPastest mucous membranes (hyperaemia of conjunctivae, pharynx or vagina), renal (renal impairment), liver (abnormal LFTs), blood (thrombocytopenia), or central nervous system (CNS) (disorientation without focal neurology) Other conditions must also be excluded – eg measles

3702

Next Question

Previous Question Tag Question Feedback End Review

Difficulty: Average

Peer Responses

Session Progress

Responses Correct: 1

Responses Incorrect: 152

Responses Total: 153

Responses - % Correct: 1%

Blog (https://www.pastest.com/blog) About Pastest (https://www.pastest.com/about-us) Contact Us (https://www.pastest.com/contact-us) Help (https://www.pastest.com/help)

© Pastest 2016

https://mypastest.pastest.com/Secure/TestMe/Browser/429893 2/2 8/9/2016 MyPastest

Back to Filters (/Secure/TestMe/Filter/429893/QA) Question 97 of 153

An 18-year-old woman complains of malaise, tiredness, headache and abdominal discomfort for the last 3–4 days. She was started on ampicillin 2 days ago and has developed a rash. She has lymphadenopathy and exudative tonsillitis. Her white cell count shows abnormal lymphocytosis. Which one of the following options is the most likely diagnosis?

A German measles

B Chickenpox

C Infectious mononucleosis

D Herpes simplex infection

E Cytomegalovirus infection

Explanation

Infectious mononucleosis High-risk population

Infectious mononucleosis is commonly seen in the young

Causative organism

It is caused by the Epstein–Barr virus This virus infects and replicates in the B lymphocytes and is shed in the throat following the acute disease

Transmission and incubation period

Transmission is by exchange of saliva The incubation period is between 1-2 months

Clinical features

Typically, a rash develops if ampicillin has been given https://mypastest.pastest.com/Secure/TestMe/Browser/429893 1/2 8/9/2016 MyPastest Patients may develop exudative tonsillitis, lymphadenopathy, splenomegaly and a petechial rash on the palate

1360

Next Question

Previous Question Tag Question Feedback End Review

Difficulty: Easy

Peer Responses

Session Progress

Responses Correct: 1

Responses Incorrect: 152

Responses Total: 153

Responses - % Correct: 1%

Blog (https://www.pastest.com/blog) About Pastest (https://www.pastest.com/about-us) Contact Us (https://www.pastest.com/contact-us) Help (https://www.pastest.com/help)

© Pastest 2016

https://mypastest.pastest.com/Secure/TestMe/Browser/429893 2/2 8/9/2016 MyPastest

Back to Filters (/Secure/TestMe/Filter/429893/QA) Question 98 of 153

An HIV-positive woman on anti-retrovirals with an undetectable viral load becomes pregnant. Which of the following courses of action is recommended to reduce the risk of vertical transmission?

A Admit to hospital for close monitoring

B Avoid fetal blood sampling

C Change the anti-retroviral regimen to zidovudine, lamivudine and efavirenz

D Stop anti-retroviral therapy immediately and recommence at the time of birth

E Take vitamin supplements

Explanation

Mother-to-child transmission of human immunodeficiency virus (HIV) Factors reducing risk of vertical transmission

The combination of anti-retroviral therapy perinatally, an elective caesarean section and avoidance of breast-feeding are the key factors in reducing vertical transmission Planned caesarean section at 38–39 weeks’ gestation is, however, offered to all HIV- infected women with detectable plasma viral loads While caesarean section still offers reduction in transmission in women with low level viraemia (< 1000 HIV RNA copies/ml), an additional effect of caesarean section at viral loads of < 50 RNA copies/ml is not established Some women choose vaginal delivery, and this may be a particular consideration for those returning to areas where subsequent caesarean sections are not widely available. For vaginal deliveries, if possible, membranes should be left intact until delivery and fetal blood sampling and scalp electrodes avoided Provided she is experiencing no side-effects to therapy the anti-retrovirals should be continued throughout pregnancy, since generally there are no contraindications to their use in pregnancy

https://mypastest.pastest.com/Secure/TestMe/Browser/429893 1/2 8/9/2016 MyPastest Management

A switch at this stage from a regimen that is efficacious would be unwise

Role of supplements

Folate supplements may reduce neural tube defects but are not known to reduce the risk of vertical transmission

763

Next Question

Previous Question Tag Question Feedback End Review

Difficulty: Average

Peer Responses

Session Progress

Responses Correct: 1

Responses Incorrect: 152

Responses Total: 153

Responses - % Correct: 1%

Blog (https://www.pastest.com/blog) About Pastest (https://www.pastest.com/about-us) Contact Us (https://www.pastest.com/contact-us) Help (https://www.pastest.com/help) © Pastest 2016

https://mypastest.pastest.com/Secure/TestMe/Browser/429893 2/2 8/9/2016 MyPastest

Back to Filters (/Secure/TestMe/Filter/429893/QA) Question 99 of 153

A 10-year-old boy is complaining of pain in his right leg. He is pyrexial and a diagnosis of osteomyelitis has been made. Which one of the following is the most likely pathogen?

A Streptococcus viridans

B Staphylococcus aureus

C Cornebacterium diphtheriae

D Neisseria meningitides

E Brucellosis

Explanation

Acute osteomyelitis Causative organism

Acute osteomyelitis is almost always due to Staphylococcus aureus

High-risk group

It occurs predominantly in males between the age of 3 and 12 years and usually affects the lower limbs

Clinical features

The child is usually febrile, irritable and has leukocytosis

Laboratory findings

Blood cultures are positive in 75% of patients

1365

Next Question https://mypastest.pastest.com/Secure/TestMe/Browser/429893 1/2 8/9/2016 MyPastest

Previous Question Tag Question Feedback End Review

Difficulty: Easy

Peer Responses

Session Progress

Responses Correct: 1

Responses Incorrect: 152

Responses Total: 153

Responses - % Correct: 1%

Blog (https://www.pastest.com/blog) About Pastest (https://www.pastest.com/about-us) Contact Us (https://www.pastest.com/contact-us) Help (https://www.pastest.com/help)

© Pastest 2016

https://mypastest.pastest.com/Secure/TestMe/Browser/429893 2/2 8/9/2016 MyPastest

Back to Filters (/Secure/TestMe/Filter/429893/QA) Question 100 of 153

A 30-year-old man has been unwell for the last 3 weeks and now developed a rash in the past 24 hours. Chickenpox is diagnosed. Which one of the following is the most appropriate treatment?

A Aciclovir

B Erythromycin

C Doxycycline

D Ampicillin

E No medication

Explanation

Treatment of chickenpox

The UK VZV guidelines suggest that treatment of chickenpox is beneficial in adults provided it is started within 24 h of the appearance of the rash Early treatment of shingles with an antiviral can lessen the occurrence of post-herpetic neuralgia

Pregnant patients

Aciclovir is not licensed in pregnancy, but most physicians would prescribe it for pregnant women in the second and third trimester, and many also in the first trimester – there has been no documented evidence of harm despite extensive use

Herpes simplex infection

Sporadic recurrences of genital herpes simplex virus (HSV) are not usually treated, as they tend to settle rapidly with no treatment However frequent recurrences, or recurrences in immunocompromised patients are treated with long-term (6–12 months) prophylactic aciclovir https://mypastest.pastest.com/Secure/TestMe/Browser/429893 1/2 8/9/2016 MyPastest

3689

Next Question

Previous Question Tag Question Feedback End Review

Difficulty: Average

Peer Responses

Session Progress

Responses Correct: 1

Responses Incorrect: 152

Responses Total: 153

Responses - % Correct: 1%

Blog (https://www.pastest.com/blog) About Pastest (https://www.pastest.com/about-us) Contact Us (https://www.pastest.com/contact-us) Help (https://www.pastest.com/help) © Pastest 2016

https://mypastest.pastest.com/Secure/TestMe/Browser/429893 2/2 8/9/2016 MyPastest

Back to Filters (/Secure/TestMe/Filter/429893/QA) Question 101 of 153

A 19-year-old woman presents with fever, rash and cough, and is pyrexial, tachycardic and tachypnoeic. She has a florid erythematous rash on her face, trunk and arms, with scattered whitish papular lesions on the buccal mucosa. Which one of the following options is the most likely diagnosis?

A Meningococcaemia

B Rubella

C Parvovirus B19

D Secondary syphilis

E Measles

Explanation

Measles Epidemiology

Adult measles is unusual except in non-immunised persons

Clinical features

The classic features are: a maculopapular rash that starts on the face and migrates caudally and Koplik’s spots present in the mouth

Differential diagnosis

In meningococcaemia, there is a petechial/purpuric macular rash with no buccal lesions, and there is no typical migratory pattern In rubella the lesions are smaller and not intense or darkly coloured: they can also spread caudally but there is no desquamation Parvovirus infection produces the typical raised erythema on the cheeks (‘slapped cheek’) and usually occurs in young children: bizarre variations in the rash are https://mypastest.pastest.com/Secure/TestMe/Browser/429893 1/2 8/9/2016 MyPastest sometimes seen The rash of secondary syphilis usually affects the upper trunk, palms and soles and flexural extremities Mucosal lesions are usually erythematous and coalesce (‘snail track’ ulcers)

298

Next Question

Previous Question Tag Question Feedback End Review

Difficulty: Easy

Peer Responses

Session Progress

Responses Correct: 1

Responses Incorrect: 152

Responses Total: 153

Responses - % Correct: 1%

Blog (https://www.pastest.com/blog) About Pastest (https://www.pastest.com/about-us) Contact Us (https://www.pastest.com/contact-us) Help (https://www.pastest.com/help)

© Pastest 2016

https://mypastest.pastest.com/Secure/TestMe/Browser/429893 2/2 8/9/2016 MyPastest

Back to Filters (/Secure/TestMe/Filter/429893/QA) Question 102 of 153

A 35-year-old man attends the Emergency Department complaining of haemoptysis. He gives a history of HIV infection treated in another hospital and multiple recent problems including cerebral toxoplasmosis, CMV retinitis, cryptosporidium-related diarrhoea, oesophageal candidiasis, Kaposi’s sarcoma and pulmonary tuberculosis. He provides a list of his current medication, which includes AZT, stavudine, saquinavir, nelfinavir and ritonavir (atazanavir), isoniazid, rifampicin, pyrazinamide, ethambutol, ganciclovir, sulfadiazine, fluconazole and co- trimoxazole. On examination, he appears well. His pulse is 70 bpm, BP 120/80 mmHg. The abdomen is soft and non-tender. Respiratory, cardiovascular and neurological examinations, including ophthalmoscopy, are unremarkable. There is no lymphadenopathy. Mouth and skin examination is normal. Chest X-ray is normal. Hb is 13.5 g/dl, the WCC is 7.9 × 109 /l (lymphocytes 2.4 × 1096 /l) and platelets are 420 × 10 /l. A recent MRI brain is unremarkable. Which one of the following options is the most appropriate next step?

A Arrange a CT chest and bronchoscopy

B Arrange a CT pulmonary angiogram

C Arrange for three repeat sputum samples for AFB staining

D Place him in isolation and observe overnight

E Psychiatric review

Explanation

Fictitious disease presentation

The patient in the present case has fictitious human immunodeficiency virus (HIV) infection He has no findings on examination to support what he has said, and he is on a bizarre antiretroviral regime (zidovudine (AZT) is not usually used with stavudine and three protease inhibitors are not usually combined)

Management considerations

A phone call to the patient’s treating hospital should be made to verify his story https://mypastest.pastest.com/Secure/TestMe/Browser/429893 1/2 8/9/2016 MyPastest Magnetic resonance imaging (MRI) is very sensitive and if the patient has only recently been started on therapy, it would be unlikely to be normal The patient should be referred to psychiatry, although many patients who present in this way will abscond from the ward prior to psychiatric review An HIV test would also be appropriate

602

Next Question

Previous Question Tag Question Feedback End Review

Difficulty: Difficult

Peer Responses

Session Progress

Responses Correct: 1

Responses Incorrect: 152

Responses Total: 153

Responses - % Correct: 1%

Blog (https://www.pastest.com/blog) About Pastest (https://www.pastest.com/about-us) Contact Us (https://www.pastest.com/contact-us) Help (https://www.pastest.com/help) © Pastest 2016

https://mypastest.pastest.com/Secure/TestMe/Browser/429893 2/2 8/9/2016 MyPastest

Back to Filters (/Secure/TestMe/Filter/429893/QA) Question 103 of 153

What is the cause of myocarditis in diphtheria?

A Hypoxia

B Superinfection with streptococci

C Toxins

D Massive bacteraemia

E A virus

Explanation

Diphtheria Causative organism

Corynebacterium diphtheriae are pleomorphic, Gram-positive rods or clubs The clinical manifestations of diphtheria are caused by an exotoxin produced by virulent corynebacteria

Cardiac complications

The toxin causes fatty degeneration of cardiac muscle (myocarditis) and infiltration of the interstitium with leukocytes, which may affect the conduction fibres Parenchymal necrosis is rare Generally the heart can recover completely from these effects, although severe fibrosis and scarring may lead to death in late convalescence Mural endocarditis may cause embolism leading to cerebral infarction and hemiplegia Valvular endocarditis is extremely uncommon Neuritic changes may be seen in the nerves to the heart during the late paralytic stage of the disease

1325 https://mypastest.pastest.com/Secure/TestMe/Browser/429893 1/2 8/9/2016 MyPastest Next Question

Previous Question Tag Question Feedback End Review

Difficulty: Easy

Peer Responses

Session Progress

Responses Correct: 1

Responses Incorrect: 152

Responses Total: 153

Responses - % Correct: 1%

Blog (https://www.pastest.com/blog) About Pastest (https://www.pastest.com/about-us) Contact Us (https://www.pastest.com/contact-us) Help (https://www.pastest.com/help)

© Pastest 2016

https://mypastest.pastest.com/Secure/TestMe/Browser/429893 2/2 8/9/2016 MyPastest

Back to Filters (/Secure/TestMe/Filter/429893/QA) Question 104 of 153

A 28-year-old traveller returned from the tropics 5 days ago. She felt unwell on the plane, complaining of headache, loss of appetite and sweats. Her temperature was 39.5 °C 2 days ago; however, it is now normal. Which one of the following options is the most important investigation to conduct?

A Repeated thick and thin blood smears

B Blood cultures

C Lumbar puncture

D Coombs’ test

E Erythrocyte sedimentation rate

Explanation

Acute febrile illness Causative organism

The pathogenic species of Plasmodium cause acute febrile illness

Clinical presentation

The illness is characterised by periodic febrile paroxysms occurring every 48 or 72 h, with afebrile asymptomatic intervals and a tendency to relapse over periods of months or even years

Steps in diagnosis of the underlying condition

Malaria can present with a wide range of symptoms and signs, none of them diagnostic It must be excluded by repeated thick and thin blood smears in any patient with acute fever and an appropriate history of exposure Until malaria is confirmed, or an alternative diagnosis emerges, smears should be repeated every 8–12 hours https://mypastest.pastest.com/Secure/TestMe/Browser/429893 1/2 8/9/2016 MyPastest However, if the patient is severely ill, or the symptoms persist or deteriorate, a therapeutic trial of antimalarial chemotherapy must not be delayed

2711

Next Question

Previous Question Tag Question Feedback End Review

Difficulty: Easy

Peer Responses

Session Progress

Responses Correct: 1

Responses Incorrect: 152

Responses Total: 153

Responses - % Correct: 1%

Blog (https://www.pastest.com/blog) About Pastest (https://www.pastest.com/about-us) Contact Us (https://www.pastest.com/contact-us) Help (https://www.pastest.com/help)

© Pastest 2016

https://mypastest.pastest.com/Secure/TestMe/Browser/429893 2/2 8/9/2016 MyPastest

Back to Filters (/Secure/TestMe/Filter/429893/QA) Question 105 of 153

A 65-year-old man develops fevers and lower lumbar back pain. An MRI scan shows destruction of the L4 vertebra with a small paraspinal collection consistent with osteomyelitis. Which one of the following options is the most appropriate empirical antibiotic treatment?

A Amoxicillin

B Cefotaxime and metronidazole

C Flucloxacillin

D Meropenem

E Vancomycin and ciprofloxacin

Explanation

Osteomyelitis Causative organisms

The most likely organism to cause osteomyelitis is Staphylococcus aureus Infection with more than one organism is unusual

Management considerations

Prolonged high-dose parenteral therapy should be commenced In such circumstances broad-spectrum cover with cefotaxime and metronidazole is justified but carries the significant risk of Clostridium difficile diarrhoea in the elderly Vancomycin and ciprofloxacin would be appropriate therapy in a patient with a penicillin allergy or MRSA (methicillin-resistant Staphylococcus aureus) infection

765

Next Question

https://mypastest.pastest.com/Secure/TestMe/Browser/429893 1/2 8/9/2016 MyPastest Previous Question Tag Question Feedback End Review

Difficulty: Difficult

Peer Responses

Session Progress

Responses Correct: 1

Responses Incorrect: 152

Responses Total: 153

Responses - % Correct: 1%

Blog (https://www.pastest.com/blog) About Pastest (https://www.pastest.com/about-us) Contact Us (https://www.pastest.com/contact-us) Help (https://www.pastest.com/help) © Pastest 2016

https://mypastest.pastest.com/Secure/TestMe/Browser/429893 2/2 8/9/2016 MyPastest

Back to Filters (/Secure/TestMe/Filter/429893/QA) Question 106 of 153

A 55-year-old man of no fixed address is admitted to the hospital because of self-neglect. A chest X-ray has shown bilateral apical cavitation and hyperinflated lung fields consistent with COPD. Sputum cultures have grown Mycobacterium avium complex (MAC). Which one of the following statements is correct?

A Patient should be treated with a standard 6 months of anti-tuberculosis drugs

B Patient should be notified within 1 week of diagnosis

C Patient should be barrier nursed for 2 weeks

D This organism is most likely to affect patients with pre-existing lung disease

E Surgery has no role in management

Explanation

Mycobacterium avium complex (MAC) infection

MAC is an opportunist mycobacterium found in soil, water, dust, milk, etc It usually attacks patients with pre-existing lung disease such as chronic obstructive pulmonary disease (COPD), bronchiectasis, etc, or patients who are immunosuppressed It is a very low-grade pathogen and hence there is no need for notification, contact tracing or barrier nursing

Diagnosis

Diagnosis should be made in the presence of an abnormal chest X-ray for which no other cause is found, and if MAC can be isolated from multiple isolates

Treatment

Treatment is with rifampicin and ethambutol for 24 months Isoniazid may be added to the treatment regimen Ciprofloxacin or clarithromycin may be added to the regimen in case of treatment https://mypastest.pastest.com/Secure/TestMe/Browser/429893 1/2 8/9/2016 MyPastest failure In otherwise fit patients surgical resection of the disease area is an option

1367

Next Question

Previous Question Tag Question Feedback End Review

Difficulty: Average

Peer Responses

Session Progress

Responses Correct: 1

Responses Incorrect: 152

Responses Total: 153

Responses - % Correct: 1%

Blog (https://www.pastest.com/blog) About Pastest (https://www.pastest.com/about-us) Contact Us (https://www.pastest.com/contact-us) Help (https://www.pastest.com/help)

© Pastest 2016

https://mypastest.pastest.com/Secure/TestMe/Browser/429893 2/2 8/9/2016 MyPastest

Back to Filters (/Secure/TestMe/Filter/429893/QA) Question 107 of 153

A 57-year-old woman who presented with gastrointestinal haemorrhage was transfused with 3 units of blood. A few weeks later she presents with jaundice and symptoms of hepatitis. You suspect an infection; Which one of the following agents is most likely to be responsible?

A Hepatitis B

B Hepatitis E

C Hepatitis C

D Parvovirus

E Cytomegalovirus

Explanation

Infection following blood transfusion Viral infections

The incidence of infection with hepatitis B or C from blood transfusion is very small, and is put at around 1 per 200 000 units in the UK Other viruses which may cause post-transfusion hepatitis include cytomegalovirus (CMV) and Epstein–Barr virus Risks of viral transmission increase when patients are given products made from pooled samples such as coagulation factor products Transfusion infection is still a very major problem in the developing world

Bacterial infections

Bacterial contamination of blood is rare, although it is a major cause of death associated with haemolytic transfusion reactions Yersinia in particular is able to proliferate in red-cell concentrates, even though they are stored at around 4° C

5237 https://mypastest.pastest.com/Secure/TestMe/Browser/429893 1/2 8/9/2016 MyPastest

Next Question

Previous Question Tag Question Feedback End Review

Difficulty: Average

Peer Responses

Session Progress

Responses Correct: 1

Responses Incorrect: 152

Responses Total: 153

Responses - % Correct: 1%

Blog (https://www.pastest.com/blog) About Pastest (https://www.pastest.com/about-us) Contact Us (https://www.pastest.com/contact-us) Help (https://www.pastest.com/help)

© Pastest 2016

https://mypastest.pastest.com/Secure/TestMe/Browser/429893 2/2 8/9/2016 MyPastest

Back to Filters (/Secure/TestMe/Filter/429893/QA) Question 108 of 153

A 23-year-old homosexual develops painless lymphadenopathy after a trip to Thailand. He presents, 1 month after returning to the UK, with a generalised skin rash and orogenital ulceration. Serological tests are performed which show HIV Ab-positive, VDRL-positive, TPA- positive and anti-HCV Ab-positive. What does the serology indicate?

A Active syphilis

B False-positive serology

C HIV-seroconversion syndrome

D Previous exposure to syphilis

E Yaws

Explanation

Syphilis Specific tests

VDRL (Venereal Disease Research Laboratory)-positive indicates active primary disease It becomes negative following treatment and in late disease Malaria, systemic lupus erythematosus (SLE), tuberculosis and leprosy are causes of a false-positive VDRL TPA (Treponema pallidum assay) is positive in primary disease and remains so for life It may be positive in other treponemal diseases such as Yaws

Differential diagnosis

HIV-seroconversion syndrome is a differential diagnosis, although orogenital ulceration would be unusual and the serology suggests prior exposure to HIV

789

Next Question https://mypastest.pastest.com/Secure/TestMe/Browser/429893 1/2 8/9/2016 MyPastest

Previous Question Tag Question Feedback End Review

Difficulty: Average

Peer Responses

Session Progress

Responses Correct: 1

Responses Incorrect: 152

Responses Total: 153

Responses - % Correct: 1%

Blog (https://www.pastest.com/blog) About Pastest (https://www.pastest.com/about-us) Contact Us (https://www.pastest.com/contact-us) Help (https://www.pastest.com/help)

© Pastest 2016

https://mypastest.pastest.com/Secure/TestMe/Browser/429893 2/2 8/9/2016 MyPastest

Back to Filters (/Secure/TestMe/Filter/429893/QA) Question 109 of 153

A 29-year-old Catholic priest returns from a trip to Brazil with fevers and deranged LFTs. He has an ALT of 2500 U/l and bilirubin of 75 μmol/l. He attended a travel clinic and was vaccinated prior to travel. He also took mefloquine malaria prophylaxis. Which one of the following options is the most likely diagnosis?

A Malaria

B Hepatitis A

C Hepatitis B

D Hepatitis E

E Dengue fever

Explanation

Hepatitis E Diagnostic considerations in the present case

Hepatitis A vaccine is routinely offered for the prevention of hepatitis A and is very effective Acute hepatitis B would be less likely as it is transmitted by blood or sexual contact, and a priest is less likely to have it because of the vow of celibacy and a low chance of exposure to infected blood Both malaria and dengue may cause deranged liver function tests but not to the level in the present case

Complications

Hepatitis E has no chronic form but has a higher mortality (1–2%) than hepatitis A (0.1%), which has a similar clinical presentation and is also acquired by the faeco-oral route Most deaths in patients with hepatitis E occur in women during the last trimester of https://mypastest.pastest.com/Secure/TestMe/Browser/429893 1/2 8/9/2016 MyPastest pregnancy Both hepatitis A and E may cause fulminant liver failure in patients with underlying liver disease

598

Next Question

Previous Question Tag Question Feedback End Review

Difficulty: Difficult

Peer Responses

Session Progress

Responses Correct: 1

Responses Incorrect: 152

Responses Total: 153

Responses - % Correct: 1%

Blog (https://www.pastest.com/blog) About Pastest (https://www.pastest.com/about-us) Contact Us (https://www.pastest.com/contact-us) Help (https://www.pastest.com/help) © Pastest 2016

https://mypastest.pastest.com/Secure/TestMe/Browser/429893 2/2 8/9/2016 MyPastest

Back to Filters (/Secure/TestMe/Filter/429893/QA) Question 110 of 153

A 22-year-old farm worker on an arable farm is admitted to hospital with a 2-day history of headache, fever, severe myalgia and a petechial rash. He is known to have mild asthma, which is well controlled by inhaled steroids. He is jaundiced, has a tachycardia and has not passed urine for over 14 h. His urea level is raised and liver function tests indicate hepatocellular damage. Which one of the following options is the most likely diagnosis?

A Brucellosis

B Weil’s disease

C Lyme disease

D Rat-bite fever

E Septicaemic shock

Explanation

Weil’s disease Causative organism

Weil’s disease (Leptospirosis) is caused by Leptospira icterohaemorrhagiae The causative organism is excreted in the urine of infected rodents and rats and penetrates the skin or mucosa of humans

High-risk groups

It is commonly seen in abattoir and farm workers

Clinical presentation

Patients present with fever, myalgia, conjunctival suffusion, anorexia and vomiting A petechial rash develops and hepatomegaly and splenomegaly occur Hepatitis, renal tubular necrosis, myocarditis and meningitis can develop in severe infections https://mypastest.pastest.com/Secure/TestMe/Browser/429893 1/2 8/9/2016 MyPastest Diagnosis

Diagnosis is made by a rising titre of leptospiral antibodies

Management

Treatment is with penicillin

1362

Next Question

Previous Question Tag Question Feedback End Review

Difficulty: Average

Peer Responses

Session Progress

Responses Correct: 1

Responses Incorrect: 152

Responses Total: 153

Responses - % Correct: 1%

Blog (https://www.pastest.com/blog) About Pastest (https://www.pastest.com/about-us) Contact Us (https://www.pastest.com/contact-us) Help (https://www.pastest.com/help) © Pastest 2016

https://mypastest.pastest.com/Secure/TestMe/Browser/429893 2/2 8/9/2016 MyPastest

Back to Filters (/Secure/TestMe/Filter/429893/QA) Question 111 of 153

An elderly woman is admitted with pneumonia and commenced on intravenous antibiotics. After making a good initial response to treatment she becomes febrile again and has become dehydrated with profuse watery diarrhoea. She has raised inflammatory markers with a neutrophilia. A repeat chest X-ray is not significantly changed from admission with patchy consolidation in the left lower lobe. Urine microscopy shows 5–20 white blood cells/ml but cultures are negative. Which one of the following is most likely to have happened to explain her deterioration?

A Clostridium difficile diarrhoea

B Drug reaction

C Empyema

D MRSA colonisation

E UTI

Explanation

Clostridium difficile diarrhoea

Clostridium difficile diarrhoea usually occurs in the context of high-dose intravenous cephalosporins in the elderly

Clinical features

A neutrophilia is typical and diarrhoea can quite quickly lead to dehydration

Differential diagnosis

The absence of a rash or eosinophilia makes a drug reaction less likely Urinary tract infections are probably overdiagnosed and negative cultures also make this unlikely Hospital-acquired infection with MRSA (meticillin-resistant Staphylococcus aureus) is

https://mypastest.pastest.com/Secure/TestMe/Browser/429893 1/2 8/9/2016 MyPastest an important consideration, but colonisation from surface swabs is insufficient for this diagnosis

770

Next Question

Previous Question Tag Question Feedback End Review

Difficulty: Easy

Peer Responses

Session Progress

Responses Correct: 1

Responses Incorrect: 152

Responses Total: 153

Responses - % Correct: 1%

Blog (https://www.pastest.com/blog) About Pastest (https://www.pastest.com/about-us) Contact Us (https://www.pastest.com/contact-us) Help (https://www.pastest.com/help)

© Pastest 2016

https://mypastest.pastest.com/Secure/TestMe/Browser/429893 2/2 8/9/2016 MyPastest

Back to Filters (/Secure/TestMe/Filter/429893/QA) Question 112 of 153

Elderly patients (age 65 or above) are more vulnerable to community-acquired infections. Against which one of the following pathogens is immunisation most likely to be given on a yearly basis?

A Legionella

B Campylobacter

C Influenza virus

D Pneumococcus

E Measles

Explanation

Influenza vaccination

Influenza vaccination represents one of the most important health promotion and disease prevention measures in the elderly population Vaccination is currently recommended for all persons over the age of 65 At highest risk are those elderly individuals with underlying chronic diseases, particularly those with cardiorespiratory disorders as well as those with metabolic or neoplastic diseases Although influenza vaccination is recognised as perhaps the most cost-effective medical intervention in the elderly population, protection rates are reduced from approximately 90% in younger adults to anywhere between 25% and 44% in the elderly population The vaccination does not change the incidence of upper respiratory tract symptoms, but significantly reduces hospital admission and mortality rates, by 72% and 87%, respectively, in community-dwelling elderly people The efficacy is greatly reduced in nursing-home residents, who tend to have a greater burden of chronic disease, such that there may be no difference between vaccinated and unvaccinated residents in long-term care https://mypastest.pastest.com/Secure/TestMe/Browser/429893 1/2 8/9/2016 MyPastest

2706

Next Question

Previous Question Tag Question Feedback End Review

Difficulty: Easy

Peer Responses

Session Progress

Responses Correct: 1

Responses Incorrect: 152

Responses Total: 153

Responses - % Correct: 1%

Blog (https://www.pastest.com/blog) About Pastest (https://www.pastest.com/about-us) Contact Us (https://www.pastest.com/contact-us) Help (https://www.pastest.com/help) © Pastest 2016

https://mypastest.pastest.com/Secure/TestMe/Browser/429893 2/2 8/9/2016 MyPastest

Back to Filters (/Secure/TestMe/Filter/429893/QA) Question 113 of 153

A HIV-positive man presents with a 2-week history of lethargy, confusion and personality change. A computed tomography (CT) scan shows multiple ring-enhancing lesions in both hemispheres. Which one of the following options is the most appropriate treatment?

A Ketoconazole

B Cefuroxime and gentamicin

C Rifampicin

D Corticosteroids

E Pyrimethamine and sulfadiazine

Explanation

Cerebral toxoplasmosis in AIDS Epidemiology

Cerebral infection with the intracellular protozoan Toxoplasma gondii is the most frequent infection of the central nervous system in AIDS when the CD4 lymphocyte count is below 200/mm3

Pathogenesis

It usually results from reactivation of Toxoplasma cysts in the brain, leading to the formation of focal lesions that are typically multiple but may be single

Clinical presentation

Symptoms develop subacutely and include focal neurological disturbance, headache, confusion, fever and convulsions

Radiological features

https://mypastest.pastest.com/Secure/TestMe/Browser/429893 1/2 8/9/2016 MyPastest

On computed tomography (CT) scanning, the lesions appear as ring-enhancing masses with surrounding oedema Magnetic resonance imaging (MRI) is more sensitive and frequently detects lesions not visible on the CT scan

Treatment and prognosis

The condition responds well if treatment is started early; a combination of sulfadiazine at 4–6 g/day and pyrimethamine at 50–75 mg/day is the treatment of choice

5519

Next Question

Previous Question Tag Question Feedback End Review

Difficulty: Easy

Peer Responses

Session Progress

Responses Correct: 1

Responses Incorrect: 152

Responses Total: 153

Responses - % Correct: 1%

Blog (https://www.pastest.com/blog) About Pastest (https://www.pastest.com/about-us) Contact Us (https://www.pastest.com/contact-us) Help (https://www.pastest.com/help)

© Pastest 2016

https://mypastest.pastest.com/Secure/TestMe/Browser/429893 2/2 8/9/2016 MyPastest

Back to Filters (/Secure/TestMe/Filter/429893/QA) Question 114 of 153

A native of Ghana has been diagnosed as having epidemic typhus. By which one of the following vectors is he most likely to have become infected?

A Hard tick

B Human body louse

C Trombiculid mite

D Aedes aegypti mosquito

E Rat flea

Explanation

Vectors in infectious diseases

The chief vector of epidemic typhus is the human body louse The hard tick is a vector for a number of rickettsial diseases Larval trombiculid mites spread scrub typhus Rat fleas are responsible for the spread of endemic (murine) typhus The Aedes aegypti mosquito is the vector for dengue fever

3581

Next Question

Previous Question Tag Question Feedback End Review

Difficulty: Difficult

Peer Responses

https://mypastest.pastest.com/Secure/TestMe/Browser/429893 1/2 8/9/2016 MyPastest

Session Progress

Responses Correct: 1

Responses Incorrect: 152

Responses Total: 153

Responses - % Correct: 1%

Blog (https://www.pastest.com/blog) About Pastest (https://www.pastest.com/about-us) Contact Us (https://www.pastest.com/contact-us) Help (https://www.pastest.com/help)

© Pastest 2016

https://mypastest.pastest.com/Secure/TestMe/Browser/429893 2/2 8/9/2016 MyPastest

Back to Filters (/Secure/TestMe/Filter/429893/QA) Question 115 of 153

A 45-year-old man is referred to hospital having returned from a 3-month business trip to Thailand. He went on an elephant safari just before returning to the UK. He is married and denies other sexual contacts. He complains of a generalised itch, dry cough and diarrhoea. Blood results are normal, other than a peripheral blood eosinophilia of 1.6 × 109 /l. Chest X-ray reveals bilateral small areas of opacification. Which one of the following treatment options is most likely to be effective?

A Ciprofloxacin

B Metronidazole

C Ivermectin

D Diethylcarbamazine

E Prednisolone

Explanation

Helminthic infections

Peripheral blood eosinophilia is the hallmark of a tissue-invasive helminth infection The most likely diagnosis is a Strongyloides infection, which commonly causes a marked eosinophilia However, this may not occur in cases of overwhelming infection Immunosuppressed patients are at risk of infection and HIV should be considered Ivermectin is associated with the highest rates of eradication with a success rate of 97% and is therfore the first line therapy of choice, mebendazole is an alternative if ivermectin is unavailable

609

Next Question

https://mypastest.pastest.com/Secure/TestMe/Browser/429893 1/2 8/9/2016 MyPastest Previous Question Tag Question Feedback End Review

Difficulty: Average

Peer Responses

Session Progress

Responses Correct: 1

Responses Incorrect: 152

Responses Total: 153

Responses - % Correct: 1%

Blog (https://www.pastest.com/blog) About Pastest (https://www.pastest.com/about-us) Contact Us (https://www.pastest.com/contact-us) Help (https://www.pastest.com/help) © Pastest 2016

https://mypastest.pastest.com/Secure/TestMe/Browser/429893 2/2 8/9/2016 MyPastest

Back to Filters (/Secure/TestMe/Filter/429893/QA) Question 116 of 153

A 72-year-old man is brought to the Emergency Department. He has returned from Thailand where he has been living for the past 20 years to seek medical care in the UK. His carers are concerned that he has been increasingly confused and has begun to have problems with falls, such that he has had to use a wheelchair at home. Examination reveals a BP of 142/82 mmHg, pulse is 70/min and regular. He has a systolic murmur on auscultation of his chest. Neurological examination reveals dorsal column sensory loss and a reduced MMSE score of 21. There are a number of fibrous/nodular skin lesions affecting both the upper and lower limbs and the trunk. Investigations;

Hb 11.0 g/dl

MCV 103 fl

WCC 9.4 x109 /l

PLT 201 x109 /l

Na+ 138 mmol/l

K+ 4.3 mmol/l

Creatinine 112 micromol/l

ESR 55 mm/1st hour

TPHA positive

VDRL positive

CXR Possible mediastinal widening

Which of the following is the most appropriate intervention?

A IM B12

B IM Penicillin

C IV Ceftriaxone

https://mypastest.pastest.com/Secure/TestMe/Browser/429893 1/2 8/9/2016 MyPastest

D IV Hydrocortisone

E IV Thiamine

Explanation The answer is IM Penicillin -

This patient has tertiary syphillis as evidenced by neurological dysfunction, possible aortitis and syphillitic gummae. The TPHA and VDRL positive tests support the diagnosis. Management of choice is IM Penicillin given at a dose of 2.4 U/day for 17 days, with probenecid cover. In patients who are Penicillin allergic, Doxycycline can be given twice daily for 28 days. Whilst he may have a significant problem with alcohol addiction given the raised MCV, in this case it is more likely that the neurological picture is related to syphillis rather than to B12 deficiency. 38087

Next Question

Previous Question Tag Question Feedback End Review

Difficulty: Average

Peer Responses

Session Progress

Responses Correct: 1

Responses Incorrect: 152

Responses Total: 153

Responses - % Correct: 1%

Blog (https://www.pastest.com/blog) About Pastest (https://www.pastest.com/about-us) Contact Us (https://www.pastest.com/contact-us) Help (https://www.pastest.com/help)

© Pastest 2016

https://mypastest.pastest.com/Secure/TestMe/Browser/429893 2/2 8/9/2016 MyPastest

Back to Filters (/Secure/TestMe/Filter/429893/QA) Question 117 of 153

The start of symptomatic food poisoning is the fastest after ingestion of which one of the following?

A Salmonella enteritis

B Staphylococcus aureus toxin

C Clostridium botulinum

D Vibrio cholerae

E Shigella sonnei

Explanation

Food poisoning Transmission

Often staphylococcus is introduced by contamination from a small abscess, whitlow or other discharging lesion present during the preparation of food which is allowed to remain warm and not fully cooked before serving

Incubation period

Staphylococcus aureus food poisoning has a very short incubation period of 1–6 h The incubation period of Salmonella spp is 12–48 h, Clostridium botulinum 12–36 h, Shigella spp 2–3 days, Vibrio cholerae 1–5 days.

Clinical picture

Staphylococcal food poisoning presents with severe vomiting, nausea and abdominal cramps, often followed by diarrhoea Fevers are uncommon Episodes of staphylococcal food poisoning are short-lived and usually resolve within 24 hours https://mypastest.pastest.com/Secure/TestMe/Browser/429893 1/2 8/9/2016 MyPastest

1303

Next Question

Previous Question Tag Question Feedback End Review

Difficulty: Average

Peer Responses

Session Progress

Responses Correct: 1

Responses Incorrect: 152

Responses Total: 153

Responses - % Correct: 1%

Blog (https://www.pastest.com/blog) About Pastest (https://www.pastest.com/about-us) Contact Us (https://www.pastest.com/contact-us) Help (https://www.pastest.com/help)

© Pastest 2016

https://mypastest.pastest.com/Secure/TestMe/Browser/429893 2/2 8/9/2016 MyPastest

Back to Filters (/Secure/TestMe/Filter/429893/QA) Question 118 of 153

A man visits the travel clinic prior to an overseas trip. After giving his medical history, including two previous episodes of severe viral meningitis, he is told that he should not receive a live attenuated vaccine as it would not be suitable. Which one of the following is a live, attenuated vaccine?

A Influenza

B Yellow fever

C Diphtheria

D Tetanus

E Hepatitis B

Explanation

Vaccines

Yellow fever vaccine is a live attenuated vaccine Traditional influenza vaccine is a trivalent inactivated vaccine, although new intra-nasal live attenuated vaccines are currently in clinical trials Hepatitis B vaccine is produced from purified hepatitis B surface antigen Tetanus vaccine is a toxoid vaccine, as is diphtheria vaccine The recently introduced varicella vaccine is another example of a live attenuated vaccine

20783

Next Question

Previous Question Tag Question Feedback End Review

https://mypastest.pastest.com/Secure/TestMe/Browser/429893 1/2 8/9/2016 MyPastest

Difficulty: Easy

Peer Responses

Session Progress

Responses Correct: 1

Responses Incorrect: 152

Responses Total: 153

Responses - % Correct: 1%

Blog (https://www.pastest.com/blog) About Pastest (https://www.pastest.com/about-us) Contact Us (https://www.pastest.com/contact-us) Help (https://www.pastest.com/help) © Pastest 2016

https://mypastest.pastest.com/Secure/TestMe/Browser/429893 2/2 8/9/2016 MyPastest

Back to Filters (/Secure/TestMe/Filter/429893/QA) Question 119 of 153

A 25-year-old homosexual man complains of a 9-day history of mucopurulent anal discharge, anal bleeding and pain while opening his bowels. Which one of the following options is the most likely diagnosis?

A Candidiasis

B Gonorrhoea

C Crohn’s disease

D Salmonella infection

E Chancroid

Explanation

Gonorrhoea Constipation, a mucopurulent anal discharge, anal bleeding, perianal discomfort or pruritus ani, and, in severe cases, pain and tenesmus are symptoms of gonorrhoea in homosexual men. 2686

Next Question

Previous Question Tag Question Feedback End Review

Difficulty: Easy

Peer Responses

https://mypastest.pastest.com/Secure/TestMe/Browser/429893 1/2 8/9/2016 MyPastest

Session Progress

Responses Correct: 1

Responses Incorrect: 152

Responses Total: 153

Responses - % Correct: 1%

Blog (https://www.pastest.com/blog) About Pastest (https://www.pastest.com/about-us) Contact Us (https://www.pastest.com/contact-us) Help (https://www.pastest.com/help)

© Pastest 2016

https://mypastest.pastest.com/Secure/TestMe/Browser/429893 2/2 8/9/2016 MyPastest

Back to Filters (/Secure/TestMe/Filter/429893/QA) Question 120 of 153

An 18-year-old man who has not received measles, mumps and rubella (MMR) vaccine presents with meningism, orchitis and unilateral parotitis. Which one of the following options is the most likely diagnosis?

A Epstein–Barr virus infection

B HIV

C Measles

D Rubella

E Mumps

Explanation

Mumps

The patient in the present case has mumps

Diagnostic tests in mumps and related conditions

Electron microscopy of serum is used routinely only in the diagnosis of acute parvovirus B19 Virus can be recovered from cerebrospinal fluid, urine, throat swabs and nasopharyngeal aspirates by culture and polymerase chain reaction Immunofluorescence on nasopharyngeal aspirates is successful, but rarely done for mumps (although it is routine for influenza, respiratory syncytial virus, parainfluenza and adenovirus)

3687

Next Question

https://mypastest.pastest.com/Secure/TestMe/Browser/429893 1/2 8/9/2016 MyPastest Previous Question Tag Question Feedback End Review

Difficulty: Easy

Peer Responses

Session Progress

Responses Correct: 1

Responses Incorrect: 152

Responses Total: 153

Responses - % Correct: 1%

Blog (https://www.pastest.com/blog) About Pastest (https://www.pastest.com/about-us) Contact Us (https://www.pastest.com/contact-us) Help (https://www.pastest.com/help) © Pastest 2016

https://mypastest.pastest.com/Secure/TestMe/Browser/429893 2/2 8/9/2016 MyPastest

Back to Filters (/Secure/TestMe/Filter/429893/QA) Question 121 of 153

Bacterial endocarditis is treated with penicillin. What single additional step is undertaken to prevent penicillin resistance?

A Double the penicillin dose

B Give a suitable β-lactam combination

C Add macrolides

D Add aminoglycosides

E Interrupt penicillin treatment

Explanation

Bacterial endocarditis

In those patients who have been chronically unwell for many weeks, antibiotic treatment can be deferred until the blood cultures are positive and the pathogen known In patients who are acutely ill, antibiotic treatment should be started after taking blood cultures, using a broad-spectrum combination that can be adjusted when the pathogen is known However, in many who are acutely ill with a native-valve infection, endocarditis is often not suspected initially because there may be no obvious signs and antibiotics are started for treating septicaemia There are many possible therapeutic combinations for acutely ill patients, but intravenous vancomycin and gentamicin will encompass most possible pathogens When methicillin-resistant staphylococci (whether Staphylococcus aureus or coagulase- negative staphylococci) are likely pathogens, vancomycin or teicoplanin are an essential component of any combination

1312

https://mypastest.pastest.com/Secure/TestMe/Browser/429893 1/2 8/9/2016 MyPastest Next Question

Previous Question Tag Question Feedback End Review

Difficulty: Average

Peer Responses

Session Progress

Responses Correct: 1

Responses Incorrect: 152

Responses Total: 153

Responses - % Correct: 1%

Blog (https://www.pastest.com/blog) About Pastest (https://www.pastest.com/about-us) Contact Us (https://www.pastest.com/contact-us) Help (https://www.pastest.com/help)

© Pastest 2016

https://mypastest.pastest.com/Secure/TestMe/Browser/429893 2/2 8/9/2016 MyPastest

Back to Filters (/Secure/TestMe/Filter/429893/QA) Question 122 of 153

A 19-year-old athlete was admitted to hospital after having been found wandering around in a confused state. His girlfriend is concerned that his behaviour has been very odd during the day. No illicit drugs are found on him. Clinically, he is disoriented and vague about his history but does admit to headaches. His temperature is high at 38.3 °C. On review by the medical team, his level of consciousness deteriorated, his Glasgow Coma Score (GCS) being 11 out of 15. A CT scan of his head was normal. His CSF was clear, but the opening pressure was raised

at 23 cmH2 O. His CSF protein concentration was 0.9 g/l and the glucose level was normal. The CSF showed 300 white cells, mainly lymphocytes. No organisms were seen on Gram staining. The results of CSF PCR is awaited. Which one of the following treatments would you start immediately?

A Intravenous benzylpenicillin

B Anti-tuberculous therapy

C Intravenous anti-fungal therapy

D Intravenous steroid therapy

E Intravenous aciclovir

Explanation

Encephalitis

The young man in the present case appears to have encephalitis

Aetiological agents

The likely viral agents include herpes simplex virus (most common), varicella zoster virus and HIV

Clinical features

The onset is somewhat acute; he has headaches, fever and altered level of consciousness https://mypastest.pastest.com/Secure/TestMe/Browser/429893 1/3 8/9/2016 MyPastest Other significant findings

His cerebrospinal fluid (CSF) pressure is high, macroscopically it is clear and a Gram stain is negative The CSF protein level is mildly raised, white cells are high and mainly lymphocytes and the glucose level is not low

Diagnosis

The symptoms highly favour a viral aetiology: CSF PCR will be helpful when available A magnetic resonance scan may be more helpful and may show non-specific changes in the temporal lobes

Treatment

Empirical treatment with intravenous aciclovir should be commenced for HSV as soon as possible, as it may improve the outcome

Prognosis

Severe brain damage and even death become likely once the Glasgow Coma Scale (GCS) score drops below 8

1368

Next Question

Previous Question Tag Question Feedback End Review

Difficulty: Easy

Peer Responses

Session Progress

Responses Correct: 1

Responses Incorrect: 152

Responses Total: 153

Responses - % Correct: 1% https://mypastest.pastest.com/Secure/TestMe/Browser/429893 2/3 8/9/2016 MyPastest

Blog (https://www.pastest.com/blog) About Pastest (https://www.pastest.com/about-us) Contact Us (https://www.pastest.com/contact-us) Help (https://www.pastest.com/help) © Pastest 2016

https://mypastest.pastest.com/Secure/TestMe/Browser/429893 3/3 8/9/2016 MyPastest

Back to Filters (/Secure/TestMe/Filter/429893/QA) Question 123 of 153

A 50-year-old Ghanaian man visits Ghana for a funeral, having been continuously resident in the UK for 15 years. A week after his return to the UK he develops fever and a blood test confirms the presence of malaria. The laboratory technician is happy that she can see all stages of the parasite and not just trophozoites and gametocytes. Less than 1% of erythrocytes are parasitised. Which one of the following options is the most appropriate treatment?

A Erythromycin

B Quinidine

C Chloroquine

D Blood transfusion

E Praziquantel

Explanation Malaria Type of malaria

The present patient is likely to have Plasmodium ovale malaria because all stages are visible in the peripheral blood (cf P. falciparum where only trophozoite–ring forms and gametocytes are usually seen) It is unlikely to be P. vivax as West Africans lack the Duffy red-cell antigen that is the receptor for P. vivax

Treatment and complications

The treatment of choice is chloroquine followed by primaquine to eradicate the liver hypnozoites If the patient had taken mefloquine there is an increased risk of symptomatic cerebral malaria because of low levels of transport of mefloquine across the blood brain barrier Patients with full-blown cerebral malaria are at high risk of fitting https://mypastest.pastest.com/Secure/TestMe/Browser/429893 1/2 8/9/2016 MyPastest Exchange transfusion is recommended for a parasitaemia > 10% with complications (or > 30% if no other complications) Treatment of the malaria is with IV quinine, which increases the sensitivity of cells to insulin and can cause hypoglycaemia Malaria itself can cause hypoglycaemia too, so blood glucose should be monitored 2- hourly

3677

Next Question

Previous Question Tag Question Feedback End Review

Difficulty: Average

Peer Responses

Session Progress

Responses Correct: 1

Responses Incorrect: 152

Responses Total: 153

Responses - % Correct: 1%

Blog (https://www.pastest.com/blog) About Pastest (https://www.pastest.com/about-us) Contact Us (https://www.pastest.com/contact-us) Help (https://www.pastest.com/help) © Pastest 2016

https://mypastest.pastest.com/Secure/TestMe/Browser/429893 2/2 8/9/2016 MyPastest

Back to Filters (/Secure/TestMe/Filter/429893/QA) Question 124 of 153

A 42-year-old woman who has an indwelling tunnelled catheter for haemodialysis presents to the Emergency Department. She has been feeling unwell for the past few days and is worried about a fever and sweats. On examination she is pyrexial 38.2 °C. Cardiovascular and respiratory examination is unremarkable. Investigations:

Hb 10.5 g /dl

WCC 10.9 × 109 /l

PLT 142 × 109 /l

Na+ 139 mmol/l

K+ 4.4 mmol/l

Creatinine 630 μmol/l (pre-dialysis)

CRP 87 mg/l (<10)

Urine red cells negative

Urine white cells negative (still passes around 80–100ml urine/day)

You think a line infection is the most likely problem. Which one of the following bacteria is most likely to be responsible?

A S. pyogenes

B S. epidermidis

C E. coli

D Klebsiella spp

E S. faecalis

Explanation https://mypastest.pastest.com/Secure/TestMe/Browser/429893 1/3 8/9/2016 MyPastest Infection in dialysis patients Causative organism

Staphylococcus epidermidis is a common cause of infection in dialysis patients

Diagnosis

Given the fact that in the present case the urine sample is negative, and respiratory examination is unremarkable, a line infection is likely

Management

Typical regimens used for S. epidermidis infection include rifampicin and vancomycin, vancomycin and gentamicin, and linezolid Line removal is necessary A prolonged period of antibiotic therapy for around 6 weeks or more is recommended

22438

Next Question

Previous Question Tag Question Feedback End Review

Difficulty: Average

Peer Responses

Session Progress

Responses Correct: 1

Responses Incorrect: 152

Responses Total: 153

Responses - % Correct: 1%

Blog (https://www.pastest.com/blog) About Pastest (https://www.pastest.com/about-us) Contact Us (https://www.pastest.com/contact-us) Help (https://www.pastest.com/help)

https://mypastest.pastest.com/Secure/TestMe/Browser/429893 2/3 8/9/2016 MyPastest © Pastest 2016

https://mypastest.pastest.com/Secure/TestMe/Browser/429893 3/3 8/9/2016 MyPastest

Back to Filters (/Secure/TestMe/Filter/429893/QA) Question 125 of 153

A 19-year-old man visits his GP with a severe rash, which is identified as chickenpox. He wishes to fly to the United States on a skiing holiday and wants to know when he will be safe to fly. The spots began appearing about 5 days earlier.

Which one of the following options represents the best advice?

A He is no longer infective 3 days after the first spot has appeared

B He is no longer infective 1 week after the first spot has appeared

C He is no longer infective 2 days after the last spot has appeared

D He is no longer infective after 1 week

E He is no longer infective when all lesions have crusted over and are dry.

Explanation

Infectious period in chickenpox

The prodromal period for chickenpox infection is usually less than 2 weeks; lesions may then appear for up to a week and may number a handful of spots or a very large number Patients still shed virus until all of the lesions are crusted and dry, so that definitive advice in terms of a period of time within which travel should be avoided cannot be given If travel or exposure to the public cannot be avoided, any wet lesions should be covered with a non-adherent dressing

8297

Next Question

Previous Question Tag Question Feedback End Review https://mypastest.pastest.com/Secure/TestMe/Browser/429893 1/2 8/9/2016 MyPastest

Difficulty: Easy

Peer Responses

Session Progress

Responses Correct: 1

Responses Incorrect: 152

Responses Total: 153

Responses - % Correct: 1%

Blog (https://www.pastest.com/blog) About Pastest (https://www.pastest.com/about-us) Contact Us (https://www.pastest.com/contact-us) Help (https://www.pastest.com/help) © Pastest 2016

https://mypastest.pastest.com/Secure/TestMe/Browser/429893 2/2 8/9/2016 MyPastest

Back to Filters (/Secure/TestMe/Filter/429893/QA) Question 126 of 153

A patient presents with a row of ulcers on his right leg. A biopsy from the affected area is cultured on Sabouraud’s dextrose agar. Which one of the following options is the most likely organism that may be responsible for these features?

A Sporothrix schenckii

B Cladosporium spp

C Nocardia brasiliensis

D Blastomyces dermatitidis

E Madurella mycetomi

Explanation

Differential diagnosis of infected leg ulcers Lymphangitic sporotrichosis

This is the most common infection due to Sporothrix schenckii This begins as a reddish, non-tender maculopapular lesion at the site of inoculation Over the next several weeks, similar nodules form along proximal lymphatic channels These break down to form a row of ulcers

Madurella mycetoma

This is a fungal (class Eumycetes) subcutaneous infection Infection results in local swelling that may discharge through sinuses

Cladosporium infection

This presents initially as a small papule, usually at the site of a previous injury, which persists for several months before ulcerating Itching is frequent

https://mypastest.pastest.com/Secure/TestMe/Browser/429893 1/2 8/9/2016 MyPastest Nocardia infection

This presents as a painless swelling followed by an area of ulceration with yellowish- white grains on the surface

Blastomycosis

This presents as non-itchy papular lesions that develop into ulcers with red, verrucous margins Atrophy and scarring may occur

3584

Next Question

Previous Question Tag Question Feedback End Review

Difficulty: Difficult

Peer Responses

Session Progress

Responses Correct: 1

Responses Incorrect: 152

Responses Total: 153

Responses - % Correct: 1%

Blog (https://www.pastest.com/blog) About Pastest (https://www.pastest.com/about-us) Contact Us (https://www.pastest.com/contact-us) Help (https://www.pastest.com/help)

© Pastest 2016

https://mypastest.pastest.com/Secure/TestMe/Browser/429893 2/2 8/9/2016 MyPastest

Back to Filters (/Secure/TestMe/Filter/429893/QA) Question 127 of 153

A blood culture bottle from a patient with endocarditis, so far of unknown agent, becomes positive after 2 weeks culture. The organism is a Gram-negative rod on microscopy. Which one of the following is the likeliest organism?

A Streptococcus bovis

B Pseudomonas aeruginosa

C Treponema pallidum

D Acinetobacterspp.

E Eikenella corrodens

Explanation

HACEK organisms

Eikenella corrodens is one of the HACEK organisms, fastidious oral Gram-negative rods that are documented rare causes of endocarditis The others are Haemophilus aphrophilus, Actinobacillus actinomycetemcomitans, Cardiobacterium hominis and Kingella kingae The long lead time for positive blood culture is indicative of a HACEK organism

3695

Next Question

Previous Question Tag Question Feedback End Review

Difficulty: Difficult

Peer Responses

https://mypastest.pastest.com/Secure/TestMe/Browser/429893 1/2 8/9/2016 MyPastest

Session Progress

Responses Correct: 1

Responses Incorrect: 152

Responses Total: 153

Responses - % Correct: 1%

Blog (https://www.pastest.com/blog) About Pastest (https://www.pastest.com/about-us) Contact Us (https://www.pastest.com/contact-us) Help (https://www.pastest.com/help)

© Pastest 2016

https://mypastest.pastest.com/Secure/TestMe/Browser/429893 2/2 8/9/2016 MyPastest

Back to Filters (/Secure/TestMe/Filter/429893/QA) Question 128 of 153

A 20-year-old white woman presents with jaundice and malaise of 2 weeks’ duration. Her boyfriend had some form of hepatitis several months before. Initial laboratory studies reveal alanine transaminase (ALT) of 211 U/l, aspartate transaminase (AST) of 194 U/l and bilirubin of 5.4 mg/dl. HBsAg and anti-HBc IgM are positive. Which one of the following statements regarding acute hepatitis B is false?

A About 90% of patients with acute hepatitis B will recover completely

B About 1% of patients with acute hepatitis B can experience fulminant hepatic failure

C Chronic hepatitis B carrier state will develop in 10% of patients

D Interferon administration in the acute phase of infection prevents the development of the chronic hepatitis B carrier state

E Around 5–10% of patients may develop chronic hepatitis B infection

Explanation

Hepatitis B Epidemiology

The hepatitis B virus may be carried by up to 350 million people globally Carriage rate in the UK is low at some 0.5%, but rises to around 15% in Africa, the Far East and the Middle East

Prognosis after acute infection

Ninety to ninety-five per cent of otherwise healthy adult patients with acute hepatitis B recover completely and become HBsAg negative About 1% experience massive necrosis and 5–10% of patients who remain HBsAg positive beyond 6 months are at increased risk of chronic hepatitis

Treatment

https://mypastest.pastest.com/Secure/TestMe/Browser/429893 1/2 8/9/2016 MyPastest

Interferon given during acute hepatitis B infection has not shown any benefit, but is effective in the treatment of chronic hepatitis B side-effects include an acute flu-like illness occurring some 6–8 h after first injection

5262

Next Question

Previous Question Tag Question Feedback End Review

Difficulty: Difficult

Peer Responses

Session Progress

Responses Correct: 1

Responses Incorrect: 152

Responses Total: 153

Responses - % Correct: 1%

Blog (https://www.pastest.com/blog) About Pastest (https://www.pastest.com/about-us) Contact Us (https://www.pastest.com/contact-us) Help (https://www.pastest.com/help) © Pastest 2016

https://mypastest.pastest.com/Secure/TestMe/Browser/429893 2/2 8/9/2016 MyPastest

Back to Filters (/Secure/TestMe/Filter/429893/QA) Question 129 of 153

An injecting drug user presents with jaundice and a hepatocellular blood picture. Which one of the following tests when done in isolation would confirm a cause of acute active hepatitis?

A Hepatitis A total antibody

B Hepatitis B surface antigen

C Hepatitis B anticore IgM

D Hepatitis C IgG

E Hepatitis C RNA detection

Explanation

Test results in active hepatitis

The hepatitis A total antibody test detects IgM and IgG and does not differentiate between the two

A hepatitis A IgM assay should be performed

The hepatitis C virus (HCV) IgG usually become positive some time after the acute hepatitis stage, so the diagnosis of acute HCV should be made by HCV RNA detection at the time of a negative HCV IgG, or by conversion from HCV IgG negative to positive over a few weeks after the episode of hepatitis HBsAg is present in acute and chronic hepatitis B virus (HBV) (and may even sometimes have disappeared by the time the patient presents with jaundice due to acute HBV) HB anticore IgM, if strongly positive, confirms recent, acute HBV

3680

Next Question

https://mypastest.pastest.com/Secure/TestMe/Browser/429893 1/2 8/9/2016 MyPastest

Previous Question Tag Question Feedback End Review

Difficulty: Difficult

Peer Responses

Session Progress

Responses Correct: 1

Responses Incorrect: 152

Responses Total: 153

Responses - % Correct: 1%

Blog (https://www.pastest.com/blog) About Pastest (https://www.pastest.com/about-us) Contact Us (https://www.pastest.com/contact-us) Help (https://www.pastest.com/help) © Pastest 2016

https://mypastest.pastest.com/Secure/TestMe/Browser/429893 2/2 8/9/2016 MyPastest

Back to Filters (/Secure/TestMe/Filter/429893/QA) Question 130 of 153

Which one of the following options is the most appropriate prophylaxis against Pneumocystis jirovecii infection in patients with a CD4 count below 200/mm3 ?

A Ampicillin

B Erythromycin

C Immunoglobulins

D Co-trimoxazole

E α-Interferon

Explanation

Management of low CD4 count in human immunodeficiency virus infections

The risk of developing an opportunistic infection rises greatly once the peripheral CD4 lymphocyte count falls consistently below 200/mm3 It is standard practice to introduce low-dose co-trimoxazole prophylaxis for Pneumocystis pneumonia at this stage (Note: Pneumocystis jirovecii used to be called Pneumocystis carinii) This also reduces the risk of cerebral toxoplasmosis and may prevent bacterial pneumonia

2703

Next Question

Previous Question Tag Question Feedback End Review

Difficulty: Easy

Peer Responses https://mypastest.pastest.com/Secure/TestMe/Browser/429893 1/2 8/9/2016 MyPastest

Session Progress

Responses Correct: 1

Responses Incorrect: 152

Responses Total: 153

Responses - % Correct: 1%

Blog (https://www.pastest.com/blog) About Pastest (https://www.pastest.com/about-us) Contact Us (https://www.pastest.com/contact-us) Help (https://www.pastest.com/help)

© Pastest 2016

https://mypastest.pastest.com/Secure/TestMe/Browser/429893 2/2 8/9/2016 MyPastest

Back to Filters (/Secure/TestMe/Filter/429893/QA) Question 131 of 153

A 28-year-old farm labourer presents with an aching, stiff lower back and an inability to open his mouth fully. He subsequently suffers what is described as a generalised rigid spasm by Emergency Department staff. This was not associated with loss of consciousness, confusion or fever, and he was treated with intravenous diazepam. You are unable to find any more problems on examination. What is the next most important step?

A Broad-spectrum antibiotic treatment

B Anti-tetanus immunoglobulin treatment

C Lumbar puncture

D Brain CT scan

E Call the police

Explanation

Tetanus Diagnosis

The patient in the present case has tetanus (the main differential diagnosis of which in the UK is strychnine poisoning)

Immediate management

Anti-tetanus immunoglobulin must be given urgently on clinical suspicion of the disease The initial entry wound may be trivial and is often not identified Penicillin and metronidazole should also be started, but the priority is for the immunoglobulin

3715

Next Question

https://mypastest.pastest.com/Secure/TestMe/Browser/429893 1/2 8/9/2016 MyPastest

Previous Question Tag Question Feedback End Review

Difficulty: Average

Peer Responses

Session Progress

Responses Correct: 1

Responses Incorrect: 152

Responses Total: 153

Responses - % Correct: 1%

Blog (https://www.pastest.com/blog) About Pastest (https://www.pastest.com/about-us) Contact Us (https://www.pastest.com/contact-us) Help (https://www.pastest.com/help)

© Pastest 2016

https://mypastest.pastest.com/Secure/TestMe/Browser/429893 2/2 8/9/2016 MyPastest

Back to Filters (/Secure/TestMe/Filter/429893/QA) Question 132 of 153

A 24-year-old woman came to the GU clinic complaining of urethral discharge. She admitted to three episodes of unprotected sex with different male partners over the past 3 months. Investigations: microbiology samples – Gram-negative diplococci seen

She was treated with cephalosporin, but no resolution of symptoms was apparent. With which one of the following organisms is there likely to be co-infection?

A Candida spp

B Chlamydia trachomatis

C HSV

D Syphilis

E Trichomonas vaginalis

Explanation

Chlamydia Diagnosis

The most likely organism responsible for co-infection in the present case is Chlamydia trachomatis, which may cause urethral discharge and pain in women Chlamydia needs to be grown on special culture media; as such it may be missed if conventional samples are collected

Treatment and complications

Treatment of chlamydia is with either doxycycline or azithromycin, and contact tracing should be attempted where possible Untreated chlamydia may lead to chronic pelvic inflammatory disease and tubal damage over the longer term

20787 https://mypastest.pastest.com/Secure/TestMe/Browser/429893 1/2 8/9/2016 MyPastest Next Question

Previous Question Tag Question Feedback End Review

Difficulty: Easy

Peer Responses

Session Progress

Responses Correct: 1

Responses Incorrect: 152

Responses Total: 153

Responses - % Correct: 1%

Blog (https://www.pastest.com/blog) About Pastest (https://www.pastest.com/about-us) Contact Us (https://www.pastest.com/contact-us) Help (https://www.pastest.com/help)

© Pastest 2016

https://mypastest.pastest.com/Secure/TestMe/Browser/429893 2/2 8/9/2016 MyPastest

Back to Filters (/Secure/TestMe/Filter/429893/QA) Question 133 of 153

A 30-year-old homosexual man has been diagnosed with anal carcinoma. Which one of the following pathogens is responsible for this?

A Human herpesvirus 8 (HHV8)

B HumanT-cell lymphotrophic virus type I (HTLV-I)

C HCV

D CMV

E Human papillomavirus (HPV)

Explanation

Viral causes of cancer

Human herpesvirus (HHV) 8 is associated with Kaposi’s sarcoma and body cavity (primary effusion) lymphoma in human immunodeficiency virus (HIV) infection Human T-cell lymphotrophic virus type I (HTLV-I) infection is associated with a 2–5% lifetime risk of acute T-cell lymphoma/leukaemia Human papilloma virus is considered an aetiological agent in carcinoma of anal canal in homosexual men

3693

Next Question

Previous Question Tag Question Feedback End Review

Difficulty: Average

Peer Responses

https://mypastest.pastest.com/Secure/TestMe/Browser/429893 1/2 8/9/2016 MyPastest

Session Progress

Responses Correct: 1

Responses Incorrect: 152

Responses Total: 153

Responses - % Correct: 1%

Blog (https://www.pastest.com/blog) About Pastest (https://www.pastest.com/about-us) Contact Us (https://www.pastest.com/contact-us) Help (https://www.pastest.com/help)

© Pastest 2016

https://mypastest.pastest.com/Secure/TestMe/Browser/429893 2/2 8/9/2016 MyPastest

Back to Filters (/Secure/TestMe/Filter/429893/QA) Question 134 of 153

An aid worker in West Africa for the past 5 years sees his local doctor with a fever. Blood films for malaria parasites are negative but an eosinophilia is noted. Blood films at night demonstrate microfilariae. Which one of the following is the most likely aetiological agent?

A Brugia malaya

B Loa loa

C Onchocerca volvulus

D Schistosoma haematobium

E Wuchereria bancrofti

Explanation

Tropical parasitic infections

All of tropical human parasitic infections listed in the question are important causes of eosinophilia Onchocerca volvulus is the cause of river blindness and may lead to characteristic skin nodules

Skin snips and slit-lamp examination may identify the organism

Loa loa can cause characteristic hot, soft-tissue swellings called Calabar swellings

Microfilariae are best demonstrated during the day

Brugia malaya and Wuchereria bancrofti may present with a febrile illness and acute lymphangitis, and later lymphatic obstruction can occur with elephantiasis and massive scrotal swelling Brugia malaya is not found in West Africa; it is endemic in South-East Asia

782

Next Question https://mypastest.pastest.com/Secure/TestMe/Browser/429893 1/2 8/9/2016 MyPastest

Previous Question Tag Question Feedback End Review

Difficulty: Average

Peer Responses

Session Progress

Responses Correct: 1

Responses Incorrect: 152

Responses Total: 153

Responses - % Correct: 1%

Blog (https://www.pastest.com/blog) About Pastest (https://www.pastest.com/about-us) Contact Us (https://www.pastest.com/contact-us) Help (https://www.pastest.com/help) © Pastest 2016

https://mypastest.pastest.com/Secure/TestMe/Browser/429893 2/2 8/9/2016 MyPastest

Back to Filters (/Secure/TestMe/Filter/429893/QA) Question 135 of 153

An Australian backpacker returned to the UK 2 months ago following an extended trip to east and southern Africa. He now presents with fever, giant urticaria, a headache and bloody diarrhoea. Blood tests show an eosinophil count of 7 × 109 /l. Which one of the following options is the most likely infectious agent causing his illness?

A Ascaris lumbricoides

B Strongyloides stercoralis

C Entamoeba histolytica

D Giardia lamblia

E Schistosoma mansoni

Explanation

Travel-related infections

Katayama fever develops some 6 weeks to 3 months after infection with schistosomiasis (usually caught by swimming in Lake Malawi) The present patient also has a schistosomal colitis, which can be very severe in Western travellers

Diagnosis

Diagnosis is made on serology and by rectal biopsy (looking for the eggs) Ascaris and Strongyloides cause an eosinophilia, but not these severe symptoms Entamoeba histolytica can cause a chronic colitis, but not an eosinophilia Giardia causes non-bloody diarrhoea without an eosinophilia

3707

Next Question

https://mypastest.pastest.com/Secure/TestMe/Browser/429893 1/2 8/9/2016 MyPastest

Previous Question Tag Question Feedback End Review

Difficulty: Difficult

Peer Responses

Session Progress

Responses Correct: 1

Responses Incorrect: 152

Responses Total: 153

Responses - % Correct: 1%

Blog (https://www.pastest.com/blog) About Pastest (https://www.pastest.com/about-us) Contact Us (https://www.pastest.com/contact-us) Help (https://www.pastest.com/help) © Pastest 2016

https://mypastest.pastest.com/Secure/TestMe/Browser/429893 2/2 8/9/2016 MyPastest

Back to Filters (/Secure/TestMe/Filter/429893/QA) Question 136 of 153

A 60-year-old woman is convalescing in hospital following total right knee replacement surgery undertaken 3 weeks ago. She develops headache, chills and a fever of 39.2°C. On examination the right knee is red, hot and very tender. Synovial fluid aspirate reports the growth of Gram-positive cocci. Which one of the following is the most likely causal organism?

A Staphylococcus epidermidis

B Pseudomonas aeruginosa

C Streptococcus pneumoniae

D Staphylococcus aureus

E Haemophilus influenzae

Explanation

Infected prosthesis

Bacterial infection of a prosthetic joint is a rare, but devastating and costly event

Risk of infection

Hip and knee replacements, which account for most of the joint replacement operations performed, have a 1–2% chance of becoming infected over the life of the patient or replacement

Aetiology

The most common source of infection is seeding from an infected skin lesion especially when infection occurs early in the first 12 weeks (postoperative period) Gram-positive staphylococci comprise 75–90% of the Gram-positive bacteria found in infected prosthetic joints, with Staphylococcus epidermidis being more common than Staphylococcus aureus, in contrast with septic arthritis in natural joints, where Staph. aureus predominates 1633 https://mypastest.pastest.com/Secure/TestMe/Browser/429893 1/2 8/9/2016 MyPastest 1633

Next Question

Previous Question Tag Question Feedback End Review

Difficulty: Average

Peer Responses

Session Progress

Responses Correct: 1

Responses Incorrect: 152

Responses Total: 153

Responses - % Correct: 1%

Blog (https://www.pastest.com/blog) About Pastest (https://www.pastest.com/about-us) Contact Us (https://www.pastest.com/contact-us) Help (https://www.pastest.com/help) © Pastest 2016

https://mypastest.pastest.com/Secure/TestMe/Browser/429893 2/2 8/9/2016 MyPastest

Back to Filters (/Secure/TestMe/Filter/429893/QA) Question 137 of 153

A 24-year-old man returns from a holiday in Goa. He presents straight to the Emergency Department as he was bitten by a street dog on his hand early in the morning of the second to last day of his holiday. On examination the wound looks clean and not infected. Three days have now elapsed since the bite. Investigations;

Hb 13.1 g/dl

WCC 7.2 × 109 /l

PLT 201 × 109 /l

Na+ 141 mmol/l

K+ 4.9 mmol/l

Creatinine 100 μmol/l

Which one of the following options is the most appropriate management for him?

A IV rabies immunoglobulin and course of antibiotics

B IV rabies immunoglobulin and course of rabies vaccination

C Rabies immunoglobulin infiltrated around wound and course of rabies vaccination

D Rabies immunoglobulin infiltrated around wound, course of rabies vaccination and antibiotics

E Rabies immunoglobulin infiltrated around wound and antibiotics

Explanation

Management of dog bites

In the present case, there is no indication of acute bacterial infection after 3 days, hence antibiotic therapy is almost certainly unnecessary https://mypastest.pastest.com/Secure/TestMe/Browser/429893 1/2 8/9/2016 MyPastest Human rabies immunoglobulin provides immediate protection from rabies, with a half- life of around 21 days As much as possible of the immunoglobulin should be infiltrated in and around the wound itself, with any remaining immunoglobulin given intramuscularly Rabies vaccination should also be administered, although this takes between seven and 10 days to provide adequate immunity

22419

Next Question

Previous Question Tag Question Feedback End Review

Difficulty: Difficult

Peer Responses

Session Progress

Responses Correct: 1

Responses Incorrect: 152

Responses Total: 153

Responses - % Correct: 1%

Blog (https://www.pastest.com/blog) About Pastest (https://www.pastest.com/about-us) Contact Us (https://www.pastest.com/contact-us) Help (https://www.pastest.com/help) © Pastest 2016

https://mypastest.pastest.com/Secure/TestMe/Browser/429893 2/2 8/9/2016 MyPastest

Back to Filters (/Secure/TestMe/Filter/429893/QA) Question 138 of 153

A HIV-positive patient presents with watery diarrhoea, nausea, vomiting and fever. On examination he looks dehydrated. Which one of the following options is the most likely diagnosis?

A Toxoplasmosis

B Cryptosporidiosis

C Salmonellosis

D Pneumocystisinfection

E Ulcerative colitis

Explanation

Cryptosporidiosis

Patients with cryptosporidiosis present with watery diarrhoea associated with abdominal pain, nausea, vomiting, low-grade fever, malaise, and anorexia Faecal output may be voluminous and dehydrating in immunocompromised patients, particularly those with underlying human immunodeficiency virus (HIV) infection Symptoms usually resolve by 5–10 days

3720

Next Question

Previous Question Tag Question Feedback End Review

Difficulty: Easy

Peer Responses

https://mypastest.pastest.com/Secure/TestMe/Browser/429893 1/2 8/9/2016 MyPastest

Session Progress

Responses Correct: 1

Responses Incorrect: 152

Responses Total: 153

Responses - % Correct: 1%

Blog (https://www.pastest.com/blog) About Pastest (https://www.pastest.com/about-us) Contact Us (https://www.pastest.com/contact-us) Help (https://www.pastest.com/help)

© Pastest 2016

https://mypastest.pastest.com/Secure/TestMe/Browser/429893 2/2 8/9/2016 MyPastest

Back to Filters (/Secure/TestMe/Filter/429893/QA) Question 139 of 153

A 55-year-old man complains of lethargy, fever, dry cough, headache, chest pain and increasing shortness of breath. His occupational history reveals that he installs and repairs air- conditioning systems. Which one of the following organisms is most likely to have caused this infection?

A Staphylococcus aureus

B Legionella spp

C Chlamydia psittaci

D Mycoplasma spp

E Streptococcus spp

Explanation

Legionellosis Source of infection

Legionella has been found in small numbers in water distribution systems, through which they can colonise artificial habitats, again as part of the biofilm from which they are shed into the water Factors that encourage colonisation and multiplication are temperature (20–45 °C) and stagnation The most common sites in buildings in which Legionella have been found are hot-water calorifiers and storage tanks Piped water, especially hot water from the calorifiers in large buildings and industrial complexes with long runs of pipework, is a potential source of infection Other well-recognised sources include:

Recirculating water in air-conditioning and cooling systems Whirlpool spas and other warm-water baths Decorative fountains

https://mypastest.pastest.com/Secure/TestMe/Browser/429893 1/2 8/9/2016 MyPastest Nebulisers and humidifier reservoirs of hospital ventilation machines if topped up with contaminated tap water

Transmission of infection

Dissemination of infection is by contaminated water droplets (aerosol), which are then inhaled To cause infection the droplets must be of a size (less than 5 μm in diameter) that can reach the alveoli of the lungs Taps and showerheads produce very localised aerosols, whereas the water droplets (drift) contained in the airstream released from a cooling tower may be carried a considerable distance and expose a greater number of people to risk

1295

Next Question

Previous Question Tag Question Feedback End Review

Difficulty: Easy

Peer Responses

Session Progress

Responses Correct: 1

Responses Incorrect: 152

Responses Total: 153

Responses - % Correct: 1%

Blog (https://www.pastest.com/blog) About Pastest (https://www.pastest.com/about-us) Contact Us (https://www.pastest.com/contact-us) Help (https://www.pastest.com/help) © Pastest 2016

https://mypastest.pastest.com/Secure/TestMe/Browser/429893 2/2 8/9/2016 MyPastest

Back to Filters (/Secure/TestMe/Filter/429893/QA) Question 140 of 153

A 65-year-old diabetic man who is taking a non-steroidal anti-inflammatory drug presents with fever, hypotension and a swollen, exquisitely tender thigh with subcutaneous crepitus. There is no history of trauma or injection into the leg. Which one of the following options is the most appropriate management step after broad-spectrum antibiotics have been initiated?

A Referral for urgent surgical exploration

B Ultrasound scan

C Magnetic resonance imaging (MRI) scan

D Observe closely over the next 24 hours

E Gallium scan

Explanation

Necrotising fasciitis

The description of the present case fits that of necrotising fasciitis An improved prognosis is dependent on immediate and extensive surgical debridement Imaging has not been shown to help determine the extent of debridement that will be needed and merely delays operation

3694

Next Question

Previous Question Tag Question Feedback End Review

Difficulty: Easy

Peer Responses

https://mypastest.pastest.com/Secure/TestMe/Browser/429893 1/2 8/9/2016 MyPastest

Session Progress

Responses Correct: 1

Responses Incorrect: 152

Responses Total: 153

Responses - % Correct: 1%

Blog (https://www.pastest.com/blog) About Pastest (https://www.pastest.com/about-us) Contact Us (https://www.pastest.com/contact-us) Help (https://www.pastest.com/help)

© Pastest 2016

https://mypastest.pastest.com/Secure/TestMe/Browser/429893 2/2 8/9/2016 MyPastest

Back to Filters (/Secure/TestMe/Filter/429893/QA) Question 141 of 153

You are asked to review a 54-year-old asylum seeker from eastern Europe. He has a cough and has lost weight of some 14 kg over the past few months. He admits to occasional night sweats. He is a heavy smoker of some 40 cigarettes per day. Blood testing reveals that he is HIV-positive. Chest X-ray reveals multiple calcified lymph nodes, fibrosis and hilar retraction. Initial sputum culture is unremarkable. Which one of the following options is the most likely diagnosis in this case?

A Bronchial carcinoma

B Sarcoidosis

C Silicosis

D Pulmonary tuberculosis

E Histoplasmosis

Explanation

Pulmonary tuberculosis Risk factors

The man in the present case is likely to have lived in cramped conditions where tuberculosis (TB) is endemic His human immunodeficiency virus (HIV) positivity also puts him at increased risk of infection In the UK, the peak incidence of TB is now in migrants to the UK and patients with HIV positivity, although some reactivation cases are still diagnosed in the elderly.

Radiographic features

His chest X-ray shows a picture typical of pulmonary tuberculosis The chest X-ray may also show evidence of pleural effusion in these cases, or more diffuse calcification, a ‘miliary’ pattern

https://mypastest.pastest.com/Secure/TestMe/Browser/429893 1/3 8/9/2016 MyPastest Laboratory findings

Diagnosis is confirmed via sputum for acid-fast bacilli (AFB) staining and culture Sputum may need to be induced or even obtained at bronchoscopy Gastric aspirates may be a useful alternative source of culture material Many centres now also offer PCR (polymerase chain reaction) testing for the rapid diagnosis of tuberculosis

Management and complications

Treatment of choice is combination therapy with isoniazid, rifampicin, ethambutol and pyrazinamide for 6 months In HIV-positive patients this may be extended to around 9 months Multiple drug-resistant TB is becoming an increasing problem, and is increased by prior treatment and in those patients with HIV positivity Infection in the developing world also carries an increased risk of drug resistance, as does infection acquired in institutions, eg prisons

2646

Next Question

Previous Question Tag Question Feedback End Review

Difficulty: Easy

Peer Responses

Session Progress

Responses Correct: 1

Responses Incorrect: 152

Responses Total: 153

Responses - % Correct: 1%

Blog (https://www.pastest.com/blog) About Pastest (https://www.pastest.com/about-us) https://mypastest.pastest.com/Secure/TestMe/Browser/429893 2/3 8/9/2016 MyPastest Contact Us (https://www.pastest.com/contact-us) Help (https://www.pastest.com/help) © Pastest 2016

https://mypastest.pastest.com/Secure/TestMe/Browser/429893 3/3 8/9/2016 MyPastest

Back to Filters (/Secure/TestMe/Filter/429893/QA) Question 142 of 153

You have been informed that an organism is growing in both the aerobic and the anaerobic blood-culture bottles that you obtained from a patient yesterday. A Gram-positive coccus has been isolated, which is growing in small clusters. On further laboratory testing, it is shown to cause the coagulation of fibrinogen to a fibrin clot when added to diluted plasma in a test- tube. Which one of the following is the most likely organism?

A Enterobacter cloacae

B Staphylococcus aureus

C Staphylococcus epidermidis

D Streptococcus pneumoniae

E Streptococcus pyogenes

Explanation

Features of Gram-positive and -negative bacteria

Important Gram-positive cocci include staphylococci, streptococci and enterococci Staphylococci typically form clumps and clusters in culture, whereas streptococci characteristically grow in chains of variable length Staphylococcus aureus is coagulase positive, unlike S. epidermidis. Staphylococcus species are facultative anaerobes, which means that they can grow in both aerobic and anaerobic conditions. Enterobacter spp. are Gram-negative coliforms – not to be confused with the Gram-positive enterococci, such as E. faecalis and E. faecium

1641

Next Question

Previous Question Tag Question Feedback End Review

https://mypastest.pastest.com/Secure/TestMe/Browser/429893 1/2 8/9/2016 MyPastest Difficulty: Average

Peer Responses

Session Progress

Responses Correct: 1

Responses Incorrect: 152

Responses Total: 153

Responses - % Correct: 1%

Blog (https://www.pastest.com/blog) About Pastest (https://www.pastest.com/about-us) Contact Us (https://www.pastest.com/contact-us) Help (https://www.pastest.com/help) © Pastest 2016

https://mypastest.pastest.com/Secure/TestMe/Browser/429893 2/2 8/9/2016 MyPastest

Back to Filters (/Secure/TestMe/Filter/429893/QA) Question 143 of 153

A 73-year-old woman is admitted to the Emergency department with profuse watery diarrhoea which is flecked with blood. She has recently completed a 2 week course of oral cephalosporin. Past medical history includes Type 2 diabetes, hypertension and reflux disease. On examination her BP is 100/60 mmHg, pulse 95/min and regular. Her abdomen is soft, but is generally tender with active bowel sounds.

Investigations;

Hb 12.1 g/dl

WCC 8.3 x109 /l

PLT 203 x109 /l

Na+ 138 mmol/l

K+ 3.3 mmol/l

Creatinine 145 micromol/l

Stool C. difficile toxin positive

Which of the following of her medications is most likely to have contributed to the development of C. difficile apart from the cephalosporin?

A Alendronate

B Gliclazide

C Indapamide

D Metformin

E Omeprazole

Explanation The answer is Omeprazole

https://mypastest.pastest.com/Secure/TestMe/Browser/429893 1/2 8/9/2016 MyPastest Omeprazole and other proton pump inhibitors were the subject of an FDA warning at the beginning of 2012, because of the possibility that altering the pH of the stomach increases the risk of C difficile infection. Whilst alendronate is associated with GI symptoms, the association is with oesophagitis / ulceration. Metformin is associated with diarrhoea due to bile acid malabsorption. 34349

Next Question

Previous Question Tag Question Feedback End Review

Difficulty: Average

Peer Responses

Session Progress

Responses Correct: 1

Responses Incorrect: 152

Responses Total: 153

Responses - % Correct: 1%

Blog (https://www.pastest.com/blog) About Pastest (https://www.pastest.com/about-us) Contact Us (https://www.pastest.com/contact-us) Help (https://www.pastest.com/help) © Pastest 2016

https://mypastest.pastest.com/Secure/TestMe/Browser/429893 2/2 8/9/2016 MyPastest

Back to Filters (/Secure/TestMe/Filter/429893/QA) Question 144 of 153

A patient presents with high fever, neck stiffness and a rash on both legs. A lumbar puncture reveals Gram-positive bacteria. Which one of the following options is the most likely pathogen?

A Neisseria meningitidis

B Haemophilus influenzae

C Streptococcus pneumoniae

D Pseudomonas aeruginosa

E Escherichia coli

Explanation

Pneumococcal meningitis Causative organism

Streptococcus pneumoniae is a Gram-positive, lanceolate-shaped diplococcus that was isolated independently by Sternberg and Pasteur in 1881

Symptoms

Fever and headache are the usual presenting symptoms of pneumococcal meningitis Headache usually comes on gradually over a few hours; it is generalised and may be very severe Nausea, backache and photophobia may develop, and convulsions may occur Confusion may be the most prominent symptom in elderly patients, and failure to feed the first symptom in infants Patients with pneumococcal meningitis are febrile and toxaemic

Signs

Neck stiffness and a positive Kernig’s sign are usually found in adults and in older https://mypastest.pastest.com/Secure/TestMe/Browser/429893 1/3 8/9/2016 MyPastest children Impairment of consciousness is often present, which varies in severity from drowsiness and confusion to deep coma Bradycardia and hypertension may indicate the presence of raised intracranial pressure, but papilloedema is rarely seen Bulging of the anterior fontanelle may be present in infants Cranial nerve palsies, most frequently of the VIth or of the IIIrd cranial nerve, may be found on presentation and, occasionally, other peripheral localising neurological signs are present An associated pneumococcal lesion, such as otitis media or pneumonia, may be detected Petechiae are rarely seen Herpes labialis may be present H. influenzae, E. coli and Pseudomonas are all Gram-negative rods and Neisseria are Gram-negative diplococci

1309

Next Question

Previous Question Tag Question Feedback End Review

Difficulty: Average

Peer Responses

Session Progress

Responses Correct: 1

Responses Incorrect: 152

Responses Total: 153

Responses - % Correct: 1%

Blog (https://www.pastest.com/blog) About Pastest (https://www.pastest.com/about-us) Contact Us (https://www.pastest.com/contact-us) Help (https://www.pastest.com/help) https://mypastest.pastest.com/Secure/TestMe/Browser/429893 2/3 8/9/2016 MyPastest © Pastest 2016

https://mypastest.pastest.com/Secure/TestMe/Browser/429893 3/3 8/9/2016 MyPastest

Back to Filters (/Secure/TestMe/Filter/429893/QA) Question 145 of 153

A 45-year-old man comes to the Emergency Department feeling very unwell. He returned a week earlier from a walking holiday in Corsica. He has had headache, fever, myalgias and diarrhoea for the past 2 days. On examination he has a scar on his left lower leg with a blackened area, and a maculopapular rash. He has a mild pyrexia at 37.6 °C, BP of 110/70 mmHg and a pulse of 92/min.

Investigations:

Hb 13.1 g/dl

WCC 10.9 × 109 /l

PLT 210 × 109 /l

Na+ 141 mmol/l

K+ 4.5 mmol/l

Creatinine 110 μmol/l

CRP 87 mg/l (<10)

Which one of the following options is the most likely diagnosis?

A Influenza

B Mediterranean spotted fever

C Kawasaki’s disease

D Malaria

E Q fever

Explanation

Mediterranean spotted fever

https://mypastest.pastest.com/Secure/TestMe/Browser/429893 1/3 8/9/2016 MyPastest

The history in the present case is relatively typical for Mediterranean spotted fever

Causative organism

The condition, caused by Rickettsia conorii is found among countries of southern Europe and north Africa which border the Mediterranean

Laboratory investigations

In rare cases, R. conorii can be isolated from the eschar site, although polymerase chain reaction (PCR) is a more reliable way to determine infection status

Treatment

Doxycycline 100 mg bd is the treatment of choice for R. conorii, with both quinolones and macrolides as possible alternative therapies

22445

Next Question

Previous Question Tag Question Feedback End Review

Difficulty: Average

Peer Responses

Session Progress

Responses Correct: 1

Responses Incorrect: 152

Responses Total: 153

Responses - % Correct: 1%

Blog (https://www.pastest.com/blog) About Pastest (https://www.pastest.com/about-us) Contact Us (https://www.pastest.com/contact-us) Help (https://www.pastest.com/help) https://mypastest.pastest.com/Secure/TestMe/Browser/429893 2/3 8/9/2016 MyPastest © Pastest 2016

https://mypastest.pastest.com/Secure/TestMe/Browser/429893 3/3 8/9/2016 MyPastest

Back to Filters (/Secure/TestMe/Filter/429893/QA) Question 146 of 153

A 28-year-old man presents to the GP with paroxysms of fever, sweating and rigors some 5 months after returning from a holiday in tropical India. He tells you that he was ill with fevers while in India but recovered and he put this down to an episode of flu. On further questioning he admits that his compliance with anti-malarials was patchy. On examination he looks unwell with pyrexia of 38.8 °C.

Investigations;

Hb 10.9 g/dl

WCC 13.1 × 109 /l

PLT 105 × 109 /l

Na+ 136 mmol/l

K+ 4.9 mmol/l

Creatinine 140 μmol/l

ALT 110 U/l

Thick and thin film Plasmodium vivax identified

Which one of the following options would be the most effective agent in combination to prevent re-emergence of parasites from his liver?

A Chloroquine

B Quinine

C Mefloquine

D Primaquine

E Proguanil

Explanation https://mypastest.pastest.com/Secure/TestMe/Browser/429893 1/2 8/9/2016 MyPastest Plasmodium vivax malaria

Proguanil, mefloquine, quinine and chloroquine are not effective in treating the hypnozoite form of Plasmodium vivax malaria It is likely that the patient in the present case has had a re-emergence of parasites from his liver; this is known to occur up to 6 months after initial infection with P. vivax and may happen repeatedly thereafter The other agents treat the erythrocytic stage of malarial infection and primaquine is combined with them if P. vivax or P. ovale infection is suspected As such the treatment of choice is dual therapy utilising both chloroquine and primaquine

18699

Next Question

Previous Question Tag Question Feedback End Review

Difficulty: Average

Peer Responses

Session Progress

Responses Correct: 1

Responses Incorrect: 152

Responses Total: 153

Responses - % Correct: 1%

Blog (https://www.pastest.com/blog) About Pastest (https://www.pastest.com/about-us) Contact Us (https://www.pastest.com/contact-us) Help (https://www.pastest.com/help) © Pastest 2016

https://mypastest.pastest.com/Secure/TestMe/Browser/429893 2/2 8/9/2016 MyPastest

Back to Filters (/Secure/TestMe/Filter/429893/QA) Question 147 of 153

A 52 year-old man has asked the practice nurse about the applicability of vaccines prior to taking a holiday abroad. He has a history of asthma and has required 20 mg of prednisolone therapy per day for the previous 2 months, which was recently increased to 40 mg during an exacerbation. Which one of following vaccinations would be contraindicated in this man?

A Yellow fever

B Diphtheria toxoid

C Hepatitis B

D Meningococcus

E Tetanus toxoid

Explanation

Travel vaccination

Yellow fever vaccine is a live vaccine that consists of the 17-d strain of the virus grown in hens’ eggs

Its use is not recommended in patients with a history of impaired immune responsiveness or anaphylactic reaction to egg In addition, live vaccines should not be given to individuals who are currently undergoing corticosteroid therapy

All the other vaccines listed in the question are not live vaccines

Hepatitis B and meningococcal vaccines are surface antigen based Tetanus and diphtheria vaccines are toxoids and contain no active components

3905

Next Question

https://mypastest.pastest.com/Secure/TestMe/Browser/429893 1/2 8/9/2016 MyPastest

Previous Question Tag Question Feedback End Review

Difficulty: Easy

Peer Responses

Session Progress

Responses Correct: 1

Responses Incorrect: 152

Responses Total: 153

Responses - % Correct: 1%

Blog (https://www.pastest.com/blog) About Pastest (https://www.pastest.com/about-us) Contact Us (https://www.pastest.com/contact-us) Help (https://www.pastest.com/help) © Pastest 2016

https://mypastest.pastest.com/Secure/TestMe/Browser/429893 2/2 8/9/2016 MyPastest

Back to Filters (/Secure/TestMe/Filter/429893/QA) Question 148 of 153

A 25-year-old woman refugee has had a swollen neck lymph node since 6 weeks. Serology for cytomegalovirus, toxoplasmosis and infectious mononucleosis is negative. A lymph node excision biopsy shows caseating granulomas and PCR is positive for Mycobacterium tuberculosis. After 4 weeks the lymph node culture is positive for Mycobacterium tuberculosis. Which one of the following options is the most appropriate therapy?

A Await spontaneous recovery

B Therapy depends on the tuberculin test

C Isoniazid therapy for 6 months

D Isoniazid, rifampicin, pyrazinamide and ethambutol for 2 months followed by isoniazid and rifampicin for 4 months

E Isoniazid, rifampicin, pyrazinamide and ethambutol for 18 months

Explanation

Treatment of tuberculosis

Regimens currently used for the treatment of tuberculosis are partly based on the results of trials conducted by the British Medical Research Council over the past 30 years By combining drugs that target both rapidly growing bacillary populations and slow- growing or semi-dormant organisms within cells, modern short-course chemotherapy can successfully cure drug-susceptible pulmonary tuberculosis in 6 months Treatment of extrapulmonary tuberculosis is generally for the same duration as for pulmonary disease, with the exceptions of bone, joint and central nervous system tuberculosis, which are treated for 12 months HIV-related tuberculosis is treated for 9 months

1324

Next Question https://mypastest.pastest.com/Secure/TestMe/Browser/429893 1/2 8/9/2016 MyPastest

Previous Question Tag Question Feedback End Review

Difficulty: Easy

Peer Responses

Session Progress

Responses Correct: 1

Responses Incorrect: 152

Responses Total: 153

Responses - % Correct: 1%

Blog (https://www.pastest.com/blog) About Pastest (https://www.pastest.com/about-us) Contact Us (https://www.pastest.com/contact-us) Help (https://www.pastest.com/help) © Pastest 2016

https://mypastest.pastest.com/Secure/TestMe/Browser/429893 2/2 8/9/2016 MyPastest

Back to Filters (/Secure/TestMe/Filter/429893/QA) Question 149 of 153

Which one of the following is a major side-effect of combination interferon and ribavirin therapy for the treatment of chronic hepatitis C?

A Raised serum amylase level

B Raised serum creatinine level

C Raised WCC

D Decreased haemoglobin level

E Decreased serum calcium level

Explanation

Side-effects of pharmacological treatment of hepatitis C

The most common side-effect of ribavirin is haemolytic anaemia, with the average decrease in haemoglobin being 20 g/l This therapy is best avoided in older patients, those with concomitant heart disease or patients with pre-existing haematological disorders Side-effects of interferon therapy are a flu-like syndrome, emotional lability, mood changes and sometimes thyroiditis Ribavirin is teratogenic, therefore pregnancy must be prevented during and for up to 1 year after administration, whichever sex is being treated

1310

Next Question

Previous Question Tag Question Feedback End Review

Difficulty: Average https://mypastest.pastest.com/Secure/TestMe/Browser/429893 1/2 8/9/2016 MyPastest Peer Responses

Session Progress

Responses Correct: 1

Responses Incorrect: 152

Responses Total: 153

Responses - % Correct: 1%

Blog (https://www.pastest.com/blog) About Pastest (https://www.pastest.com/about-us) Contact Us (https://www.pastest.com/contact-us) Help (https://www.pastest.com/help) © Pastest 2016

https://mypastest.pastest.com/Secure/TestMe/Browser/429893 2/2 8/9/2016 MyPastest

Back to Filters (/Secure/TestMe/Filter/429893/QA) Question 150 of 153

A 78-year-old alcoholic man presents with a 2-month history of cough with occasional haemoptysis, fever, night sweats and weight loss. Chest X-ray (CXR) shows extensive bilateral apical cavitation. Which one of the following options is the most likely diagnosis?

A Lung cancer

B Chronic pulmonary disease (COPD)

C Asthma

D Tuberculosis

E Pneumonia

Explanation

Tuberculosis

The patient in the present case has cavitating tuberculosis

Diagnostic features

A history of cough, fever, weight loss and haemoptysis and the chest X-ray appearance are strongly suggestive of tuberculosis Whilst lung cancer would be the alternative diagnosis, the constellation of symptoms here point one towards tuberculosis

Complications of treatment

In older patients, the development of multidrug-resistant TB (MDRTB) (resistance to rifampicin and isoniazid) is extremely rare, especially in those who are fully compliant with therapy An acute psychosis can occur with isoniazid Ethambutol ocular toxicity is extremely rare (1% at a dose of 15 mg/kg) but as it is renally excreted toxicity can occur in the elderly if there is unsuspected renal https://mypastest.pastest.com/Secure/TestMe/Browser/429893 1/2 8/9/2016 MyPastest impairment Drug-induced hepatitis is likely An alanine aminotransferase of five times the upper limit of normal necessitates interrupting therapy Pyrazinamide elevates serum urate Frank gout is rare

3711

Next Question

Previous Question Tag Question Feedback End Review

Difficulty: Easy

Peer Responses

Session Progress

Responses Correct: 1

Responses Incorrect: 152

Responses Total: 153

Responses - % Correct: 1%

Blog (https://www.pastest.com/blog) About Pastest (https://www.pastest.com/about-us) Contact Us (https://www.pastest.com/contact-us) Help (https://www.pastest.com/help) © Pastest 2016

https://mypastest.pastest.com/Secure/TestMe/Browser/429893 2/2 8/9/2016 MyPastest

Back to Filters (/Secure/TestMe/Filter/429893/QA) Question 151 of 153

A 35-year-old woman returns from a 2-year work placement in west Africa. She has had numerous insect bites. She has oedematous, red, itchy lesions on her forearms and complains of fever. Microfilariae of loa loa are found in the blood film. Which one of the following options is the most appropriate treatment?

A Penicillin

B Erythromycin

C Diethylcarbamazine

D Prednisolone

E Quinidine

Explanation

Treatment of loiasis

Diethylcarbamazine remains the treatment of choice Adequate dosage will kill adult worms A small, single dose will clear blood microfilariae temporarily Sensitivity reactions to filarial antigen, both local and systemic, are common in infected people and simulate some of the acute manifestations of the infection; they necessitate care and supervision in the initial stages Treatment should be started at 1 mg/kg on the first day, increasing over 3 or more days to 6 mg/kg in divided doses, this dose then being continued for 21 days

3708

Next Question

Previous Question Tag Question Feedback End Review

https://mypastest.pastest.com/Secure/TestMe/Browser/429893 1/2 8/9/2016 MyPastest

Difficulty: Average

Peer Responses

Session Progress

Responses Correct: 1

Responses Incorrect: 152

Responses Total: 153

Responses - % Correct: 1%

Blog (https://www.pastest.com/blog) About Pastest (https://www.pastest.com/about-us) Contact Us (https://www.pastest.com/contact-us) Help (https://www.pastest.com/help)

© Pastest 2016

https://mypastest.pastest.com/Secure/TestMe/Browser/429893 2/2 8/9/2016 MyPastest

Back to Filters (/Secure/TestMe/Filter/429893/QA) Question 152 of 153

A patient with recurrent venous thromboembolic disease is on long-term warfarin. He is admitted with an infectious disease and commenced on antibiotics. Subsequently his INR comes down to 1.4 despite remaining on his usual dose of warfarin. Which antibiotic is most likely to be responsible?

A Aztreonam

B Ciprofloxacin

C Erythromycin

D Metronidazole

E Rifampicin

Explanation

Drug interactions

Rifampicin is a potent enzyme inducer, thereby reducing the anticoagulant effect of warfarin Aztreonam, ciprofloxacin, erythromycin and metronidazole are enzyme inhibitors

766

Next Question

Previous Question Tag Question Feedback End Review

Difficulty: Average

Peer Responses

https://mypastest.pastest.com/Secure/TestMe/Browser/429893 1/2 8/9/2016 MyPastest

Session Progress

Responses Correct: 1

Responses Incorrect: 152

Responses Total: 153

Responses - % Correct: 1%

Blog (https://www.pastest.com/blog) About Pastest (https://www.pastest.com/about-us) Contact Us (https://www.pastest.com/contact-us) Help (https://www.pastest.com/help)

© Pastest 2016

https://mypastest.pastest.com/Secure/TestMe/Browser/429893 2/2 8/9/2016 MyPastest

Back to Filters (/Secure/TestMe/Filter/429893/QA) Question 153 of 153

An 5-year-old boy is admitted with a temperature of 39.6°C and a rash consisting of numerous dusky pink macules and papules. He became unwell 6 days ago, when his mother noticed that he had a dry cough, red eyes and a temperature. The rash started 2 days prior to admission, appearing on his face initially, but then spreading to the trunk and limbs. He was in contact with a boy with a similar rash 10 days ago. There is no significant past medical history. He had not received all his childhood immunisations due to parental concerns regarding vaccine safety. Which one of the following options is the likely cause of his rash?

A Central retinal vein occlusion

B Measles virus

C Parvovirus B19

D Rubella virus

E Mumps virus

Explanation

Measles

The history in the present case is typical of measles

Clinical features

Measles has an incubation period of 8–12 days, followed by a 2–6 day prodrome with coryzal symptoms, conjunctivitis, dry cough and fever before the rash appears Koplik’s spots (small red lesions with a bluish white centre on the buccal mucosa opposite the 2nd molar teeth) appear during the prodrome

Differential diagnosis

Rubella has a longer incubation period (12–21 days) than measles and is a milder illness, typically with a low-grade (or absent) fever and no significant respiratory symptoms

https://mypastest.pastest.com/Secure/TestMe/Browser/429893 1/2 8/9/2016 MyPastest (apart from coryza) Parvovirus B19 infection typically causes the ‘slapped-cheek’ appearance, but can mimic rubella EBV is usually asymptomatic in young children, and mumps does not cause a rash

1640

End Session

Previous Question Tag Question Feedback End Review

Difficulty: Easy

Peer Responses

Session Progress

Responses Correct: 1

Responses Incorrect: 152

Responses Total: 153

Responses - % Correct: 1%

Blog (https://www.pastest.com/blog) About Pastest (https://www.pastest.com/about-us) Contact Us (https://www.pastest.com/contact-us) Help (https://www.pastest.com/help)

© Pastest 2016

https://mypastest.pastest.com/Secure/TestMe/Browser/429893 2/2 8/9/2016 MyPastest

Back to Filters (/Secure/TestMe/Filter/429893/QA) Question 1 of 151

An epidemic of diarrhoea and vomiting has broken out on the elderly care wards. Your catering suppliers assure you that their food is unlikely to be responsible as they follow strict hygiene procedures. A total of 15 patients on the ward have become unwell with a sudden onset of diarrhoea and vomiting. Patients infected earlier have recovered with rehydration therapy after about 48 h. Which one of the following viruses is most likely to be responsible for this outbreak?

A Enteric adenovirus

B Picornavirus

C Norovirus

D Astrovirus

E Rotavirus

Explanation

Norovirus infection Causative organism

Norovirus is the commonest cause of viral gastroenteritis:

it is said to account for the vast majority of cases, around 96% according to the CDC it accounts for a significant burden in the National Health Service (NHS), particularly during the winter months

Rotavirus is so named because of its characteristic circle with radiating spokes outline but is not the most likely cause here

Epidemiology

Approximately 3000 patients are admitted to hospital with norovirus per year It is thought to be responsible for approximately 15% of cases of infectious GI upset in https://mypastest.pastest.com/Secure/TestMe/Browser/429893 1/2 8/9/2016 MyPastest the community Cyclical outbreaks have led to it being given the name of the winter vomiting virus

Diagnosis

Diagnosis is established via ELISA (enzyme-linked immunosorbent assay) or PCR

Treatment and prevention

Treatment is aimed at adequate rehydration

2660

Next Question

revious Question Tag Question Feedback End Review

Difficulty: Average

Peer Responses

Session Progress

Responses Correct: 0

Responses Incorrect: 151

Responses Total: 151

Responses - % Correct: 0%

Blog (https://www.pastest.com/blog) About Pastest (https://www.pastest.com/about-us) Contact Us (https://www.pastest.com/contact-us) Help (https://www.pastest.com/help)

© Pastest 2016

https://mypastest.pastest.com/Secure/TestMe/Browser/429893 2/2 8/9/2016 MyPastest

Back to Filters (/Secure/TestMe/Filter/429893/QA) Question 2 of 151

A 30-year-old man presents with an acute onset of pain and blurred vision of his right eye. On examination there is conjunctival injection, and dendritic ulceration is seen on his cornea. Which one of the following options is the most likely diagnosis?

A Herpes simplex virus keratitis

B Foreign body

C Candida keratitis

D Trachoma

E Glaucoma

Explanation

Herpes simplex virus keratitis

Herpes simplex virus (HSV) keratitis is characterised by the acute onset of pain, blurred vision, conjunctival injection and dendritic ulceration of the cornea HSV keratitis can cause corneal blindness and its treatment is urgent

2691

Next Question

Previous Question Tag Question Feedback End Review

Difficulty: Easy

Peer Responses

https://mypastest.pastest.com/Secure/TestMe/Browser/429893 1/2 8/9/2016 MyPastest

Session Progress

Responses Correct: 0

Responses Incorrect: 151

Responses Total: 151

Responses - % Correct: 0%

Blog (https://www.pastest.com/blog) About Pastest (https://www.pastest.com/about-us) Contact Us (https://www.pastest.com/contact-us) Help (https://www.pastest.com/help)

© Pastest 2016

https://mypastest.pastest.com/Secure/TestMe/Browser/429893 2/2 8/9/2016 MyPastest

Back to Filters (/Secure/TestMe/Filter/429893/QA) Question 3 of 151

A 34-year-old coronary care nurse accidentally stabs himself with a used needle from a patient infected with the hepatitis C virus. He attends the occupational health department the following day, (later than specified by hospital policy), and asks for advice. Which one of the following options would be the most appropriate next step suggested by the occupational health doctor?

A Monthly hepatitis C antibody testing

B Monthly hepatitis C RT PCR/PCR testing

C 6 months’ ribavirin therapy

D 6 months’ lamivudine therapy

E 6 months of weekly interferon therapy

Explanation

Hepatitis C - Investigations -

The most appropriate test is with polymerase chain reaction (PCR) as this becomes positive prior to IgG levels

Treatment

Acute hepatitis C is now amenable to treatment, and very high success rates have been reported in a relatively recent landmark paper published in the New England Journal of Medicine If seroconversion occurs, the nurse would be offered interferon (usually in combination with ribavirin), which would afford a greater than 90% chance of cure Length of therapy depends on the hepatitis C genotype Lamivudine is used for hepatitis B and HIV treatment

591 https://mypastest.pastest.com/Secure/TestMe/Browser/429893 1/2 8/9/2016 MyPastest

Next Question

Previous Question Tag Question Feedback End Review

Difficulty: Average

Peer Responses

Session Progress

Responses Correct: 0

Responses Incorrect: 151

Responses Total: 151

Responses - % Correct: 0%

Blog (https://www.pastest.com/blog) About Pastest (https://www.pastest.com/about-us) Contact Us (https://www.pastest.com/contact-us) Help (https://www.pastest.com/help) © Pastest 2016

https://mypastest.pastest.com/Secure/TestMe/Browser/429893 2/2 8/9/2016 MyPastest

Back to Filters (/Secure/TestMe/Filter/429893/QA) Question 4 of 151

A 33-year-old man has recently returned from a holiday in Pakistan. He is complaining of episodes of abdominal spasms followed by loose stools containing blood and mucus. Which one of the following pathogens is not likely to be causative of his disorder?

A Entamoeba histolytica

B Shigella dysenteriae

C Salmonella typhi

D Campylobacter jejuni

E Schistosoma mansoni

Explanation

Schistosomiasis Causative organism

Schistosoma mansoni is found mainly in Africa and South America

Clinical features

Schistosomiasis affects the colon and produces erythema and superficial ulceration Progressive fibrosis leads to rigidity and stricture formation in the intestine Patients can also develop hepatitis, followed by periportal fibrosis and portal hypertension, hepatosplenomegaly and oesophageal varices

1366

Next Question

Previous Question Tag Question Feedback End Review

https://mypastest.pastest.com/Secure/TestMe/Browser/429893 1/2 8/9/2016 MyPastest

Difficulty: Average

Peer Responses

Session Progress

Responses Correct: 0

Responses Incorrect: 151

Responses Total: 151

Responses - % Correct: 0%

Blog (https://www.pastest.com/blog) About Pastest (https://www.pastest.com/about-us) Contact Us (https://www.pastest.com/contact-us) Help (https://www.pastest.com/help) © Pastest 2016

https://mypastest.pastest.com/Secure/TestMe/Browser/429893 2/2 8/9/2016 MyPastest

Back to Filters (/Secure/TestMe/Filter/429893/QA) Question 5 of 151

A 17-year-old girl is admitted with a non-blanching rash suspicious of meningococcal septicaemia. According to her boyfriend she has had symptoms of a sore throat and head cold over the past few days, and over the past 12 h has becoming increasingly drowsy and confused. The GP administered iv benzylpenicillin at home while awaiting the ambulance. On examination she is pyrexial at 38.6 °C, her BP is 95/60 mmHg, pulse 105/min, and she has an extensive petechial rash consistent with meningococcal septicaemia, including over the area you are considering for lumbar puncture. She is drowsy and photophobic but you manage to get a view of her optic discs and there is evidence of papilloedema. Investigations;

Hb 12.1 g/dl

WCC 15.6 × 109 /l

PLT 210 × 109 /l

Na+ 138 mmol/l

K+ 4.4 mmol/l

Creatinine 134 μmol/l

CT head slight ventricular enlargement

Which one of the following options is the most appropriate way to confirm the diagnosis of meningococcal infection?

A Blood culture

B CSF microscopy and culture

C Skin lesion culture

D PCR of whole blood for meningococcus

E Meningococcal serology

https://mypastest.pastest.com/Secure/TestMe/Browser/429893 1/3 8/9/2016 MyPastest Explanation

Meningococcal infection Appropriate investigations

Lumbar puncture (LP) to gain a cerebrospinal fluid (CSF) sample is not recommended when there is raised intracranial pressure or evidence of a meningococcal skin rash over the area where LP is being considered Additionally, the yield from CSF culture is diminished after antibiotic therapy has been commenced As such polymerase chain reaction (PCR) of blood, taken as soon as possible, preferably just before introduction of IV antibiotics, is the optimal investigation

Treatment

Treatment is with a broad-spectrum cephalosporin such as ceftriaxone

21328

Next Question

Previous Question Tag Question Feedback End Review

Difficulty: Average

Peer Responses

Session Progress

Responses Correct: 0

Responses Incorrect: 151

Responses Total: 151

Responses - % Correct: 0%

Blog (https://www.pastest.com/blog) About Pastest (https://www.pastest.com/about-us) Contact Us (https://www.pastest.com/contact-us) Help (https://www.pastest.com/help)

https://mypastest.pastest.com/Secure/TestMe/Browser/429893 2/3 8/9/2016 MyPastest © Pastest 2016

https://mypastest.pastest.com/Secure/TestMe/Browser/429893 3/3 8/9/2016 MyPastest

Back to Filters (/Secure/TestMe/Filter/429893/QA) Question 6 of 151

HIV patients with a CD4 count below 200/mm3 should receive appropriate prophylaxis against Pneumocystis jirovecii (formerly called Pneumocystis carinii) pneumonia. Which one of the following options is the most appropriate medication?

A Ampicillin

B Erythromycin

C Co-trimoxazole

D Corticosteroids

E Cefaclor

Explanation

Pneumocystis jirovecii pneumonia

All human immunodeficiency virus (HIV)-infected patients with a CD4 count below 200/mm3 should receive effective prophylaxis with low-dose suppressive therapy to prevent Pneumocystis jirovecii pneumonia Co-trimoxazole is the preferred agent Dapsone and inhaled pentamidine are also used

1292

Next Question

Previous Question Tag Question Feedback End Review

Difficulty: Easy

Peer Responses

https://mypastest.pastest.com/Secure/TestMe/Browser/429893 1/2 8/9/2016 MyPastest

Session Progress

Responses Correct: 0

Responses Incorrect: 151

Responses Total: 151

Responses - % Correct: 0%

Blog (https://www.pastest.com/blog) About Pastest (https://www.pastest.com/about-us) Contact Us (https://www.pastest.com/contact-us) Help (https://www.pastest.com/help)

© Pastest 2016

https://mypastest.pastest.com/Secure/TestMe/Browser/429893 2/2 8/9/2016 MyPastest

Back to Filters (/Secure/TestMe/Filter/429893/QA) Question 7 of 151

A 27-year-old man with a history of intravenous drug use was found to have abnormal liver function tests. Further work-up, including serological tests for viral hepatitis, show:

hepatitis B surface antibody (HBsAb) negative hepatitis B surface antigen (HBsAg) positive hepatitis B core antibody (HBcAb) positive hepatitis B e antibody (HBeAb) positive hepatitis B e antigen (HBeAg) negative hepatitis B DNA is negative

Which one of the following statements is true regarding this patient?

A He is a chronic hepatitis B virus (HBV) carrier with high infectivity

B He is in the incubation period of HBV

C He is a chronic HBV carrier with low infectivity

D He has recovered from HBV infection and is immune to HBV

E He has chronic active hepatitis

Explanation

Interpretation of hepatitis B blood results In the interpretation of results of hepatitis B serological tests, the following facts should be considered;

during the incubation period (ie, before the onset of clinical manifestations):

HBsAg, HBeAg and HBV DNA become detectable in the serum

At the onset of clinical symptoms (eg, jaundice), an increase in the serum transaminases antibodies occurs and antibodies to HBc become detectable (HBc antibodies)

https://mypastest.pastest.com/Secure/TestMe/Browser/429893 1/3 8/9/2016 MyPastest

Initially, the HBc antibodies are IgM and thereafter IgG; these latter antibodies persist for years HBs antibodies become detectable late in convalescence A rise in HBs antibodies in combination with a loss of HBsAg, HBeAg and HBV DNA indicate the presence of immunity to HBV HBeAg and HBV DNA are markers of active viral replication and so indicate high infectivity The loss of HBeAg and appearance of anti-HBeAg with persistence of HBsAg indicates a chronic carrier state with low infectivity

Not all patients who lose HBeAg and seroconvert to anti-HBe antibody go into sustained remission of HBV replication and liver disease activity

A variable proportion of the HBeAg-negative and anti-HBe positive patients, depending upon HBV genotype as well as other factors, retains or redevelops high serum HBV- DNA levels and persistent or intermittent elevations in alanine aminotransferase (ALT) activity Such patients harbour replication-competent HBV variants that are unable to produce HBeAg due to some mutations either in the precore or the basic core promoter region of the HBV genome This form of chronic hepatitis B is also referred to as HBeAg-negative, or anti-HBe- positive chronic hepatitis B

it represents a potentially severe and progressive form of liver disease, with frequent development of cirrhosis and chronic hepatitis C

5255

Next Question

Previous Question Tag Question Feedback End Review

Difficulty: Average

Peer Responses

Session Progress https://mypastest.pastest.com/Secure/TestMe/Browser/429893 2/3 8/9/2016 MyPastest

Responses Correct: 0

Responses Incorrect: 151

Responses Total: 151

Responses - % Correct: 0%

Blog (https://www.pastest.com/blog) About Pastest (https://www.pastest.com/about-us) Contact Us (https://www.pastest.com/contact-us) Help (https://www.pastest.com/help) © Pastest 2016

https://mypastest.pastest.com/Secure/TestMe/Browser/429893 3/3 8/9/2016 MyPastest

Back to Filters (/Secure/TestMe/Filter/429893/QA) Question 8 of 151

A 22-year-old man presents complaining of severe itching and erythematous lesions in his groin with a white scaly border for the past month. The lesions have well-defined margins and the more mature ones have normal looking skin at their centre. Which one of the following is most likely to be the causative agent?

A Trichophyton rubrum

B Candida albicans

C Candida glabrata

D Malassezia furfur

E Phthiriasis pubis

Explanation

Differential diagnosis of groin lesions

The classic appearance of tinea cruris (caused by T. rubrum) infection is a red scaly lesion However, the patient’s lesions are typical of this condition Candida infection presents as small pustules with satellite lesions that have a typical frayed peeling edge on rupture

superficial white or creamy pseudomembranous plaques appear that can be scraped off, leaving raw areas underneath

Malassezia furfur causes pityriasis versicolor that is asymptomatic and not inflammatory

the lesions are more likely to occur on the trunk than the genital area

Pubic lice (Phthiriasis pubis) present with itching, especially at night

scaly lesions are usually not seen

Trichophyton rubrum is a dermatophyte commonly associated with fungal infection of https://mypastest.pastest.com/Secure/TestMe/Browser/429893 1/2 8/9/2016 MyPastest the nail (onychomycosis)

3591

Next Question

Previous Question Tag Question Feedback End Review

Difficulty: Average

Peer Responses

Session Progress

Responses Correct: 0

Responses Incorrect: 151

Responses Total: 151

Responses - % Correct: 0%

Blog (https://www.pastest.com/blog) About Pastest (https://www.pastest.com/about-us) Contact Us (https://www.pastest.com/contact-us) Help (https://www.pastest.com/help) © Pastest 2016

https://mypastest.pastest.com/Secure/TestMe/Browser/429893 2/2 8/9/2016 MyPastest

Back to Filters (/Secure/TestMe/Filter/429893/QA) Question 9 of 151

A patient presents with sudden onset of nausea and vomiting associated with watery diarrhoea. The diarrhoea has intensified and now is of a rice watery character. Which one of the following options is the most likely diagnosis?

A Diphtheria

B Cholera

C Tetanus

D Ulcerative colitis

E Salmonellosis

Explanation

Clinical manifestations of cholera

Vomiting and watery diarrhoea are the initial signs of cholera Diarrhoea may be modest at first – and consist of faecal matter and watery stool In some patients, the diarrhoea becomes profound – exceeding 200 ml/kg body weight per day In these patients the stool will become ‘rice-watery’ in character – in other words, it resembles the opaque white water discarded after rice has been washed – it will not contain faecal matter, and is not malodorous The diarrhoea is painless and patients are often incontinent of stool In the absence of antimicrobial treatment, the total stool volume during the illness can exceed total body weight

3714

Next Question

https://mypastest.pastest.com/Secure/TestMe/Browser/429893 1/2 8/9/2016 MyPastest Previous Question Tag Question Feedback End Review

Difficulty: Easy

Peer Responses

Session Progress

Responses Correct: 0

Responses Incorrect: 151

Responses Total: 151

Responses - % Correct: 0%

Blog (https://www.pastest.com/blog) About Pastest (https://www.pastest.com/about-us) Contact Us (https://www.pastest.com/contact-us) Help (https://www.pastest.com/help) © Pastest 2016

https://mypastest.pastest.com/Secure/TestMe/Browser/429893 2/2 8/9/2016 MyPastest

Back to Filters (/Secure/TestMe/Filter/429893/QA) Question 10 of 151

A 35-year-old sailor presents with a painless swelling on the sole of his foot that has progressively increased in size. There is an area of ulceration with yellowish-white grains on the surface. Gram staining of a smear from the ulcer shows Gram-positive branching organisms. Which one of the following options is the most probable causative agent?

A Madurella mycetomi

B Cladosporium spp

C Nocardia brasiliensis

D Sporothrix schenckii

E Blastomyces dermatitidis

Explanation

Nocardia infection

The clinical features and investigative findings in the present case are suggestive of infection with a species of Nocardia

Differential diagnosis

Madurella mycetoma is a subcutaneous fungal (class Eumycetes) infection

Infection results in local swelling that may discharge through sinuses

Cladosporium infection presents initially as a small papule usually at the site of a previous injury

This persists for several months before ulcerating Itching injury severity scores (ISS) frequent

Sporotrichosis due to Sporothrix schenckii begins as a reddish, non-tender maculopapular lesion Blastomycosis presents as non-itchy papular lesions that develop into ulcers with red, https://mypastest.pastest.com/Secure/TestMe/Browser/429893 1/2 8/9/2016 MyPastest verrucous margins

Atrophy and scarring may occur

3577

Next Question

Previous Question Tag Question Feedback End Review

Difficulty: Difficult

Peer Responses

Session Progress

Responses Correct: 0

Responses Incorrect: 151

Responses Total: 151

Responses - % Correct: 0%

Blog (https://www.pastest.com/blog) About Pastest (https://www.pastest.com/about-us) Contact Us (https://www.pastest.com/contact-us) Help (https://www.pastest.com/help) © Pastest 2016

https://mypastest.pastest.com/Secure/TestMe/Browser/429893 2/2 8/9/2016 MyPastest

Back to Filters (/Secure/TestMe/Filter/429893/QA) Question 11 of 151

A 25-year-old man presents with 4 days of fever, retro-orbital pain and severe myalgia following travel to the Indian subcontinent. He has red eyes and a faint, blanching, maculopapular rash. A peripheral smear for malarial parasites is negative and his white cell count and chest X-ray are normal. Which one of the following options is the most likely diagnosis?

A Malaria

B Typhoid fever

C Bubonic plague

D Dengue fever

E Tuberculosis

Explanation

Dengue fever Causative organism

Dengue fever is transmitted from the bite of infected Aedes aegypti mosquitoes

High-risk areas

It is commonly seen in the tropics where epidemics occur

Clinical presentation

After an incubation period of 3–4 days, patients develop fever, severe myalgias and headaches (mainly retro-orbital) Painful red eyes are commonly seen, along with lymphadenopathy A faint, blanching, maculopapular rash is characteristic

Laboratory findings https://mypastest.pastest.com/Secure/TestMe/Browser/429893 1/3 8/9/2016 MyPastest

White cell counts are normal and platelets may be low, along with mildly abnormal liver function tests It can be diagnosed during the fever by detection of the viral genome by reverse transcriptase polymerase chain reaction (RT-PCR) Paired sera can provide a retrospective diagnosis

Prognosis and management

A small proportion of patients may develop dengue haemorrhagic fever with disseminated intravascular coagulation Treatment is supportive

Bubonic plague is associated with lymphatic swelling "buboes", and is spread by rodent fleas, not therefore the presentation seen here. 1359

Next Question

Previous Question Tag Question Feedback End Review

Difficulty: Average

Peer Responses

Session Progress

Responses Correct: 0

Responses Incorrect: 151

Responses Total: 151

Responses - % Correct: 0%

Blog (https://www.pastest.com/blog) About Pastest (https://www.pastest.com/about-us) Contact Us (https://www.pastest.com/contact-us) Help (https://www.pastest.com/help) © Pastest 2016

https://mypastest.pastest.com/Secure/TestMe/Browser/429893 2/3 8/9/2016 MyPastest

https://mypastest.pastest.com/Secure/TestMe/Browser/429893 3/3 8/9/2016 MyPastest

Back to Filters (/Secure/TestMe/Filter/429893/QA) Question 12 of 151

A patient with AIDS attends with purplish, nodular skin lesions that have been increasing in size since 2 months. They started as small, pink macules and some have ulcerated. Which one of the following options is the most likely diagnosis?

A Malignant melanoma

B Tuberculosis

C Papillomavirus infection

D Keratosis

E Kaposi’s sarcoma

Explanation

Kaposi’s sarcoma Clinical presentation

Kaposi’s sarcoma characteristically presents as multiple, purplish nodular skin lesions The lesions start as small, pink, deep-purple or brown macules, and develop into nodules or plaques that may ulcerate They also occur on mucosal surfaces, most commonly on the hard palate Local or regional oedema and lymph node enlargement may occur

Consequences and complications

Mucocutaneous lesions are cosmetically and psychologically compromising for the patient but are rarely clinically significant However, visceral disease, which most commonly affects the lungs and gastrointestinal tract, is a cause of morbidity and even mortality Lung lesions cause dyspnoea, cough or haemoptysis, and gut involvement may cause abdominal pain, bleeding or a rare protein-losing enteropathy Extensive visceral involvement can cause constitutional symptoms such as fevers, night https://mypastest.pastest.com/Secure/TestMe/Browser/429893 1/2 8/9/2016 MyPastest sweats and weight loss Kaposi’s sarcoma rarely affects the central nervous system

2702

Next Question

Previous Question Tag Question Feedback End Review

Difficulty: Easy

Peer Responses

Session Progress

Responses Correct: 0

Responses Incorrect: 151

Responses Total: 151

Responses - % Correct: 0%

Blog (https://www.pastest.com/blog) About Pastest (https://www.pastest.com/about-us) Contact Us (https://www.pastest.com/contact-us) Help (https://www.pastest.com/help)

© Pastest 2016

https://mypastest.pastest.com/Secure/TestMe/Browser/429893 2/2 8/9/2016 MyPastest

Back to Filters (/Secure/TestMe/Filter/429893/QA) Question 13 of 151

A 26-year-old man presents to the sexually transmitted diseases clinic. He has returned from a trip in eastern Europe and he admits to an episode of unprotected sex with a woman he met in a bar some 2 weeks earlier. He complains of pain on passing urine, arthritic-type pain affecting predominantly his knees, wrists, ankles and the small joints of his hands. There is associated conjunctivitis and a psoriatic-type rash on his palms and the soles of his feet.

Investigations:

Hb 13.1 g/dl

WCC 6.1 × 109 /l

PLT 301 × 109 /l

Na+ 140 mmol/l

K+ 4.9 mmol/l

Creatinine 100 μmol/l

ESR 52 mm/h

Which one of the following options is the HLA-subtype most commonly associated with this condition?

A HLA-DR2

B HLA-DR3

C HLA-DR4

D HLA-B26

E HLA-B27

Explanation

Reactive arthritis https://mypastest.pastest.com/Secure/TestMe/Browser/429893 1/3 8/9/2016 MyPastest Underlying disease

Reactive arthritis is associated with both gastrointestinal infection (Shigella, Salmonella and Campylobacter) and non-specific urethritis

Diagnosis

Human leukocyte antigen (HLA)-B27 testing is positive in around 65% of cases of reactive arthritis, although it is not a routine part of the diagnostic workup

Treatment

Non-steroidals and corticosteroids are the mainstay of therapy for the condition The role of antibiotics in shortening the duration of the condition is controversial – while positive results have been seen where Chlamydia is the cause, they do not appear to shorten the condition where gastrointestinal infection is thought to be the precipitant

18605

Next Question

Previous Question Tag Question Feedback End Review

Difficulty: Average

Peer Responses

Session Progress

Responses Correct: 0

Responses Incorrect: 151

Responses Total: 151

Responses - % Correct: 0%

Blog (https://www.pastest.com/blog) About Pastest (https://www.pastest.com/about-us) Contact Us (https://www.pastest.com/contact-us) Help (https://www.pastest.com/help) https://mypastest.pastest.com/Secure/TestMe/Browser/429893 2/3 8/9/2016 MyPastest © Pastest 2016

https://mypastest.pastest.com/Secure/TestMe/Browser/429893 3/3 8/9/2016 MyPastest

Back to Filters (/Secure/TestMe/Filter/429893/QA) Question 14 of 151

An HIV-positive patient with a CD4 count of 550 cells/μl attends clinic to seek advice on vaccinations prior to travel to sub-Saharan Africa. Which of the following vaccines should be avoided?

A BCG

B Hepatitis A

C Polio (Salk)

D Meningococcal

E Tetanus

Explanation

Vaccinations in human immunodeficiency virus (HIV) infection

BCG, yellow fever and oral typhoid are all live vaccines, which should be avoided in HIV- positive patients, even if the CD4 count is well preserved Hepatitis A, polio (Salk) and meningococcal vaccines are inactivated vaccines, while tetanus is a toxoid Measles, mumps and rubella (MMR) and polio (Sabin) are live vaccines deemed safe for use in HIV-positive patients

787

Next Question

Previous Question Tag Question Feedback End Review

Difficulty: Average

Peer Responses https://mypastest.pastest.com/Secure/TestMe/Browser/429893 1/2 8/9/2016 MyPastest

Session Progress

Responses Correct: 0

Responses Incorrect: 151

Responses Total: 151

Responses - % Correct: 0%

Blog (https://www.pastest.com/blog) About Pastest (https://www.pastest.com/about-us) Contact Us (https://www.pastest.com/contact-us) Help (https://www.pastest.com/help)

© Pastest 2016

https://mypastest.pastest.com/Secure/TestMe/Browser/429893 2/2 8/9/2016 MyPastest

Back to Filters (/Secure/TestMe/Filter/429893/QA) Question 15 of 151

A 28-year-old commercial sex worker is diagnosed as being HIV-positive. Her CD4 count is > 200 and she presented with salmonella gastroenteritis. This is successfully managed. In addition to highly active anti-retroviral therapy (HAART), against which one of the following organisms would prophylaxis be most useful over the long term?

A Toxoplasma gondii

B Cytomegalovirus

C Pneumocystis jirovecii

D Salmonellae

E Cryptococci

Explanation

Pharmacotherapy in human immunodeficiency virus (HIV)-positive individuals

Primary prophylaxis has been shown to be effective in reducing the risk of P. jirovecii (previously known as P. carinii) infection and toxoplasmosis With the introduction of highly active anti-retroviral therapy (HAART) and immune reconstitution, ongoing prophylaxis for previously life-threatening conditions (eg P. jirovecii, cytomegalovirus, Toxoplasma and cryptococcal infections) can be discontinued in patients with CD4 counts that remain consistently above 200 and who have a low viral load Long-term therapy is recommended to prevent a recurrence in people diagnosed with salmonellosis. This is because despite HAART, patients infected with HIV appear to mount an ineffective antibody response to salmonella infection

Ciprofloxacin is usually the first choice

3589

Next Question https://mypastest.pastest.com/Secure/TestMe/Browser/429893#Top 1/2 8/9/2016 MyPastest

Previous Question Tag Question Feedback End Review

Difficulty: Difficult

Peer Responses

Session Progress

Responses Correct: 0

Responses Incorrect: 151

Responses Total: 151

Responses - % Correct: 0%

Blog (https://www.pastest.com/blog) About Pastest (https://www.pastest.com/about-us) Contact Us (https://www.pastest.com/contact-us) Help (https://www.pastest.com/help)

© Pastest 2016

https://mypastest.pastest.com/Secure/TestMe/Browser/429893#Top 2/2 8/9/2016 MyPastest

Back to Filters (/Secure/TestMe/Filter/429893/QA) Question 16 of 151

A 19-year-old student presents to his GP. For the past few days he has had a low-grade fever, sore throat, malaise and headache. Over the past 24–36 h he has experienced pain and swelling over his left parotid. He presents today as the right parotid has now become tender. He is having difficulty opening his jaw, and speaking in particular. Which one of the following options is the most likely diagnosis in this case?

A Paramyxovirus infection

B Bilateral parotid salivary stone formation

C Cytomegalovirus infection

D Influenza A

E Sjögren syndrome

Explanation

Mumps Epidemiology

Outbreaks of paramyxovirus infection (mumps in this case) are occasionally seen among students In fact, there has been a recent increase in the number of cases of mumps cases in the late teenage years, as a result of individuals who may have missed the mumps vaccination before the introduction of MMR The number of cases seen in this age group may increase further if MMR (measles, mumps, rubella) vaccination continues to fall out of favour

Transmission of infection

Mumps is spread via respiratory contact and patients are contagious from 48 h before, to 9 days after parotid swelling.

Clinical presentation https://mypastest.pastest.com/Secure/TestMe/Browser/429893 1/3 8/9/2016 MyPastest

There is a prodromal period characterised by low-grade fever, malaise, anorexia and headache This is followed by parotid swelling and tenderness, which is usually the first sign of overt infection It is unilateral in 25% of cases

Complications

Central nervous system (CNS) involvement may lead to meningitis in 1–10% of cases Encephalitis is very rare, but may be seen early, or late in the second week after the onset of parotitis as a postinfectious demyelinating process Other CNS complications include cerebellar ataxia, transverse myelitis and Guillain– Barré syndrome Epididymo-orchitis occurs in 30–38% of postpubertal males who suffer mumps Oophoritis may occur in 5% of postpubertal women suffering from the condition Deafness is usually transient, but unilateral permanent high-tone hearing loss may occur in around 1 in 20,000 cases

Treatment

Treatment is generally of a supportive nature with fluids and analgesics, and the majority of patients recover with no long-term consequences

2647

Next Question

Previous Question Tag Question Feedback End Review

Difficulty: Easy

Peer Responses

Session Progress

Responses Correct: 0

Responses Incorrect: 151

Responses Total: 151 https://mypastest.pastest.com/Secure/TestMe/Browser/429893 2/3 8/9/2016 MyPastest

Responses - % Correct: 0%

Blog (https://www.pastest.com/blog) About Pastest (https://www.pastest.com/about-us) Contact Us (https://www.pastest.com/contact-us) Help (https://www.pastest.com/help) © Pastest 2016

https://mypastest.pastest.com/Secure/TestMe/Browser/429893 3/3 8/9/2016 MyPastest

Back to Filters (/Secure/TestMe/Filter/429893/QA) Question 17 of 151

A 30-year-old from the New Forest, southern England, develops a febrile illness and 1 month later Lyme serology, in keeping with a recent infection, is positive. Which one of the following is the rarest finding for this infection?

A Arthritis

B Carditis

C Erythema chronicum migrans

D Radiculoneuritis

E Transmission by the Ixodes tick

Explanation

Lyme disease Causative organism

Lyme disease is an infection caused by the spirochaete Borrelia burgdorferi, which is transmitted by contact with ticks of the genus Ixodes

Clinical features

Erythema chronicum migrans is usually the first symptom along with malaise, arthralgia and lymphadenopathy Up to 60% of patients develop chronic oligoarthritis Radiculoneuritis is another feature in around 15%, often involving the cranial nerves

Complications

Lymphocytic meningitis is a recognised complication Carditis is rare (< 10%) especially with European strains of the disease compared with those found in North America Heart block and arrhythmias may occur https://mypastest.pastest.com/Secure/TestMe/Browser/429893#Top 1/2 8/9/2016 MyPastest

776

Next Question

Previous Question Tag Question Feedback End Review

Difficulty: Difficult

Peer Responses

Session Progress

Responses Correct: 0

Responses Incorrect: 151

Responses Total: 151

Responses - % Correct: 0%

Blog (https://www.pastest.com/blog) About Pastest (https://www.pastest.com/about-us) Contact Us (https://www.pastest.com/contact-us) Help (https://www.pastest.com/help)

© Pastest 2016

https://mypastest.pastest.com/Secure/TestMe/Browser/429893#Top 2/2 8/9/2016 MyPastest

Back to Filters (/Secure/TestMe/Filter/429893/QA) Question 18 of 151

A 34-year-old man with AIDS on a combination anti-retroviral therapy was brought into casualty because he had a fit lasting approximately 15 minutes at home. The day before he complained of headaches and fever. On examination he is confused but has no neurological symptoms. A CT scan of his brain shows ring-enhancing masses with surrounding oedema. Given the likely diagnosis, which one of the following options is the most appropriate treatment?

A Erythromycin

B Ampicillin

C Aspirin and clopidogrel

D Sulfadiazine and pyrimethamine

E Fluconazole

Explanation

Cerebral toxoplasmosis Risk factors

Cerebral infection with the intracellular protozoan Toxoplasma gondii is the most frequent infection of the central nervous system in patients with AIDS when their CD4 lymphocyte count is below 200/mm3

Treatment

The condition responds well if treatment is started early; a combination of sulfadiazine at 4–6 g/day and pyrimethamine at 50–75 mg/day is the treatment of choice More than 40% of patients experience adverse effects, especially rash and nephrotoxicity caused by sulfadiazine The haematological toxicity of pyrimethamine may be reduced by adding folinic acid (10 mg/day)

2701 https://mypastest.pastest.com/Secure/TestMe/Browser/429893#Top 1/2 8/9/2016 MyPastest 2701

Next Question

Previous Question Tag Question Feedback End Review

Difficulty: Easy

Peer Responses

Session Progress

Responses Correct: 0

Responses Incorrect: 151

Responses Total: 151

Responses - % Correct: 0%

Blog (https://www.pastest.com/blog) About Pastest (https://www.pastest.com/about-us) Contact Us (https://www.pastest.com/contact-us) Help (https://www.pastest.com/help)

© Pastest 2016

https://mypastest.pastest.com/Secure/TestMe/Browser/429893#Top 2/2 8/9/2016 MyPastest

Back to Filters (/Secure/TestMe/Filter/429893/QA) Question 19 of 151

An 18-year-old student presents to his GP with a 1-day history of rash, which has followed a 3- day history of cold-like symptoms and conjunctivitis. The rash began as a maculopapular eruption in the postauricular region, but has rapidly spread to his face and upper body. On examination white papules are visible inside his mouth. Which one of the following diagnoses fits best with this clinical picture?

A Scarlet fever

B Rubella

C Measles

D Enterovirus infection

E Adenovirus infection

Explanation

Measles Epidemiology

Measles occasionally occurs in outbreaks in young adults who have not been vaccinated There is an incubation period of around 14 days in children, although this may extend to up to 21 days in adults

Clinical presentation

A prodrome of cold-like symptoms and conjunctivitis precedes the development of a rash by between 2 and 4 days The rash begins as a maculopapular outbreak behind the ears, rapidly spreading across the upper body and down to the buttocks over 2–3 days After 3 days the rash commonly fades in the order of its appearance, becoming copper brown and then desquamating Koplik’s spots are diagnostic of measles and occur as white papules with an https://mypastest.pastest.com/Secure/TestMe/Browser/429893#Top 1/3 8/9/2016 MyPastest erythematous base, appearing on the buccal mucosa and lips

Investigations

Blood testing often reveals a leukopenia ELISA (enzyme-linked immunosorbent assay) testing may be useful for detecting measles antibodies, these appear shortly after the development of the rash and peak 3– 4 weeks later

Complications

Complications of measles are rare, but include otitis media, laryngitis, pneumonia, encephalitis (a chronic process resulting in the death of up to 15% of those with encephalitis), myocarditis, pericarditis and hepatitis; and conjunctival suffusion may occur

Management

Supportive therapy is often all that is required, although mortality is appreciably higher in the immunocompromised and patients from the developing world

2648

Next Question

Previous Question Tag Question Feedback End Review

Difficulty: Easy

Peer Responses

Session Progress

Responses Correct: 0

Responses Incorrect: 151

Responses Total: 151

Responses - % Correct: 0%

https://mypastest.pastest.com/Secure/TestMe/Browser/429893#Top 2/3 8/9/2016 MyPastest

Blog (https://www.pastest.com/blog) About Pastest (https://www.pastest.com/about-us) Contact Us (https://www.pastest.com/contact-us) Help (https://www.pastest.com/help) © Pastest 2016

https://mypastest.pastest.com/Secure/TestMe/Browser/429893#Top 3/3 8/9/2016 MyPastest

Back to Filters (/Secure/TestMe/Filter/429893/QA) Question 20 of 151

A 30-year-old Indian woman presents with a widespread nodular rash, loss of eyebrows and burns on her hands. Her ulnar nerves are thickened and exquisitely tender. Skin biopsy shows the presence of numerous acid-fast bacilli (AFBs). Which one of the following options is the most likely diagnosis?

A Tuberculosis

B Leprosy

C Scleroderma

D Motor neurone disease

E HIV

Explanation

Leprosy

The woman in the present case has leprosy towards the lepromatous end of the spectrum (high bacterial load, low cell-mediated immunity) The nodular rash is typical of lepromatous leprosy, and there is generally less nerve involvement at the lepromatous end of the spectrum

Treatment

Initial treatment for multi-bacillary leprosy should, however, be rifampicin, dapsone and clofazimine Clofazimine is not needed in paucibacillary leprosy

3701

Next Question

https://mypastest.pastest.com/Secure/TestMe/Browser/429893#Top 1/2 8/9/2016 MyPastest Previous Question Tag Question Feedback End Review

Difficulty: Easy

Peer Responses

Session Progress

Responses Correct: 0

Responses Incorrect: 151

Responses Total: 151

Responses - % Correct: 0%

Blog (https://www.pastest.com/blog) About Pastest (https://www.pastest.com/about-us) Contact Us (https://www.pastest.com/contact-us) Help (https://www.pastest.com/help) © Pastest 2016

https://mypastest.pastest.com/Secure/TestMe/Browser/429893#Top 2/2 8/9/2016 MyPastest

Back to Filters (/Secure/TestMe/Filter/429893/QA) Question 21 of 151

A young man presents with an ulcer over his forehead 3 months after returning from a holiday in Israel. On examination there is a single ulcerating lesion 2 cm in diameter over his forehead. Which infectious aetiology is most likely?

A HSV

B Leishmania donovani

C Leishmania tropica

D Molluscum contagiosum

E Mycobacterium chelonae

Explanation

Leishmaniasis Causative organisms

Leishmania tropica, found in the Middle East, is one of several Leishmania species that causes cutaneous leishmaniasis It is an important cause of ulcerative skin lesions occurring over skin exposed to the bite of phlebotamine sandflies

Differential diagnosis of ulcers

Leishmania donovani causes visceral leishmaniasis Mycobacterium chelonae is a fast-growing environmental ‘atypical’ mycobacterium that rarely causes ulceration in immunocompetent adults Herpes simplex virus (HSV) and Molluscum contagiosum usually cause lesions in the orogenital region The lesion should be biopsied, not only to perform polymerase chain reaction (PCR) testing for Leishmania spp, but also to exclude other important causes of ulceration, such as squamous-cell carcinoma

780 https://mypastest.pastest.com/Secure/TestMe/Browser/429893#Top 1/2 8/9/2016 MyPastest 780

Next Question

Previous Question Tag Question Feedback End Review

Difficulty: Difficult

Peer Responses

Session Progress

Responses Correct: 0

Responses Incorrect: 151

Responses Total: 151

Responses - % Correct: 0%

Blog (https://www.pastest.com/blog) About Pastest (https://www.pastest.com/about-us) Contact Us (https://www.pastest.com/contact-us) Help (https://www.pastest.com/help) © Pastest 2016

https://mypastest.pastest.com/Secure/TestMe/Browser/429893#Top 2/2 8/9/2016 MyPastest

Back to Filters (/Secure/TestMe/Filter/429893/QA) Question 22 of 151

A 7-year-old boy who has come from India presented with mild fever (37.5 °C), maculopapular rash and pharyngitis. He has had difficulty swallowing, even his own saliva. On examination there is a greyish membrane surrounding the tonsils. There is also regional lymphadenopathy. Which one of the following options is the most likely diagnosis?

A Infectious mononucleosis

B Diphtheria

C Rubella

D Measles

E Streptococcus pyogenes

Explanation

Diphtheria Epidemiology

In countries with active programmes of immunisation, diphtheria has virtually disappeared, but cases still occur in the developing world

Clinical presentation

The most common presentation is with pharyngitis, but any part of the respiratory tract may be involved Areas of grey or white fibrinous exudates coalesce to produce a tough grey–yellow membrane that bleeds when removed Cutaneous diphtheria may occur; eventually the maculopapular rash begins to ulcerate, with characteristic grey–white exudates at the base of the ulcers The main clinical clue is presence of the characteristic pseudomembrane; dysphagia and relative lack of fever are other signs that help to distinguish this from streptococcal or Epstein–Barr virus infection

https://mypastest.pastest.com/Secure/TestMe/Browser/429893#Top 1/2 8/9/2016 MyPastest Treatment and side-effects

Treatment is with diphtheria antitoxin and benzylpenicillin iv for 1 week Administration of antitoxin has been associated with both the development of anaphylaxis and serum sickness after 2–3 weeks

5081

Next Question

Previous Question Tag Question Feedback End Review

Difficulty: Average

Peer Responses

Session Progress

Responses Correct: 0

Responses Incorrect: 151

Responses Total: 151

Responses - % Correct: 0%

Blog (https://www.pastest.com/blog) About Pastest (https://www.pastest.com/about-us) Contact Us (https://www.pastest.com/contact-us) Help (https://www.pastest.com/help)

© Pastest 2016

https://mypastest.pastest.com/Secure/TestMe/Browser/429893#Top 2/2 8/9/2016 MyPastest

Back to Filters (/Secure/TestMe/Filter/429893/QA) Question 23 of 151

An HIV-positive patient whose CD4 count is heading down towards 250/mm3 is being started on zidovudine. Which one of the following options is the most severe side-effect to watch out for?

A Lactic acidosis

B Renal colic

C Nightmares

D Hyperlipidaemia

E Peripheral neuropathy

Explanation

Side-effects of anti-retroviral agents

Side-effects of anti-retroviral agents are now a major concern As a broad rule the nucleoside reverse transcriptase inhibitors (NRTIs) cause mitochondrial DNA damage and lactic acidosis and peripheral neuropathy, apart from lamivudine, which is relatively free of side-effects The protease inhibitors interact with part of the cytochrome P450 system and therefore have major drug interactions

They also cause metabolic and lipid changes

Lipodystrophy is a major concern to patients

It is not yet clear what the main cause of this is, but stavudine is implicated

3685

Next Question

https://mypastest.pastest.com/Secure/TestMe/Browser/429893#Top 1/2 8/9/2016 MyPastest Previous Question Tag Question Feedback End Review

Difficulty: Average

Peer Responses

Session Progress

Responses Correct: 0

Responses Incorrect: 151

Responses Total: 151

Responses - % Correct: 0%

Blog (https://www.pastest.com/blog) About Pastest (https://www.pastest.com/about-us) Contact Us (https://www.pastest.com/contact-us) Help (https://www.pastest.com/help) © Pastest 2016

https://mypastest.pastest.com/Secure/TestMe/Browser/429893#Top 2/2 8/9/2016 MyPastest

Back to Filters (/Secure/TestMe/Filter/429893/QA) Question 24 of 151

A 26-year-old traveller has just returned from South America. He notices several erythematous nodules all over his body. Some have a golden crust. What is the mode of transmission?

A Mosquitoes

B Sandfly

C Food

D Sexually

E Dogs

Explanation

Leishmaniasis Causative organisms and transmission of infection

Leishmaniasis is caused by parasites of the genus Leishmania The parasites are transmitted by phlebotomine sandflies The infection may be anthroponotic or zoonotic

Disease sites

In humans, the disease is usually either cutaneous or visceral The most important variant is mucosal leishmaniasis of South and Central America

Epidemiology

In certain places the disease is common and important, but there are few accurate statistics The World Health Organization estimates 500 000 cases of visceral leishmaniasis and 1.5–2 million cases of cutaneous leishmaniasis annually, with 200 million people at risk of each disease https://mypastest.pastest.com/Secure/TestMe/Browser/429893#Top 1/2 8/9/2016 MyPastest

3713

Next Question

Previous Question Tag Question Feedback End Review

Difficulty: Easy

Peer Responses

Session Progress

Responses Correct: 0

Responses Incorrect: 151

Responses Total: 151

Responses - % Correct: 0%

Blog (https://www.pastest.com/blog) About Pastest (https://www.pastest.com/about-us) Contact Us (https://www.pastest.com/contact-us) Help (https://www.pastest.com/help)

© Pastest 2016

https://mypastest.pastest.com/Secure/TestMe/Browser/429893#Top 2/2 8/9/2016 MyPastest

Back to Filters (/Secure/TestMe/Filter/429893/QA) Question 25 of 151

After initial infection with HIV what is the median time for untreated patients to develop AIDs?

A 6 months

B 1 year

C 5 years

D 10 years

E 15 years

Explanation

Human immunodeficiency virus (HIV) infection The latency period

Following the acute syndrome or subclinical seroconversion, there usually follows an asymptomatic period, which without anti-retroviral therapy lasts an average of 10 years Although a time of clinical latency, there is intense viral turnover: 10910 –10 viral particles are replaced daily and the half-life of circulating CD4 lymphocytes is substantially reduced During the asymptomatic period, physical examination may be normal, but about one- third of patients have persistent generalised lymphadenopathy The enlarged nodes, caused by a non-specific follicular hyperplasia, are usually symmetrical, mobile and non-tender The cervical and axillary nodes are most commonly affected Markedly asymmetrical, painful or rapidly enlarging nodes should be biopsied to exclude tumours such as lymphoma and opportunistic infections such as tuberculosis

Management

Symptoms of progressive HIV infection can be prevented by highly active anti-retroviral https://mypastest.pastest.com/Secure/TestMe/Browser/429893#Top 1/3 8/9/2016 MyPastest treatment (HAART)

Complications

In the absence of treatment, patients often develop minor opportunistic conditions affecting the skin and mucous membranes; these are also common throughout the later stages of HIV disease They include a range of infections:

fungal (eg tinea, Pityrosporum spp) viral (eg warts, molluscum contagiosum herpes simplex, herpes zoster) bacterial (eg folliculitis, impetigo) eczema, seborrhoeic dermatitis and psoriasis

1288

Next Question

Previous Question Tag Question Feedback End Review

Difficulty: Difficult

Peer Responses

Session Progress

Responses Correct: 0

Responses Incorrect: 151

Responses Total: 151

Responses - % Correct: 0%

Blog (https://www.pastest.com/blog) About Pastest (https://www.pastest.com/about-us) Contact Us (https://www.pastest.com/contact-us) Help (https://www.pastest.com/help) © Pastest 2016 https://mypastest.pastest.com/Secure/TestMe/Browser/429893#Top 2/3 8/9/2016 MyPastest

https://mypastest.pastest.com/Secure/TestMe/Browser/429893#Top 3/3 8/9/2016 MyPastest

Back to Filters (/Secure/TestMe/Filter/429893/QA) Question 26 of 151

A 29-year-old woman returns from a trip to the jungles of northern Thailand with body ache, severe myalgia of her back, arms and legs and a maculopapular rash which began on her limbs and has now spread to involve the trunk. She had fevers of up to 40 °C for the first few days, but these have now subsided. Malaria films are negative. Which one of the following diagnoses fits best with this clinical picture?

A Dengue fever

B Malaria

C Hepatitis A

D Influenza

E Yellow fever

Explanation

Dengue fever Epidemiology

Dengue is the commonest arthropod borne viral infection in humans 50–100 million cases occur every year in the tropics Around 10 000 deaths occur per year from dengue haemorrhagic fever

Causative organism and transmission

It is caused by a flavivirus, and is reported mainly in Asia, South America and Africa Although initial epidemics were located in urban areas, increased dengue spread has involved suburban and rural locales The disease is transmitted by the daytime feeding mosquito Aedes aegypti Humans are also infective during the first 3 days of the illness

Clinical course

https://mypastest.pastest.com/Secure/TestMe/Browser/429893#Top 1/3 8/9/2016 MyPastest

Classic dengue fever is characterised by abrupt onset of fever, malaise, headache, facial flushing, severe muscular backache and conjunctival suffusion Lymphadenopathy, petechiae on the soft palate and a morbilliform rash which begins on the limbs and later spreads to the trunk may also occur Cough is uncommon The fever subsides after 3–4 days and patients then have a normal or even subnormal temperature

Dengue haemorrhagic fever

Dengue haemorrhagic fever is a much more severe form of the disease, thought to be due to sequential infection with different dengue sub-types

Diagnosis

Diagnosis is usually via complement techniques or enzyme-linked immunosorbent assay (ELISA), and treatment is supportive

5234

Next Question

Previous Question Tag Question Feedback End Review

Difficulty: Average

Peer Responses

Session Progress

Responses Correct: 0

Responses Incorrect: 151

Responses Total: 151

Responses - % Correct: 0%

https://mypastest.pastest.com/Secure/TestMe/Browser/429893#Top 2/3 8/9/2016 MyPastest Blog (https://www.pastest.com/blog) About Pastest (https://www.pastest.com/about-us) Contact Us (https://www.pastest.com/contact-us) Help (https://www.pastest.com/help) © Pastest 2016

https://mypastest.pastest.com/Secure/TestMe/Browser/429893#Top 3/3 8/9/2016 MyPastest

Back to Filters (/Secure/TestMe/Filter/429893/QA) Question 27 of 151

A woman recently diagnosed with acute myeloid leukaemia presents with a rash 3 weeks after completing a first course of chemotherapy. She is febrile and leucopenic. A maculopapular rash is noted over much of her trunk, several vesicles are noted. Previous serology to VZV is positive for IgG. Which one of the following options is the most likely diagnosis?

A Chickenpox

B HSV infection

C Multi-dermatomal shingles

D Orf

E Parvovirus infection

Explanation

Shingles

Immunosuppression is a common cause of atypical presentations of shingles, which may be disseminated and multi-dermatomal

Differential diagnosis

Chickenpox may also present as a severe infection in these patients, but serology to varicella zoster virus (VZV) IgG would be negative since it is due to primary VZV infection Orf is a poxvirus causing a pustular dermatitis usually affecting the hands: it is acquired from contact with sheep Parvoviruses can lead to chronic infection in immunosuppressed people with anaemia; arthropathy is a feature

785

https://mypastest.pastest.com/Secure/TestMe/Browser/429893#Top 1/2 8/9/2016 MyPastest Next Question

Previous Question Tag Question Feedback End Review

Difficulty: Average

Peer Responses

Session Progress

Responses Correct: 0

Responses Incorrect: 151

Responses Total: 151

Responses - % Correct: 0%

Blog (https://www.pastest.com/blog) About Pastest (https://www.pastest.com/about-us) Contact Us (https://www.pastest.com/contact-us) Help (https://www.pastest.com/help)

© Pastest 2016

https://mypastest.pastest.com/Secure/TestMe/Browser/429893#Top 2/2 8/9/2016 MyPastest

Back to Filters (/Secure/TestMe/Filter/429893/QA) Question 28 of 151

Which one of the following is the most appropriate management step in a ward outbreak of MRSA?

A Close ward until infection clear

B Treat all positive members of ward with iv antibiotics

C Improve hand washing hygiene among staff

D Exclude staff with positive MRSA tests

E Clean rooms and walls with alcohol

Explanation

Meticillin-resistant Staphylococcus aureus (MRSA) Significance

MRSA is a major cause of hospital-acquired infection It may be carried asymptomatically, (nasal, perineum) and may cause pneumonia, wound infections and septicaemia Ward outbreaks are common and it is important to consult the local policy for management

Infection control and management

Eradication is with mupirocin nasally and triclosan washes Intravenous treatment is only required if MRSA is causing an intercurrent illness The most important measure is to ensure adequate hand washing and washing of equipment, eg stethoscopes, which are likely to spread infection Staff and visitors should also wear a gown and gloves when treating MRSA-positive patients

13856

https://mypastest.pastest.com/Secure/TestMe/Browser/429893#Top 1/2 8/9/2016 MyPastest Next Question

Previous Question Tag Question Feedback End Review

Difficulty: Easy

Peer Responses

Session Progress

Responses Correct: 0

Responses Incorrect: 151

Responses Total: 151

Responses - % Correct: 0%

Blog (https://www.pastest.com/blog) About Pastest (https://www.pastest.com/about-us) Contact Us (https://www.pastest.com/contact-us) Help (https://www.pastest.com/help)

© Pastest 2016

https://mypastest.pastest.com/Secure/TestMe/Browser/429893#Top 2/2 8/9/2016 MyPastest

Back to Filters (/Secure/TestMe/Filter/429893/QA) Question 29 of 151

You review a 48-year-old man presenting with a bacterial infection. Which one of the following micro-organisms is generally sensitive to penicillin?

A Bordetella pertussis

B Cryptococcus neoformans

C Mycoplasma pneumoniae

D Legionella pneumophila

E Streptococcus pneumoniae

Explanation

Antibiotic sensitivity

Erythromycin is the antibiotic of choice for whooping cough (Bordetella pertussis), although treatment after the catarrhal stage with antibiotics is ineffective Amphotericin B is the treatment of choice for Cryptococcus neoformans Clarithromycin is the usual therapy for mycoplasmosis and for legionellosis In patients severely ill with legionnaire’s disease, rifampicin may be added Penicillins are the treatment of choice for streptococcal infections

3904

Next Question

Previous Question Tag Question Feedback End Review

Difficulty: Easy

Peer Responses

https://mypastest.pastest.com/Secure/TestMe/Browser/429893#Top 1/2 8/9/2016 MyPastest

Session Progress

Responses Correct: 0

Responses Incorrect: 151

Responses Total: 151

Responses - % Correct: 0%

Blog (https://www.pastest.com/blog) About Pastest (https://www.pastest.com/about-us) Contact Us (https://www.pastest.com/contact-us) Help (https://www.pastest.com/help)

© Pastest 2016

https://mypastest.pastest.com/Secure/TestMe/Browser/429893#Top 2/2 8/9/2016 MyPastest

Back to Filters (/Secure/TestMe/Filter/429893/QA) Question 30 of 151

Infection with which one virus is the most frequent cause of blindness in patients with AIDS?

A Herpes simplex virus

B Varicella zoster virus

C Epstein–Barr virus

D Cytomegalovirus

E Papillomavirus

Explanation

CMV retinitis in AIDS Prevalence

Without anti-retroviral therapy, up to 30% of patients with acquired immune deficiency syndrome (AIDS) (and a CD4 lymphocyte count below 50/mm3 ) develop reactivation of cytomegalovirus (CMV) in the form of a destructive and blinding retinitis This is rare in other types of immunosuppression

Clinical presentation

Patients usually present with blurring of vision, scotomas, floaters or flashing lights The characteristic retinal changes are patches of irregular retinal pallor, caused by oedema and necrosis, and haemorrhages in a perivascular distribution The retinitis usually starts peripherally and progresses rapidly to involve the macula and whole retina, leading to blindness

Complications

Complications include retinal detachment, branch retinal artery occlusion, persistent iritis and cataract.

https://mypastest.pastest.com/Secure/TestMe/Browser/429893#Top 1/2 8/9/2016 MyPastest Treatment

First-line treatment is with intravenous ganciclovir, which may cause severe neutropenia and thrombocytopenia that are dose-limiting in about 10% of patients Foscarnet (phosphonoformate) is a relatively toxic second-line agent that causes dose- limiting reversible renal impairment and symptoms of hypocalcaemia in about 20% of patients

1326

Next Question

Previous Question Tag Question Feedback End Review

Difficulty: Average

Peer Responses

Session Progress

Responses Correct: 0

Responses Incorrect: 151

Responses Total: 151

Responses - % Correct: 0%

Blog (https://www.pastest.com/blog) About Pastest (https://www.pastest.com/about-us) Contact Us (https://www.pastest.com/contact-us) Help (https://www.pastest.com/help)

© Pastest 2016

https://mypastest.pastest.com/Secure/TestMe/Browser/429893#Top 2/2 8/9/2016 MyPastest

Back to Filters (/Secure/TestMe/Filter/429893/QA) Question 31 of 151

A woman attends clinic after passing a ‘worm’ in her stool. After examination of the stool a diagnosis of Taenia saginata infection is made. Which one of the following statements is true?

A Humans are a dead-end host

B Neurocysticercosis is a complication

C Proglottids are helpful diagnostically

D Resistance to niclosamide

E The source of the infection was infected pork

Explanation

Taenia saginata infection Life-cycle

Taenia saginata is the beef tapeworm acquired by eating cysts contained in infected beef These develop into adult worms in the intestine and may reach several metres in length often with few symptoms The tapeworm is made up of many repeating segments, each termed a ‘proglottid’ The proglottids are shed in the faeces and their characteristics are used to identify the species of cestode worm Adult worms are hermaphrodites and shed eggs into the faeces, which complete the life-cycle when they are ingested by cattle

Differential diagnosis

T. solium, the pork tapeworm, can lead to cysticercosis when eggs are ingested and the larval form disseminates to tissues Humans are dead-end hosts for cysticercosis

793 https://mypastest.pastest.com/Secure/TestMe/Browser/429893#Top 1/2 8/9/2016 MyPastest

Next Question

Previous Question Tag Question Feedback End Review

Difficulty: Difficult

Peer Responses

Session Progress

Responses Correct: 0

Responses Incorrect: 151

Responses Total: 151

Responses - % Correct: 0%

Blog (https://www.pastest.com/blog) About Pastest (https://www.pastest.com/about-us) Contact Us (https://www.pastest.com/contact-us) Help (https://www.pastest.com/help)

© Pastest 2016

https://mypastest.pastest.com/Secure/TestMe/Browser/429893#Top 2/2 8/9/2016 MyPastest

Back to Filters (/Secure/TestMe/Filter/429893/QA) Question 32 of 151

A man recently returned from India, presents with a 10-day history of a fever, cough, constipation and headache. His blood culture grows a Gram-negative rod. Which one of the following options is the most likely diagnosis?

A Typhoid

B Malaria

C Infectious mononucleosis

D Streptococcus pneumoniae infection

E Mumps

Explanation

Typhoid

The man in the present case has typhoid and is now in the second week of his illness ‘Typhoid’ (and typhus) is derived from ‘clouding’ and refers to the dissociated, absent mental state of these patients with advanced disease

Clinical and investigative features

Rose spots are seen in the second week An abnormal chest X-ray (CXR) is not uncommon Splenomegaly is usual

Complications

During the third week the Peyer’s patches ulcerate and can lead to bowel perforation

3699

Next Question

https://mypastest.pastest.com/Secure/TestMe/Browser/429893#Top 1/2 8/9/2016 MyPastest

Previous Question Tag Question Feedback End Review

Difficulty: Easy

Peer Responses

Session Progress

Responses Correct: 0

Responses Incorrect: 151

Responses Total: 151

Responses - % Correct: 0%

Blog (https://www.pastest.com/blog) About Pastest (https://www.pastest.com/about-us) Contact Us (https://www.pastest.com/contact-us) Help (https://www.pastest.com/help) © Pastest 2016

https://mypastest.pastest.com/Secure/TestMe/Browser/429893#Top 2/2 8/9/2016 MyPastest

Back to Filters (/Secure/TestMe/Filter/429893/QA) Question 33 of 151

A 19-year-old student comes to see the GP after developing some chickenpox spots. He has not been exposed as a child and has recently been taking prednisolone for an exacerbation of asthma. The steroids are stopped and he is discharged home, told to rest and take paracetamol; but 2 days later he returns with pyrexia, 38.2°C, and is markedly short of breath with a cough. Auscultation of the chest reveals bronchial breathing.

Investigations:

Hb 12.4 g/dl

WCC 11.2 × 109 /l

PLT 240 × 109 /l

Na+ 139 mmol/l

K+ 5.0 mmol/l

Creatinine 120 μmol/l

p (O2 ) 7.6 kPa

p (CO2 ) 3.2 kPa

Which one of the following options is the most appropriate therapy for his infection?

A Oral flucloxacillin

B IV aciclovir

C PO aciclovir

D po steroids

E Co-amoxiclav

Explanation

Chickenpox pneumonitis https://mypastest.pastest.com/Secure/TestMe/Browser/429893#Top 1/3 8/9/2016 MyPastest Diagnosis

The patient in the present case has chickenpox pneumonitis, a serious complication of varicella zoster virus (VZV) infection He has not been exposed as a child, the student environment increases exposure to VZV, and it is known that corticosteroids may increase the risk of infection

Management

The patient should be admitted, and managed on the high-dependency unit Treatment is with iv aciclovir, at a total dose of 1500mg/m2 /day for a period of 7–10 days Thankfully with appropriate management, whilst a serious complication of VZV, the pneumonitis is rarely fatal

20784

Next Question

Previous Question Tag Question Feedback End Review

Difficulty: Easy

Peer Responses

Session Progress

Responses Correct: 0

Responses Incorrect: 151

Responses Total: 151

Responses - % Correct: 0%

Blog (https://www.pastest.com/blog) About Pastest (https://www.pastest.com/about-us) Contact Us (https://www.pastest.com/contact-us) Help (https://www.pastest.com/help)

© Pastest 2016

https://mypastest.pastest.com/Secure/TestMe/Browser/429893#Top 2/3 8/9/2016 MyPastest

https://mypastest.pastest.com/Secure/TestMe/Browser/429893#Top 3/3 8/9/2016 MyPastest

Back to Filters (/Secure/TestMe/Filter/429893/QA) Question 34 of 151

A 27-year-old heroin addict presents for review. He has lost weight over the past few months and complains of severe lethargy. Sputum is sent for review. Which one of the following organisms in sputum may be associated with HIV infection?

A Cryptococcus

B Toxoplasma

C Cryptosporidium

D Microsporidium

E Isospora

Explanation

Cryptococcosis - Causative organism

Cryptococcus is widely disseminated, particularly in bird muck, and is acquired via inhalation

Clinical presentation

The most common presentation of cryptococcosis is in the context of meningitis, although pulmonary and disseminated infections may occur Onset may be insidious, associated with non-specific fevers

Investigations

Organisms can be seen with Indian ink staining Where there is cryptococcal meningitis, cerebrospinal fluid (CSF) can be tested for the cryptococcal antigen titre

Treatment and prognosis https://mypastest.pastest.com/Secure/TestMe/Browser/429893#Top 1/2 8/9/2016 MyPastest

Treatment is with intravenous amphotericin B or fluconazole Where presentation is with cryptococcal meningitis, mortality may be up to 20%

5236

Next Question

Previous Question Tag Question Feedback End Review

Difficulty: Average

Peer Responses

Session Progress

Responses Correct: 0

Responses Incorrect: 151

Responses Total: 151

Responses - % Correct: 0%

Blog (https://www.pastest.com/blog) About Pastest (https://www.pastest.com/about-us) Contact Us (https://www.pastest.com/contact-us) Help (https://www.pastest.com/help) © Pastest 2016

https://mypastest.pastest.com/Secure/TestMe/Browser/429893#Top 2/2 8/9/2016 MyPastest

Back to Filters (/Secure/TestMe/Filter/429893/QA) Question 35 of 151

A 29-year-old Sri Lankan man is admitted to hospital with a rash and high fevers. He gives a long history of pains in his hands and feet. On examination, he has an erythematous rash over his nose and cheeks. ANA testing and dsDNA are strongly positive. As part of his screening for PUO, an HIV antibody test is carried out, which proves positive. His blood results are as follows:

Na, 136 mmol/l; K, 3.7 mmol/l; urea, 3.5 mmol/l; creatine, 67 U/l; LFTs, normal; Hb, 12.9 g/dl; WCC, 2.4 × 10996 /l (lymphocytes 0.8 × 10 /l); platelets, 200 × 10 /l; ANA, positive; dsDNA, positive; CD4, 80 cells/mm3; HIV antibody, positive; blood cultures, negative; syphilis serology, negative. Which one of the following options is the most appropriate treatment for his rash and arthropathy?

A Combivir, efavirenz, co-trimoxazole

B Prednisolone

C Combivir, efavirenz, co-trimoxazole, prednisolone

D Prednisolone, co-trimoxazole

E Combivir, efavirenz

Explanation

Systemic lupus erythematosus

The patient in the present case has acute systemic lupus erythematosus (SLE), and needs to be treated accordingly.The co-existence of HIV infection and SLE seems to be rare but a number of cases have been reported; the presence of overlapping clinical and haematological features can make diagnosis difficult. Interestingly some case reports have observed aggravation of SLE symptoms once HIV immunosuppression is reversed with treatment

Further investigations

https://mypastest.pastest.com/Secure/TestMe/Browser/429893#Top 1/2 8/9/2016 MyPastest

Since there is a chance that his HIV antibody test may be falsely positive, he needs confirmation of his HIV status with proviral DNA and an RNA viral load prior to starting anti-retroviral therapy. However the low CD4 count makes HIV immunosuppression very likely

Treatment considerations

In any case, he is at significant risk of Pneumocystis jirovecii (PCJ; formerly Pneumocystis carinii) pneumonia (PCP) and it would be prudent to put him on prophylaxis for this with co-trimoxazole, especially as steroids are likely to further increase the risk of PCP

589

Next Question

Previous Question Tag Question Feedback End Review

Difficulty: Difficult

Peer Responses

Session Progress

Responses Correct: 0

Responses Incorrect: 151

Responses Total: 151

Responses - % Correct: 0%

Blog (https://www.pastest.com/blog) About Pastest (https://www.pastest.com/about-us) Contact Us (https://www.pastest.com/contact-us) Help (https://www.pastest.com/help)

© Pastest 2016

https://mypastest.pastest.com/Secure/TestMe/Browser/429893#Top 2/2 8/9/2016 MyPastest

Back to Filters (/Secure/TestMe/Filter/429893/QA) Question 36 of 151

A 29-year-old patient with progressive HIV infection complains of a 5-day history of feeling unwell associated with diarrhoea, abdominal pain and vomiting. On examination he looks anaemic. He has elevated liver function tests and his CD4 lymphocyte count is 30/mm3 . What pathogen is the most likely cause of his infection?

A Pneumocystis jirovecii

B Mycobacterium avium complex

C Campylobacter

D Helicobacter

E Lactobacillus

Explanation

Bacterial infections in advanced human immunodeficiency virus infections Main causative organism

Patients with advanced HIV infection and CD4 lymphocyte counts below 50/mm3 are at high risk of disseminated Mycobacterium avium complex (MAC) infection, particularly in industrialised countries where it is reported to develop in up to 40% of patients with acquired immune deficiency syndrome (AIDS) Mycobacterium avium is a ubiquitous environmental organism of low pathogenicity that can be isolated from domestic water supplies

Transmission of infection and clinical features

Infection is likely to be through the gastrointestinal tract MAC infection becomes widely disseminated in those with advanced HIV and causes fever, night sweats, weight loss, diarrhoea, abdominal pain, anaemia, disturbed liver function and reduced overall survival

2698 https://mypastest.pastest.com/Secure/TestMe/Browser/429893#Top 1/2 8/9/2016 MyPastest

Next Question

Previous Question Tag Question Feedback End Review

Difficulty: Average

Peer Responses

Session Progress

Responses Correct: 0

Responses Incorrect: 151

Responses Total: 151

Responses - % Correct: 0%

Blog (https://www.pastest.com/blog) About Pastest (https://www.pastest.com/about-us) Contact Us (https://www.pastest.com/contact-us) Help (https://www.pastest.com/help) © Pastest 2016

https://mypastest.pastest.com/Secure/TestMe/Browser/429893#Top 2/2 8/9/2016 MyPastest

Back to Filters (/Secure/TestMe/Filter/429893/QA) Question 37 of 151

A 23-year-old man presents to his GP complaining of fevers, headache, malaise and muscle pain. Shortly after his return to the UK last week from a walking trip in the United States, he says he used a cigarette to burn off a tick on his leg. On examination he has a rash on the palms and soles of his feet. Which one of the following diagnoses fits best with this clinical picture?

A Infectious mononucleosis

B Rocky Mountain spotted fever

C Reactive arthritis

D Influenza

E Typhoid fever

Explanation

Rocky Mountain spotted fever Definition and causative organism

Rocky Mountain spotted fever is a febrile illness caused by infection with Rickettsia rickettsii

Epidemiology

It is most prevalent in the Central, South and South-Eastern United States There is history of a tick bite in 65%, fever in 100% and a rash in 90% of cases

Clinical features

The incubation period is 3–12 days, and the first symptoms are fever, headache, malaise and myalgia Mild hepatitis, myocarditis, glomerulonephritis and encephalitis may also be associated

Diagnosis https://mypastest.pastest.com/Secure/TestMe/Browser/429893#Top 1/2 8/9/2016 MyPastest

Antibody titres to Rickettsia may provide the diagnosis, but they are not helpful in the care of the patient Diagnosis is confirmed with a fourfold rise in titres over a 2-week period

Treatment and prognosis

Treatment is with tetracycline-type antibiotics The mortality rate is between 1% and 4%, but rises appreciably if treatment is initiated after day 5 of the illness Hence, antibiotic treatment should be initiated on clinical suspicion

2656

Next Question

Previous Question Tag Question Feedback End Review

Difficulty: Easy

Peer Responses

Session Progress

Responses Correct: 0

Responses Incorrect: 151

Responses Total: 151

Responses - % Correct: 0%

Blog (https://www.pastest.com/blog) About Pastest (https://www.pastest.com/about-us) Contact Us (https://www.pastest.com/contact-us) Help (https://www.pastest.com/help) © Pastest 2016

https://mypastest.pastest.com/Secure/TestMe/Browser/429893#Top 2/2 8/9/2016 MyPastest

Back to Filters (/Secure/TestMe/Filter/429893/QA) Question 38 of 151

A 25-year-old homosexual man complains of a 9-day history of mucopurulent anal discharge, anal bleeding and pain while opening his bowels. Which one of the following options is the most appropriate next step in the diagnosis?

A Colonoscopy

B Erythrocyte sedimentation rate

C Stained specimen microscopy

D Specimen culture

E C-reactive protein

Explanation

Diagnosis of gonorrhoea

Microscopy of a suitably stained specimen is the first line in diagnosis The organisms must be Gram-negative, intracellular (within the cytoplasm of a leukocyte) and diplococci Constipation, a mucopurulent anal discharge, anal bleeding, perianal discomfort or pruritus ani, and, in severe cases, pain and tenesmus are symptoms of gonorrhoea in homosexual men

2687

Next Question

Previous Question Tag Question Feedback End Review

Difficulty: Average

Peer Responses https://mypastest.pastest.com/Secure/TestMe/Browser/429893#Top 1/2 8/9/2016 MyPastest

Session Progress

Responses Correct: 0

Responses Incorrect: 151

Responses Total: 151

Responses - % Correct: 0%

Blog (https://www.pastest.com/blog) About Pastest (https://www.pastest.com/about-us) Contact Us (https://www.pastest.com/contact-us) Help (https://www.pastest.com/help)

© Pastest 2016

https://mypastest.pastest.com/Secure/TestMe/Browser/429893#Top 2/2 8/9/2016 MyPastest

Back to Filters (/Secure/TestMe/Filter/429893/QA) Question 39 of 151

Which one of the following options is the most appropriate prophylaxis against bacterial endocarditis in patients with prosthetic heart valves undergoing dental procedures?

A Amoxicillin

B Cefaclor

C Gentamicin

D Nothing

E Cefuroxime

Explanation

Principles of antibiotic prophylaxis in bacterial endocarditis

Previous guidelines on prophylaxis for dental procedures used to recommend amoxicillin 3 g before the procedure A NICE evidence review published in 2008 suggested that there was no substantive evidence to support antibiotic prophylaxis, and as such it is no longer recommended

1291

Next Question

Previous Question Tag Question Feedback End Review

Difficulty: Average

Peer Responses

https://mypastest.pastest.com/Secure/TestMe/Browser/429893#Top 1/2 8/9/2016 MyPastest

Session Progress

Responses Correct: 0

Responses Incorrect: 151

Responses Total: 151

Responses - % Correct: 0%

Blog (https://www.pastest.com/blog) About Pastest (https://www.pastest.com/about-us) Contact Us (https://www.pastest.com/contact-us) Help (https://www.pastest.com/help)

© Pastest 2016

https://mypastest.pastest.com/Secure/TestMe/Browser/429893#Top 2/2 8/9/2016 MyPastest

Back to Filters (/Secure/TestMe/Filter/429893/QA) Question 40 of 151

An HIV-positive patient attends the Emergency Department with severe right-sided loin pain. An IVU reveals a calculus obstructing the right ureter. Which one of his medications is likely to have contributed to this?

A AZT

B Indinavir

C Co-trimoxazole

D Lamivudine

E Abacavir

Explanation

Side-effects of anti-retroviral medications

It is worthwhile learning some of the side-effects of anti-retroviral medications Indinavir leads to the formation of gelatinous renal stones, found to be composed of the precipitated drug The nucleoside reverse-transcriptase inhibitors (eg zidovudine (AZT), stavudine, abacavir, didanosine (DDI), but much less commonly lamivudine) are associated with lactic acidosis, which is thought to be related to mitochondrial dysfunction and peripheral neuropathy The non-nucleoside reverse transcriptase inhibitors (NNRTIs) may cause hepatic disturbance Efavirenz commonly causes nightmares and is associated with neuropsychiatric side- effects Nevirapine may cause a severe rash (as can abacavir) The protease inhibitors, and to a lesser extent NRTIs (especially stavudine), may be involved with fat redistribution and cholesterol and triglyceride elevation

604 https://mypastest.pastest.com/Secure/TestMe/Browser/429893#Top 1/2 8/9/2016 MyPastest

Next Question

Previous Question Tag Question Feedback End Review

Difficulty: Average

Peer Responses

Session Progress

Responses Correct: 0

Responses Incorrect: 151

Responses Total: 151

Responses - % Correct: 0%

Blog (https://www.pastest.com/blog) About Pastest (https://www.pastest.com/about-us) Contact Us (https://www.pastest.com/contact-us) Help (https://www.pastest.com/help)

© Pastest 2016

https://mypastest.pastest.com/Secure/TestMe/Browser/429893#Top 2/2 8/9/2016 MyPastest

Back to Filters (/Secure/TestMe/Filter/429893/QA) Question 41 of 151

A patient has been diagnosed with Chlamydia pneumonia. Which one of the following options is the most appropriate antibiotic therapy?

A Ampicillin

B Erythromycin

C Imipenem

D Cefuroxime

E Amikacin

Explanation

Chlamydia pneumonia

The diagnosis of chlamydia pneumonia is difficult The organism is cultivated like a virus using tissue cultures, but few laboratories offer this test Serology is difficult to interpret; the usual titres for IgM or serial changes with acute and convalescent sera are arbitrary Like Mycoplasma, this is an organism that is often suspected, infrequently proven, but easily treated empirically Macrolide antibiotics (erythromycin and clarithromycin) are the treatment of choice for chlamydia and other atypical pneumonias. Doxycycline is a reasonable alternative in this situation

1294

Next Question

Previous Question Tag Question Feedback End Review https://mypastest.pastest.com/Secure/TestMe/Browser/429893#Top 1/2 8/9/2016 MyPastest

Difficulty: Easy

Peer Responses

Session Progress

Responses Correct: 0

Responses Incorrect: 151

Responses Total: 151

Responses - % Correct: 0%

Blog (https://www.pastest.com/blog) About Pastest (https://www.pastest.com/about-us) Contact Us (https://www.pastest.com/contact-us) Help (https://www.pastest.com/help) © Pastest 2016

https://mypastest.pastest.com/Secure/TestMe/Browser/429893#Top 2/2 8/9/2016 MyPastest

Back to Filters (/Secure/TestMe/Filter/429893/QA) Question 42 of 151

A 59-year-old concert violinist is admitted to hospital with a 4-day history of confusion, severe headache and high fevers. He has not travelled abroad recently other than to visit France where he bought red wine and Brie cheese to take home. He is otherwise fit and well and is married with three children. On admission, his temperature is 39 °C and he is confused with a mini-mental test (MMT) of 6/10. There is neck stiffness and meningism. A CT brain is normal and a lumbar puncture reveals 400 white cells (mainly neutrophils). Blood cultures are reported as showing ‘diphtheroids, likely contaminant’. Which one of the following options is the most appropriate antibiotic regimen?

A Aciclovir, ceftriaxone, ampicillin

B Aciclovir, ceftriaxone, doxycycline

C Ceftriaxone, ampicillin, doxycycline

D Aciclovir, pleconaril, ceftriaxone

E Aciclovir, co-trimoxazole, ceftriaxone

Explanation

Listeria meningitis Diagnosis

The patient in the present case is quite likely to have Listeria meningitis The diphtheroids in his blood culture may actually be the Gram-positive rods of a Listeria sp

Treatment considerations

Ampicillin (or amoxicillin) should be given in high-dose iv as Listeria meningitis is resistant to cephalosporins The patient is confused and therefore viral meningoencephalitis with herpes simplex virus (HSV) or varicella zoster virus (VZV) should also be considered and treated pre- emptively with aciclovir 10–15 mg/kg three times per day https://mypastest.pastest.com/Secure/TestMe/Browser/429893#Top 1/2 8/9/2016 MyPastest Bacterial meningitis should be covered with ceftriaxone Pleconaril is a new treatment for enteroviral meningitis, although it is not yet licensed for use

611

Next Question

Previous Question Tag Question Feedback End Review

Difficulty: Difficult

Peer Responses

Session Progress

Responses Correct: 0

Responses Incorrect: 151

Responses Total: 151

Responses - % Correct: 0%

Blog (https://www.pastest.com/blog) About Pastest (https://www.pastest.com/about-us) Contact Us (https://www.pastest.com/contact-us) Help (https://www.pastest.com/help) © Pastest 2016

https://mypastest.pastest.com/Secure/TestMe/Browser/429893#Top 2/2 8/9/2016 MyPastest

Back to Filters (/Secure/TestMe/Filter/429893/QA) Question 43 of 151

A 42-year-old man who is known HIV positive presents with difficulty with short-term memory, generalised weakness and gait disturbance. He also has headaches and blurred vision. On examination he has 3/5 weakness of the left arm and 4/5 power weakness of the right leg.

Investigations:

Hb 11.5 g/dl

WCC 6.7 × 109 /l

CD4 82 cells/mm3 count

PLT 184 × 109 /l

Na+ 137 mmol/l

K+ 4.5 mmol/l

Creatinine 134 μmol/l

CSF Elevated protein

Multiple hyperintense white matter lesions seen on the T2-weighted scan, MRI brain predominantly in the frontal and parieto-occipital regions

Which one of the following options is the most likely diagnosis?

A Progressive multifocal leukoencephalopathy (PML)

B HIV encephalopathy

C CNS lymphoma

D Herpes simplex encephalitis

E Focal cerebritis

https://mypastest.pastest.com/Secure/TestMe/Browser/429893#Top 1/3 8/9/2016 MyPastest Explanation

Progressive multifocal leukoencephalopathy Definition

Progressive multifocal leukoencephalopathy occurs in human immunodeficiency virus (HIV)-positive patients with reduced CD4 count and is a progressive demyelinating disorder

Causative organism

It is thought to be caused by the JC virus which infects oligodendrocytes, the cells responsible for maintaining the myelin sheath

Histological features

Histological examination reveals multifocal demyelination, hyperchromatic enlarged oligodendrocytic nuclei and enlarged astrocytes

Prognosis

Regression of the condition has been described in association with highly active anti- retroviral therapy, but ultimately the condition is progressive in fatal

21093

Next Question

Previous Question Tag Question Feedback End Review

Difficulty: Easy

Peer Responses

Session Progress

Responses Correct: 0

Responses Incorrect: 151

Responses Total: 151

Responses - % Correct: 0% https://mypastest.pastest.com/Secure/TestMe/Browser/429893#Top 2/3 8/9/2016 MyPastest

Blog (https://www.pastest.com/blog) About Pastest (https://www.pastest.com/about-us) Contact Us (https://www.pastest.com/contact-us) Help (https://www.pastest.com/help)

© Pastest 2016

https://mypastest.pastest.com/Secure/TestMe/Browser/429893#Top 3/3 8/10/2016 MyPastest

Back to Filters (/Secure/TestMe/Filter/429893/QA) Question 44 of 151

A 60-year-old man attends the Emergency Department 3 weeks after returning from a holiday in India. Over the past 2 weeks he has had fever, sweats and weight loss. A chest X- ray is normal. He has raised inflammatory makers with a neutrophilia of 9.5 × 109 /l and deranged LFTs with an ALT 120 IU/l and bilirubin 39 μmol/l. A liver biopsy shows small, non- caseating granulomas. An organism is detected in blood cultures at day 5 after admission. Which one of the following options is the most likely finding on the stain?

A Acid–alcohol-fast bacilli on Ziehl–Neelsen stain

B Gram-negative bacillus

C Gram-negative coccus

D Gram-positive bacillus

E Indian ink stain-positive

Explanation

Brucellosis

The diagnosis in the present case is brucellosis

Causative organism

A small Gram-negative bacillus, members of the genus Brucella are zoonoses acquired by ingestion of unpasteurised milk or close contact with infected cattle The organisms are slow growing, as indicated here with cultures taking 5 days, which would be unusual for most bacterial organisms

Differential diagnosis

Tuberculosis is the key differential diagnosis, although caseating granulomas are typical and blood cultures are rarely positive and may take weeks to culture on specialised media

790 https://mypastest.pastest.com/Secure/TestMe/Browser/429893#Top 1/2 8/10/2016 MyPastest 790

Next Question

Previous Question Tag Question Feedback End Review

Difficulty: Difficult

Peer Responses

Session Progress

Responses Correct: 0

Responses Incorrect: 151

Responses Total: 151

Responses - % Correct: 0%

Blog (https://www.pastest.com/blog) About Pastest (https://www.pastest.com/about-us) Contact Us (https://www.pastest.com/contact-us) Help (https://www.pastest.com/help) © Pastest 2016

https://mypastest.pastest.com/Secure/TestMe/Browser/429893#Top 2/2 8/10/2016 MyPastest

Back to Filters (/Secure/TestMe/Filter/429893/QA) Question 45 of 151

The larvae of which one of the following helminth species undertakes a symptomatic travel through the lung before the adult worms reside in the intestine?

A Enterobius vermicularis

B Trichinella spiralis

C Ascaris lumbricoides

D Trichuris trichuria

E Taenia saginata

Explanation

Ascariasis - Causative organism

Ascariasis is an infection caused by Ascaris lumbricoides

Life-cycle

Normally, the adult worms are located in the small intestine However, in unusual circumstances, such as fever, irritation due to drugs, anaesthesia and bowel manipulation during surgery, the worms may migrate to ectopic sites where they may give rise to severe disease On ingestion, the infective larva hatches out in the small intestine and penetrates the intestinal wall to enter the portal circulation From the liver it is carried to the heart and via the pulmonary artery to the lungs In the lungs, it breaks out of the capillaries into the alveoli and undergoes another moult to become a fourth-stage larva From the lungs the larva moves up to the bronchi and then crawls over the epiglottis to enter the digestive tract In the intestine, it moults again to become a sexually mature worm https://mypastest.pastest.com/Secure/TestMe/Browser/429893#Top 1/2 8/10/2016 MyPastest The lifespan of an adult worm is approximately 1 year, after which it is spontaneously expelled In hyperendemic areas, children are being continuously infected so that as some worms are being expelled, others are maturing to take their place

1311

Next Question

Previous Question Tag Question Feedback End Review

Difficulty: Average

Peer Responses

Session Progress

Responses Correct: 0

Responses Incorrect: 151

Responses Total: 151

Responses - % Correct: 0%

Blog (https://www.pastest.com/blog) About Pastest (https://www.pastest.com/about-us) Contact Us (https://www.pastest.com/contact-us) Help (https://www.pastest.com/help)

© Pastest 2016

https://mypastest.pastest.com/Secure/TestMe/Browser/429893#Top 2/2 8/10/2016 MyPastest

Back to Filters (/Secure/TestMe/Filter/429893/QA) Question 46 of 151

Which one of the following antigens is involved in the entry of Plasmodium vivax into red cells?

A Anti-D

B Anti-S

C Duffy

D Kell

E Kidd

Explanation

Susceptibility to Plasmodium vivax infection

People of West African origin are strikingly resistant to Plasmodium vivax infection This correlates with the extreme rarity of the Duffy blood-group antigen alleles FYA and FYB encoding Fya and Fyb, which are receptors for penetration of the red cell by the merozoites

5520

Next Question

Previous Question Tag Question Feedback End Review

Difficulty: Average

Peer Responses

https://mypastest.pastest.com/Secure/TestMe/Browser/429893#Top 1/2 8/10/2016 MyPastest

Session Progress

Responses Correct: 0

Responses Incorrect: 151

Responses Total: 151

Responses - % Correct: 0%

Blog (https://www.pastest.com/blog) About Pastest (https://www.pastest.com/about-us) Contact Us (https://www.pastest.com/contact-us) Help (https://www.pastest.com/help)

© Pastest 2016

https://mypastest.pastest.com/Secure/TestMe/Browser/429893#Top 2/2 8/10/2016 MyPastest

Back to Filters (/Secure/TestMe/Filter/429893/QA) Question 47 of 151

An 18-year-old man presents to the Emergency Department with a history of painful urethral discharge and dysuria. It transpires he had sexual intercourse with a prostitute during a trip to eastern Europe with friends. Gram staining of the discharge reveals Gram-negative diplococci. Which one of the following options would be the most appropriate therapy in this case?

A Oral oxytetracycline

B Oral amoxicillin

C Cefixime im

D Oral ciprofloxacin

E IM ceftriaxone

Explanation

Management of gonorrhoea

Tetracyclines and ciprofloxacin are no longer recommended as treatment for gonorrhoea due to the emergence of resistance Single dose im ceftriaxone or oral cefixime are the standard first line choices now Azithromycin would be another reasonable alternative All sexual contacts should be treated, and if this man has a regular partner in the UK then he or she should be encouraged to seek medical attention if possible

14962

Next Question

Previous Question Tag Question Feedback End Review

Difficulty: Average https://mypastest.pastest.com/Secure/TestMe/Browser/429893#Top 1/2 8/10/2016 MyPastest Peer Responses

Session Progress

Responses Correct: 0

Responses Incorrect: 151

Responses Total: 151

Responses - % Correct: 0%

Blog (https://www.pastest.com/blog) About Pastest (https://www.pastest.com/about-us) Contact Us (https://www.pastest.com/contact-us) Help (https://www.pastest.com/help) © Pastest 2016

https://mypastest.pastest.com/Secure/TestMe/Browser/429893#Top 2/2 8/10/2016 MyPastest

Back to Filters (/Secure/TestMe/Filter/429893/QA) Question 48 of 151

A 32-year-old man returns from a cruise on the river Nile in Egypt. During the trip he made a point of trying some local food from the various stops along the river. For a few days before coming home, and since his return he has had diarrhoea that floats on the surface of the toilet bowl and is hard to flush away. He has also noticed that his abdomen feels very bloated. His partner complains that he has increased flatulence that smells disgusting. A fresh stool sample is obtained and appears to contain some cysts.

Which one of the following options represents the most likely infective cause?

A Giardia

B Salmonella

C Shigella

D Campylobacter

E Staphylococcus aureus

Explanation

Giardiasis Aetiology

The patient in the present case has a diarrhoeal illness caused by Giardia lamblia, a flagellate protozoan water-borne infection It is common in the Middle East

Investigations

Trophozoites and cysts may be seen in a fresh stool sample

Treatment

Treatment is often empirical; treatment of giardiasis is either with a one-off dose of tinidazole, or a short course of metronidazole https://mypastest.pastest.com/Secure/TestMe/Browser/429893#Top 1/2 8/10/2016 MyPastest

Complications

Lactose intolerance persists in a significant number of patients post infection, symptoms of which do resemble continued giardiasis

18602

Next Question

Previous Question Tag Question Feedback End Review

Difficulty: Easy

Peer Responses

Session Progress

Responses Correct: 0

Responses Incorrect: 151

Responses Total: 151

Responses - % Correct: 0%

Blog (https://www.pastest.com/blog) About Pastest (https://www.pastest.com/about-us) Contact Us (https://www.pastest.com/contact-us) Help (https://www.pastest.com/help)

© Pastest 2016

https://mypastest.pastest.com/Secure/TestMe/Browser/429893#Top 2/2 8/10/2016 MyPastest

Back to Filters (/Secure/TestMe/Filter/429893/QA) Question 49 of 151

A 20-year-old student presents with a 5-day history of fever and sore throat. His GP started him on penicillin, but there was no improvement. On examination his temperature is 38.8 °C, he has grey plaques on his tonsils, cervical lymphadenopathy and splenomegaly. Which one of the following options is the most likely diagnosis?

A Streptococcus infection

B Borrelia vincentii infection

C Diphtheria

D Infectious mononucleosis

E Toxoplasmosis

Explanation

Infectious mononucleosis Epidemiology

Infectious mononucleosis is associated with kissing The disease is acquired when a healthy carrier, who is shedding virus in his/her saliva, passes it during close buccal contact directly into the oropharynx of a partner who was not primarily infected as a child However as the incubation period is 1–2 months, initial contact with an individual with pharyngitis may not be remembered Although most cases occur in adolescents and young adults, children and the middle- aged may sometimes develop the disease, and rarely also the elderly Primary Epstein–Barr virus (EBV) infection giving infectious mononucleosis-like symptoms may also be transmitted by blood transfusion or organ grafting from an infected donor to a previously uninfected recipient.

Clinical presentation

https://mypastest.pastest.com/Secure/TestMe/Browser/429893#Top 1/3 8/10/2016 MyPastest

Classic infectious mononucleosis may follow some days of vague indisposition or may start abruptly It presents with sore throat, fever with sweating, anorexia, headache and fatigue, together with malaise quite out of proportion to the other complaints Dysphagia may be noticed and also brief orbital oedema Erythematous and maculopapular rashes occur in a small number of untreated patients but in many more who have been taking ampicillin for a sore throat before infectious mononucleosis has been diagnosed Rarely, tonsillar and pharyngeal oedema can cause pharyngeal obstruction The fever may rise to 40 °C but high levels and swings are not seen There is redness and oedema of the pharynx, fauces, soft palate and uvula, and about half the patients develop greyish exudates Generalised lymphadenopathy is almost always present, most marked in the cervical region; the glands are symmetrical, discrete and slightly tender, and are accompanied by splenomegaly in about 60% of cases and an enlarged liver in 10% There is usually a moderate bradycardia Besides the rash, characteristic palatal enanthematous crops of reddish petechiae are found in about one-third of patients, and jaundice occurs in about 8%

Differential diagnosis

Borrelia vincenti infection causes acute necrotising ulcerative gingivitis

1304

Next Question

Previous Question Tag Question Feedback End Review

Difficulty: Average

Peer Responses

Session Progress

Responses Correct: 0

Responses Incorrect: 151 https://mypastest.pastest.com/Secure/TestMe/Browser/429893#Top 2/3 8/10/2016 MyPastest

Responses Total: 151

Responses - % Correct: 0%

Blog (https://www.pastest.com/blog) About Pastest (https://www.pastest.com/about-us) Contact Us (https://www.pastest.com/contact-us) Help (https://www.pastest.com/help) © Pastest 2016

https://mypastest.pastest.com/Secure/TestMe/Browser/429893#Top 3/3 8/10/2016 MyPastest

Back to Filters (/Secure/TestMe/Filter/429893/QA) Question 50 of 151

Which one of the following statements is true with regard to legionnaires’ disease?

A Legionella pneumophila is a Gram-positive rod

B The urinary antigen test for Legionella species has low sensitivity and is not particularly specific

C The infection is generally confined to immunocompromised patients

D The β-lactam group of drugs is now regarded as the drug of choice against Legionella species

E Hyponatraemia occurs significantly more often in legionnaires’ disease than in other pneumonias

Explanation

Legionnaires’ disease

Legionnaires’ disease is sometimes overlooked as a possible cause of sporadic, community-acquired pneumonia

Transmission of infection

Water contaminated with the bacteria is the source of infection, and aspiration is the mode of transmission for most patients

Causative organism

Legionella pneumophila is consistently ranked among the top three or four most common causes of community-acquired pneumonia; Streptococcus pneumoniae and Haemophilus influenzae are the two most common causes L. pneumophila is a Gram-negative rod, it is not readily seen on Gram stains of sputum

Risk factors https://mypastest.pastest.com/Secure/TestMe/Browser/429893#Top 1/3 8/10/2016 MyPastest Predisposing factors for legionnaires’ disease include age > 50 years, cigarette smoking, and excessive alcohol intake The risk of contracting legionnaires’ disease is unusually high in patients who have chronic lung disease and those who are immunosuppressed

Clinical presentation

High fever and gastrointestinal symptoms are clinical clues to this disease

Laboratory testing

The urinary antigen test for Legionella species is most useful and should be available in every clinical microbiology laboratory It is sensitive and highly specific Results of this test can be available within hours after submission to the laboratory

Treatment

The treatment of choice is macrolide or quinolone antibiotics

1637

Next Question

Previous Question Tag Question Feedback End Review

Difficulty: Easy

Peer Responses

Session Progress

Responses Correct: 0

Responses Incorrect: 151

Responses Total: 151

Responses - % Correct: 0%

https://mypastest.pastest.com/Secure/TestMe/Browser/429893#Top 2/3 8/10/2016 MyPastest

Blog (https://www.pastest.com/blog) About Pastest (https://www.pastest.com/about-us) Contact Us (https://www.pastest.com/contact-us) Help (https://www.pastest.com/help) © Pastest 2016

https://mypastest.pastest.com/Secure/TestMe/Browser/429893#Top 3/3 8/10/2016 MyPastest

Back to Filters (/Secure/TestMe/Filter/429893/QA) Question 51 of 151

A 29-year-old HIV-positive Brazilian waiter presents with a 4-day history of increasing confusion, night sweats and falling to the right. A CT scan of the brain shows two 3-cm ring- enhancing lesions in the left cerebral hemisphere with surrounding oedema and midline shift. No other history is available as he is too confused. His neurological status is deteriorating steadily. Which is the most likely cause?

A Toxoplasmosis

B Tuberculosis

C Herpes simplex virus infection

D Brain metastasis

E Brain tumour

Explanation

Toxoplasma infection in acquired immune deficiency syndrome (AIDS)

Cerebral infection with the intracellular protozoan Toxoplasma gondii is the most frequent infection of the central nervous system in AIDS when the CD4 lymphocyte count is below 200/mm3

Aetiology

It usually results from reactivation of Toxoplasma cysts in the brain, leading to the formation of focal lesions that are typically multiple but may be single

Clinical presentation

Symptoms develop subacutely and include focal neurological disturbance, headache, confusion, fever, and convulsions

Investigations https://mypastest.pastest.com/Secure/TestMe/Browser/429893#Top 1/2 8/10/2016 MyPastest

On CT scanning the lesions appear as ring-enhancing masses with surrounding oedema Magnetic resonance imaging is more sensitive and frequently detects lesions not visible on the CT scan Serum antibodies to Toxoplasma spp. are usually detectable; their absence makes the diagnosis unlikely but does not exclude it

3706

Next Question

Previous Question Tag Question Feedback End Review

Difficulty: Easy

Peer Responses

Session Progress

Responses Correct: 0

Responses Incorrect: 151

Responses Total: 151

Responses - % Correct: 0%

Blog (https://www.pastest.com/blog) About Pastest (https://www.pastest.com/about-us) Contact Us (https://www.pastest.com/contact-us) Help (https://www.pastest.com/help) © Pastest 2016

https://mypastest.pastest.com/Secure/TestMe/Browser/429893#Top 2/2 8/10/2016 MyPastest

Back to Filters (/Secure/TestMe/Filter/429893/QA) Question 52 of 151

A 32-year-old woman who admits to intermittent drug use in the past 4 months presents to her general practitioner (GP) with jaundice. Screening bloods reveal a hepatocellular-type picture with a predominant rise in transaminases. Her antibody results are shown below:

HBsAg + HBeAg + Anti-HBcAb + Anti-HBeAb + HCV RNA + Anti-HBc IgM +

Which one of the following infections is the most likely cause of her jaundice?

A Chronic hepatitis B infection

B Acute hepatitis C infection

C Chronic hepatitis C infection

D Autoimmune hepatitis

E Acute hepatitis B infection

Explanation

Acute hepatitis B infection

Although the patient appears to have infection with hepatitis C, this is normally asymptomatic until late in the disease and is picked up as an incidental finding with raised transaminases It is most likely that she is in the last stages of acute hepatitis B infection with symptoms of jaundice still persisting, and hepatitis B is the cause of her symptoms The appearance of both anti-HBc and anti HBe antibodies and persistence of e and s antigens support the hypothesis that this is an acute infection with hepatitis B The last antibodies to appear are anti-HBsAbs, which appear around 3 months after https://mypastest.pastest.com/Secure/TestMe/Browser/429893#Top 1/2 8/10/2016 MyPastest acute infection and then persist

5238

Next Question

Previous Question Tag Question Feedback End Review

Difficulty: Difficult

Peer Responses

Session Progress

Responses Correct: 0

Responses Incorrect: 151

Responses Total: 151

Responses - % Correct: 0%

Blog (https://www.pastest.com/blog) About Pastest (https://www.pastest.com/about-us) Contact Us (https://www.pastest.com/contact-us) Help (https://www.pastest.com/help) © Pastest 2016

https://mypastest.pastest.com/Secure/TestMe/Browser/429893#Top 2/2 8/10/2016 MyPastest

Back to Filters (/Secure/TestMe/Filter/429893/QA) Question 53 of 151

A 40-year-old man who is normally entirely fit and well attends the Emergency Department with a sudden history of severe pain in his right thigh. His only medication is ibuprofen, which he has been taking after ‘overdoing it in the gym’. His right lower limb is exquisitely tender and his thigh is slightly swollen. His temperature is 39 °C and his blood tests reveal a white cell count of 25 × 109 /l, with a neutrophilia, and a CRP of 350 μg/l. His urea and creatinine are mildly elevated. Which one of the following options is the most appropriate management plan?

A Doppler ultrasound of his right lower limb

B MRI of his right lower limb

C Blood cultures, start iv co-amoxiclav and admit to the ward for observation

D Obtain urgent surgical opinion

E CT chest, abdomen and pelvis

Explanation

Necrotising fasciitis

The patient in the present case has a classic history of early necrotising fasciitis

Causative organisms

This infection is caused by group A streptococci

Clinical features

It is characterised by exquisite pain and tenderness, often in the absence of marked erythema of the skin This is because the infection is within the fascia, and only later in the infection does the skin become dusky/purple in colour followed by areas of bullae and necrosis

https://mypastest.pastest.com/Secure/TestMe/Browser/429893#Top 1/2 8/10/2016 MyPastest Further investigations

Radiological tests may reveal gas and inflammation in the deep tissues but this wastes valuable time A diagnostic core biopsy with a view to debridement is what is required

Treatment

The infection can be extremely fast moving and the treatment of choice is early surgery Antibiotics have a secondary role

626

Next Question

Previous Question Tag Question Feedback End Review

Difficulty: Average

Peer Responses

Session Progress

Responses Correct: 0

Responses Incorrect: 151

Responses Total: 151

Responses - % Correct: 0%

Blog (https://www.pastest.com/blog) About Pastest (https://www.pastest.com/about-us) Contact Us (https://www.pastest.com/contact-us) Help (https://www.pastest.com/help)

© Pastest 2016

https://mypastest.pastest.com/Secure/TestMe/Browser/429893#Top 2/2 8/10/2016 MyPastest

Back to Filters (/Secure/TestMe/Filter/429893/QA) Question 54 of 151

A 30-year-old immigrant from India complains of fever, night sweats and backache. He also coughed up blood. Which one of the following options is the most likely diagnosis?

A HIV

B Infectious mononucleosis

C Tuberculosis

D Malaria

E Pulmonary embolism

Explanation

Epidemiology of tuberculosis

Typically, tuberculosis affects young adults, with peak incidence in those aged 25–44 The dynamics of tuberculosis within a particular country or region, however, reflects both historical trends in tuberculosis transmission and current risk factors and practices of disease control In Western Europe, for example, tuberculosis is seen in two demographic groups: elderly native Europeans who were presumably infected many years ago and who experience reactivation of latent infections as they age or become immunocompromised, and younger immigrants from high-incidence countries in the developing world

3700

Next Question

Previous Question Tag Question Feedback End Review

https://mypastest.pastest.com/Secure/TestMe/Browser/429893#Top 1/2 8/10/2016 MyPastest

Difficulty: Easy

Peer Responses

Session Progress

Responses Correct: 0

Responses Incorrect: 151

Responses Total: 151

Responses - % Correct: 0%

Blog (https://www.pastest.com/blog) About Pastest (https://www.pastest.com/about-us) Contact Us (https://www.pastest.com/contact-us) Help (https://www.pastest.com/help) © Pastest 2016

https://mypastest.pastest.com/Secure/TestMe/Browser/429893#Top 2/2 8/10/2016 MyPastest

Back to Filters (/Secure/TestMe/Filter/429893/QA) Question 55 of 151

A nurse has a needlestick injury after taking blood from a patient known to be HIV positive. Which one of the following options is the most appropriate immediate management after hand washing for 10 minutes?

A Continue hand washing for a further 20 minutes

B Anti-retroviral therapy

C Test for hepatitis B and C

D Blood cultures

E Broad-spectrum antibiotics

Explanation Infection post-needlestick injury

Estimated risk of infection

Based on data from more than 3000 occupational exposures to HIV, the average risk of HIV infection after needlestick injury or other percutaneous exposure was calculated to be 0.3% (about 1 in 325) The risk following mucous membrane exposure has been estimated to be around 0.1% The risk of transmission is greatest for deep injuries; if there is visible blood on the device; during procedures involving direct cannulation of blood vessels; or if the source patient has advanced HIV disease In the management of occupational exposure to HIV, a careful risk assessment should be done and information provided

Post-exposure prophylaxis

A small retrospective case–control study demonstrated an 80% reduction in the likelihood of seroconversion in healthcare workers who took zidovudine soon after percutaneous exposure to HIV In view of the greater activity of anti-retroviral drug combinations but without direct evidence, it is currently recommended that high-risk occupational exposures to HIV are treated as soon as possible with Raltegravir and Truvada in combination for 28 days Nevirapine is not currently recommended in post-exposure prophylaxis regimens because of a relatively high rate of adverse reactions

https://www.gov.uk/government/uploads/system/uploads/attachment_data/file/351633/Change_to_recommended_regimen_fo (https://www.gov.uk/government/uploads/system/uploads/attachment_data/file/351633/Change_to_recommended_regimen_ 5572

Next Question

https://mypastest.pastest.com/Secure/TestMe/Browser/429893#Top 1/2 8/10/2016 MyPastest

Previous Question Tag Question Feedback End Review

Difficulty: Average

Peer Responses

Session Progress

Responses Correct: 0

Responses Incorrect: 151

Responses Total: 151

Responses - % Correct: 0%

Blog (https://www.pastest.com/blog) About Pastest (https://www.pastest.com/about-us) Contact Us (https://www.pastest.com/contact-us) Help (https://www.pastest.com/help) © Pastest 2016

https://mypastest.pastest.com/Secure/TestMe/Browser/429893#Top 2/2 8/10/2016 MyPastest

Back to Filters (/Secure/TestMe/Filter/429893/QA) Question 56 of 151

A 32-year-old former IV drug abuser comes to the clinic for review. She has been clean of heroin for the past 4 years, has a job and a stable partner and is interested in starting a family. She wants to discuss her hepatitis status with you. Investigations:

Anti-Hep C antibody negative

Anti-HBs antibody positive

HBs Ag negative

Hbe Ag negative

Anti-HBc antibody positive

HIV negative

Which of the following is the most likely diagnosis?

A Cleared hepatitis B infection

B Hepatitis B carrier status low infectivity

C Hepatitis B carrier status high infectivity

D Previous vaccination

E Recent hepatitis B infection

Explanation The answer is Cleared hepatitis B infection - This patient is antigen negative with both core and surface antibodies, indicating that she has cleared hepatitis B infection. Patients who are HBs Ag positive still have active hepatitis B. Where HBe Ag is also positive patients have a high level of infectivity, where HBe Ag is negative then infectivity is lower. Previous vaccination is only associated with HBs positivity. 38085 https://mypastest.pastest.com/Secure/TestMe/Browser/429893#Top 1/2 8/10/2016 MyPastest

Next Question

Previous Question Tag Question Feedback End Review

Difficulty: Average

Peer Responses

Session Progress

Responses Correct: 0

Responses Incorrect: 151

Responses Total: 151

Responses - % Correct: 0%

Blog (https://www.pastest.com/blog) About Pastest (https://www.pastest.com/about-us) Contact Us (https://www.pastest.com/contact-us) Help (https://www.pastest.com/help) © Pastest 2016

https://mypastest.pastest.com/Secure/TestMe/Browser/429893#Top 2/2 8/10/2016 MyPastest

Back to Filters (/Secure/TestMe/Filter/429893/QA) Question 57 of 151

A 30-year-old man who is HIV-positive presents with a 2-week history of epigastric pain,

dysphagia and occasional vomiting. His GP prescribed H2 -antagonists, however there was no relief. He currently takes zidovudine. His CD4+ lymphocyte count is 220/mm3 . What is the next step in confirming the cause of his epigastric pain?

A Oesophageal manometry

B Abdominal ultrasound

C Chest X-ray

D Therapeutic trial of aciclovir

E Oesophagogastroduodenoscopy

Explanation

Oesophagitis in HIV-positive patients Causative organism

In patients with human immunodeficiency virus (HIV), oesophagitis is most commonly caused by Candida albicans Oesophageal candidiasis indicates advanced immunosuppression and is an AIDS- defining condition

Clinical presentation

Oesophagitis presents with retrosternal pain on swallowing The diagnosis should be suspected in a patient with oral candida and dysphagia, and may be supported by a barium swallow or confirmed by endoscopy and biopsy

Management

Treatment is with oral azole antifungal agents Fluconazole may be more effective than ketoconazole https://mypastest.pastest.com/Secure/TestMe/Browser/429893#Top 1/3 8/10/2016 MyPastest Prognosis and further treatment

Oesophagitis may recur, and in patients with severe immunosuppression, candida may become resistant to prolonged azole treatment Resistance tends to develop gradually, which can be monitored by in vitro testing Such patients require treatment or continuous suppression of fungal infection with high doses of fluconazole (which is better tolerated than high doses of ketoconazole or itraconazole) or intermittent treatment with intravenous amphotericin

Other considerations

Azole-resistant oro-oesophageal candidiasis has become much less common since the advent of highly active anti-retroviral therapy (HAART) The differential diagnosis of oesophageal candidiasis includes oesophagitis caused by cytomegalovirus (CMV) or herpes simplex virus (HSV), which require specific antiviral therapy, and aphthous ulceration, which may respond to oral prednisolone or thalidomide The latter should never be used in women of child-bearing potential because of the significant teratogenic risk

1315

Next Question

Previous Question Tag Question Feedback End Review

Difficulty: Easy

Peer Responses

Session Progress

Responses Correct: 0

Responses Incorrect: 151

Responses Total: 151

Responses - % Correct: 0%

https://mypastest.pastest.com/Secure/TestMe/Browser/429893#Top 2/3 8/10/2016 MyPastest

Blog (https://www.pastest.com/blog) About Pastest (https://www.pastest.com/about-us) Contact Us (https://www.pastest.com/contact-us) Help (https://www.pastest.com/help) © Pastest 2016

https://mypastest.pastest.com/Secure/TestMe/Browser/429893#Top 3/3 8/10/2016 MyPastest

Back to Filters (/Secure/TestMe/Filter/429893/QA) Question 58 of 151

A 31-year-old lawyer currently managed with triple anti-retroviral therapy comes to the HIV clinic for review. Treatment has been unchanged since his diagnosis some 18 months earlier. Despite his taking his medication regularly and maintaining a healthy lifestyle he has begun to feel increasingly unwell with tiredness and lethargy over the past 3 months and has suffered an episode of herpes zoster. Viral load has increased to above 2000 copies per ml for the past two measurements. Which of the following is the most appropriate next step?

A Arrange genotypic resistance testing

B Arrange phenotypic resistance testing

C Maintain current regimen for a further 3 months

D Maintain current regimen for a further 6 months

E Switch to an alternative drug regimen without resistance testing

Explanation The answer is Arrange genotypic resistance testing - After resistance has developed to a first anti-retroviral therapy regimen, genotypic resistance testing to look for mutations known to lead to drug resistance in the protease, reverse transcriptase and integrase regions of the HIV virus is recommended. The kits in essence look for mutations which have first been amplified with PCR. Once any mutations have been identified, therapy can then be tailored to the patient. Phenotypic resistance testing is not generally recommended when resistance first appears, although it does have the potential advantage of measuring the aggregate effect of a number of mutations in the HIV genome on susceptability to specific therapies. It can however take considerably longer than genotype analysis. Blind switching may be ineffective, and maintaining the current drug regimen runs the risk of worsening the patient’s underlying immunosuppression significantly. 40177

Next Question

https://mypastest.pastest.com/Secure/TestMe/Browser/429893#Top 1/2 8/10/2016 MyPastest Previous Question Tag Question Feedback End Review

Difficulty: Average

Peer Responses

Session Progress

Responses Correct: 0

Responses Incorrect: 151

Responses Total: 151

Responses - % Correct: 0%

Blog (https://www.pastest.com/blog) About Pastest (https://www.pastest.com/about-us) Contact Us (https://www.pastest.com/contact-us) Help (https://www.pastest.com/help) © Pastest 2016

https://mypastest.pastest.com/Secure/TestMe/Browser/429893#Top 2/2 8/10/2016 MyPastest

Back to Filters (/Secure/TestMe/Filter/429893/QA) Question 59 of 151

A 28-year-old school teacher presents with headache, photophobia and neck stiffness following a flu-like illness. Following a dose of intramuscular penicillin, a lumbar puncture is performed. The CSF is clear, with 60 white blood cells/μl (50% lymphocytes), protein 0.8 g/l, glucose 3.5 mmol/l (serum glucose 5.0 mmol/l) and no organisms on the Gram stain. Which one of the following options is the most likely causative organism?

A Enterovirus spp

B Listeria monocytogenes

C Mycobacterium tuberculosis

D Neisseria meningitidis

E Streptococcus pneumoniae

Explanation

Meningitis Causative organisms

In young adults, meningitis is most likely due to enterovirus and meningococcus (N. meningitidis) Enterovirus is far more prevalent than Meningococcus and is often passed around schools Listeria is mostly seen in neonates and the elderly Streptococcus pneumoniae is the commonest cause of meningitis in those over 40 years of age

Investigations

In enterovirus infections, the cerebrospinal fluid (CSF) typically has a normal glucose concentration, modestly raised protein level and a pleiotropic white cell count The CSF is clear with no organisms on the Gram stain and the diagnosis is usually https://mypastest.pastest.com/Secure/TestMe/Browser/429893#Top 1/2 8/10/2016 MyPastest confirmed by identification of viral nucleic acid by polymerase chain reaction (PCR) on CSF is rare in the UK and the CSF typically shows a markedly raised protein (> 1 g/l), a low glucose (< 50% serum) and a lymphocytosis In streptococcal meningitis very high neutrophil counts are often seen in the CSF, which has a high protein and low glucose level

Immediate treatment

Given the important differential diagnosis of meningococcus, the patient should be treated with high-dose intravenous antibiotics until the diagnosis is confirmed

757

Next Question

Previous Question Tag Question Feedback End Review

Difficulty: Average

Peer Responses

Session Progress

Responses Correct: 0

Responses Incorrect: 151

Responses Total: 151

Responses - % Correct: 0%

Blog (https://www.pastest.com/blog) About Pastest (https://www.pastest.com/about-us) Contact Us (https://www.pastest.com/contact-us) Help (https://www.pastest.com/help)

© Pastest 2016

https://mypastest.pastest.com/Secure/TestMe/Browser/429893#Top 2/2 8/10/2016 MyPastest

Back to Filters (/Secure/TestMe/Filter/429893/QA) Question 60 of 151

A 41-year-old man with a history of HIV presents to the Emergency Department with a seizure. He attends with his partner who tells the staff that his boyfriend has been unwell with a diarrhoeal illness for the past 3 days after returning from a gastronomic weekend in France. On examination he is pyrexial at 38.2 °C and is drowsy. He has signs of meningism and appears to be neglecting his left side.

Investigations;

Hb 11.4 g/dl

WCC 9.1 × 109 /l

PLT 181 × 109 /l

Na+ 140 mmol/l

K+ 4.8 mmol/l

Creatinine 125 μmol/l

CSF pleiocytosis, protein moderately elevated, glucose low

Which one of the following options is the most likely diagnosis?

A Viral meningitis

B Meningococcal meningitis

C Salmonella meningitis

D Listeria meningitis

E Herpes encephalitis

Explanation

Listeria meningitis Diagnosis https://mypastest.pastest.com/Secure/TestMe/Browser/429893#Top 1/2 8/10/2016 MyPastest

The clue in the present case is the scenario, where the couple are likely to have been exposed to Listeria through consumption of French unpasteurised soft cheeses Examination of wet cerebrospinal fluid (CSF) reveals the tumbling motility of Listeria bacteria, and coupled with blood cultures is a key part of establishing the diagnosis

Clinical features

Neurological manifestations of Listeria infection range from generalised seizures to partial seizures, neglect, hemiparesis, cervical myelitis and brain abscess in up to 10% of patients

21260

Next Question

Previous Question Tag Question Feedback End Review

Difficulty: Average

Peer Responses

Session Progress

Responses Correct: 0

Responses Incorrect: 151

Responses Total: 151

Responses - % Correct: 0%

Blog (https://www.pastest.com/blog) About Pastest (https://www.pastest.com/about-us) Contact Us (https://www.pastest.com/contact-us) Help (https://www.pastest.com/help) © Pastest 2016

https://mypastest.pastest.com/Secure/TestMe/Browser/429893#Top 2/2 8/10/2016 MyPastest

Back to Filters (/Secure/TestMe/Filter/429893/QA) Question 61 of 151

A 40-year-old Indian man is visiting the UK and 1 month later develops abdominal pain and fevers. He gives a history of intermittent bouts of diarrhoea, the last occurring 6 weeks ago. On examination his temperature is 39.5 °C and he has right-sided abdominal tenderness. His white cell count in blood is 24 × 109 /l. Blood films for malaria are negative. Stool and blood cultures are negative. Urine is negative for blood and protein.

What is the next most appropriate investigation?

A Bone marrow aspirate for culture

B Electron microscopy on the stool specimen

C Urine culture

D US scan abdomen

E Widal test

Explanation

Amoebic liver abscess

The most likely diagnosis in the present case is amoebic liver abscess

Further investigations

Ultrasound and CT scans are the best tools to investigate this, and to determine the need for drainage if there is a risk of rupture

Differential diagnosis

Typhoid is an important differential diagnosis, usually identified by culturing Salmonella typhi in the blood, although a bone marrow aspirate will occasionally yield the organism

the Widal test for typhoid is largely obsolete, especially since the advent of vaccination, which gives a positive test

769 https://mypastest.pastest.com/Secure/TestMe/Browser/429893#Top 1/2 8/10/2016 MyPastest 769

Next Question

Previous Question Tag Question Feedback End Review

Difficulty: Average

Peer Responses

Session Progress

Responses Correct: 0

Responses Incorrect: 151

Responses Total: 151

Responses - % Correct: 0%

Blog (https://www.pastest.com/blog) About Pastest (https://www.pastest.com/about-us) Contact Us (https://www.pastest.com/contact-us) Help (https://www.pastest.com/help) © Pastest 2016

https://mypastest.pastest.com/Secure/TestMe/Browser/429893#Top 2/2 8/10/2016 MyPastest

Back to Filters (/Secure/TestMe/Filter/429893/QA) Question 62 of 151

The 3-year-old child of a 12-week pregnant, 25-year-old woman develops a typical chickenpox illness. The mother does not recall having had chickenpox herself. Which one of the following is the most appropriate advice you can give the mother?

A Avoid further contact with the child

B Test the mother for varicella zoster IgG

C Take aciclovir as prophylaxis

D Receive varicella zoster immune globulin (VZIG) urgently

E Consider termination of pregnancy

Explanation

Chickenpox Laboratory investigations

The mother in the present case should be tested for varicella zoster virus (VZV) IgG urgently and if negative should receive VZIG

Management

VZIG is effective in reducing the incidence of chickenpox up to 7–10 days post- exposure and probably in reducing the incidence of congenital varicella syndrome Aciclovir is not licensed as prophylaxis, but is occasionally used in severely immunocompromised VZV contacts

Complications of chickenpox in pregnancy

The incidence of congenital varicella syndrome is about 2% in mothers who develop primary chickenpox in the first half of pregnancy

3690 https://mypastest.pastest.com/Secure/TestMe/Browser/429893#Top 1/2 8/10/2016 MyPastest Next Question

Previous Question Tag Question Feedback End Review

Difficulty: Average

Peer Responses

Session Progress

Responses Correct: 0

Responses Incorrect: 151

Responses Total: 151

Responses - % Correct: 0%

Blog (https://www.pastest.com/blog) About Pastest (https://www.pastest.com/about-us) Contact Us (https://www.pastest.com/contact-us) Help (https://www.pastest.com/help)

© Pastest 2016

https://mypastest.pastest.com/Secure/TestMe/Browser/429893#Top 2/2 8/10/2016 MyPastest

Back to Filters (/Secure/TestMe/Filter/429893/QA) Question 63 of 151

A 72-year-old woman returns to the UK after a visit to see her son who is working in Kenya. She is admitted the Emergency Department straight from the plane with a fever and confusion, and has been incontinent of urine during the flight. She is usually very well; her only medication of note is indapamide which she takes for hypertension. On examination she has a fever with a pyrexia of 38.6 °C, she is hypotensive at 100/50 mmHg, and has a pulse of 95/min. She looks jaundiced. You call her son and he tells you that she has been relatively well apart from a minor cough over the past few days; he reports that during her trip they went on a safari within the past week and bathed in some local pools. Investigations:

Hb 9.9 g/dl

WCC 13.4 × 109 /l

PLT 100 × 109 /l

Na+ 134 mmol/l

K+ 5.9 mmol/l

Creatinine 201 μmol/l

Bilirubin 130 μmol/l

ALT 95 U/l

Glucose 3.9 mmol/l

Which one of the following options is the most likely diagnosis?

A Dengue fever

B Malaria

C Urinary tract infection

D Listeria meningitis

https://mypastest.pastest.com/Secure/TestMe/Browser/429893#Top 1/3 8/10/2016 MyPastest

E Weil’s disease

Explanation

Malaria Diagnosis

Fever with anaemia and decreased platelets, coupled with a raised bilirubin suggestive of haemolysis increases the suspicion of malaria as the underlying diagnosis The hyponatraemia and evolving renal failure are also consistent with malaria

Laboratory investigations

Thick and thin films remain the standard method of confirming the diagnosis Polymerase chain reaction (PCR), whilst an expensive option, is becoming increasingly available

Prognosis

Hypoglycaemia and increased white cell count are indicators of a poor prognosis IV quinine and oral doxycycline remains the regimen of choice for patients with Plasmodium falciparum malaria, the suspected type of malaria here Artemesin based compounds are recommended by the WHO in certain circumstances, but are available in the UK only on a named patient basis

22478

Next Question

Previous Question Tag Question Feedback End Review

Difficulty: Difficult

Peer Responses

Session Progress

Responses Correct: 0

Responses Incorrect: 151 https://mypastest.pastest.com/Secure/TestMe/Browser/429893#Top 2/3 8/10/2016 MyPastest

Responses Total: 151

Responses - % Correct: 0%

Blog (https://www.pastest.com/blog) About Pastest (https://www.pastest.com/about-us) Contact Us (https://www.pastest.com/contact-us) Help (https://www.pastest.com/help) © Pastest 2016

https://mypastest.pastest.com/Secure/TestMe/Browser/429893#Top 3/3 8/10/2016 MyPastest

Back to Filters (/Secure/TestMe/Filter/429893/QA) Question 64 of 151

A 25-year-old man noticed a small painless papule of the glans of his penis, and which ulcerated a few days ago. On examination there is a solitary, round, indurated ulcer. Which one of the following options is the most likely diagnosis?

A Gonorrhoea

B Syphilis

C AIDS

D Tuberculosis

E Papillomavirus infection

Explanation

Primary syphilis Clinical presentation

The first sign of primary syphilis is a small, painless papule that rapidly ulcerates The ulcer (chancre) is usually solitary, round or oval, painless and often indurated, and surrounded by a bright-red margin It is not usually secondarily infected, a feature of all open syphilitic lesions of any stage In heterosexual men, the common sites are the coronal sulcus, the glans and inner surface of the prepuce, but may be found on the shaft of the penis and beyond In homosexual men, the ulcer is usually present in the anal canal, less commonly in the mouth and genitalia In women, most chancres occur on the vulva, the labia and, more rarely, the cervix where they are liable to be overlooked

Laboratory findings

Treponema pallidum can be demonstrated in the serum from the sore, which is easily obtained after slightly abrading the base

2708 https://mypastest.pastest.com/Secure/TestMe/Browser/429893#Top 1/2 8/10/2016 MyPastest 2708

Next Question

Previous Question Tag Question Feedback End Review

Difficulty: Easy

Peer Responses

Session Progress

Responses Correct: 0

Responses Incorrect: 151

Responses Total: 151

Responses - % Correct: 0%

Blog (https://www.pastest.com/blog) About Pastest (https://www.pastest.com/about-us) Contact Us (https://www.pastest.com/contact-us) Help (https://www.pastest.com/help) © Pastest 2016

https://mypastest.pastest.com/Secure/TestMe/Browser/429893#Top 2/2 8/10/2016 MyPastest

Back to Filters (/Secure/TestMe/Filter/429893/QA) Question 65 of 151

A 56-year-old man is diagnosed with viridans-type streptococcal endocarditis. His electrocardiogram (ECG) shows an increasing PR interval. Which one of the following options is the most appropriate management?

A Urgent surgical referral

B Angiography

C Central line

D Blood transfusion

E Intravenous fluids

Explanation

Considerations in cardiovascular infections

An increasing PR interval suggests an aortic root abscess, which has a high mortality even with surgery Embolism, particularly if recurrent, is an indication for surgery Cerebral embolism and coronary artery embolism are associated with increased mortality Failure of fever to resolve after 1 week also suggests the presence of an intracardiac abscess that will usually require surgery

3696

Next Question

Previous Question Tag Question Feedback End Review

Difficulty: Easy https://mypastest.pastest.com/Secure/TestMe/Browser/429893#Top 1/2 8/10/2016 MyPastest Peer Responses

Session Progress

Responses Correct: 0

Responses Incorrect: 151

Responses Total: 151

Responses - % Correct: 0%

Blog (https://www.pastest.com/blog) About Pastest (https://www.pastest.com/about-us) Contact Us (https://www.pastest.com/contact-us) Help (https://www.pastest.com/help) © Pastest 2016

https://mypastest.pastest.com/Secure/TestMe/Browser/429893#Top 2/2 8/10/2016 MyPastest

Back to Filters (/Secure/TestMe/Filter/429893/QA) Question 66 of 151

You attend a 23-year-old student at her flat as outreach doctor with a paramedic crew. She is extremely unwell with drowiness and confusion, complaining of neck stiffness and a severe headache. Examination reveals a BP of 90/60 mmHg, pulse of 110/min and regular. There is a widespread non-blanching rash with petechial haemorrhages. There are obvious signs of meningism and bilateral papilloedema.

Which of the following is the most important next step?

A IV Benzylpenicillin

B IV Dexamethasone

C IV Mannitol

D IV Normal saline

E Transfer to the Emergency Department

Explanation The answer is IV Benzylpenicillin - This patient has suspected meningococcal meningitis. As long as it does not delay transfer to the Emergency Department, IV or IM Benzylpenicillin is therefore recommended. According to NICE pathways, antibiotics are not required in the absence of rash or if the rash is blanching. Whilst IV fluid support is of course important, antibiotic therapy pre-transfer is the most important priority. IV mannitol can be considered only if there is life threatening cerebral oedema and would be given in the hospital setting +/- corticosteroids. http://pathways.nice.org.uk/pathways/bacterial-meningitis-and-meningococcal- septicaemia/prehospital-management-for-bacterial-meningitis-and-meningococcal- disease#content=view-node%3Anodes-give-antibiotics-unless-it-will-delay-transfer-to- hospital (http://pathways.nice.org.uk/pathways/bacterial-meningitis-and-meningococcal- septicaemia/prehospital-management-for-bacterial-meningitis-and-meningococcal- disease#content=view-node%3Anodes-give-antibiotics-unless-it-will-delay-transfer-to- hospital) 40178

Next Question https://mypastest.pastest.com/Secure/TestMe/Browser/429893#Top 1/2 8/10/2016 MyPastest

Previous Question Tag Question Feedback End Review

Difficulty: Average

Peer Responses

Session Progress

Responses Correct: 0

Responses Incorrect: 151

Responses Total: 151

Responses - % Correct: 0%

Blog (https://www.pastest.com/blog) About Pastest (https://www.pastest.com/about-us) Contact Us (https://www.pastest.com/contact-us) Help (https://www.pastest.com/help) © Pastest 2016

https://mypastest.pastest.com/Secure/TestMe/Browser/429893#Top 2/2 8/10/2016 MyPastest

Back to Filters (/Secure/TestMe/Filter/429893/QA) Question 67 of 151

A 45-year-old business traveller noticed some moderate diarrhoea 3 days after he arrived in Korea. The diarrhoea lasted for 4 days. Which one of the following options is the most likely cause for his diarrhoea?

A Legionella

B Staphylococcus

C Enterotoxic Escherichia coli

D Giardia lamblia

E Entamoeba histolytica

Explanation

Traveller’s diarrhoea Aetiology

The most common cause of the type of presentation in the present case is enterotoxigenic Escherichia coli (ETEC)

Clinical course

Symptoms are usually mild, lasting only about 3–5 days, but holiday and business plans may be disrupted

Preventive measures

Strict food and water hygiene reduces the risk of gastroenteritis Heating water to 100 °C will kill most pathogens, as will chemical treatment with chlorine or iodine (iodine is contraindicated in pregnant women and some patients with thyroid disease) Water filters are useful additions

2690 https://mypastest.pastest.com/Secure/TestMe/Browser/429893#Top 1/2 8/10/2016 MyPastest 2690

Next Question

Previous Question Tag Question Feedback End Review

Difficulty: Average

Peer Responses

Session Progress

Responses Correct: 0

Responses Incorrect: 151

Responses Total: 151

Responses - % Correct: 0%

Blog (https://www.pastest.com/blog) About Pastest (https://www.pastest.com/about-us) Contact Us (https://www.pastest.com/contact-us) Help (https://www.pastest.com/help) © Pastest 2016

https://mypastest.pastest.com/Secure/TestMe/Browser/429893#Top 2/2 8/10/2016 MyPastest

Back to Filters (/Secure/TestMe/Filter/429893/QA) Question 68 of 151

A 19-year-old patient presents with an episode of diarrhoea followed a few days later, after her return to the UK, by increasing nausea, anorexia and jaundice. She has recently returned from an all inclusive holiday to Tunisia. She bought it as a last minute purchase, and did not consult her GP about travel vaccinations. Which one of the following is the most likely pathogen?

A Hepatitis A virus

B E. coli

C Salmonella

D Mycobacterium tuberculosis

E Staphylococcus

Explanation

Hepatitis A virus

This causes acute self-limited hepatitis, and is often associated with a short diarrhoeal illness, although there is no chronic viral carriage or chronic liver disease The RNA virus is acquired orally The incubation period is between 2 and 6 weeks Transmission generally follows the ingestion of food or water contaminated with faeces from an Hepatitis A virus-infected individual

3717

Next Question

Previous Question Tag Question Feedback End Review

https://mypastest.pastest.com/Secure/TestMe/Browser/429893#Top 1/2 8/10/2016 MyPastest

Difficulty: Easy

Peer Responses

Session Progress

Responses Correct: 0

Responses Incorrect: 151

Responses Total: 151

Responses - % Correct: 0%

Blog (https://www.pastest.com/blog) About Pastest (https://www.pastest.com/about-us) Contact Us (https://www.pastest.com/contact-us) Help (https://www.pastest.com/help) © Pastest 2016

https://mypastest.pastest.com/Secure/TestMe/Browser/429893#Top 2/2 8/10/2016 MyPastest

Back to Filters (/Secure/TestMe/Filter/429893/QA) Question 69 of 151

Pyrazinamide is used as part of the combination therapy for tuberculosis. Which one of the following options is the most common side-effect of pyrazinamide?

A Hepatitic dysfunction

B Hyperuricaemia

C Colour vision changes

D Dizziness

E Neurotoxicity

Explanation

Side-effects of anti-tuberculous drugs

Hepatic dysfunction is the most common adverse reaction and varies from a symptomless abnormality of hepatic cell function detected only through laboratory liver function tests, through a mild syndrome of fever, malaise and liver tenderness, to more serious reactions such as clinical jaundice and rarely acute yellow atrophy and death Pyrazinamide is also often associated with arthralgias, and it may precipitate gout Pyrazinamide inhibits the renal tubular excretion of uric acid, resulting in increased serum levels of uric acid Frank gouty arthritis is relatively uncommon with pyrazinamide use, and its frequency is reduced with intermittent dosing, however, routine use of allopurinol to prevent gout is not recommended Colour vision changes are associated with ethambutol

1320

Next Question

https://mypastest.pastest.com/Secure/TestMe/Browser/429893#Top 1/2 8/10/2016 MyPastest

Previous Question Tag Question Feedback End Review

Difficulty: Average

Peer Responses

Session Progress

Responses Correct: 0

Responses Incorrect: 151

Responses Total: 151

Responses - % Correct: 0%

Blog (https://www.pastest.com/blog) About Pastest (https://www.pastest.com/about-us) Contact Us (https://www.pastest.com/contact-us) Help (https://www.pastest.com/help) © Pastest 2016

https://mypastest.pastest.com/Secure/TestMe/Browser/429893#Top 2/2 8/10/2016 MyPastest

Back to Filters (/Secure/TestMe/Filter/429893/QA) Question 70 of 151

A 22-year-old student develops a sore throat, fever and myalgia. He is admitted with increasing abdominal pain and jaundice. On examination he is pyrexial at 39.5 °C and jaundiced with tender hepatosplenomegaly. He has a normocytic anaemia (Hb 9.5 g/dl) and raised white cell count (lymphocytosis 8.0 × 109 /l). The ALT is 250 IU/l, the bilirubin is 60 μmol/l and the amylase is normal. Serology to EBV is negative for IgG. Which one of the following options is the most likely diagnosis?

A CMV

B EBV

C Hepatitis A

D HIV-seroconversion illness

E Streptococcal infection

Explanation

Epstein–Barr virus infection Diagnostic considerations

A marked lymphocytosis in the presence of hepatosplenomegaly and anaemia points to Epstein–Barr virus (EBV) or cytomegalovirus (CMV) infection

the former is more likely, especially in the context of negative IgG antibodies

Complications

Primary EBV infection in young adults can lead to a severe illness with pharyngitis, hepatitis, anaemia and splenic rupture

Differential diagnosis

Hepatitis A is usually accompanied by gastroenteritis with no marrow suppression or pharyngitis https://mypastest.pastest.com/Secure/TestMe/Browser/429893#Top 1/2 8/10/2016 MyPastest HIV-seroconversion illness does not typically cause a hepatitis or hepatosplenomegaly, but a rash and lymphopenia would be typical Streptococcal infection usually leads to a neutrophilia These are all important differential diagnoses

786

Next Question

Previous Question Tag Question Feedback End Review

Difficulty: Average

Peer Responses

Session Progress

Responses Correct: 0

Responses Incorrect: 151

Responses Total: 151

Responses - % Correct: 0%

Blog (https://www.pastest.com/blog) About Pastest (https://www.pastest.com/about-us) Contact Us (https://www.pastest.com/contact-us) Help (https://www.pastest.com/help)

© Pastest 2016

https://mypastest.pastest.com/Secure/TestMe/Browser/429893#Top 2/2 8/10/2016 MyPastest

Back to Filters (/Secure/TestMe/Filter/429893/QA) Question 71 of 151

A patient with stable sickle-cell anaemia presents with an erythematous rash all over her body, which is associated with painful swollen joints. On examination she has clinical signs of anaemia. The full blood count shows absent reticulocytes in the peripheral blood. Infection with which one of the following pathogens is most likely to have caused her symptoms?

A Hepatitis C virus

B Hepatitis B virus

C Human immunodeficiency virus

D Rubella virus

E Parvovirus B19

Explanation

Parvovirus B19 infection Epidemiology

Parvovirus B19 is most common in children between 6 and 10 years By 20 years of age, 60–70% of the population have been infected Susceptible adults remain at risk of infection, often following exposure to B19 virus in their own children Epidemics occur every 4 or 5 years, with peaks of infection in winter and spring

Transmission of infection

Parvovirus B19 infection is usually spread by respiratory droplets; contamination of hands and surfaces may also contribute

Clinical manifestations and course

The most common specific clinical manifestation is erythema infectiosum, an erythematous rash illness https://mypastest.pastest.com/Secure/TestMe/Browser/429893#Top 1/3 8/10/2016 MyPastest The rash has an incubation period of 17–22 days Classically, the illness begins with mild fever and lassitude, followed by the facial erythema referred to as ‘slapped-cheek disease’ More than 80% of adults with B19 infection (especially women) present with painful or swollen joints An acute-onset, symmetrical polyarthritis specifically affects the small joints of the hands and feet The infection usually resolves within a few weeks

Haematological features

Interruption of erythropoiesis caused by B19 is transient and insufficient to cause clinically significant anaemia in individuals with normal red cell lifespan and function However, in those with a shortened red cell lifespan, such as patients with sickle-cell anaemia, B19 infection can rapidly lead to a more profound anaemia termed ‘an aplastic crisis’ In the acute phase there is erythroid aplasia and the absence of reticulocytes in peripheral blood and, in recovery, reticulocytosis and the appearance of giant pronormoblasts in the bone marrow

2710

Next Question

Previous Question Tag Question Feedback End Review

Difficulty: Easy

Peer Responses

Session Progress

Responses Correct: 0

Responses Incorrect: 151

Responses Total: 151

Responses - % Correct: 0%

https://mypastest.pastest.com/Secure/TestMe/Browser/429893#Top 2/3 8/10/2016 MyPastest

Blog (https://www.pastest.com/blog) About Pastest (https://www.pastest.com/about-us) Contact Us (https://www.pastest.com/contact-us) Help (https://www.pastest.com/help) © Pastest 2016

https://mypastest.pastest.com/Secure/TestMe/Browser/429893#Top 3/3 8/10/2016 MyPastest

Back to Filters (/Secure/TestMe/Filter/429893/QA) Question 72 of 151

A 41-year-old bird-watcher goes on a beach holiday in Gambia. She takes no malaria prophylaxis. On return to the UK she develops high fevers and self-medicates at home with Lemsip®. On presentation to hospital, she had a fever of 40 °C and looked markedly unwell, with a pulse of 130 bpm and BP 90/50 mmHg. She was commenced on iv ceftriaxone and quinine in casualty. Despite this, she deteriorated rapidly, and after 2 hours is found to have a GCS of 3. Her blood film is reported as showing trophozoites and schizonts of Plasmodium falciparum with a parasitaemia of 20%. Which of the following is most important as the next step in the management of this patient, once her airway, breathing and circulation have been stabilised?

A Urgent CT brain scan

B Blood glucose testing

C U&E laboratory testing

D Commence a phenytoin infusion

E Start chloroquine

Explanation

Complications of quinine treatment

The patient in the present case either has cerebral malaria or hypoglycaemia, which may well have been exacerbated by the quinine (it stimulates insulin release) as well as the malaria parasites (they metabolise glucose)

Management

Although intubation may be indicated, she may recover her Glasgow Coma Scale (GCS) score rapidly with a dextrose infusion Anticonvulsant medication may be given to prevent seizures in patients with severe malaria Rehydration is important, but it should be carried out with a degree of caution in those https://mypastest.pastest.com/Secure/TestMe/Browser/429893#Top 1/2 8/10/2016 MyPastest with malaria because of the risk of precipitating an acute respiratory distress syndrome Inotropic support may be required

593

Next Question

Previous Question Tag Question Feedback End Review

Difficulty: Average

Peer Responses

Session Progress

Responses Correct: 0

Responses Incorrect: 151

Responses Total: 151

Responses - % Correct: 0%

Blog (https://www.pastest.com/blog) About Pastest (https://www.pastest.com/about-us) Contact Us (https://www.pastest.com/contact-us) Help (https://www.pastest.com/help)

© Pastest 2016

https://mypastest.pastest.com/Secure/TestMe/Browser/429893#Top 2/2 8/10/2016 MyPastest

Back to Filters (/Secure/TestMe/Filter/429893/QA) Question 73 of 151

A 60-year-old Thai immigrant to the UK presents with a 5-day history of fever, malaise, headache and vomiting. On examination, he has splenomegaly and there is tenderness in the right hypochondrium. A blood test shows normocytic normochromic anaemia and the presence of schizonts in erythrocytes suggestive of Plasmodium vivax infection. What is the initial drug of choice for treating this infection?

A Quinine

B Fansidar (pyrimethamine/sulfadoxine)

C Chloroquine

D Mefloquine

E Malarone (atovaquone/proguanil)

Explanation

Treatment of malaria

The first line of treatment in Plasmodium vivax malaria is chloroquine Most strains are sensitive to this drug Quinine in the form of quinine sulphate may be given in chloroquine-resistant cases Primaquine is then used to eradicate the hypnozoite stage of the disease

3587

Next Question

Previous Question Tag Question Feedback End Review

Difficulty: Average

Peer Responses https://mypastest.pastest.com/Secure/TestMe/Browser/429893#Top 1/2 8/10/2016 MyPastest

Session Progress

Responses Correct: 0

Responses Incorrect: 151

Responses Total: 151

Responses - % Correct: 0%

Blog (https://www.pastest.com/blog) About Pastest (https://www.pastest.com/about-us) Contact Us (https://www.pastest.com/contact-us) Help (https://www.pastest.com/help)

© Pastest 2016

https://mypastest.pastest.com/Secure/TestMe/Browser/429893#Top 2/2 8/10/2016 MyPastest

Back to Filters (/Secure/TestMe/Filter/429893/QA) Question 74 of 151

An HIV-positive patient recently returned from the tropics presents with severe diarrhoea (up to 30 times per day). CD4 count is reduced at 170 cells/mm3 . Which one of the following micro-organisms is the most likely cause?

A Escherichia coli

B Staphylococcus aureus

C Cryptosporidium

D Entamoeba histolytica

E Salmonella

Explanation

Gastroenteritis in HIV-positive patients Causative organisms

The most common opportunistic cause of diarrhoea in human immunodeficiency virus (HIV)-positive patients is infection by the protozoan Cryptosporidium parvum, which causes a self-limiting gastroenteritis in those who are non-immunosuppressed

Clinical features

The diarrhoea may be protracted and severe in HIV-positive patients, with marked fluid and electrolyte losses

Laboratory findings

The diagnosis is made by finding cryptosporidial oocysts in the stool using a modified acid-fast stain Symptoms of cryptosporidiosis are often intermittent, as is excretion of the oocysts, so multiple stool specimens may need to be examined

https://mypastest.pastest.com/Secure/TestMe/Browser/429893#Top 1/2 8/10/2016 MyPastest Treatment considerations

Therapy with highly active anti-retroviral drugs is the most important step in treatment Symptomatic treatment with antidiarrhoeal drugs (such as loperamide), fluid and electrolyte replacement and nutritional support are required Octreotide may be useful in the most severe cases

1328

Next Question

Previous Question Tag Question Feedback End Review

Difficulty: Average

Peer Responses

Session Progress

Responses Correct: 0

Responses Incorrect: 151

Responses Total: 151

Responses - % Correct: 0%

Blog (https://www.pastest.com/blog) About Pastest (https://www.pastest.com/about-us) Contact Us (https://www.pastest.com/contact-us) Help (https://www.pastest.com/help) © Pastest 2016

https://mypastest.pastest.com/Secure/TestMe/Browser/429893#Top 2/2 8/10/2016 MyPastest

Back to Filters (/Secure/TestMe/Filter/429893/QA) Question 75 of 151

A patient is scheduled for an elective splenectomy. At least how long before the operation should Pneumovax be given?

A 1 day

B 1 week

C 2 weeks

D 2 months

E 3 months

Explanation

Vaccination and splenectomy

All patients undergoing an elective splenectomy should be immunised with polyvalent pneumococcal vaccine (Pneumovax), which currently gives variable protection against 23 strains of Streptococcus pneumoniae Where possible, it should be given at least 2 weeks and preferably at least 4 weeks prior to splenectomy to allow immunoglobulin G (IgG) antibody production

5573

Next Question

Previous Question Tag Question Feedback End Review

Difficulty: Average

Peer Responses

https://mypastest.pastest.com/Secure/TestMe/Browser/429893#Top 1/2 8/10/2016 MyPastest

Session Progress

Responses Correct: 0

Responses Incorrect: 151

Responses Total: 151

Responses - % Correct: 0%

Blog (https://www.pastest.com/blog) About Pastest (https://www.pastest.com/about-us) Contact Us (https://www.pastest.com/contact-us) Help (https://www.pastest.com/help)

© Pastest 2016

https://mypastest.pastest.com/Secure/TestMe/Browser/429893#Top 2/2 8/10/2016 MyPastest

Back to Filters (/Secure/TestMe/Filter/429893/QA) Question 76 of 151

A 23-year-old man had unprotected sexual intercourse with a commercial sex worker. Some 2 weeks later, he developed a painless indurated ulcer on the glans that exuded clear serum on pressure. Inguinal lymph nodes in both groins were enlarged and non-tender. Given the suspected diagnosis, which one of the following options is the most appropriate diagnostic test in this case?

A Gram stain of discharge from the ulcer

B Dark-field microscopy of the discharge

C Tissue culture

D Enzyme immunoassay for Chlamydia trachomatis LGV 1, 2 and 3

E Polymerase chain reaction technique

Explanation

Investigations in syphilis

A painless indurated ulcer with non-tender inguinal lymphadenopathy indicates syphilitic infection The best investigation would be dark-field microscopy of the discharge to show the presence of the spirochaete, Treponema pallidum

3576

Next Question

Previous Question Tag Question Feedback End Review

Difficulty: Average

Peer Responses https://mypastest.pastest.com/Secure/TestMe/Browser/429893#Top 1/2 8/10/2016 MyPastest

Session Progress

Responses Correct: 0

Responses Incorrect: 151

Responses Total: 151

Responses - % Correct: 0%

Blog (https://www.pastest.com/blog) About Pastest (https://www.pastest.com/about-us) Contact Us (https://www.pastest.com/contact-us) Help (https://www.pastest.com/help)

© Pastest 2016

https://mypastest.pastest.com/Secure/TestMe/Browser/429893#Top 2/2 8/10/2016 MyPastest

Back to Filters (/Secure/TestMe/Filter/429893/QA) Question 77 of 151

A 23-year-old man who lives with his male partner consults you for an opinion. He has had anal discharge and pruritus for the past 3 days. There are also some symptoms of dysuria. A urethral smear reveals intracellular diplococci. Which one of the following is the most likely infective agent to fit with this clinical picture?

A Neisseria gonorrhoeae

B Chlamydia trachomatis

C Treponema pallidum

D Herpes simplex-type 1

E Herpes simplex-type 2

Explanation

Gonorrhoea - Definition

Gonorrhoea is a sexually transmitted bacterial infection

Causative organism

The cause is the Gram-negative intracellular diplococcus Neisseria gonorrhoeae

Clinical presentation

It may manifest with urethritis, cervicitis, salpingitis or anorectal symptoms Symptoms in men may be severe and include purulent discharge from the anterior urethra and dysuria, with rectal discharge where anal intercourse has taken place Symptoms in women are often mild, with urethritis and cervicitis occurring a few days after exposure In around 20% of cases, uterine invasion may occur with of endometritis or salpingitis https://mypastest.pastest.com/Secure/TestMe/Browser/429893#Top 1/3 8/10/2016 MyPastest Bartholin’s glands may become inflamed

Diagnosis

The presence of N. gonorrhoeae in male urethral smears is diagnostic, although there is a false-negative rate of 60–70% in samples from women Gonococci require culture in anaerobic media in an increased carbon dioxide environment Patients should of course also receive screening for other sexually transmitted diseases

Treatment

First line treatment of choice is a single IM dose of ceftriaxone, with quinolone antibiotics the alternative, but local protocols should be referred to

2653

Next Question

Previous Question Tag Question Feedback End Review

Difficulty: Easy

Peer Responses

Session Progress

Responses Correct: 0

Responses Incorrect: 151

Responses Total: 151

Responses - % Correct: 0%

Blog (https://www.pastest.com/blog) About Pastest (https://www.pastest.com/about-us) Contact Us (https://www.pastest.com/contact-us) Help (https://www.pastest.com/help) © Pastest 2016

https://mypastest.pastest.com/Secure/TestMe/Browser/429893#Top 2/3 8/10/2016 MyPastest

https://mypastest.pastest.com/Secure/TestMe/Browser/429893#Top 3/3 8/10/2016 MyPastest

Back to Filters (/Secure/TestMe/Filter/429893/QA) Question 78 of 151

A young man presents with a round, slowly enlarging erythema on his thigh. He also complains of joint discomfort and fatigue. Lyme disease is suspected. Which one of the following options is the most appropriate laboratory test to confirm this diagnosis?

A Erythrocyte sedimentation rate

B AntiBorrelia burgdorferi titre

C Anti-nuclear antibodies

D Culture of joint fluids

E Blood cultures

Explanation

Serological testing in borreliosis

In patients with suspected extracutaneous Lyme borreliosis, serological testing is essential to support the diagnosis Patients with extracutaneous Lyme borreliosis almost always have diagnostic serum antibodies to Borrelia burgdorferi, except for some patients with early VIIth nerve palsy or occasional patients in whom antibodies to Borrelia burgdorferi are only present in cerebrospinal fluid A two-step approach to serological diagnosis has recently been proposed in the United States (and is being studied in Europe) to increase the specificity of a positive test A positive or equivocal first-stage test (usually an enzyme-linked immunosorbent assay (ELISA) or indirect immunofluorescence assay (IFA)) is followed on the same serum sample by a second-stage test (immunoblot) Two-step testing, however, is not indicated for those with little or no clinical evidence of Lyme borreliosis because of a low positive-predictive value Since IgM and IgG antibodies to Borrelia burgdorferi may persist in the serum for years after clinical recovery, serology has no role in measuring the response to treatment The culture of Borrelia burgdorferi is a highly insensitive diagnostic technique for https://mypastest.pastest.com/Secure/TestMe/Browser/429893#Top 1/2 8/10/2016 MyPastest patients with extracutaneous Lyme borreliosis, presumably because of the inaccessibility of tissues that contain the organism

1322

Next Question

Previous Question Tag Question Feedback End Review

Difficulty: Easy

Peer Responses

Session Progress

Responses Correct: 0

Responses Incorrect: 151

Responses Total: 151

Responses - % Correct: 0%

Blog (https://www.pastest.com/blog) About Pastest (https://www.pastest.com/about-us) Contact Us (https://www.pastest.com/contact-us) Help (https://www.pastest.com/help)

© Pastest 2016

https://mypastest.pastest.com/Secure/TestMe/Browser/429893#Top 2/2 8/10/2016 MyPastest

Back to Filters (/Secure/TestMe/Filter/429893/QA) Question 79 of 151

A 40-year-old patient presents with a slowly enlarging round erythema on her thigh. A few weeks later she develops a lymphocytic meningitis and polyneuritis. Which one of the following options is the most likely diagnosis?

A Coxsackievirus infection

B Lyme disease

C Herpes simplex virus infection

D Generalised candida infection

E Relapse of a varicella zoster infection

Explanation

Lyme disease Causative organism

Lyme borreliosis (also called Lyme disease) is an infection caused by the spirochaete Borrelia burgdorferi, which is transmitted to humans by the usually asymptomatic bite of certain ticks of the genus Ixodes

Epidemiology

Lyme borreliosis occurs equally in males and females, and affects people of all ages There is a bimodal age distribution, with the highest rates in children aged 5–9 years and adults over 30 years of age

Clinical presentation

Erythema migrans, the clinical hallmark of Lyme borreliosis, occurs in approximately 90% of patients with objective evidence of B. burgdorferi infection Typically, erythema migrans begins as a red macule or papule at the site of a tick bite that occurred 7–10 days earlier https://mypastest.pastest.com/Secure/TestMe/Browser/429893#Top 1/2 8/10/2016 MyPastest The rash expands over days to weeks Central clearing may or may not be present Secondary cutaneous lesions may develop after haematogenous spread of spirochaetes Systemic complaints in patients with erythema migrans include fatigue, myalgia, arthralgia, headache, fever and/or chills, as well as a stiff neck

Differential diagnosis

Erythema migrans must be distinguished from local tick-bite reactions, tinea, insect and spider bites, bacterial cellulitis, and plant dermatitis

1296

Next Question

Previous Question Tag Question Feedback End Review

Difficulty: Easy

Peer Responses

Session Progress

Responses Correct: 0

Responses Incorrect: 151

Responses Total: 151

Responses - % Correct: 0%

Blog (https://www.pastest.com/blog) About Pastest (https://www.pastest.com/about-us) Contact Us (https://www.pastest.com/contact-us) Help (https://www.pastest.com/help) © Pastest 2016

https://mypastest.pastest.com/Secure/TestMe/Browser/429893#Top 2/2 8/10/2016 MyPastest

Back to Filters (/Secure/TestMe/Filter/429893/QA) Question 80 of 151

A 44-year-old African woman presents to the clinic. She is known to be HIV positive, and has been visiting her son who is a student in the UK. He has attended the clinic with her as she has become increasingly tired, drowsy and intermittently confused over the past few weeks. Ophthalmoscopy reveals evidence of choroidoretinitis. Her CD4 count is noted to be 10/mm3 (normal >500) . Contrast CT brain reveals ring-enhancing lesions. CSF reveals mononuclear pleocytosis and elevated protein.

Which one of the following is the most likely diagnosis?

A Tuberculous meningitis

B CMV encephalitis

C Pneumocystis jirovecii pneumonia

D Cryptococcal infection

E Toxoplasmosis

Explanation

Toxoplasmosis Clinical, radiological and laboratory features

The combination of choroidoretinitis, gradual onset confusion, cerebrospinal fluid (CSF) mononuclear pleocytosis and enhancing lesions on computed tomography (CT) scan in a patient with acquired immune deficiency syndrome (AIDS) is typical of toxoplasmosis In immunocompromised individuals presentation usually represents reactivation of previous infection

Treatment and prognosis

Patients should be treated with high-dose pyrimethamine and sulfadiazine therapy for a period of 6 weeks, followed by lifelong dual suppressive therapy at lower doses Unfortunately relapse is frequent in patients who are immunocompromised

18603 https://mypastest.pastest.com/Secure/TestMe/Browser/429893#Top 1/2 8/10/2016 MyPastest 18603

Next Question

Previous Question Tag Question Feedback End Review

Difficulty: Easy

Peer Responses

Session Progress

Responses Correct: 0

Responses Incorrect: 151

Responses Total: 151

Responses - % Correct: 0%

Blog (https://www.pastest.com/blog) About Pastest (https://www.pastest.com/about-us) Contact Us (https://www.pastest.com/contact-us) Help (https://www.pastest.com/help) © Pastest 2016

https://mypastest.pastest.com/Secure/TestMe/Browser/429893#Top 2/2 8/10/2016 MyPastest

Back to Filters (/Secure/TestMe/Filter/429893/QA) Question 81 of 151

A 21-year-old student vet presents to the Emergency Department for review. She was forced to return early from an attachment on a farm. She has been feeling increasingly unwell, with a 10-day history of malaise and bloody diarrhoea. Now she has noticed swelling of her legs and petechial haemorrhages. On examination she was very pale with peripheral oedema and puffiness around her face. There was pulmonary oedema, tachycardia and elevated blood pressure at 165/95 mmHg. Investigations:

Hb 8.2 g/dl

White cell count 12.9 × 109 /l

Neutrophils 7.9 × 109 /l

Platelets 32 × 109 /l

PT 11 s

APTT 34 s

Fibrinogen 4g/dl

Sodium 140 mmol/l

Potassium 6.0 mmol/l

Urea 35 mmol/l

Creatinine 420 μmol/l

Albumin 25g/l

Urine dipstick shows blood and protein

Which one of the following investigations is most likely to provide the diagnosis in this case?

A Trans-oesophageal echocardiogram

B Stool culture

C Urine cytology

https://mypastest.pastest.com/Secure/TestMe/Browser/429893#Top 1/3 8/10/2016 MyPastest D Renal tract ultrasound scan

E Urine microscopy

Explanation

Haemolytic uraemic syndrome History and diagnosis

The major clue in this question is the fact that the patient works as a student vet and has recently completed a farm placement The clinical picture seen here is highly suggestive of haemolytic uraemic syndrome (HUS)

Aetiopathogenesis

In North America and Western Europe over 70% of cases are due to Escherichia coli 157 This particular E. coli subtype is thought to alter endothelial cell adhesion properties leading to microvascular thrombosis Prodromal gastroenteritis is seen in over 80% of HUS patients, acute renal failure is said to be seen in 97% of patients, other prominent features include pallor, hypertension and oedema as seen here

Management and prognosis

Management is with supportive care including renal replacement therapy which may be required for some days Antibiotics are often prescribed although there are few randomised controlled trials to absolutely confirm a benefit Over time, 85% of patients are said to recover their renal function

14965

Next Question

Previous Question Tag Question Feedback End Review

Difficulty: Average

Peer Responses

https://mypastest.pastest.com/Secure/TestMe/Browser/429893#Top 2/3 8/10/2016 MyPastest

Session Progress

Responses Correct: 0

Responses Incorrect: 151

Responses Total: 151

Responses - % Correct: 0%

Blog (https://www.pastest.com/blog) About Pastest (https://www.pastest.com/about-us) Contact Us (https://www.pastest.com/contact-us) Help (https://www.pastest.com/help)

© Pastest 2016

https://mypastest.pastest.com/Secure/TestMe/Browser/429893#Top 3/3 8/10/2016 MyPastest

Back to Filters (/Secure/TestMe/Filter/429893/QA) Question 82 of 151

An intravenous drug user sees his doctor with a history of lethargy. LFTs demonstrate an ALT of 80 IU/l and a bilirubin of 18 μmol/l. Serological tests for hepatitis viruses show HepB surface Ag-positive and anti-HepB core IgG Ab-positive. Which one of the following options is the most appropriate interpretation of the serological studies?

A Acute hepatitis B

B Carrier of hepatitis B

C Recovery from past hepatitis B infection

D Requires hepatitis B vaccine

E Vaccinated from hepatitis B

Explanation

Serology of hepatitis B

In acute hepatitis B, the surface antigen appears first followed by the e antigen, which indicates high infectivity The transaminases (AST/ALT) may rise 2-log fold The persistence of the surface antigen indicates a carrier status, which occurs in 10% of cases IgG antibodies to the core antigen imply past infection, while antibodies to the surface antigen alone indicate seroconversion to the vaccine

795

Next Question

Previous Question Tag Question Feedback End Review

Difficulty: Average https://mypastest.pastest.com/Secure/TestMe/Browser/429893#Top 1/2 8/10/2016 MyPastest

Peer Responses

Session Progress

Responses Correct: 0

Responses Incorrect: 151

Responses Total: 151

Responses - % Correct: 0%

Blog (https://www.pastest.com/blog) About Pastest (https://www.pastest.com/about-us) Contact Us (https://www.pastest.com/contact-us) Help (https://www.pastest.com/help) © Pastest 2016

https://mypastest.pastest.com/Secure/TestMe/Browser/429893#Top 2/2 8/10/2016 MyPastest

Back to Filters (/Secure/TestMe/Filter/429893/QA) Question 83 of 151

A 15-year-old girl who has recently returned from a holiday in India complains of a sore throat. On examination she has a severe pharyngitis and a thick exudate. A throat swab reveals diphtheria. Which one of the following options is the next most appropriate action?

A Examine the cerebrospinal fluid (CSF)

B Blood cultures

C Ceftriaxone

D Antitoxin

E Hydrocortisone

Explanation

Diphtheria Diagnosis

In areas where diphtheria is relatively common it should be suspected in any child with exudate in the throat Clinical diagnosis is much more difficult where diphtheria is rare

Treatment

If the exudate is thick and discoloured the child should be given antitoxin

Differential diagnosis

The differential diagnosis includes infectious mononucleosis, streptococcal or viral tonsillitis, peritonsillar abscess, oral thrush, and leukaemia and other blood dyscrasias The bull-neck of malignant diphtheria may be mistaken for mumps In adults, secondary syphilis can sometimes cause a glairy (resembling egg-white) exudate on the tonsils, and may be accompanied by rash and laryngitis

5564 https://mypastest.pastest.com/Secure/TestMe/Browser/429893#Top 1/2 8/10/2016 MyPastest 5564

Next Question

Previous Question Tag Question Feedback End Review

Difficulty: Average

Peer Responses

Session Progress

Responses Correct: 0

Responses Incorrect: 151

Responses Total: 151

Responses - % Correct: 0%

Blog (https://www.pastest.com/blog) About Pastest (https://www.pastest.com/about-us) Contact Us (https://www.pastest.com/contact-us) Help (https://www.pastest.com/help) © Pastest 2016

https://mypastest.pastest.com/Secure/TestMe/Browser/429893#Top 2/2 8/10/2016 MyPastest

Back to Filters (/Secure/TestMe/Filter/429893/QA) Question 84 of 151

A 40-year-old man presents with a swollen erythematous finger and right axillary tenderness, 24 hours after being bitten by his pet dog. Which one of the following options is the best treatment?

A Metronidazole

B Aciclovir

C Fluconazole

D Flucloxacillin

E Co-amoxiclav

Explanation Infectious complications of dog bites Epidemiology

Dog bites become infected in 10% of cases

Causative organisms

The commonest organisms are anaerobic mouth flora and Pasteurella multocida Other organisms include Capnocytophaga spp and Streptococcus pyogenes

Pharmacotherapeutic considerations

Co-amoxiclav (a mixture of amoxicillin and clavulanic acid) is appropriate as it is effective against all the most likely organisms Herpes simplex infection is most unusual in cases of dog bites, though monkey bites often transmit this virus Although dogs harbour fungal infections including Blastomyces, Microsporidium and Trichosporon spp, these are dermatophytes and only produce superficial skin rashes, and therefore the antifungal agent, fluconazole, is not indicated in this patient https://mypastest.pastest.com/Secure/TestMe/Browser/429893#Top 1/2 8/10/2016 MyPastest Flucloxacillin is not active against anaerobic bacteria Metronidazole, a good antibiotic against anaerobic bacteria, does not cover aerobic Gram-negative organisms

301

Next Question

Previous Question Tag Question Feedback End Review

Difficulty: Average

Peer Responses

Session Progress

Responses Correct: 0

Responses Incorrect: 151

Responses Total: 151

Responses - % Correct: 0%

Blog (https://www.pastest.com/blog) About Pastest (https://www.pastest.com/about-us) Contact Us (https://www.pastest.com/contact-us) Help (https://www.pastest.com/help)

© Pastest 2016

https://mypastest.pastest.com/Secure/TestMe/Browser/429893#Top 2/2 8/10/2016 MyPastest

Back to Filters (/Secure/TestMe/Filter/429893/QA) Question 85 of 151

A 32-year-old woman presents to the Emergency Department with a fever, lower abdominal pain and foul smelling bloody vaginal discharge some 6 days after the birth of her second child by caesarean section. She was discharged some 3 days earlier under continued follow up of the community midwife and had an area of erythema at one end of her wound before she left the hospital. Examination reveals a pyrexia of 38.6°C, and a BP of 90/60 mmHg. There is a generalised maculopapular rash and evidence of a wound infection and underlying abscess affecting the right hand side of the scar. Her abdomen is generally tender across the lower half, and she is drowsy and agitated. Investigations:

Hb 10.2 g/dl

WCC 14.5 x109 /l

PLT 64 x109 /l

PT 19.1 s

APTT 34.1 s

Na+ 138 mmol/l

K+ 4.3 mmol/l

Creatinine 158 micromol/l

Urea 14.2 mmol/l

USS abdomen evidence of fluid collection under right side of caesarean scar

You begin fluid resuscitation Which of the following is the most important intervention?

A Activated protein C

B IV corticosteroids

C Incision and drainage of the wound abscess

https://mypastest.pastest.com/Secure/TestMe/Browser/429893#Top 1/3 8/10/2016 MyPastest

D Platelet transfusion

E IV Ceftriaxone

Explanation The answer is Incision and drainage of the wound abscess -

This patient has evidence of toxic shock syndrome. Without drainage of the wound abscess even broad-spectrum antibiotics will be ineffective. Activated protein C was shown to have no benefit in the treatment of septic shock and was withdrawn in the majority of major countries. IV corticosteroids are also not recommended in the surviving sepsis guidelines where adequate fluid replacement is available. At a platelet count of 64 x109 /l, transfusion would not be recommended. http://link.springer.com/article/10.1007/s00134-012-2769-8#/page-1 (http://link.springer.com/article/10.1007/s00134-012-2769-8#/page-1) 38090

Next Question

Previous Question Tag Question Feedback End Review

Difficulty: Average

Peer Responses

Session Progress

Responses Correct: 0

Responses Incorrect: 151

Responses Total: 151

Responses - % Correct: 0%

Blog (https://www.pastest.com/blog) About Pastest (https://www.pastest.com/about-us) Contact Us (https://www.pastest.com/contact-us) Help (https://www.pastest.com/help) © Pastest 2016 https://mypastest.pastest.com/Secure/TestMe/Browser/429893#Top 2/3 8/10/2016 MyPastest

https://mypastest.pastest.com/Secure/TestMe/Browser/429893#Top 3/3 8/10/2016 MyPastest

Back to Filters (/Secure/TestMe/Filter/429893/QA) Question 86 of 151

A patient wishes to travel by plane having commenced on quadruple anti-tuberculous therapy 1 week ago for smear-positive pulmonary tuberculosis. The infection was acquired in the UK. The patient is HIV antibody-negative and has never received treatment for tuberculosis previously. Which of the following statements accords with British Thoracic Society guidelines?

A Await 2 weeks’ therapy and a negative sputum culture

B Await 2 weeks’ therapy and a negative sputum smear

C Normal chest X-ray prior to travel

D Wait until symptoms have resolved

E Wait until therapy is completed

Explanation

Pulmonary tuberculosis

Fully sensitive pulmonary tuberculosis should be no longer communicable after 2 weeks’ therapy Smear-negative disease is much less infectious and is an indication that the therapy is working HIV-positive patients have a slower response to treatment and previous failed therapy and country of acquisition (Eastern Europe and Russia in particular) are risk factors for multidrug-resistant disease Cultures may take 8 weeks to grow and symptoms often take several months to resolve The chest X-ray frequently shows scarring and calcification for life

773

Next Question

https://mypastest.pastest.com/Secure/TestMe/Browser/429893#Top 1/2 8/10/2016 MyPastest Previous Question Tag Question Feedback End Review

Difficulty: Average

Peer Responses

Session Progress

Responses Correct: 0

Responses Incorrect: 151

Responses Total: 151

Responses - % Correct: 0%

Blog (https://www.pastest.com/blog) About Pastest (https://www.pastest.com/about-us) Contact Us (https://www.pastest.com/contact-us) Help (https://www.pastest.com/help) © Pastest 2016

https://mypastest.pastest.com/Secure/TestMe/Browser/429893#Top 2/2 8/10/2016 MyPastest

Back to Filters (/Secure/TestMe/Filter/429893/QA) Question 87 of 151

A 23-year-old ex-intravenous drug user is seen in the medical outpatients’ clinic following a deep venous thrombosis 2 months ago for which he is on warfarin therapy. He has the following blood results: U&E, normal; albumin, 39 g/l; ALT, 170 U/l; GGT, 20 U/l; LDH, 500 U/l; bilirubin, 23 μmol/l; INR, 2.5; Hb, 13.3 g/dl; WCC, 4.2 × 1096 /l; platelets, 300 × 10 /l.

You suggest that he should be screened for hepatitis B and C and HIV. The results subsequently come back as follows: HIV-1 and -2 antibody tests, negative; hepatitis B core IgG, positive; hepatitis B surface antigen, positive; hepatitis B e antigen, negative; hepatitis C IgG and PCR, negative.

Which one of the following tests is most likely to lead to the correct diagnosis with respect to his viral status?

A Hepatitis B DNA load

B Hepatitis D test

C VDRL

D AFP

E Hepatitis E test

Explanation

Hepatitis B

The patient here is quite likely to have the precore mutant form of hepatitis B This is hepatitis B, which, although it replicates actively, does not produce e antigen, either as a result of a mutation in the gene producing e antigen itself, or as a result of a mutation in one of the gene upregulators Hepatitis B DNA can be measured quantitatively, and a high level in the patient in the present case would be suggestive of precore mutant hepatitis B

590 https://mypastest.pastest.com/Secure/TestMe/Browser/429893#Top 1/2 8/10/2016 MyPastest

Next Question

Previous Question Tag Question Feedback End Review

Difficulty: Easy

Peer Responses

Session Progress

Responses Correct: 0

Responses Incorrect: 151

Responses Total: 151

Responses - % Correct: 0%

Blog (https://www.pastest.com/blog) About Pastest (https://www.pastest.com/about-us) Contact Us (https://www.pastest.com/contact-us) Help (https://www.pastest.com/help)

© Pastest 2016

https://mypastest.pastest.com/Secure/TestMe/Browser/429893#Top 2/2 8/10/2016 MyPastest

Back to Filters (/Secure/TestMe/Filter/429893/QA) Question 88 of 151

A patient is hospitalised with cavitary pulmonary tuberculosis. Combination chemotherapy is started and he is placed in isolation. Which one of the following offers the best description of when isolation can be discontinued?

A 2 weeks after onset of treatment

B For the duration of treatment

C Until the patient shows clinical improvement and has three negative sputum samples

D If there is improvement on the chest X-ray

E 3 months after onset of treatment

Explanation

Length of isolation

Criteria for discontinuation of respiratory isolation are:

treatment given for at least 14 days clinically responsive to therapy including improvement in fever and cough acid-fast smears of sputum negative on three occasions

Guidelines for taking patients out of isolation in the hospital are strict, and are intended to protect other vulnerable patients and hospital staff from any exposure to the disease

1321

Next Question

Previous Question Tag Question Feedback End Review

Difficulty: Average

https://mypastest.pastest.com/Secure/TestMe/Browser/429893#Top 1/2 8/10/2016 MyPastest Peer Responses

Session Progress

Responses Correct: 0

Responses Incorrect: 151

Responses Total: 151

Responses - % Correct: 0%

Blog (https://www.pastest.com/blog) About Pastest (https://www.pastest.com/about-us) Contact Us (https://www.pastest.com/contact-us) Help (https://www.pastest.com/help) © Pastest 2016

https://mypastest.pastest.com/Secure/TestMe/Browser/429893#Top 2/2 8/10/2016 MyPastest

Back to Filters (/Secure/TestMe/Filter/429893/QA) Question 89 of 151

A 46-year-old patient has been chronically infected with the hepatitis B virus for the past 8 years. He has an increased risk of developing which one of the following diseases?

A Coronary artery disease

B COPD

C Pancreatic cancer

D Hepatocellular cancer

E Malignant melanoma

Explanation

Hepatocellular cancer - Incidence

The incidence of hepatocellular cancer in patients with chronic hepatitis B is high, probably a 100-fold increase over non-infected controls

Complications

Most, but not all, patients with hepatocellular cancer will have cirrhosis

2694

Next Question

Previous Question Tag Question Feedback End Review

Difficulty: Easy

Peer Responses https://mypastest.pastest.com/Secure/TestMe/Browser/429893#Top 1/2 8/10/2016 MyPastest

Session Progress

Responses Correct: 0

Responses Incorrect: 151

Responses Total: 151

Responses - % Correct: 0%

Blog (https://www.pastest.com/blog) About Pastest (https://www.pastest.com/about-us) Contact Us (https://www.pastest.com/contact-us) Help (https://www.pastest.com/help)

© Pastest 2016

https://mypastest.pastest.com/Secure/TestMe/Browser/429893#Top 2/2 8/10/2016 MyPastest

Back to Filters (/Secure/TestMe/Filter/429893/QA) Question 90 of 151

A young farmer presents with a 10-day history of blurred vision in his left eye. He says that 2 weeks ago his left eye was damaged by vegetation. On examination there is an ulcerative lesion in the cornea, the base of which has a raised creamy infiltrate. The slough of the ulcer appears dry, with a feathery border surrounded by a yellow line of demarcation. There are a few satellite lesions as well. Which one of the following options is the most likely cause for his condition?

A Staphylococcus aureus

B Herpes simplex virus

C Chlamydia trachomatis

D Moraxella sp

E Aspergillus sp

Explanation

Differential diagnosis of corneal ulcers

The clinical features in the present case are highly suggestive of corneal ulceration caused by aspergillus infection Bacterial corneal ulcers would present with much more florid symptoms of pain, photophobia and lacrimation Dendritic ulcers are characteristic of herpes simplex virus infection Trachoma caused by Chlamydia trachomatis is usually bilateral and begins in the conjunctiva with marked follicular inflammation and scarring Ulcers due to infection with Moraxella spp are shallow, slightly infiltrated and often multiple, and may be accompanied by neuralgic pain in the face and head

3579

Next Question

https://mypastest.pastest.com/Secure/TestMe/Browser/429893#Top 1/2 8/10/2016 MyPastest

Previous Question Tag Question Feedback End Review

Difficulty: Difficult

Peer Responses

Session Progress

Responses Correct: 0

Responses Incorrect: 151

Responses Total: 151

Responses - % Correct: 0%

Blog (https://www.pastest.com/blog) About Pastest (https://www.pastest.com/about-us) Contact Us (https://www.pastest.com/contact-us) Help (https://www.pastest.com/help) © Pastest 2016

https://mypastest.pastest.com/Secure/TestMe/Browser/429893#Top 2/2 8/10/2016 MyPastest

Back to Filters (/Secure/TestMe/Filter/429893/QA) Question 91 of 151

During the World Cup, a 27-year-old Brazilian football player collapsed on the field. He is brought to the Emergency Department where he is found to have evidence of cardiac failure with cardiomegaly and bi-basal shadowing on chest X-ray. Which one of the following infections is most likely to be responsible?

A Trypanosoma cruzi

B Trypanosoma brucei

C Influenza

D Mycoplasma pneumoniae

E Haemophilus aphrophilus

Explanation

Trypanosomiasis Causative organism

Trypanosoma cruzi, a protozoan parasite, is spread by reduviid bugs; these are known as ‘kissing bugs’ as they often bite around the lips during the night They frequently live in thatched roofs and occur in much of Central and South America

Complications

After an asymptomatic period, often of many years, patients may develop cardiomyopathy and cardiac failure with arrhythmias The illness may also cause mega-oesophagus and megacolon Acute Chagas’ disease may present with periocular swelling (Romana’s sign) and a chagoma (area of localised swelling and regional lymphadenopathy at the inoculation site) Meningitis and myocarditis may also present acutely, but these are very rare manifestations of the illness

https://mypastest.pastest.com/Secure/TestMe/Browser/429893#Top 1/2 8/10/2016 MyPastest Treatments such as nifurtimox and benznidazole are often ineffective and are also toxic

608

Next Question

Previous Question Tag Question Feedback End Review

Difficulty: Average

Peer Responses

Session Progress

Responses Correct: 0

Responses Incorrect: 151

Responses Total: 151

Responses - % Correct: 0%

Blog (https://www.pastest.com/blog) About Pastest (https://www.pastest.com/about-us) Contact Us (https://www.pastest.com/contact-us) Help (https://www.pastest.com/help) © Pastest 2016

https://mypastest.pastest.com/Secure/TestMe/Browser/429893#Top 2/2 8/10/2016 MyPastest

Back to Filters (/Secure/TestMe/Filter/429893/QA) Question 92 of 151

A 22-year-old man has returned from a period travelling, during which he visited central/sub- Saharan Africa. He presents to the GP complaining of urinary frequency, perineal itching and inflammation and also of painless haematuria. Which one of the following diagnoses fits best with this clinical picture?

A Infection with Schistosoma mansoni

B Infection with Schistosoma japonicum

C Infection with Schistosoma haematobium

D Syphilis

E Gonorrhoea

Explanation

Schistosomiasis Epidemiology and transmission of infection

S. haematobium infection occurs across a wide belt of central Africa It occurs after swimming in water containing the water-snail vector for the S. haematobium water fluke The final destination for S. haematobium after infection is the vesicular plexus S. mansoni, S. japonicum have the mesenteric veins as their final destination, and hence cause bowel symptoms

Clinical features

After a symptom-free period of 3–4 weeks, systemic allergic features may develop, including fever, rash, myalgia and pneumonitis The commonest presenting symptom though for S. haematobium infection is painless terminal haematuria As bladder inflammation progresses there may be urinary frequency and groin pain

https://mypastest.pastest.com/Secure/TestMe/Browser/429893#Top 1/2 8/10/2016 MyPastest Obstructive uropathy leading to hydronephrosis and renal failure may develop

Diagnosis

Diagnosis is via a filtered mid-day urine sample to look for parasites

Treatment

Praziquantel (40 mg/kg) as a single dose is the chemotherapeutic agent of choice, but re-infection is common in endemic areas

2662

Next Question

Previous Question Tag Question Feedback End Review

Difficulty: Average

Peer Responses

Session Progress

Responses Correct: 0

Responses Incorrect: 151

Responses Total: 151

Responses - % Correct: 0%

Blog (https://www.pastest.com/blog) About Pastest (https://www.pastest.com/about-us) Contact Us (https://www.pastest.com/contact-us) Help (https://www.pastest.com/help)

© Pastest 2016

https://mypastest.pastest.com/Secure/TestMe/Browser/429893#Top 2/2 8/10/2016 MyPastest

Back to Filters (/Secure/TestMe/Filter/429893/QA) Question 93 of 151

A 19-year-old student returns from a field trip abroad complaining of fever, malaise and worsening cough. He also has a headache and sore throat. On examination there is a mild pharyngitis and wheeze on auscultation of the chest. He is pyrexial, 38 °C, and his blood pressure is 110/70 mmHg, with a pulse of 105/min.

Investigations:

Hb 13.7 g/dl

WCC 8.9 × 109 /l

PLT 202 × 109 /l

Na+ 134 mmol/l

K+ 4.8 mmol/l

Creatinine 100 μmol/l

CXR – Extensive bilateral consolidation

PCR – Positive for Mycoplasma

You elect to begin treatment with oral erythromycin, but he returns 2 days later complaining of severe diarrhoea. Which one of the following options would be the most appropriate alternative antibiotic?

A Penicillin V

B Ciprofloxacin

C Clarithromycin

D Doxycycline

E Co-amoxiclav

https://mypastest.pastest.com/Secure/TestMe/Browser/429893#Top 1/2 8/10/2016 MyPastest Explanation

Mycoplasma infection

Given that the patient in the present case has experienced diarrhoea on erythromycin, he may experience similar symptoms with clarithromycin Mycoplasma is not sensitive to penicillins or ciprofloxacin; this means that doxycycline is the most appropriate option for the patient in the present case Debate exists as to the length of antibiotic course required, but most clinicians give 10 days to 2 weeks of therapy

20476

Next Question

Previous Question Tag Question Feedback End Review

Difficulty: Average

Peer Responses

Session Progress

Responses Correct: 0

Responses Incorrect: 151

Responses Total: 151

Responses - % Correct: 0%

Blog (https://www.pastest.com/blog) About Pastest (https://www.pastest.com/about-us) Contact Us (https://www.pastest.com/contact-us) Help (https://www.pastest.com/help)

© Pastest 2016

https://mypastest.pastest.com/Secure/TestMe/Browser/429893#Top 2/2 8/10/2016 MyPastest

Back to Filters (/Secure/TestMe/Filter/429893/QA) Question 94 of 151

A 20-year-old student presents to the Emergency Department with fever and difficulty swallowing. On examination she has enlarged tonsils, palpable submandibular lymph nodes and hepatosplenomegaly. Laboratory examination reveals a threefold upper limit elevation of the transaminases and lymphocytosis with atypical monocytes. Which one of the following options is the most appropriate next step in reaching a diagnosis?

A Anti-streptolysin-O titre

B Bone marrow examination

C Abdominal ultrasound

D Screening for hepatitis

E Serologic test for Epstein–Barr virus

Explanation

Infectious mononucleosis

The patient in the present case has infectious mononucleosis

Serology

A rapid screening test (Monospot test) can be used to detect the presence of heterophil antibodies in her serum Although these heterophil antibodies are not directed against viral-coded proteins they are present in up to 85% of acute infectious mononucleosis sera Cases of Monospot-negative infectious mononucleosis tend to be those outside the classic 15–25 year age range, and false-positive tests may occur in pregnancy and autoimmune disease However, the presence of serum IgM antibodies to Epstein–Barr virus-capsid antigen (VCA) is diagnostic of infectious mononucleosis

Other laboratory findings https://mypastest.pastest.com/Secure/TestMe/Browser/429893#Top 1/2 8/10/2016 MyPastest

An additional important feature of infectious mononucleosis is the presence of a lymphocytosis of up to 15 × 109 /l, with the majority of cells having an atypical morphology Anti-streptolysin-O titre is used for the diagnosis of Streptococcus pyogenes

1313

Next Question

Previous Question Tag Question Feedback End Review

Difficulty: Easy

Peer Responses

Session Progress

Responses Correct: 0

Responses Incorrect: 151

Responses Total: 151

Responses - % Correct: 0%

Blog (https://www.pastest.com/blog) About Pastest (https://www.pastest.com/about-us) Contact Us (https://www.pastest.com/contact-us) Help (https://www.pastest.com/help) © Pastest 2016

https://mypastest.pastest.com/Secure/TestMe/Browser/429893#Top 2/2 8/10/2016 MyPastest

Back to Filters (/Secure/TestMe/Filter/429893/QA) Question 95 of 151

A 45-year-old man presents with a 1-week history of progressive breathlessness. He gives a history of past intravenous drug use and is hepatitis C antibody-positive. He desaturates with minimal exertion with an oxygen saturation of 75% on air. He is lymphopenic and a chest X- ray demonstrates subtle bilateral interstitial shadowing. Which one of the following options is the most likely cause of his breathlessness?

A Congestive cardiac failure

B Cytomegalovirus pneumonia

C Pneumocystis jirovecii pneumonia

D Pulmonary aspergillosis

E Streptococcus pneumoniae pneumonia

Explanation

Immunosuppression-associated pneumonia Aetiology

The present case has strong indicators for HIV infection, given the risk factors of intravenous drug use and the use of contaminated needles, as indicated by co-infection with hepatitis C and a lymphopenia Severe immunosuppression with a CD4 count less than 200 cell/μl carries a significantly increased risk of infection with Pneumocystis jirovecii (new name for Pneumocystis carinii)

desaturation with exercise and bilateral interstitial shadowing are features of this infection

Overall, the commonest cause of community-acquired pneumonia is Streptococcus pneumoniae, though typically lobar consolidation is seen Aspergillosis and cytomegalovirus are rarer causes of pneumonia in the immunosuppressed, the former may lead to cavitation with an aspergilloma https://mypastest.pastest.com/Secure/TestMe/Browser/429893#Top 1/2 8/10/2016 MyPastest Differential diagnosis

Congestive cardiac failure is an important non-infectious differential diagnosis

760

Next Question

Previous Question Tag Question Feedback End Review

Difficulty: Easy

Peer Responses

Session Progress

Responses Correct: 0

Responses Incorrect: 151

Responses Total: 151

Responses - % Correct: 0%

Blog (https://www.pastest.com/blog) About Pastest (https://www.pastest.com/about-us) Contact Us (https://www.pastest.com/contact-us) Help (https://www.pastest.com/help) © Pastest 2016

https://mypastest.pastest.com/Secure/TestMe/Browser/429893#Top 2/2 8/10/2016 MyPastest

Back to Filters (/Secure/TestMe/Filter/429893/QA) Question 96 of 151

A man attends the clinic with an itchy rash over his foot. He has recently returned from a beach holiday in Jamaica. On examination there is a serpiginous tract at the site of the rash. He has eosinophilia. Which one of the following is the likely aetiological agent?

A Ascaris lumbricoides

B Enterobius vermicularis

C Ancylostoma braziliense

D Trichinella spiralis

E Trichuris trichiura

Explanation

Hookworm infection

The patient in this question has cutaneous larva migrans Along with Enterobius vermicularis, Trichuris trichiura and Trichinella spiralis, all the organism listed in the question are nematode worms

Pathological features

The filariform larvae of human, dog and cat hookworms can burrow into exposed skin and are usually acquired from walking barefoot on a contaminated beach The larvae can be seen to migrate through the skin (cutaneous larva migrans) and occasionally secondary bacterial infection may occur

Complications

The most common cause of cutaneous larva migrans is Ancylostoma braziliense, which is a dog and cat hookworm found in the USA, Central America, South America and the Caribbean Other reported, less common, animal roundworms that cause cutaneous larva migrans https://mypastest.pastest.com/Secure/TestMe/Browser/429893#Top 1/2 8/10/2016 MyPastest include Ancylostoma tubaeforme, Ancylostoma caninum, Ancylostoma ceylanicum and Uncinaria stenocephala (ie dog hookworms); Bunostomum phlebotomum (ie cattle hookworm); Gnathostoma species (ie cat, dog, and pig roundworms); Capillaria species (ie whipworms found in rodents, cats, dogs, and poultry); and Strongyloides myopotami, Strongyloides papillosus and Strongyloides westeri (found in the small intestine of mammals). Nematodes that use a human as a definitive host (such as Ancylostoma duodenale, Strongyloides stercoralis, and Necator americanus) are rare causes of cutaneous larva migrans. A. duodenale and N americanus usually cause ground itch. S stercoralis is usually associated with larva currens. S. stercoralis can lead to pneumonitis and enteritis and hyperinfection in the immunosuppressed

781

Next Question

Previous Question Tag Question Feedback End Review

Difficulty: Difficult

Peer Responses

Session Progress

Responses Correct: 0

Responses Incorrect: 151

Responses Total: 151

Responses - % Correct: 0%

Blog (https://www.pastest.com/blog) About Pastest (https://www.pastest.com/about-us) Contact Us (https://www.pastest.com/contact-us) Help (https://www.pastest.com/help) © Pastest 2016

https://mypastest.pastest.com/Secure/TestMe/Browser/429893#Top 2/2 8/10/2016 MyPastest

Back to Filters (/Secure/TestMe/Filter/429893/QA) Question 97 of 151

A 40-year-old man is admitted with a cerebral abscess. After completing a course of intravenous antibiotics he is converted to oral antibiotics prior to discharge. Which one of the following antibiotics provides the best and broadest cover for anaerobic organisms that may be responsible for this presentation?

A Amoxicillin

B Ciprofloxacin

C Co-amoxiclav

D Flucloxacillin

E Trimethoprim

Explanation

Antibiotics in anaerobic infections

Amoxicillin is a penicillin-based antibiotic with excellent bioavailability and cover against Gram-positive organisms but no activity against anaerobes However, the addition of clavulanic acid in co-amoxiclav will provide the additional anaerobic cover required Ciprofloxacin has activity against Gram-negative organisms, such as many enteric bacteria Similarly, trimethoprim has activity against Gram-negatives but with a narrower spectrum and is mainly used for the first-line treatment of UTIs Flucloxacillin is predominantly used as an anti-staphylococcal agent

791

Next Question

Previous Question Tag Question Feedback End Review https://mypastest.pastest.com/Secure/TestMe/Browser/429893#Top 1/2 8/10/2016 MyPastest

Difficulty: Average

Peer Responses

Session Progress

Responses Correct: 0

Responses Incorrect: 151

Responses Total: 151

Responses - % Correct: 0%

Blog (https://www.pastest.com/blog) About Pastest (https://www.pastest.com/about-us) Contact Us (https://www.pastest.com/contact-us) Help (https://www.pastest.com/help) © Pastest 2016

https://mypastest.pastest.com/Secure/TestMe/Browser/429893#Top 2/2 8/10/2016 MyPastest

Back to Filters (/Secure/TestMe/Filter/429893/QA) Question 98 of 151

Which one of the following statements with regard to Kaposi’s sarcoma (KS) is true?

A The incidence of KS in AIDS has been in progressive decline since the early 1990s

B There is a 400 times increased risk of KS among patients with congenital immune deficiency

C Respiratory tract disease is regarded as the most common initial manifestation

D In recent years, in the UK the incidence of KS in AIDS in heterosexual men has exceeded that in homosexual and bisexual men

E KS is never encountered in organ transplant patients

Explanation

Kaposi’s sarcoma Epidemiology

While the incidence of Kaposi’s sarcoma (KS) as an acquired immune deficiency syndrome (AIDS)-defining event has steadily declined from > 30% in the mid-1980s to < 15% by the mid-1990s, it remains the most common AIDS-related malignancy With the advent of highly active anti-retroviral therapy, the prevalence of KS and other opportunistic infections and malignancies has significantly dropped over the past 2 years It remains to be determined whether the incidence of KS will increase again as more patients fail anti-retroviral therapy Reports have documented that KS is ten times more common in homosexual and bisexual men than in heterosexual men infected with HIV AIDS-related KS remains relatively uncommon among heterosexual intravenous drug abusers, women and those acquiring HIV infections through blood products, suggesting a sexually transmitted agent has a causative role Human herpes virus 8 (HHV-8) genomic sequences have been identified using polymerase chain reaction in more than 90% of all types of KS lesions (including https://mypastest.pastest.com/Secure/TestMe/Browser/429893#Top 1/2 8/10/2016 MyPastest epidemic and endemic forms), suggesting a causative role. Co-infection with HIV and HHV-8 (Kaposi sarcoma-associated herpesvirus (KSHV1)), is commoner in homosexual men The importance of an altered immune system in the pathogenesis of KS is underscored by its 400 times increased risk in organ transplant patients

Clinical presentation

Although AIDS-associated KS may involve any organ, mucocutaneousdisease is the most common initial manifestation

1638

Next Question

Previous Question Tag Question Feedback End Review

Difficulty: Average

Peer Responses

Session Progress

Responses Correct: 0

Responses Incorrect: 151

Responses Total: 151

Responses - % Correct: 0%

Blog (https://www.pastest.com/blog) About Pastest (https://www.pastest.com/about-us) Contact Us (https://www.pastest.com/contact-us) Help (https://www.pastest.com/help) © Pastest 2016

https://mypastest.pastest.com/Secure/TestMe/Browser/429893#Top 2/2 8/10/2016 MyPastest

Back to Filters (/Secure/TestMe/Filter/429893/QA) Question 99 of 151

A sexually active 19-year-old Nigerian man presents with a 2-day history of small, painful ulcers on the glans penis and bilateral tender inguinal lymphadenopathy. Gram stain of penile discharge reveals no organisms. Which one of the following options is the most likely diagnosis?

A Herpes simplex virus

B Lymphogranuloma venereum

C Syphilis

D Chancroid

E Granuloma inguinale

Explanation

Genital herpes

The presentation in the question is typical of primary genital HSV Ulcers are painless in syphilis The others listed in the questions (lymphogranuloma venereum, chancroid, granuloma inguinale) tend to present with a longer history Chancroid also produces painful genital ulcers and lymphadenopathy but is much rarer than HSV infection It is also more likely to be associated with unilateral lymphadenopathy, and you might expect to see gram negative rods on microscopy of collected samples

3721

Next Question

Previous Question Tag Question Feedback End Review https://mypastest.pastest.com/Secure/TestMe/Browser/429893#Top 1/2 8/10/2016 MyPastest

Difficulty: Difficult

Peer Responses

Session Progress

Responses Correct: 0

Responses Incorrect: 151

Responses Total: 151

Responses - % Correct: 0%

Blog (https://www.pastest.com/blog) About Pastest (https://www.pastest.com/about-us) Contact Us (https://www.pastest.com/contact-us) Help (https://www.pastest.com/help) © Pastest 2016

https://mypastest.pastest.com/Secure/TestMe/Browser/429893#Top 2/2 8/10/2016 MyPastest

Back to Filters (/Secure/TestMe/Filter/429893/QA) Question 100 of 151

A 40-year-old man is admitted with a 5-day history of fevers, abdominal pain and diarrhoea 1 week after returning from a business trip to Rome. He is noted to have a dry cough and a temperature of 40 °C. Blood tests show a normal total white cell count with a lymphopenia and his serum Na concentration is 125 mmol/l. A chest X-ray shows some consolidation in the right lower lobe. A presumptive diagnosis is made and the patient started on antibiotics. Which one of the following tests would most rapidly confirm the diagnosis?

A Blood cultures

B Gram stain on bronchoalveolar lavage

C Serology

D Urinary antigen test

E Urine MCS

Explanation

Legionnaire’s disease

The most likely diagnosis in the present patient is legionnaire’s disease

Causative organism

Legionella pneumophila, although a Gram-negative organism, is poorly detected by Gram stain and may culture after prolonged incubation in blood cultures

Clinical features

The incubation period is between 2 and 10 days with a flu-like prodrome, high fevers, dry cough and often gastrointestinal symptoms Hyponatraemia and lymphopenia without leukocytosis are typical Outbreaks can occur and cases may be traced to hotels and institutions

Further investigations https://mypastest.pastest.com/Secure/TestMe/Browser/429893#Top 1/2 8/10/2016 MyPastest

The Legionella urinary antigen test is highly specific and sensitive if carried out in the first 10 days of illness Thereafter a rising antibody titre may confirm the diagnosis: often, however, this does not rise until some weeks after the onset

778

Next Question

Previous Question Tag Question Feedback End Review

Difficulty: Easy

Peer Responses

Session Progress

Responses Correct: 0

Responses Incorrect: 151

Responses Total: 151

Responses - % Correct: 0%

Blog (https://www.pastest.com/blog) About Pastest (https://www.pastest.com/about-us) Contact Us (https://www.pastest.com/contact-us) Help (https://www.pastest.com/help) © Pastest 2016

https://mypastest.pastest.com/Secure/TestMe/Browser/429893#Top 2/2 8/10/2016 MyPastest

Back to Filters (/Secure/TestMe/Filter/429893/QA) Question 101 of 151

A 45-year-old Christian missionary returned from a tropical assignment in Central America 10 months ago. Since then he has undertaken no foreign travel. He presents with fever, malaise and rigors. On examination he has mild jaundice, liver tenderness and a palpable spleen. Blood testing reveals that he is anaemic. Which one of the following infective agents fits best with this clinical picture?

A Plasmodium falciparum

B Plasmodium malariae

C Tropical sprue

D Plasmodium vivax

E Plasmodium ovale

Explanation

Malaria

This man in the present case has malaria

Causative organisms

Two subtypes of Plasmodium (P. vivax and P. ovale) have the capacity to lie dormant in the liver as hypnozoites Plasmodium vivax infection usually occurs in Central America or the Indian subcontinent Plasmodium ovale infection usually occurs in Africa Plasmodium malariae is the rarest subtype, existing in most areas, but especially in Central and West Africa

Diagnosis

The diagnosis of P. vivax malaria is based upon identification of so-called ‘Signet ring’ https://mypastest.pastest.com/Secure/TestMe/Browser/429893#Top 1/2 8/10/2016 MyPastest trophozoites on a blood film Blood samples should be collected over 3 days and examined by a technician experienced in looking for malaria parasites

Treatment and prevention

Chloroquine treatment is effective for red blood cell-phase organisms, but the addition of primaquine is necessary for eradication of the hepatic-phase organisms Malaria prophylaxis is best, but resistance patterns do alter and so a specialist information source should be consulted to determine the best prophylactic treatment

2652

Next Question

Previous Question Tag Question Feedback End Review

Difficulty: Difficult

Peer Responses

Session Progress

Responses Correct: 0

Responses Incorrect: 151

Responses Total: 151

Responses - % Correct: 0%

Blog (https://www.pastest.com/blog) About Pastest (https://www.pastest.com/about-us) Contact Us (https://www.pastest.com/contact-us) Help (https://www.pastest.com/help) © Pastest 2016

https://mypastest.pastest.com/Secure/TestMe/Browser/429893#Top 2/2 8/10/2016 MyPastest

Back to Filters (/Secure/TestMe/Filter/429893/QA) Question 102 of 151

A Nigerian student in the UK presents with haemoptysis. He otherwise feels well. A chest X- ray demonstrates two cavitating lesions in the right lower lobe. A bronchoscopy is performed for transbronchial biopsies and a bronchoalveolar lavage (BAL). A Ziehl–Neelsen stain on the BAL is negative and also culture for acid-fast bacilli is negative. However, granulomas are noted on the transbronchial biopsy. An eosinophilia is noted. Which one of the following options is the most likely unifying diagnosis?

A Hydatid disease

B Lymphoma

C Paragonimiasis

D Sarcoidosis

E Tuberculosis

Explanation

Paragonimiasis Diagnostic considerations

Although the history in the present case at a glance suggests tuberculosis, as a unifying diagnosis several features are not typical These include the lack of systemic symptoms (fever, weight loss), lower lobe cavitating lesions, an eosinophilia and the failure to demonstrate acid-fast bacilli, despite intensive sampling Hydatid disease due to Echinococcus species has many similar features with the formation of hydatid cysts, although a granulomatous reaction is not typical

Aetiopathogenesis

Paragonimiasis is an infection due to a trematode fluke, Paragonimus westermani This fluke is acquired when metacercaria are ingested from an infected crustacean The metacercaria penetrate the gut wall, the peritoneum and the lungs https://mypastest.pastest.com/Secure/TestMe/Browser/429893#Top 1/2 8/10/2016 MyPastest Here worms produce a cellular reaction, eosinophilia and haemorrhage A granulomatous response to the eggs may occur

792

Next Question

Previous Question Tag Question Feedback End Review

Difficulty: Difficult

Peer Responses

Session Progress

Responses Correct: 0

Responses Incorrect: 151

Responses Total: 151

Responses - % Correct: 0%

Blog (https://www.pastest.com/blog) About Pastest (https://www.pastest.com/about-us) Contact Us (https://www.pastest.com/contact-us) Help (https://www.pastest.com/help)

© Pastest 2016

https://mypastest.pastest.com/Secure/TestMe/Browser/429893#Top 2/2 8/10/2016 MyPastest

Back to Filters (/Secure/TestMe/Filter/429893/QA) Question 103 of 151

A 65-year-old patient with COPD who continues to smoke is housebound due to his disability. Which one of the following vaccinations should he receive on a yearly basis?

A Clostridium tetani

B Haemophilus influenzae type b

C Meningococcus type c virus

D Influenza virus

E Streptococcus pneumoniae

Explanation

Influenza vaccination Vaccine features

Influenza vaccines contain antigens from the two subtypes of human influenza A (H3N2 and N1N1) and B viruses To keep abreast of changes in the surface antigens of the virus, the composition of the virus must be modified on an annual basis and therefore annual re-immunisation is required To make global recommendations on vaccine composition and to maintain surveillance for the emergence of influenza viruses with pandemic potential, the World Health Organization maintains a global network of collaborating laboratories

Target patient groups

Influenza vaccination is recommended for those groups at the highest risk of morbidity, eg: patients in chronic care facilities (especially the elderly) those with chronic cardiopulmonary, lung or renal diseases, diabetes mellitus, haemoglobinopathies https://mypastest.pastest.com/Secure/TestMe/Browser/429893#Top 1/2 8/10/2016 MyPastest the immunocompromised

1314

Next Question

Previous Question Tag Question Feedback End Review

Difficulty: Average

Peer Responses

Session Progress

Responses Correct: 0

Responses Incorrect: 151

Responses Total: 151

Responses - % Correct: 0%

Blog (https://www.pastest.com/blog) About Pastest (https://www.pastest.com/about-us) Contact Us (https://www.pastest.com/contact-us) Help (https://www.pastest.com/help) © Pastest 2016

https://mypastest.pastest.com/Secure/TestMe/Browser/429893#Top 2/2 8/10/2016 MyPastest

Back to Filters (/Secure/TestMe/Filter/429893/QA) Question 104 of 151

An immigrant from the Philippines presents with high-grade fever of 1-week duration with spells of chills and rigor. Auscultation of his chest reveals bilateral crepitations with scattered rhonchi. Multiple subcutaneous nodules are found on the extensor surface of his left forearm and left leg. Gram-staining of a smear of the pus aspirated from a nodule shows plenty of Gram-negative bacilli. On blood culture, distinct, rough corrugated, grey–white colonies are seen. The organisms are motile and oxidase-positive. Which one of the following options is the most likely diagnosis?

A Plague

B Melioidosis

C Trench fever

D Actinomycosis

E Tuberculosis

Explanation

Differential diagnosis of a nodular infection Melioidosis infection

This is caused by Burkholderia pseudomallei Infection follows inhalation or direct inoculation, with the majority of clinical cases occurring in South-East Asia Symptoms include cough, pleuritic chest pains, bone pain and cellulitis IV Ceftazidime is the current drug of choice

Trench fever

This is caused by Bartonella quintana There is an abrupt onset of fever followed by the sudden development of profound anaemia Multiple abscesses are not seen in this disease https://mypastest.pastest.com/Secure/TestMe/Browser/429893#Top 1/3 8/10/2016 MyPastest The tetracycline group of antibiotics are the initial drug of choice

Plague

This is caused by Yersinia pestis, a Gram-negative bacillus that is non-motile and oxidase-negative A spectrum of symptoms occur in conjunction with plague, these include coryzal symptoms, massive lymph node swelling leading to formation of buboes, and coagulopathy Doxycycline is effective as monotherapy for Yersinia infection

Actinomycosis

Actinomycetes are Gram-positive branching bacteria Infection is associated with poor dental hygiene, abdominal surgery and large areas of devascularised tissue Symptoms include fever, cough, and formation of infected nodules which may later form discharging sinuses Penicillin, tetracyclines and macrolides are all potential options for treatment

3585

Next Question

Previous Question Tag Question Feedback End Review

Difficulty: Difficult

Peer Responses

Session Progress

Responses Correct: 0

Responses Incorrect: 151

Responses Total: 151

Responses - % Correct: 0%

https://mypastest.pastest.com/Secure/TestMe/Browser/429893#Top 2/3 8/10/2016 MyPastest

Blog (https://www.pastest.com/blog) About Pastest (https://www.pastest.com/about-us) Contact Us (https://www.pastest.com/contact-us) Help (https://www.pastest.com/help) © Pastest 2016

https://mypastest.pastest.com/Secure/TestMe/Browser/429893#Top 3/3 8/10/2016 MyPastest

Back to Filters (/Secure/TestMe/Filter/429893/QA) Question 105 of 151

A 26-year-old man is brought to the Emergency Department from the airport, having just returned on a flight from Bangladesh. Around 1 hour before landing he collapsed after severe vomiting and diarrhoea on the plane. His travelling partner reports that he became ill 1 day before leaving Bangladesh with high volume, painless watery diarrhoea. Blood pressure is 95/60 mmHg and his pulse is 100/min and regular. Dark-field microscopy of a fresh stool specimen reveals Gram-negative bacilli. Given the likeliest diagnosis, which one of the following antibiotic choices would be most appropriate in the treatment of the patient in the present case?

A Ciprofloxacin

B Metronidazole

C Amoxicillin

D Co-trimoxazole

E Co-amoxiclav

Explanation

Cholera

The patient in the present case has cholera.

Causative organism

Cholera is caused by Vibrio cholerae, a comma-shaped Gram-negative bacillus

Pathogenetic mechanism

It causes clinical disease by releasing an enterotoxin that affects the small bowel, leading to secretion of huge amounts of fluid and electrolytes into the intestinal lumen

Diagnosis

https://mypastest.pastest.com/Secure/TestMe/Browser/429893#Top 1/3 8/10/2016 MyPastest

Diagnosis is via dark ground microscopy of a fresh stool specimen

Treatment

While rehydration is the primary therapy for cholera, use of antibiotics has been proved to shorten the duration and severity of the illness Antibiotic choices may include quinolones such as ciprofloxacin, tetracyclines, erythromycin, and co-trimoxazole

Co-trimoxazole is not widely used though now due to associated risks of blood dyscrasias Problems with resistance to tetracyclines are emerging in patients from the Indian subcontinent in particular

Prognosis

With appropriate rehydration and antibiotic therapy where required, patients usually show signs of recovery over a few days

10702

Next Question

Previous Question Tag Question Feedback End Review

Difficulty: Average

Peer Responses

Session Progress

Responses Correct: 0

Responses Incorrect: 151

Responses Total: 151

Responses - % Correct: 0%

https://mypastest.pastest.com/Secure/TestMe/Browser/429893#Top 2/3 8/10/2016 MyPastest Blog (https://www.pastest.com/blog) About Pastest (https://www.pastest.com/about-us) Contact Us (https://www.pastest.com/contact-us) Help (https://www.pastest.com/help) © Pastest 2016

https://mypastest.pastest.com/Secure/TestMe/Browser/429893#Top 3/3 8/10/2016 MyPastest

Back to Filters (/Secure/TestMe/Filter/429893/QA) Question 106 of 151

Which one of the following pathogens can grow in contaminated food if stored at a fridge temperature of 4 °C?

A Listeria monocytogenes

B Escherichia coli

C Bacillus cereus

D Clostridium botulinum type A

E Staphylococcus aureus

Explanation

Listeria monocytogenes

Listeria species are non-sporing, facultatively anaerobic, Gram-positive bacilli that are ubiquitous in the environment and distributed worldwide Listeria monocytogenes is the major pathogen Enrichment and selective methods are now well established for the isolation of Listeria from food or the environment; immunoassays and nucleic acid-amplification techniques have also been used Several typing methods are used to trace food sources, distinguish relapses from re- infections and investigate outbreaks The ability to multiply at temperatures of 0–40 °C and tolerate preserving agents makes Listeria of particular concern if present in refrigerated foods that are consumed without further cooking

1306

Next Question

https://mypastest.pastest.com/Secure/TestMe/Browser/429893#Top 1/2 8/10/2016 MyPastest Previous Question Tag Question Feedback End Review

Difficulty: Difficult

Peer Responses

Session Progress

Responses Correct: 0

Responses Incorrect: 151

Responses Total: 151

Responses - % Correct: 0%

Blog (https://www.pastest.com/blog) About Pastest (https://www.pastest.com/about-us) Contact Us (https://www.pastest.com/contact-us) Help (https://www.pastest.com/help) © Pastest 2016

https://mypastest.pastest.com/Secure/TestMe/Browser/429893#Top 2/2 8/10/2016 MyPastest

Back to Filters (/Secure/TestMe/Filter/429893/QA) Question 107 of 151

A 42-year-old gay man comes to the clinic with a skin rash. He has multiple pink/red maculopapular lesions on his skin, they range in size from a few mm across to 2–3 cm, and involve the oral mucosa as well. You suspect that it may be Kaposi’s sarcoma Kaposi’s sarcoma is associated with which one of the following viruses?

A Human herpesvirus 8 (HHV8)

B Epstein–Barr virus (EBV)

C Human herpesvirus 6 (HHV6)

D Human T-lymphotropic virus (HTLV)

E Human papillomavirus (HPV)

Explanation

Kaposi’s sarcoma and human herpesvirus infection

Seroconversion to human herpesvirus 8 (HHV8) pre-dates the development of Kaposi’s sarcoma by a period of 5–10 years Where HIV infection has already occurred a long period before exposure to HHV8, this time period for development of Kaposi’s lesions is shortened Cigarette smoking may also have a role in increasing the risk of developing the lesions The introduction of highly active anti-retroviral treatment (HAART) has reduced the incidence and improved the prognosis of Kaposi’s sarcoma Systemic therapy with interferon alfa has also improved its clinical course Local excision, local injection of vinca alkaloids, or local radiotherapy may also be useful for particularly troublesome lesions

20786

Next Question

https://mypastest.pastest.com/Secure/TestMe/Browser/429893#Top 1/2 8/10/2016 MyPastest

Previous Question Tag Question Feedback End Review

Difficulty: Average

Peer Responses

Session Progress

Responses Correct: 0

Responses Incorrect: 151

Responses Total: 151

Responses - % Correct: 0%

Blog (https://www.pastest.com/blog) About Pastest (https://www.pastest.com/about-us) Contact Us (https://www.pastest.com/contact-us) Help (https://www.pastest.com/help) © Pastest 2016

https://mypastest.pastest.com/Secure/TestMe/Browser/429893#Top 2/2 8/10/2016 MyPastest

Back to Filters (/Secure/TestMe/Filter/429893/QA) Question 108 of 151

A 22-year-old man is admitted to the medical ward with a vesicular rash all over his body. He has a fever and says that the rash began the previous evening, a few days after symptoms of a cold. On examination his BP is 125/77 mmHg, his temperature is 38.2 °C. He has a vesicular rash covering his trunk. Respiratory examination is unremarkable and his saturation on air is 96%.

Which one of the following options is the most appropriate management for him?

A Varicella zoster immunoglobulins (VZIG)

B Aciclovir

C Flucloxacillin

D Normal saline rehydration

E Paracetamol

Explanation

Treatment of chickenpox in patients over 12 years of age

A consensus statement published in the Journal of Infection (August 2008) states that while trials of aciclovir are underpowered to demonstrate an outcomes benefit in the treatment of chickenpox, treatment of all adolescents and adults is recommended on cost-effectiveness grounds A 1-week course of aciclovir is recommended Varicella zoster immunoglobulins (VZIG) is recommended in patients who are at risk of severe chickenpox infection, who are seronegative for varicella zoster antibodies and have a significant exposure to either chickenpox or shingles

21091

Next Question

https://mypastest.pastest.com/Secure/TestMe/Browser/429893#Top 1/2 8/10/2016 MyPastest

Previous Question Tag Question Feedback End Review

Difficulty: Average

Peer Responses

Session Progress

Responses Correct: 0

Responses Incorrect: 151

Responses Total: 151

Responses - % Correct: 0%

Blog (https://www.pastest.com/blog) About Pastest (https://www.pastest.com/about-us) Contact Us (https://www.pastest.com/contact-us) Help (https://www.pastest.com/help) © Pastest 2016

https://mypastest.pastest.com/Secure/TestMe/Browser/429893#Top 2/2 8/10/2016 MyPastest

Back to Filters (/Secure/TestMe/Filter/429893/QA) Question 109 of 151

A 17-year-old young woman, who is sexually active, attends the Emergency Department with a history of dysuria and abdominal pain. A pregnancy test is negative and a urine dipstick test is positive for leukocytes. A course of trimethoprim is commenced. One week later she returns to the Emergency Department with persistent symptoms. Standard charcoal vaginal swabs and urine culture taken on the first visit are negative. Which one of the following options is the most likely aetiological agent?

A Chlamydia trachomatis

B Escherichia coli

C Haemophilus ducreyi

D Neisseria gonorrhoea

E Trichomonas vaginalis

Explanation

Sexually transmitted diseases Diagnostic considerations

Dysuria and abdominal pain are common presentations for sexually transmitted diseases (STDs) Urinary tract infection is a likely differential diagnosis, but a negative urine culture further raises the possibility of STDs Gonorrhoea and chlamydia are the most likely STDs

Laboratory investigations

Gonorrhoea is generally cultured and seen on Gram staining The diagnosis of chlamydia relies on:

specialised culture techniques use of a chlamydia antigen test

https://mypastest.pastest.com/Secure/TestMe/Browser/429893#Top 1/2 8/10/2016 MyPastest Differential diagnosis

Haemophilus ducreyi is the cause of chancroid, which leads to painful ulceration and local lymphadenopathy Trichomonas vaginalis is a protozoan that often leads to an offensive smelling vaginal discharge.

794

Next Question

Previous Question Tag Question Feedback End Review

Difficulty: Average

Peer Responses

Session Progress

Responses Correct: 0

Responses Incorrect: 151

Responses Total: 151

Responses - % Correct: 0%

Blog (https://www.pastest.com/blog) About Pastest (https://www.pastest.com/about-us) Contact Us (https://www.pastest.com/contact-us) Help (https://www.pastest.com/help) © Pastest 2016

https://mypastest.pastest.com/Secure/TestMe/Browser/429893#Top 2/2 8/10/2016 MyPastest

Back to Filters (/Secure/TestMe/Filter/429893/QA) Question 110 of 151

A 55-year-old man of Pakistani origin presents with a cough, weight loss and worsening lymphadenopathy. History of note is that he commenced anti-HIV treatment some 4 weeks earlier while visiting relatives in Pakistan. He is diagnosed on this occasion with disseminated tuberculosis. Sputum and lymph node cultures confirm fully sensitive Mycobacterium tuberculosis. He is commenced on quadruple anti-tuberculous therapy. However, 1 month later he complains of enlarging masses in his axillae. Which one of the following options is the most likely explanation of the lymph node enlargement?

A Immune reconstitution disease

B Lymphoma

C Multidrug-resistant mycobacteria

D Non-compliance

E Staphylococcal infection

Explanation

Immune reconstitution disease Pathogenesis

Immune reconstitution disease typically occurs a few weeks after commencing anti- retroviral therapy in a patient with underlying tuberculosis

Rationale for treatment

The strong, T-cell-based inflammatory response to tuberculosis infection may itself lead to important symptoms if local enlargement impinges on adjacent critical structures, such as the brain in cerebral tuberculosis This is one rationale for the concomitant use of steroids in such patients

771

Next Question https://mypastest.pastest.com/Secure/TestMe/Browser/429893#Top 1/2 8/10/2016 MyPastest

Previous Question Tag Question Feedback End Review

Difficulty: Average

Peer Responses

Session Progress

Responses Correct: 0

Responses Incorrect: 151

Responses Total: 151

Responses - % Correct: 0%

Blog (https://www.pastest.com/blog) About Pastest (https://www.pastest.com/about-us) Contact Us (https://www.pastest.com/contact-us) Help (https://www.pastest.com/help)

© Pastest 2016

https://mypastest.pastest.com/Secure/TestMe/Browser/429893#Top 2/2 8/10/2016 MyPastest

Back to Filters (/Secure/TestMe/Filter/429893/QA) Question 111 of 151

Which is the single best way to reduce meticillin-resistant Staphylococcus aureus (MRSA) transmission?

A Hand washing

B Corticosteroids

C Prophylactic antibiotics

D Control of water temperature

E Water disinfection

Explanation

Preventing transmission of meticillin-resistant Staphylococcus aureus (MRSA) infection

Epidemic outbreaks are usually amenable to measures that interrupt the spread of infection, such as use of gowns and gloves and careful hand washing by those attending patients Transfer of colonised or infected patients to a single room or an isolation ward is a physical means of preventing spread Patients infected with the same organism can be grouped together and attended to by a cohort of nurses not involved with uninfected patients Identification of additional carriers and elimination of colonisation may be necessary for some epidemic outbreaks Controlled trials demonstrating the efficacy of such measures have not been made, but many observational studies support their use

5637

Next Question

https://mypastest.pastest.com/Secure/TestMe/Browser/429893#Top 1/2 8/10/2016 MyPastest Previous Question Tag Question Feedback End Review

Difficulty: Easy

Peer Responses

Session Progress

Responses Correct: 0

Responses Incorrect: 151

Responses Total: 151

Responses - % Correct: 0%

Blog (https://www.pastest.com/blog) About Pastest (https://www.pastest.com/about-us) Contact Us (https://www.pastest.com/contact-us) Help (https://www.pastest.com/help) © Pastest 2016

https://mypastest.pastest.com/Secure/TestMe/Browser/429893#Top 2/2 8/10/2016 MyPastest

Back to Filters (/Secure/TestMe/Filter/429893/QA) Question 112 of 151

A 49-year-old man presents with an episode of acute self-limiting hepatitis. Hepatitis A is diagnosed. Which one of the following options is the most likely mode of transmission?

A Sexual

B Blood transfusion

C Needle-stick injury

D Contaminated food

E Mosquitoes

Explanation

Features of hepatitis A infection

Hepatitis A causes an acute self-limited hepatitis, but not chronic viral carriage or chronic liver disease The RNA virus is acquired orally The incubation period is between 2 and 6 weeks Transmission generally follows the ingestion of food or water contaminated with faeces from a hepatitis A virus (HAV)-infected individual Viral shedding in the faeces ceases at approximately the time of onset of clinical symptoms Transmission may occur in epidemics, following floods or after sewage contamination of shellfish beds

2693

Next Question

Previous Question Tag Question Feedback End Review https://mypastest.pastest.com/Secure/TestMe/Browser/429893#Top 1/2 8/10/2016 MyPastest

Difficulty: Easy

Peer Responses

Session Progress

Responses Correct: 0

Responses Incorrect: 151

Responses Total: 151

Responses - % Correct: 0%

Blog (https://www.pastest.com/blog) About Pastest (https://www.pastest.com/about-us) Contact Us (https://www.pastest.com/contact-us) Help (https://www.pastest.com/help) © Pastest 2016

https://mypastest.pastest.com/Secure/TestMe/Browser/429893#Top 2/2 8/10/2016 MyPastest

Back to Filters (/Secure/TestMe/Filter/429893/QA) Question 113 of 151

A 23-year-old woman presents with fever, diarrhoea and myalgia. She is currently menstruating. On examination her temperature is 40 °C, blood pressure 90/50 mmHg, pulse 140/min. Which one of the following is the most likely organism responsible for the toxins?

A Staphylococcus

B Pseudomonas

C E. coli

D Herpes simplex virus

E HIV

Explanation

Toxic shock syndrome Epidemiology

Toxic shock syndrome may be fatal A mortality rate of around 5% was reported during the ‘tampon epidemic’

Complications

Bacteraemia has rarely been reported in toxic shock syndrome

Management

Since the syndrome is mediated by toxin (toxic shock syndrome toxin), the mainstay of treatment is supportive Anti-staphylococcal antibiotics should be given to eradicate Staphylococcus aureus from the local site The staphylococci isolated are usually resistant only to penicillin

3716 https://mypastest.pastest.com/Secure/TestMe/Browser/429893#Top 1/2 8/10/2016 MyPastest Next Question

Previous Question Tag Question Feedback End Review

Difficulty: Easy

Peer Responses

Session Progress

Responses Correct: 0

Responses Incorrect: 151

Responses Total: 151

Responses - % Correct: 0%

Blog (https://www.pastest.com/blog) About Pastest (https://www.pastest.com/about-us) Contact Us (https://www.pastest.com/contact-us) Help (https://www.pastest.com/help)

© Pastest 2016

https://mypastest.pastest.com/Secure/TestMe/Browser/429893#Top 2/2 8/10/2016 MyPastest

Back to Filters (/Secure/TestMe/Filter/429893/QA) Question 114 of 151

A 40-year-old farmer complains of having had a flu-like illness for the past 2 weeks. Blood culture results are negative. Complement fixation tests reveal Coxiella burnetii infection. By which one of the following routes is he most likely to have become infected?

A Pasteurised milk

B Aerosols

C Bite of infected ticks

D Infected mites

E Direct entry via skin abrasions

Explanation

Q fever

The man in the present case has Q fever

Causative organism

The causative organism is Coxiella burnetii It is an obligate intracellular organism and does not grow on standard culture media

Transmission of the infection

C. burnetii is usually spread between domestic animals by ticks, which are a reservoir of infection Transmission to humans is by inhalation of infected dust and aerosols and drinking unpasteurised milk from infected cows

3573

Next Question

https://mypastest.pastest.com/Secure/TestMe/Browser/429893#Top 1/2 8/10/2016 MyPastest

Previous Question Tag Question Feedback End Review

Difficulty: Difficult

Peer Responses

Session Progress

Responses Correct: 0

Responses Incorrect: 151

Responses Total: 151

Responses - % Correct: 0%

Blog (https://www.pastest.com/blog) About Pastest (https://www.pastest.com/about-us) Contact Us (https://www.pastest.com/contact-us) Help (https://www.pastest.com/help) © Pastest 2016

https://mypastest.pastest.com/Secure/TestMe/Browser/429893#Top 2/2 8/10/2016 MyPastest

Back to Filters (/Secure/TestMe/Filter/429893/QA) Question 115 of 151

A student returns from a backpacking holiday in South America having developed abdominal pain, diarrhoea and fevers 1 week prior to his return. On examination he has a fever of 38.5 °C and diffuse abdominal pain. Stool microscopy shows pus cells and red blood cells, culture is awaited. Which one of the following options is the most likely organism?

A Plasmodium falciparum

B Norwalk virus

C Rotavirus

D Salmonella spp

E Vibrio cholerae

Explanation

Traveller’s diarrhoea

All fevers in returning travellers from regions where malaria is endemic should have three blood films to assess for malaria However, in this case the presence of bloody diarrhoea indicates an enteroinvasive organism The commonest are Salmonella spp, Shigella spp, enteroinvasive Escherichia coli and Entamoeba histolytica

768

Next Question

Previous Question Tag Question Feedback End Review

Difficulty: Average

https://mypastest.pastest.com/Secure/TestMe/Browser/429893#Top 1/2 8/10/2016 MyPastest Peer Responses

Session Progress

Responses Correct: 0

Responses Incorrect: 151

Responses Total: 151

Responses - % Correct: 0%

Blog (https://www.pastest.com/blog) About Pastest (https://www.pastest.com/about-us) Contact Us (https://www.pastest.com/contact-us) Help (https://www.pastest.com/help) © Pastest 2016

https://mypastest.pastest.com/Secure/TestMe/Browser/429893#Top 2/2 8/10/2016 MyPastest

Back to Filters (/Secure/TestMe/Filter/429893/QA) Question 116 of 151

A 19-year-old student visits you complaining of fevers and headaches over the past week or two. She has just started university after a world tour during her gap-year. There is also complaint of muscle ache, a sore throat, general malaise and of a general lack of appetite and vague abdominal pain. She remembers a short period of diarrhoea 10 days ago. On examination there are a few faint maculopapular blanching lesions on the chest. Which one of the following diagnoses fits best with this clinical picture?

A Malaria

B Tuberculosis

C Brucellosis

D Amoebic liver abscess

E Typhoid fever

Explanation

Typhoid Definition and causative organism

Typhoid fever is a systemic infection caused by Salmonella typhi or S. paratyphi

Clinical presentation

There is an incubation period of a few days to a few weeks after initial infection and there may be a short period of diarrhoea or constipation at the beginning of the illness Fever, headache, malaise and myalgia are common symptoms, with characteristic maculopapular ‘rose spots’ appearing over the chest or abdomen gastrointestinal bleeding may occur as a consequence of the ulceration of Peyer’s patches in the ileum Mental status changes occur only rarely

Laboratory findings https://mypastest.pastest.com/Secure/TestMe/Browser/429893#Top 1/2 8/10/2016 MyPastest

Blood cultures are more likely to be positive early in the disease, stool and urine culture later in the disease Retrospective serology using the Widal test may be useful in confirming that an infection has taken place Bone marrow biopsy is positive in 90%, but is rarely undertaken Neutropenia on full blood count is common

Treatment and prognosis

Ciprofloxacin is the antibiotic of choice, chronic carriage may occur due to residual organisms colonising the gallbladder The relapse rate is approximately 10%

2658

Next Question

Previous Question Tag Question Feedback End Review

Difficulty: Average

Peer Responses

Session Progress

Responses Correct: 0

Responses Incorrect: 151

Responses Total: 151

Responses - % Correct: 0%

Blog (https://www.pastest.com/blog) About Pastest (https://www.pastest.com/about-us) Contact Us (https://www.pastest.com/contact-us) Help (https://www.pastest.com/help) © Pastest 2016

https://mypastest.pastest.com/Secure/TestMe/Browser/429893#Top 2/2 8/10/2016 MyPastest

Back to Filters (/Secure/TestMe/Filter/429893/QA) Question 117 of 151

A 17-year-old young woman is undertaking a summer placement at a nursery school before applying to study medicine. She has received a full programme of childhood vaccinations. Her main complaints are difficulty swallowing, sore throat, malaise and fever. On examination she has 5–10 grey ulcers on her buccal mucosa. There is also a vesicular rash affecting her hands and feet. Which one of the following options is the most likely cause of this clinical picture?

A Erythema multiforme

B Herpes simplex infection

C Gonorrhoea

D Pemphigus

E Coxsackievirus infection

Explanation

Hand, foot and mouth disease

Hand, foot and mouth disease presents with the typical picture described in the present question

Epidemiology

It occurs with highest frequency in children of kindergarten age or younger, during the summer months especially Outbreaks may occur in adult care workers who deal with children

Causative organisms

Coxsackievirus group A, type 16 was the first, and is the commonest, viral agent isolated Viral subtypes A5, A7, A9, A10, B2 and B5 have also been implicated

https://mypastest.pastest.com/Secure/TestMe/Browser/429893#Top 1/3 8/10/2016 MyPastest Clinical presentation

The disease has a 3–5-day incubation period, followed by cold-like symptoms and the appearance of grey-coloured ulcers in the mouth and a vesicular rash on the hands and feet

Complications

The disease is usually self-limiting, but in rare cases encephalitis, meningitis or myocarditis may develop The rate of spontaneous abortion is increased if the infection is acquired in early pregnancy

Management

One small study without a control group claimed to show some benefit of aciclovir therapy

2654

Next Question

Previous Question Tag Question Feedback End Review

Difficulty: Average

Peer Responses

Session Progress

Responses Correct: 0

Responses Incorrect: 151

Responses Total: 151

Responses - % Correct: 0%

Blog (https://www.pastest.com/blog) About Pastest (https://www.pastest.com/about-us) Contact Us (https://www.pastest.com/contact-us) Help (https://www.pastest.com/help) https://mypastest.pastest.com/Secure/TestMe/Browser/429893#Top 2/3 8/10/2016 MyPastest © Pastest 2016

https://mypastest.pastest.com/Secure/TestMe/Browser/429893#Top 3/3 8/10/2016 MyPastest

Back to Filters (/Secure/TestMe/Filter/429893/QA) Question 118 of 151

A child has been scratched by a cat and develops axillary lymphadenopathy. He has no fever. Which organism is most likely responsible for this?

A Bartonella henselae

B Staphylococcus aureus

C Streptococcus pyogenes

D Toxoplasma gondii

E Pasteurella multicida

Explanation

Cat-scratch disease Causative organism

The aetiological agent of this disease is Bartonella henselae

Definition

Cat-scratch disease, in most patients, is an acute, self-limiting infection characterised by development of a papule at the site of inoculation by a cat, followed by regional adenopathy that may persist for 1–4 months

Complications

In a small percentage of patients, serious systemic complications may arise, including involvement of the central nervous system, liver, spleen, lung, bone, eyes, and skin

5635

Next Question

https://mypastest.pastest.com/Secure/TestMe/Browser/429893#Top 1/2 8/10/2016 MyPastest

Previous Question Tag Question Feedback End Review

Difficulty: Average

Peer Responses

Session Progress

Responses Correct: 0

Responses Incorrect: 151

Responses Total: 151

Responses - % Correct: 0%

Blog (https://www.pastest.com/blog) About Pastest (https://www.pastest.com/about-us) Contact Us (https://www.pastest.com/contact-us) Help (https://www.pastest.com/help) © Pastest 2016

https://mypastest.pastest.com/Secure/TestMe/Browser/429893#Top 2/2 8/10/2016 MyPastest

Back to Filters (/Secure/TestMe/Filter/429893/QA) Question 119 of 151

An Indian man is referred with peripheral oedema of the arms and recurrent rhinitis. On examination there is deformity of the nose with collapse of the nasal bridge and multiple skin plaques. A skin biopsy reveals multiple acid–alcohol-fast bacilli. Which one of the following options is the most likely diagnosis?

A Lepromatous leprosy

B ‘Atypical’ mycobacteria infection

C Tertiary syphilis

D Tuberculoid leprosy

E Yaws

Explanation

Lepromatous leprosy Causative organism

Leprosy is due to infection with Mycobacterium leprae, which are slow-growing, acid– alcohol-fast bacilli

Clinical features

The manifestations of the disease are dependent on the host immune response Tuberculoid leprosy is characterised by a strong T-cell immune response with no bacilli detected in affected areas The typical skin lesion for lepromatous leprosy is a hypopigmented, anaesthetic patch Many acid-fast bacilli may be seen in the skin plaques Due to the poor immune response, progressive disease occurs with manifestations such as collapse of the nasal bridge and leonine facies

Differential diagnosis

https://mypastest.pastest.com/Secure/TestMe/Browser/429893#Top 1/2 8/10/2016 MyPastest

The gumma is the characteristic lesion of tertiary syphilis Yaws is a spirochaete illness like syphilis, affecting the palms and feet initially; the latter may lead to plantar keratosis

788

Next Question

Previous Question Tag Question Feedback End Review

Difficulty: Average

Peer Responses

Session Progress

Responses Correct: 0

Responses Incorrect: 151

Responses Total: 151

Responses - % Correct: 0%

Blog (https://www.pastest.com/blog) About Pastest (https://www.pastest.com/about-us) Contact Us (https://www.pastest.com/contact-us) Help (https://www.pastest.com/help) © Pastest 2016

https://mypastest.pastest.com/Secure/TestMe/Browser/429893#Top 2/2 8/10/2016 MyPastest

Back to Filters (/Secure/TestMe/Filter/429893/QA) Question 120 of 151

A 70-year-old man known to have NIDDM was admitted with pain and swelling in the left ear and face. On examination the external ear is red, tender and swollen. There is a small amount of purulent discharge from the external auditory canal with crust covering the skin. The left side of the face is swollen, with tenderness over the left temporal bone. The primary microorganism most probably responsible for this infection is:

A Pseudomonas aeruginosa

B Staphylococcus aureus

C Streptococcus pneumoniae

D Listeria monocytogenes

E Haemophilus influenzae

Explanation

Malignant external otitis

Malignant external otitis is an infection of the external auditory canal, commonly seen in elderly patients with diabetes

Causative organism

It is caused by Pseudomonas aeruginosa.

Complications

The infection might spread from the outer ear to the soft tissues below the temporal bone and invade the parotid gland, temporomandibular joint, masseter muscle and temporal bone Necrotising osteitis of the temporal bone develops The high mortality rate originally reported for the condition (approximately 40%) led to the use of the adjective ‘malignant’ for this form of temporal bone infection

1636 https://mypastest.pastest.com/Secure/TestMe/Browser/429893#Top 1/2 8/10/2016 MyPastest 1636

Next Question

Previous Question Tag Question Feedback End Review

Difficulty: Difficult

Peer Responses

Session Progress

Responses Correct: 0

Responses Incorrect: 151

Responses Total: 151

Responses - % Correct: 0%

Blog (https://www.pastest.com/blog) About Pastest (https://www.pastest.com/about-us) Contact Us (https://www.pastest.com/contact-us) Help (https://www.pastest.com/help) © Pastest 2016

https://mypastest.pastest.com/Secure/TestMe/Browser/429893#Top 2/2 8/10/2016 MyPastest

Back to Filters (/Secure/TestMe/Filter/429893/QA) Question 121 of 151

You are asked to examine some records from a number of elderly patients admitted on the medical take in the past 3 days. They have attended a convention for retired soldiers in a mountainous region of Spain. During the week they spent some time walking in the countryside. All the patients had symptoms of cough, shortness of breath, chills and diarrhoea. Four of the patients appeared mildly confused on admission. All the six patients had stayed at neighbouring hotels. Blood testing revealed mild hyponatraemia and elevated creatinine; in addition, microscopic haematuria was seen in five of the six patients. Which one of the following infective organisms fits best with this clinical picture?

A Mycoplasma pneumoniae

B Legionella pneumophila

C Streptococcus pneumonia

D Borrelia Burgdorferi

E Chlamydia pneumoniae

Explanation

Legionnaire’s disease

The six patients in the question show features of legionnaire’s disease

Clinical features

Prominent clinical features of legionnaire’s include temperature >40 °C, headache, mental confusion or lethargy, loose stools or diarrhoea, abdominal pain and acute renal failure

Radiological and laboratory findings

Asymmetrical pulmonary infiltrates are seen on chest X-ray Elevated levels of liver enzymes, creatinine, C-reactive protein (CRP) and creatine kinase (CK) are also seen https://mypastest.pastest.com/Secure/TestMe/Browser/429893#Top 1/2 8/10/2016 MyPastest Urinary antigen testing is a rapid and effective way to make the diagnosis

Treatment and prognosis

Quinolone or macrolide antibiotics are both appropriate choices for treatment Mortality increases with age and associated pre-existing respiratory disease

14964

Next Question

Previous Question Tag Question Feedback End Review

Difficulty: Easy

Peer Responses

Session Progress

Responses Correct: 0

Responses Incorrect: 151

Responses Total: 151

Responses - % Correct: 0%

Blog (https://www.pastest.com/blog) About Pastest (https://www.pastest.com/about-us) Contact Us (https://www.pastest.com/contact-us) Help (https://www.pastest.com/help)

© Pastest 2016

https://mypastest.pastest.com/Secure/TestMe/Browser/429893#Top 2/2 8/10/2016 MyPastest

Back to Filters (/Secure/TestMe/Filter/429893/QA) Question 122 of 151

A 35-year-old homosexual man presents with a 3-week history of fever and malaise. On examination he has generalised lymphadenopathy and tenderness in the left upper quadrant. He has had unprotected sexual intercourse with number of sexual partners in the past few weeks. He has not noticed any rash or mucosal ulceration. An HIV antibody test is negative. A Paul–Bunnell test is very weakly positive. His liver function tests (LFTs) are mildly deranged, p-24 is normal. Which one of the following options is the most likely diagnosis?

A Primary HIV infection

B Secondary syphilis

C Acute hepatitis A

D Acute EBV infection

E Acute CMV infection

Explanation

Diagnosis of acute cytomegalovirus infection

The symptoms and findings in the present case are typical of acute cytomegalovirus (CMV) infection, particularly in this patient group The human immunodeficiency virus (HIV) antibody test would be expected to be positive 3 weeks into a seroconversion illness if this were HIV, although a negative test doesn't completely rule out infection The very low positive PB test suggests this is not EBV, although does not exclude it (the specificity of the modern test is high) The symptoms would fit with secondary syphilis, which is increasingly common recently, however the absence of a rash would be unusual Acute hepatitis A would not be expected to produce lymphadenopathy but could produce the rest of the findings

3692

https://mypastest.pastest.com/Secure/TestMe/Browser/429893#Top 1/2 8/10/2016 MyPastest Next Question

Previous Question Tag Question Feedback End Review

Difficulty: Difficult

Peer Responses

Session Progress

Responses Correct: 0

Responses Incorrect: 151

Responses Total: 151

Responses - % Correct: 0%

Blog (https://www.pastest.com/blog) About Pastest (https://www.pastest.com/about-us) Contact Us (https://www.pastest.com/contact-us) Help (https://www.pastest.com/help)

© Pastest 2016

https://mypastest.pastest.com/Secure/TestMe/Browser/429893#Top 2/2 8/10/2016 MyPastest

Back to Filters (/Secure/TestMe/Filter/429893/QA) Question 123 of 151

A 48-year-old oil executive has returned from a spell working out in the Middle East. He was keen to try out the local food and had some infections during his stay. He complains of abdominal bloating and diffuse vague tenderness and of diarrhoea, which is hard to flush away. He has had episodes of night sweats during a couple of nights in the past few weeks. On examination he looks tanned, and appears to have glossitis. Blood testing reveals megaloblastic anaemia. He is weighed and has lost 7 kg over the past 3 months. Which one of the following diagnoses fits best with this clinical picture?

A Coeliac disease

B Amoebiasis

C Tropical sprue

D Crohn’s disease

E Whipple’s disease

Explanation

Tropical sprue Clinical features

Clinical features of tropical sprue include anorexia, diarrhoea, weight loss, abdominal pain and steatorrhoea These symptoms can develop in travellers even some months after returning to temperate regions

Investigations

Work-up involves a detailed history (including travel history), physical examination, d- xylose absorption testing and consideration of jejunal biopsy Blood testing reveals megaloblastic anaemia in more than 50% of cases, with coexistent vitamin B12 or folate deficiency

https://mypastest.pastest.com/Secure/TestMe/Browser/429893#Top 1/2 8/10/2016 MyPastest Histological features

Jejunal biopsy may show blunting, atrophy or the disappearance of villi, with subepithelial lymphocytic infiltration

Treatment and prognosis

Treatment is with oral folic acid supplementation, im vitamin B12 , and 4–6 weeks’ treatment with a tetracycline (or ampicillin where there is tetracycline intolerance) There is complete recovery with appropriate antibiotic therapy

2645

Next Question

Previous Question Tag Question Feedback End Review

Difficulty: Average

Peer Responses

Session Progress

Responses Correct: 0

Responses Incorrect: 151

Responses Total: 151

Responses - % Correct: 0%

Blog (https://www.pastest.com/blog) About Pastest (https://www.pastest.com/about-us) Contact Us (https://www.pastest.com/contact-us) Help (https://www.pastest.com/help)

© Pastest 2016

https://mypastest.pastest.com/Secure/TestMe/Browser/429893#Top 2/2 8/10/2016 MyPastest

Back to Filters (/Secure/TestMe/Filter/429893/QA) Question 124 of 151

How is tuberculosis most commonly spread?

A Ingestion of contaminated milk

B Contamination of skin abrasions in healthcare workers

C Inhalation of droplet nuclei

D Sexual contact

E Blood transfusion

Explanation

Spread of tuberculosis

The development of active tuberculosis, like all infectious diseases, is a function of the quantity and virulence of the invading organism and the relative resistance or susceptibility of the host to the pathogen

Transmission of the bacilli

Tubercle bacilli are transmitted between people by aerosols generated by coughing or otherwise expelling infectious pulmonary or laryngeal secretions into the air Mycobacterium tuberculosis bacilli excreted by this action are contained within droplet nuclei, extremely small particles (less than 1 μm) that remain airborne for long periods and are disseminated by diffusion and convection until they are deposited on surfaces, diluted or inactivated by ultraviolet radiation People breathing air containing these droplet nuclei are at risk of becoming infected if inhaled nuclei are deposited in their alveoli Transmission of tuberculous infection by other routes, such as inoculation in laboratories and aerosolisation of bacilli from tissues in hospitals, has been documented, but these are an insignificant means of spread

1319 https://mypastest.pastest.com/Secure/TestMe/Browser/429893#Top 1/2 8/10/2016 MyPastest 1319

Next Question

Previous Question Tag Question Feedback End Review

Difficulty: Easy

Peer Responses

Session Progress

Responses Correct: 0

Responses Incorrect: 151

Responses Total: 151

Responses - % Correct: 0%

Blog (https://www.pastest.com/blog) About Pastest (https://www.pastest.com/about-us) Contact Us (https://www.pastest.com/contact-us) Help (https://www.pastest.com/help)

© Pastest 2016

https://mypastest.pastest.com/Secure/TestMe/Browser/429893#Top 2/2 8/10/2016 MyPastest

Back to Filters (/Secure/TestMe/Filter/429893/QA) Question 125 of 151

A patient with hereditary elliptocytosis will be undergoing an elective splenectomy. He has enlarged tender spleen on abdominal examination. When should the patient in the present case receive conjugate pneumococcal vaccination?

A 1 week before operation

B 5 days before operation

C 2 weeks before operation

D 1 month after operation

E Postoperatively

Explanation

Hereditary elliptocytosis: pre-splenectomy precautions

It is recommended that conjugate pneumococcal vaccination is given at least 2 weeks before the patient undergoes splenectomy If the patient has already received a polysaccharide pneumococcal vaccine then they potentially have coverage against infection for at least 5 years Meningococcal and Hib vaccinations are also recommended pre-splenectomy if they have not already been given Unfortunately though, patients are still at increased risk of subsequent bacterial infection because of the loss of splenic function

20785

Next Question

Previous Question Tag Question Feedback End Review

https://mypastest.pastest.com/Secure/TestMe/Browser/429893#Top 1/2 8/10/2016 MyPastest

Difficulty: Average

Peer Responses

Session Progress

Responses Correct: 0

Responses Incorrect: 151

Responses Total: 151

Responses - % Correct: 0%

Blog (https://www.pastest.com/blog) About Pastest (https://www.pastest.com/about-us) Contact Us (https://www.pastest.com/contact-us) Help (https://www.pastest.com/help) © Pastest 2016

https://mypastest.pastest.com/Secure/TestMe/Browser/429893#Top 2/2 8/10/2016 MyPastest

Back to Filters (/Secure/TestMe/Filter/429893/QA) Question 126 of 151

A 19-year-old woman presents with a severe sore throat, fever and malaise. She has marked cervical lymphadenopathy, gross splenomegaly and scattered petechiae on the soft palate, with enlarged tonsils covered by a confluent white exudate. Her white cell count is mildly elevated, her serum ALT and AST concentrations are twice normal and her alkaline phosphatase concentration is slightly elevated. Which one of the following investigations is most likely to help guide your management?

A Fine-needle aspiration of a lymph node

B Hepatitis-B surface antigen

C Cytomegalovirus IgM

D Heterophilic antibodies

E HIV test

Explanation

Infectious mononucleosis

The history in the present case is strongly suggestive of infectious mononucleosis It would be unusual for the acute HIV retroviral syndrome to present with marked exudative tonsillitis and splenomegaly, although a reported sexual history negative for risk factors does not exclude HIV infection

Further investigations

The history can be confirmed by a heterophilic antibody test (Monospot) Around 80% of patients at this stage of the illness will have a positive test If the Monospot test is negative, it should be repeated a few days later Specific serological tests to detect the presence of Epstein–Barr virus (EBV) should be requested if it is still negative and there is a high clinical suspicion of glandular fever If the EBV tests are negative then other causes for heterophilic-negative acute mononucleosis – including human immunodeficiency virus (HIV), cytomegalovirus and https://mypastest.pastest.com/Secure/TestMe/Browser/429893#Top 1/2 8/10/2016 MyPastest toxoplasmosis – should be considered Liver function tests (LFTs) are not sufficiently elevated in this patient for acute viral hepatitis Other features of a systemic viral infection are present and therefore there is no indication for requesting lymph node aspiration

Management considerations

Clearly, the management of this patient depends on a correct diagnosis The most frequent misdiagnosis is of bacterial tonsillitis and antibiotic treatment given to someone with glandular fever might result in a severe drug reaction Confirmation of glandular fever would suggest the need for corticosteroid management of the patient in the present case, who is toxic and has profound thrombocytopenia

300

Next Question

Previous Question Tag Question Feedback End Review

Difficulty: Average

Peer Responses

Session Progress

Responses Correct: 0

Responses Incorrect: 151

Responses Total: 151

Responses - % Correct: 0%

Blog (https://www.pastest.com/blog) About Pastest (https://www.pastest.com/about-us) Contact Us (https://www.pastest.com/contact-us) Help (https://www.pastest.com/help) © Pastest 2016

https://mypastest.pastest.com/Secure/TestMe/Browser/429893#Top 2/2 8/10/2016 MyPastest

Back to Filters (/Secure/TestMe/Filter/429893/QA) Question 127 of 151

You are consulted on a 32-year-old obese patient with cellulitis who is not responding to treatment with flucloxacillin and penicillin in combination used over the past 3 days. She has a past history of varicose veins, but nil else of note. She is not keen to come into hospital.

Investigations:

Hb 12.1 g/dl

WCC 13.4 × 109 /l

PLT 201 × 109 /l

Na+ 139 mmol/l

K+ 4.5 mmol/l

Creatinine 130 μmol/l

Which one of the following options is the most appropriate next treatment step?

A Oral co-trimoxazole

B IV metronidazole

C IV gentamicin

D Oral clindamycin

E Oral vancomycin

Explanation

Treatment of cellulitis

Clindamycin has excellent oral penetration and would be the most appropriate next treatment step here It has activity against both penicillinase and non-penicillinase producing staphylococci, https://mypastest.pastest.com/Secure/TestMe/Browser/429893#Top 1/2 8/10/2016 MyPastest and streptococci apart from Strep. faecalis The major problem of clindamycin use is the increased risk of development of pseudomembranous colitis due to Clostridium difficile infection If significant bowel symptoms develop during therapy then there must be a low index of suspicion for C. difficile infection, and both metronidazole and oral vancomycin may be used for treatment

20782

Next Question

Previous Question Tag Question Feedback End Review

Difficulty: Average

Peer Responses

Session Progress

Responses Correct: 0

Responses Incorrect: 151

Responses Total: 151

Responses - % Correct: 0%

Blog (https://www.pastest.com/blog) About Pastest (https://www.pastest.com/about-us) Contact Us (https://www.pastest.com/contact-us) Help (https://www.pastest.com/help) © Pastest 2016

https://mypastest.pastest.com/Secure/TestMe/Browser/429893#Top 2/2 8/10/2016 MyPastest

Back to Filters (/Secure/TestMe/Filter/429893/QA) Question 128 of 151

A 38-year-old woman is referred to casualty with bilateral weakness in her legs. She also complains of general malaise. Three weeks previously she had returned from a 4-week tour of eastern Europe. On examination she appeared unwell and was pyrexial (38.9 °C). She has large palpable cervical lymph nodes bilaterally. Her pharynx was inflamed with areas of exudate on the pharyngeal wall giving the appearance of a membrane. Neurological examination revealed global weakness of both legs and absent reflexes, as well as mild bilateral facial weakness. She also seems to have some palatal weakness although examination is difficult due to her neck swelling and pain. Which one of the following options should be suspected?

A Cytomegalovirus infection

B Diphtheria

C Epstein–Barr virus infection

D Hodgkin’s disease

E Streptococcal tonsillitis

Explanation

Diphtheria Definition

Diphtheria is an infection of the mucous membranes or skin

Causative organism

It is caused by Corynebacterium diphtheriae

Epidemiology

It is vanishingly rare – in the USA, the most recent case in which infection had not originated abroad was reported in 1988

https://mypastest.pastest.com/Secure/TestMe/Browser/429893#Top 1/3 8/10/2016 MyPastest Clinical presentation

It commonly presents as a pharyngitis with grey or white exudates that coalesce to form a pseudomembrane Strains that produce diphtheria toxin may result in nausea, vomiting, polyneuropathy and vascular collapse The neuritis frequently follows a bulbar distribution

Diagnosis

Diagnosis is based on the culture of mucosal lesions or discharge

Treatment

Treatment is with diphtheria antitoxin and appropriate antibiotics (either penicillin, or usually erythromycin in penicillin-allergic individuals)

3892

Next Question

Previous Question Tag Question Feedback End Review

Difficulty: Average

Peer Responses

Session Progress

Responses Correct: 0

Responses Incorrect: 151

Responses Total: 151

Responses - % Correct: 0%

Blog (https://www.pastest.com/blog) About Pastest (https://www.pastest.com/about-us) Contact Us (https://www.pastest.com/contact-us) Help (https://www.pastest.com/help)

https://mypastest.pastest.com/Secure/TestMe/Browser/429893#Top 2/3 8/10/2016 MyPastest © Pastest 2016

https://mypastest.pastest.com/Secure/TestMe/Browser/429893#Top 3/3 8/10/2016 MyPastest

Back to Filters (/Secure/TestMe/Filter/429893/QA) Question 129 of 151

A 52-year-old man wishes to commence therapy for chronic hepatitis C virus (HCV) infection with a regimen which has the best chance of conferring sustained virological success. Which one of the following treatment options would you recommend?

A α-Interferon alone

B α-Interferon with ribavirin

C Ribavirin alone

D Ribavirin with lamivudine

E Lamivudine alone

Explanation

Treatment of chronic hepatitis C virus (HCV) infection

Combination therapy with subcutaneous α-interferon (three times weekly) and oral ribavirin (daily) would be the most effective regimen from the options in the question Ribavirin alone has little or no effect Lamivudine is licensed for the treatment of HBV not HCV (it is also used in HIV therapy) New therapies for HCV are rapidly emerging, including pegylated interferons

1644

Next Question

Previous Question Tag Question Feedback End Review

Difficulty: Easy

https://mypastest.pastest.com/Secure/TestMe/Browser/429893#Top 1/2 8/10/2016 MyPastest Peer Responses

Session Progress

Responses Correct: 0

Responses Incorrect: 151

Responses Total: 151

Responses - % Correct: 0%

Blog (https://www.pastest.com/blog) About Pastest (https://www.pastest.com/about-us) Contact Us (https://www.pastest.com/contact-us) Help (https://www.pastest.com/help) © Pastest 2016

https://mypastest.pastest.com/Secure/TestMe/Browser/429893#Top 2/2 8/10/2016 MyPastest

Back to Filters (/Secure/TestMe/Filter/429893/QA) Question 130 of 151

A 40-year-old woman presents with progressive confusion and mild neck stiffness. She has suffered from increasing headaches over the past 4–6 weeks, and over the past week she has begun to suffer from worsening drowsiness and confusion, with neck stiffness. A CT scan showed basal meningeal enhancement. A lumbar puncture showed an opening pressure of

200 mmH2 O, a turbid CSF with 500 leukocytes/ml (90% lymphocytes), a glucose concentration of 1 mmol/l and negative results with Gram, Indian ink and Ziehl–Neelsen stains. CSF protein is elevated at 1.05g/l (< 0.50). Which one of the following options is the best treatment?

A Rifampicin + INAH + pyrazinamide + streptomycin + ethambutol + corticosteroids

B Ceftriaxone

C Aciclovir

D Corticosteroids

E Liposomal amphotericin B

Explanation

Tuberculous meningitis Diagnosis

Tuberculous meningitis (TBM) the most likely diagnosis in the present case, based on the subacute history, computed tomography (CT) findings and the modest lymphocytic lymphocytosis accompanied by severe hypoglycorrhachia A negative Ziehl–Neelsen stain is not unusual in such cases, except when large volumes of spun cerebrospinal fluid (CSF) are examined Almost without exception, patients with bacterial meningitis have a high neutrophil pleocytosis and a few days’ history at presentation Apparently, the patient had not been receiving antibiotics, which can modify the CSF in those with bacterial meningitis

Differential diagnosis https://mypastest.pastest.com/Secure/TestMe/Browser/429893#Top 1/3 8/10/2016 MyPastest

Viral meningitis is unlikely since in viral meningitis the sugar concentration would be normal or only slightly reduced (except in patients with mumps meningitis); the cell count would by lymphocytic but generally less than the cell count here. There is no reason to suspect fungal meningitis as she has no immunocompromising conditions, although the CSF changes may be indistinguishable from TBM However, cryptococcal meningitis occasionally presents in a normal host, but here the Indian-ink stain is usually positive Bacterial meningitis is treated with ceftriaxone, viral meningitis with aciclovir and fungal meningitis with liposomal amphotericin B

Treatment

Rifampicin + INAH (isonicotinic acid hydrazide) + pyrazinamide + streptomycin + ethambutol + corticosteroids are used to treat TBM

299

Next Question

Previous Question Tag Question Feedback End Review

Difficulty: Average

Peer Responses

Session Progress

Responses Correct: 0

Responses Incorrect: 151

Responses Total: 151

Responses - % Correct: 0%

Blog (https://www.pastest.com/blog) About Pastest (https://www.pastest.com/about-us) Contact Us (https://www.pastest.com/contact-us) Help (https://www.pastest.com/help) © Pastest 2016 https://mypastest.pastest.com/Secure/TestMe/Browser/429893#Top 2/3 8/10/2016 MyPastest

https://mypastest.pastest.com/Secure/TestMe/Browser/429893#Top 3/3 8/10/2016 MyPastest

Back to Filters (/Secure/TestMe/Filter/429893/QA) Question 131 of 151

A 76-year-old woman is found to have gonorrhoea. She lives in a residential home and has profound dementia. Which one of the following options is the most appropriate action to take after treating her?

A Send an official report to the residential home

B Discuss with your senior doctor

C Do nothing

D Inform her partner

E Call the police

Explanation

Management of sexually transmitted infection in elderly people in residential care

In this case the management is difficult, clearly there may well be a problem with the home and it is important to gain the advice of a senior as to what to do In general it would be prudent to trace sexual partners but it would be advisable in this case to consider the possibility of sexual assault, hence the imperative need to seek advice

5639

Next Question

Previous Question Tag Question Feedback End Review

Difficulty: Average

Peer Responses

https://mypastest.pastest.com/Secure/TestMe/Browser/429893#Top 1/2 8/10/2016 MyPastest

Session Progress

Responses Correct: 0

Responses Incorrect: 151

Responses Total: 151

Responses - % Correct: 0%

Blog (https://www.pastest.com/blog) About Pastest (https://www.pastest.com/about-us) Contact Us (https://www.pastest.com/contact-us) Help (https://www.pastest.com/help)

© Pastest 2016

https://mypastest.pastest.com/Secure/TestMe/Browser/429893#Top 2/2 8/10/2016 MyPastest

Back to Filters (/Secure/TestMe/Filter/429893/QA) Question 132 of 151

A sputum sample of a 27-year-old Russian man is found to be strongly smear positive for acid–alcohol fast bacilli. He has recently been released from a 5-year prison sentence in Russia where he was first diagnosed as having tuberculosis 2 years ago. Since diagnosis he has had a variety of short courses of treatment for tuberculosis, none successful. Which one of the following options is the most appropriate next step?

A Start ethambutol and streptomycin

B Start immediate treatment with standard quadruple tuberculosis therapy

C Refer to expert microbiologist and start at least 4 drug therapy including 2nd line agents

D Start prednisolone

E Arrange urgent out-patient appointment

Explanation

Multidrug-resistant tuberculosis

It is highly likely the patient in the present case now has multidrug-resistant tuberculosis

Epidemiology

This is currently a huge problem in Russian prisons and increasingly imported to the UK

Management

Treatment should be discussed with an expert in the management of multidrug- resistant tuberculosis, and should comprise multiple agents, at least three of which he has not previously received CCDC is responsible for contact tracing The rifampicin-resistance gene is present in > 90% of rifampicin-resistant isolates and https://mypastest.pastest.com/Secure/TestMe/Browser/429893#Top 1/2 8/10/2016 MyPastest can provide a guide to treatment, but a negative result does not exclude rifampicin resistance It is now standard practice to offer all tuberculosis patients a human immunodeficiency virus (HIV) test

3712

Next Question

Previous Question Tag Question Feedback End Review

Difficulty: Average

Peer Responses

Session Progress

Responses Correct: 0

Responses Incorrect: 151

Responses Total: 151

Responses - % Correct: 0%

Blog (https://www.pastest.com/blog) About Pastest (https://www.pastest.com/about-us) Contact Us (https://www.pastest.com/contact-us) Help (https://www.pastest.com/help)

© Pastest 2016

https://mypastest.pastest.com/Secure/TestMe/Browser/429893#Top 2/2 8/10/2016 MyPastest

Back to Filters (/Secure/TestMe/Filter/429893/QA) Question 133 of 151

A 28-year-old sewage worker presents with a high temperature and myalgia, especially in his legs. After a short improvement, he develops jaundice 6 days later. On examination his temperature is 39 °C, he is jaundiced and has hepatosplenomegaly. Leukocytes 17 × 109 /l, bilirubin 325 mmol/l, AST 70 U/l, ALT 45 U/l, creatinine 248 mmol/l, HBsAg-negative.

Which one of the following options is the most likely diagnosis?

A Hepatitis A

B Infectious mononucleosis

C Cytomegalovirus infection

D Leptospirosis

E Budd–Chiari syndrome

Explanation

Leptospirosis Epidemiology

Leptospirosis is a worldwide zoonosis infection It may be asymptomatic 5–15% of cases are severe or fatal

Complications

Failure to diagnose leptospirosis is particularly unfortunate: severely ill patients often recover completely with prompt treatment, but renal failure or death are likely to ensue if therapy is delayed or not given

Causative organism

Leptospira nest in the renal tubules of mammalian hosts and are shed in the urine https://mypastest.pastest.com/Secure/TestMe/Browser/429893#Top 1/3 8/10/2016 MyPastest They can survive for several months in the environment under moist conditions, particularly in the warmth.

Transmission of infection

Transmission of infection from animal to humans usually occurs through contact with contaminated water or moist soil Recreational exposure to fresh water (canoeing, sailing, water-skiing) and animal contact at home have replaced occupational exposure as the chief source of disease

Clinical presentation

The disease typically begins with the abrupt onset of intense headache, fever, chills and myalgia Fever often exceeds 40 °C and is preceded by rigors Muscle pain can be excruciating and occurs most commonly in the thighs, calves, lumbosacral region and abdomen Abdominal wall pain, accompanied by palpation tenderness, can mimic an acute surgical abdomen Nausea, vomiting, diarrhoea and sore throat are other frequent symptoms Conjunctival suffusion is a helpful diagnostic clue; this usually appears 2–3 days after the onset of fever and involves the bulbar conjunctiva Pus and serous secretions are absent, and there is no matting of the eyelashes and eyelids Mild suffusion can easily be overlooked Less common and less distinctive signs include pharyngeal injection, splenomegaly, hepatomegaly, lymphadenopathy and skin lesions

Disease progression

Within a week most patients become asymptomatic After several days of apparent recovery, the illness resumes in some individuals Manifestations of the second stage are more variable and mild than those of the initial illness and usually last for 2–4 days Leptospira disappear from the blood, cerebrospinal fluid and tissues but appear in the urine

Other investigative findings

The white blood cell count varies but neutrophilia is usually found Urinalysis may show proteinuria, pyuria and microscopic haematuria Enzyme markers of skeletal muscle damage, such as creatine kinase and aldolase, are elevated in the sera of 50% of patients during the first week of illness Chest X-rays from patients with pulmonary manifestations show a variety of abnormalities, but none are of leptospirosis The most common finding is small, patchy, snowflake-like lesions in the periphery of the https://mypastest.pastest.com/Secure/TestMe/Browser/429893#Top 2/3 8/10/2016 MyPastest lung fields

Differential diagnosis

Patients with the Budd–Chiari syndrome have hepatic venous obstruction

1301

Next Question

Previous Question Tag Question Feedback End Review

Difficulty: Easy

Peer Responses

Session Progress

Responses Correct: 0

Responses Incorrect: 151

Responses Total: 151

Responses - % Correct: 0%

Blog (https://www.pastest.com/blog) About Pastest (https://www.pastest.com/about-us) Contact Us (https://www.pastest.com/contact-us) Help (https://www.pastest.com/help) © Pastest 2016

https://mypastest.pastest.com/Secure/TestMe/Browser/429893#Top 3/3 8/10/2016 MyPastest

Back to Filters (/Secure/TestMe/Filter/429893/QA) Question 134 of 151

An elderly patient with an indwelling urinary catheter presented with severe suprapubic pain. Urinalysis showed numerous pus cells and the presence of Staphylococcus epidermidis. He was given empirical treatment with amoxicillin, which has failed to clear the infection. On removal of the catheter, it is noticed that a slimy substance is present along its length. Which one phenomenon from the following options is most likely to be responsible for antibiotic resistance in bacteria due to slime production?

A Co-aggregation

B Biofilm formation

C Mutation involving an altered target site for antibiotics

D Mutation involving a target bypass mechanism

E Mutation leading to enzyme inactivation

Explanation

Indwelling urinary catheter infection

Slime is an amorphous viscid colloidal material that is secreted extracellularly by some bacteria (eg Staphylococcus epidermidis) The slime remains around the bacteria and provides a matrix in which biofilm formation can take place The biofilm enables the bacterial colony to spread very thinly over devices such as catheters and prostheses This biofilm restricts the access of drugs to the bacterium

3586

Next Question

Previous Question Tag Question Feedback End Review https://mypastest.pastest.com/Secure/TestMe/Browser/429893#Top 1/2 8/10/2016 MyPastest

Difficulty: Easy

Peer Responses

Session Progress

Responses Correct: 0

Responses Incorrect: 151

Responses Total: 151

Responses - % Correct: 0%

Blog (https://www.pastest.com/blog) About Pastest (https://www.pastest.com/about-us) Contact Us (https://www.pastest.com/contact-us) Help (https://www.pastest.com/help) © Pastest 2016

https://mypastest.pastest.com/Secure/TestMe/Browser/429893#Top 2/2 8/10/2016 MyPastest

Back to Filters (/Secure/TestMe/Filter/429893/QA) Question 135 of 151

An intravenous drug user was diagnosed with HIV and commenced on anti-retroviral therapy 1 year ago. Initial follow-up blood tests at 3 and 6 months demonstrated an undetectable viral load, however, the latest results show a viral load of 2000 copies/ml. Which one of the following options is the most likely cause for the virological failure?

A Co-morbidity

B Drug interaction

C Poor compliance

D Re-infection with a new strain of virus

E Viral resistance

Explanation

Failed anti-retroviral therapy

Compliance is a key issue to address in all patients placed on anti-retroviral agents Hence the considerable efforts by HIV clinics to harness the support of specially trained nurses and social workers to facilitate compliance The chaotic lifestyle of many intravenous drug users militates against this effort Besides poor compliance, viral resistance, re-infection with a new strain of virus, drug interaction and co-morbidity a all also must be considered when faced with the clinical problem as illustrated in the present case. HIV genotyping is possible when the HIV load rises above 2000 copies/ml, and should be performed to assess for drug resistance

761

Next Question

Previous Question Tag Question Feedback End Review https://mypastest.pastest.com/Secure/TestMe/Browser/429893#Top 1/2 8/10/2016 MyPastest

Difficulty: Average

Peer Responses

Session Progress

Responses Correct: 0

Responses Incorrect: 151

Responses Total: 151

Responses - % Correct: 0%

Blog (https://www.pastest.com/blog) About Pastest (https://www.pastest.com/about-us) Contact Us (https://www.pastest.com/contact-us) Help (https://www.pastest.com/help) © Pastest 2016

https://mypastest.pastest.com/Secure/TestMe/Browser/429893#Top 2/2 8/10/2016 MyPastest

Back to Filters (/Secure/TestMe/Filter/429893/QA) Question 136 of 151

A 33-year-old homosexual patient is HIV positive but in the stable phase of the disease. What marker is the best indicator of risk of opportunistic infection?

A C-reactive protein

B CD4 lymphocyte count

C Erythrocyte sedimentation rate

D Polymerase chain reaction

E Blood cultures

Explanation

Monitoring in human immunodeficiency virus infection

The CD4 lymphocyte (T-helper cell) count is a reliable indicator of human immunodeficiency virus (HIV)-related immune impairment CD4 counts, normal at or above 600/mm3 , vary considerably, even in the absence of HIV infection Without anti-retroviral treatment, a fall in the CD4 lymphocyte count to below 200/mm‑ is associated with a risk of opportunistic infections of about 80% over 3 years However, progression is variable and a minority of patients remain well for several years with stable low CD4 counts This variability is explained partly by differences in HIV viral load The level of CD4 lymphopenia generally determines the spectrum potential of infections For instance, whereas oral and oesophageal candidiasis and Pneumocystis pneumonia are frequent at CD4 counts of 100–200/mm3 , disseminated Mycobacterium avium complex infection and cytomegalovirus retinitis are rarely seen until the CD4 count is below 50/mm3

2696 https://mypastest.pastest.com/Secure/TestMe/Browser/429893#Top 1/2 8/10/2016 MyPastest

Next Question

Previous Question Tag Question Feedback End Review

Difficulty: Easy

Peer Responses

Session Progress

Responses Correct: 0

Responses Incorrect: 151

Responses Total: 151

Responses - % Correct: 0%

Blog (https://www.pastest.com/blog) About Pastest (https://www.pastest.com/about-us) Contact Us (https://www.pastest.com/contact-us) Help (https://www.pastest.com/help) © Pastest 2016

https://mypastest.pastest.com/Secure/TestMe/Browser/429893#Top 2/2 8/10/2016 MyPastest

Back to Filters (/Secure/TestMe/Filter/429893/QA) Question 137 of 151

A 31-year-old obese woman is brought to the Emergency department by ambulance. She has been treated for the past 48hrs by oral flucloxacillin from her GP for cellulitis, but according to her husband she has become progressively more unwell with drowsiness and vomiting and is now unable to stand. On examination her BP is 85/50 mmHg, pulse is 95/min and regular. Her temperature is 39.2o C, she has obvious left lower leg cellulitis and a maculopapular rash over her torso and abdomen. She is obese with a BMI of 33. Investigations;

Hb 12.9 g/dl

WCC 15.3 x109 /l

PLT 181 x109 /l

Na+ 138 mmol/l

K+ 5.4 mmol/l

Creatinine 189 micromol/l

PaO2 7.9 kPa

PaCO2 3.9 kPa

pH 7.31

Skin swab sent by GP reveals staphylococcus aureus Which of the following is the most appropriate antibiotic choice?

A Ciprofloxacin and gentamicin

B Flucloxacillin and Vancomycin

C Cefuroxime and gentamicin

D Ceufuroxime and metronidazole

E Doxycycline and flucloxacillin https://mypastest.pastest.com/Secure/TestMe/Browser/429893#Top 1/2 8/10/2016 MyPastest

Explanation The answer is Flucloxacillin and vancomycin This patient has signs of generalised sepsis with hypotension and tachycardia. Given the swab result and extensive left leg cellulitis, the most likely diagnosis is Staphylococcal septicaemia. Dual anti-staphylocccal therapy with combination flucloxacillin and vancomycin is therefore initial management of choice. Of note is the marked hypotension and raised creatinine, this suggests aggressive fluid replacement is also indicated, coupled with central venous pressure monitoring. 34357

Next Question

Previous Question Tag Question Feedback End Review

Difficulty: Average

Peer Responses

Session Progress

Responses Correct: 0

Responses Incorrect: 151

Responses Total: 151

Responses - % Correct: 0%

Blog (https://www.pastest.com/blog) About Pastest (https://www.pastest.com/about-us) Contact Us (https://www.pastest.com/contact-us) Help (https://www.pastest.com/help) © Pastest 2016

https://mypastest.pastest.com/Secure/TestMe/Browser/429893#Top 2/2 8/10/2016 MyPastest

Back to Filters (/Secure/TestMe/Filter/429893/QA) Question 138 of 151

An elderly diabetic man has left-sided orbital cellulitis. A CT scan of the paranasal sinuses shows evidence of left maxillary sinusitis. A Gram-stained smear of the orbital exudates shows irregularly branching septate hyphae. Which one of the following options is the most likely aetiological agent?

A Aspergillus

B Rhizopus

C Mucor

D Candida

E Actinomyces

Explanation

Differential diagnosis of cellulitis

Septate hyphae and the involvement of sinuses suggest a diagnosis of aspergillus infection Both Rhizopus and Mucor are fungi of the class Zygomycetes and can cause mucormycosis

Diabetes is a predisposing factor for this disease Rhinocerebral mucormycosis is the most common form Nasal stuffiness, facial pain and oedema and necrotic black nasal turbinates are characteristic

In candidiasis the yeast form is infectious and hence hyphae are not seen Actinomyces species are Gram-positive, branching, higher bacteria that are normal mouth and intestinal commensals

Sinuses and tracts develop in this disease with the discharge of sulphur granules

3583 https://mypastest.pastest.com/Secure/TestMe/Browser/429893#Top 1/2 8/10/2016 MyPastest

Next Question

Previous Question Tag Question Feedback End Review

Difficulty: Difficult

Peer Responses

Session Progress

Responses Correct: 0

Responses Incorrect: 151

Responses Total: 151

Responses - % Correct: 0%

Blog (https://www.pastest.com/blog) About Pastest (https://www.pastest.com/about-us) Contact Us (https://www.pastest.com/contact-us) Help (https://www.pastest.com/help) © Pastest 2016

https://mypastest.pastest.com/Secure/TestMe/Browser/429893#Top 2/2 8/10/2016 MyPastest

Back to Filters (/Secure/TestMe/Filter/429893/QA) Question 139 of 151

A young man with epilepsy presents to the Emergency Department with a 3-day history of cough and increasing shortness of breath. He has foul-smelling sputum. He had a generalised tonic–clonic seizure 1 week ago. On examination his temperature is 40 °C, he has right-sided crepitations and his chest X-ray shows a right upper lobe infiltration. Which one of the following options is the most likely cause?

A Mycoplasma pneumonia

B Chemical pneumonitis

C Pneumonia due to Gram-negative aerobes

D Pneumonia due to Gram-positive aerobes

E Pneumonia due to anaerobes

Explanation

Anaerobic pleuropulmonary infection

Anaerobic pleuropulmonary infections include aspiration pneumonia, necrotising pneumonitis, lung abscess and empyema, as well as infection secondary to bronchiectasis and bronchial carcinoma

Causative organisms

The anaerobes involved in these infections are the oropharyngeal commensals

Seeding of infection

Anaerobic pleuropulmonary infection usually results from oropharyngeal aspiration, but occasionally from haematogenous seeding

History

Patients with an anaerobic lung abscess will usually admit to the revolting taste (as well https://mypastest.pastest.com/Secure/TestMe/Browser/429893#Top 1/2 8/10/2016 MyPastest as smell) of their sputum

Confirmation of diagnosis

Definitive bacteriological diagnosis of anaerobic pleuropulmonary infection usually requires culture of an invasive specimen, obtained either by bronchoscopy or percutaneous transthoracic aspiration Expectorated sputum is rarely suitable Specimens should preferably be obtained before antibiotics are given

1316

Next Question

Previous Question Tag Question Feedback End Review

Difficulty: Average

Peer Responses

Session Progress

Responses Correct: 0

Responses Incorrect: 151

Responses Total: 151

Responses - % Correct: 0%

Blog (https://www.pastest.com/blog) About Pastest (https://www.pastest.com/about-us) Contact Us (https://www.pastest.com/contact-us) Help (https://www.pastest.com/help)

© Pastest 2016

https://mypastest.pastest.com/Secure/TestMe/Browser/429893#Top 2/2 8/10/2016 MyPastest

Back to Filters (/Secure/TestMe/Filter/429893/QA) Question 140 of 151

Which one of the following degenerative diseases of the central nervous system is caused by an infectious protein called a prion?

A Alzheimer’s disease

B Parkinson’s disease

C Creutzfeldt–Jakob disease

D Guillain–Barré syndrome

E Amyotrophic lateral sclerosis

Explanation

Creutzfeldt–Jakob disease Epidemiology

Creutzfeldt–Jakob disease (CJD) was discovered many years ago in particular areas of Slovakia and in Libyan-born Israelis The incidence in these populations was found to be 60–100 times greater than expected Possible explanations for these clusters included excessive dietary exposure to sheep scrapie and a high coefficient of inbreeding CJD has also been transmitted accidentally in the course of medical treatment by neurosurgical instruments, corneal grafts, cadaveric dura mater grafts and human pituitary-derived hormones There is no evidence that CJD has been transmitted iatrogenically through non-central nervous system tissues such as blood, blood products or organ transplantation, but continued vigilance is necessary as many of the mechanisms of iatrogenic transmission of CJD were not predicted The possibility of secondary transmission of variant CJD (vCJD) through blood transfusion, the use of fractionated blood derivatives, organ transplantation or

https://mypastest.pastest.com/Secure/TestMe/Browser/429893#Top 1/3 8/10/2016 MyPastest contaminated surgical instruments is a matter of continuing concern

Incubation period

The incubation period also varies according to the route of exposure to infection With central exposure the mean incubation period ranges from about 18 months, similar to the incubation periods in primates after experimental intracerebral inoculation, to 6 years with dura mater grafts With a peripheral route of exposure the mean incubation period is about 12 years, but may extend to over 30 years, which is similar to the extended incubation periods in kuru, a human prion disease also caused by a peri pheral route of exposure to infection.

Preventive measures

Measures to reduce the risk of iatrogenic transmission of CJD have been introduced in many countries There are strict selection criteria for obtaining corneal grafts, recombinant growth hormone replaced human growth hormone in 1985, and human dura mater grafts have not been licensed in the UK since the early 1990s

1290

Next Question

Previous Question Tag Question Feedback End Review

Difficulty: Easy

Peer Responses

Session Progress

Responses Correct: 0

Responses Incorrect: 151

Responses Total: 151

Responses - % Correct: 0%

https://mypastest.pastest.com/Secure/TestMe/Browser/429893#Top 2/3 8/10/2016 MyPastest

Blog (https://www.pastest.com/blog) About Pastest (https://www.pastest.com/about-us) Contact Us (https://www.pastest.com/contact-us) Help (https://www.pastest.com/help) © Pastest 2016

https://mypastest.pastest.com/Secure/TestMe/Browser/429893#Top 3/3 8/10/2016 MyPastest

Back to Filters (/Secure/TestMe/Filter/429893/QA) Question 141 of 151

Which one of the following options is the most appropriate test in patients with hepatitis C virus infection to determine the need for treatment?

A Alanine aminotransferase (ALT)

B Liver biopsy

C Abdominal ultrasound

D CRP

E Serum HCV IgG level

Explanation

Treatment considerations in hepatitis C infection

Treatment is recommended for patients with moderate to severe hepatitis C (defined as histological evidence of significant fibrosis or significant necro-inflammation) The duration of therapy depends on both HCV genotype and viral load A lower baseline HCV viral load is associated with a better response to treatment Serum alanine aminotransferase (ALT) is usually, but not always, abnormally elevated in patients with significant degrees of hepatitis The primary aims of treatment of patients with chronic HCV are to achieve acceptable ALT levels and clearance of HCV, with both sustained for at least 6 months after treatment cessation Serum HCV IgG level is irrelevant

3681

Next Question

Previous Question Tag Question Feedback End Review https://mypastest.pastest.com/Secure/TestMe/Browser/429893#Top 1/2 8/10/2016 MyPastest

Difficulty: Difficult

Peer Responses

Session Progress

Responses Correct: 0

Responses Incorrect: 151

Responses Total: 151

Responses - % Correct: 0%

Blog (https://www.pastest.com/blog) About Pastest (https://www.pastest.com/about-us) Contact Us (https://www.pastest.com/contact-us) Help (https://www.pastest.com/help) © Pastest 2016

https://mypastest.pastest.com/Secure/TestMe/Browser/429893#Top 2/2 8/10/2016 MyPastest

Back to Filters (/Secure/TestMe/Filter/429893/QA) Question 142 of 151

Which one of the following malignancies has been associated with Epstein–Barr virus infection?

A Cervical cancer

B Colon cancer

C Nasopharyngeal carcinoma

D Glioma

E Carcinoma of the liver

Explanation

Nasopharyngeal carcinoma Epidemiology

Although nasopharyngeal carcinoma is seen rare throughout the world, it has a remarkably high incidence in southern Chinese and in the Inuit and related circum- Arctic races In high-incidence areas, nasopharyngeal carcinoma is the most common cancer in men and the second most common in women The tumour usually occurs in middle or old age, but in North Africa it has bimodal age peaks: one involving young people up to 20 years of age and a second much later in life

Aetiology

This tumour is restricted to the postnasal space, where it arises from squamous epithelial cells Irrespective of geographical region, nasopharyngeal carcinoma cells always carry the EBV genome.

Clinical presentation https://mypastest.pastest.com/Secure/TestMe/Browser/429893#Top 1/3 8/10/2016 MyPastest

Nasopharyngeal carcinoma causes nasal obstruction, discharge or bleeding, deafness, tinnitus or earache, headache and ocular paresis from tumour spread to involve cranial nerves Patients may present with either a single symptom caused locally by the tumour or with several symptoms, while about one-third complain only of cervical lymph-node enlargement due to metastatic spread from an occult primary tumour

Metastatic features

Direct spread from the primary tumour may involve the soft tissues, bone, parotid gland, buccal cavity and oropharynx The neoplasm may extend into the nasal fossae, the paranasal sinuses or the orbit, and can invade the eustachian canal or the parapharyngeal space where cranial nerves IX, X, XI and XII can be involved Invasion of the skull or cranial foramina may damage cranial nerves II, IV, V and VI Lymphatic spread causes enlarged cervical lymph nodes and subsequently extends to the supraclavicular glands Blood-borne metastases most frequently occur in the bones, liver and lungs, but may be found in any organ

Prognosis

Untreated nasopharyngeal carcinoma progresses inexorably to death

1293

Next Question

Previous Question Tag Question Feedback End Review

Difficulty: Easy

Peer Responses

Session Progress

Responses Correct: 0

Responses Incorrect: 151 https://mypastest.pastest.com/Secure/TestMe/Browser/429893#Top 2/3 8/10/2016 MyPastest

Responses Total: 151

Responses - % Correct: 0%

Blog (https://www.pastest.com/blog) About Pastest (https://www.pastest.com/about-us) Contact Us (https://www.pastest.com/contact-us) Help (https://www.pastest.com/help) © Pastest 2016

https://mypastest.pastest.com/Secure/TestMe/Browser/429893#Top 3/3 8/10/2016 MyPastest

Back to Filters (/Secure/TestMe/Filter/429893/QA) Question 143 of 151

A 19-year-old college student presents with violent vomiting, abdominal cramps and watery diarrhoea within 4 hours of having drunk a glass of warm milk in his hall of residence’s canteen. Given the likely bacterial infection, which one of the following microbiological mechanisms is most likely to be responsible for his symptoms?

A Toxins formed in the intestine

B Rapid multiplication of organisms in the gut

C Preformed toxins in the milk

D Action of toxins directly on the emetic centre

E Growth of organisms in the milk

Explanation

Food poisoning Aetiological agent

The man in the present case most probably has Staphylococcus aureus food poisoning S. aureus is a common cause of food-borne gastroenteritis in Europe and the USA Outbreaks usually occur as a result of poor food hygiene

Disease onset

The toxin (heat-stable enterotoxin B) is preformed in the contaminated food; hence the onset of symptoms is rapid, often within 2–4 hours of consumption

Underlying pathology

The toxin causes a massive secretion of fluid into the intestinal lumen, principally because of its effect on the autonomic nervous system

3574

https://mypastest.pastest.com/Secure/TestMe/Browser/429893#Top 1/2 8/10/2016 MyPastest Next Question

Previous Question Tag Question Feedback End Review

Difficulty: Average

Peer Responses

Session Progress

Responses Correct: 0

Responses Incorrect: 151

Responses Total: 151

Responses - % Correct: 0%

Blog (https://www.pastest.com/blog) About Pastest (https://www.pastest.com/about-us) Contact Us (https://www.pastest.com/contact-us) Help (https://www.pastest.com/help)

© Pastest 2016

https://mypastest.pastest.com/Secure/TestMe/Browser/429893#Top 2/2 8/10/2016 MyPastest

Back to Filters (/Secure/TestMe/Filter/429893/QA) Question 144 of 151

A patient with acute lymphoblastic leukaemia is admitted to the haematology ward with breathlessness and a cough. A bronchoscopy and bronchoalveolar lavage are performed. Which one of the following pathogens, if found, would allow her to remain on the haematology ward rather than requiring isolation?

A Aspergillus flavus

B Cytomegalovirus

C Rifampicin-sensitive Mycobacterium tuberculosis

D Mycobacterium tuberculosis

E Parainfluenza

Explanation

Nosocomial infection

Nosocomial infection is a particular risk to the increasing population of immunosuppressed patients Haematology wards may have a number of side-rooms that are usually under positive- pressure ventilation to accommodate especially vulnerable patients However, isolation in a negative-pressure ventilation room should be sought with the advent of more communicable and pathogenic organisms All the pathogens listed in the question and in pulmonary samples should be suitably isolated except cytomegalovirus, which is usually due to reactivation of latent disease or primary disease acquired through organ transplantation to a seronegative recipient

772

Next Question

Previous Question Tag Question Feedback End Review https://mypastest.pastest.com/Secure/TestMe/Browser/429893#Top 1/2 8/10/2016 MyPastest

Difficulty: Average

Peer Responses

Session Progress

Responses Correct: 0

Responses Incorrect: 151

Responses Total: 151

Responses - % Correct: 0%

Blog (https://www.pastest.com/blog) About Pastest (https://www.pastest.com/about-us) Contact Us (https://www.pastest.com/contact-us) Help (https://www.pastest.com/help) © Pastest 2016

https://mypastest.pastest.com/Secure/TestMe/Browser/429893#Top 2/2 8/10/2016 MyPastest

Back to Filters (/Secure/TestMe/Filter/429893/QA) Question 145 of 151

A 29-year-old man is brought to the clinic by his girlfriend. He is a long term user of intravenous heroin. Over the past few days he has become confused and aggressive and has had a fit. Investigations reveal ring lesions on contrast CT scan, his CD4 count is 45/mm3 (normal >500) and an HIV test is positive. You suspect he has toxoplasmosis.

Which one of the following options is the most appropriate therapy for him?

A Fansidar

B Sulfadiazine and pyrimethamine

C Clindamycin

D Atovaquone

E Azithromycin

Explanation

Treatment of toxoplasmosis

Sulfadiazine and pyrimethamine is seen as combination therapy of choice for patients with toxoplasmosis in the setting of human immunodeficiency virus (HIV) infection Sulfadiazine may, however, be associated with adverse effects (commonly rash) in up to 40% of users Clindamycin may be substituted in these cases Clarithromycin or azithromycin may be additional alternatives but used in combination with pyrimethamine Whilst anti-retroviral therapy may be associated with a decreased viral load and improved CD4 count, there are limited data on discontinuing long-term maintenance therapy against toxoplasmosis, therefore most patients continue permanent therapy at a lower dose

18604

https://mypastest.pastest.com/Secure/TestMe/Browser/429893#Top 1/2 8/10/2016 MyPastest Next Question

Previous Question Tag Question Feedback End Review

Difficulty: Easy

Peer Responses

Session Progress

Responses Correct: 0

Responses Incorrect: 151

Responses Total: 151

Responses - % Correct: 0%

Blog (https://www.pastest.com/blog) About Pastest (https://www.pastest.com/about-us) Contact Us (https://www.pastest.com/contact-us) Help (https://www.pastest.com/help)

© Pastest 2016

https://mypastest.pastest.com/Secure/TestMe/Browser/429893#Top 2/2 8/10/2016 MyPastest

Back to Filters (/Secure/TestMe/Filter/429893/QA) Question 146 of 151

A previously well 35-year-old man presents in the summer months with a community- acquired pneumonia and mild, watery diarrhoea. He has failed to improve after 3 days of amoxicillin given by his GP. The temperature is 40°C; the patient is complaining of a headache and seems slightly confused and disorientated but has no clinical evidence of meningitis. The peripheral white cell count is on the upper limit of normal and Na+ is 125 mmol/l. Two other members of his office are also ill with a similar illness. Which one of the following options is the most likely causative organism?

A Mycoplasma pneumoniae

B Legionella pneumophila

C Influenza A virus

D Chlamydia pneumoniae

E Penicillin-resistant Streptococcus pneumoniae

Explanation

Legionnaire’s disease Clinical manifestations

High fever, altered mental state, hyponatraemia and watery diarrhoea are all typical of legionnaire’s disease (as is the occurrence of a cluster of cases), but can occur to some extent in any pneumonia

Differential diagnosis

Mycoplasma and Chlamydia do not usually cause such a severe pneumonia It is the wrong time of year for influenza The penicillin-resistant pneumococcus is still uncommon in the UK, but is on the increase worldwide

3698

https://mypastest.pastest.com/Secure/TestMe/Browser/429893#Top 1/2 8/10/2016 MyPastest Next Question

Previous Question Tag Question Feedback End Review

Difficulty: Easy

Peer Responses

Session Progress

Responses Correct: 0

Responses Incorrect: 151

Responses Total: 151

Responses - % Correct: 0%

Blog (https://www.pastest.com/blog) About Pastest (https://www.pastest.com/about-us) Contact Us (https://www.pastest.com/contact-us) Help (https://www.pastest.com/help)

© Pastest 2016

https://mypastest.pastest.com/Secure/TestMe/Browser/429893#Top 2/2 8/10/2016 MyPastest

Back to Filters (/Secure/TestMe/Filter/429893/QA) Question 147 of 151

A 30-year-old man presents with an acute onset of pain and blurred vision of his right eye. On examination there is conjunctival injection and dendritic ulceration is seen on his cornea. Given the likely diagnosis, which one of the following options is the most important treatment?

A Topical steroids

B Topical aciclovir

C Ampicillin ointment

D Topical nystatin

E Oral fluconazole

Explanation

Herpes simplex virus keratitis

The likely diagnosis in the present case is herpes simplex virus (HSV) keratitis Topical aciclovir is the drug of choice; topical steroids may make the infection worse and are contraindicated Herpes simplex virus (HSV) can also cause an acute necrotising retinitis, usually seen in immunosuppressed patients such as those with HIV infection, but rarely in immunocompetent people

2692

Next Question

Previous Question Tag Question Feedback End Review

Difficulty: Easy

https://mypastest.pastest.com/Secure/TestMe/Browser/429893#Top 1/2 8/10/2016 MyPastest Peer Responses

Session Progress

Responses Correct: 0

Responses Incorrect: 151

Responses Total: 151

Responses - % Correct: 0%

Blog (https://www.pastest.com/blog) About Pastest (https://www.pastest.com/about-us) Contact Us (https://www.pastest.com/contact-us) Help (https://www.pastest.com/help) © Pastest 2016

https://mypastest.pastest.com/Secure/TestMe/Browser/429893#Top 2/2 8/10/2016 MyPastest

Back to Filters (/Secure/TestMe/Filter/429893/QA) Question 148 of 151

An Asian man has intermittent diarrhoea, abdominal bloating and discomfort and complains of the passage of bulky, malodorous stools that are difficult to flush away. Which one of the following infections is he most likely to have?

A Giardiasis

B Amoebiasis

C Cryptosporidiosis

D Balantidiasis

E Blastocystis hominis infection

Explanation

Giardiasis Clinical manifestations

Diarrhoea associated with malabsorption is characteristically seen in giardiasis

Underlying pathology

The mechanism by which the organism causes malabsorption is not fully understood, but may be due to a loss of brush-border enzyme activity

Differential diagnosis

Cryptosporidiosis presents with acute watery diarrhoea, fever and malaise that lasts 7– 10 days and is self-limiting

Malabsorption does not occur

3575

Next Question https://mypastest.pastest.com/Secure/TestMe/Browser/429893#Top 1/2 8/10/2016 MyPastest

Previous Question Tag Question Feedback End Review

Difficulty: Average

Peer Responses

Session Progress

Responses Correct: 0

Responses Incorrect: 151

Responses Total: 151

Responses - % Correct: 0%

Blog (https://www.pastest.com/blog) About Pastest (https://www.pastest.com/about-us) Contact Us (https://www.pastest.com/contact-us) Help (https://www.pastest.com/help) © Pastest 2016

https://mypastest.pastest.com/Secure/TestMe/Browser/429893#Top 2/2 8/10/2016 MyPastest

Back to Filters (/Secure/TestMe/Filter/429893/QA) Question 149 of 151

Which one of the following combination therapies is the most appropriate for the treatment of chronic hepatitis C infection?

A Amoxicillin and clavulanic acid

B Piperacillin and tazobactam

C Isoniazid and pyridoxine

D Interferon-α (IFN-α) and ribavirin

E Interferon-γ (IFN-γ) and lamivudine

Explanation

Drug treatment of hepatitis C Mechanism of action

Ribavirin is a guanosine analogue with broad anti-viral activity but which may affect the

hepatitis C virus (HCV) by inducing a shift toward a TH1 -cellular immune response Used orally alone, ribavirin returns the level of serum alanine aminotransferase (ALT) to normal in some individuals but does not substantially change HCV RNA levels However, in combination with interferon (IFN)-α-2b, 1000–1200 mg daily of oral ribavirin improves the sustained virological response rates both for people who have never been treated and those who initially responded to interferon but then relapsed (but not those who never responded) As initial treatment, approximately one-third of patients treated with IFN and ribavirin have a sustained virological response

Side-effects

Adverse reactions to ribavirin and IFN-α are similar to those with IFN-α alone, but ribavirin causes haemolytic anaemia in many patients Ribavirin is teratogenic; pregnancy must be prevented during and for up to 1 year after https://mypastest.pastest.com/Secure/TestMe/Browser/429893#Top 1/2 8/10/2016 MyPastest administration, whichever sex is being treated

1297

Next Question

Previous Question Tag Question Feedback End Review

Difficulty: Easy

Peer Responses

Session Progress

Responses Correct: 0

Responses Incorrect: 151

Responses Total: 151

Responses - % Correct: 0%

Blog (https://www.pastest.com/blog) About Pastest (https://www.pastest.com/about-us) Contact Us (https://www.pastest.com/contact-us) Help (https://www.pastest.com/help) © Pastest 2016

https://mypastest.pastest.com/Secure/TestMe/Browser/429893#Top 2/2 8/10/2016 MyPastest

Back to Filters (/Secure/TestMe/Filter/429893/QA) Question 150 of 151

A 19-year-old man undergoes splenectomy after a horse riding accident. He was given the pneumococcal vaccination 2 years previously because of a past history of asthma. Which one of the following options is true with respect to prophylaxis against pneumococcal infection?

A Penicillin is not indicated

B Penicillin prophylaxis 500 mg/day is indicated for at least a 2-year period

C Penicillin prophylaxis 1g/day is indicated for a 2-year period

D They should be immediately revaccinated against pneumococcus

E Pneumococcal vaccination should be repeated every 10 years

Explanation

Post splenectomy infection High-risk period

The greatest risk of infection post splenectomy exists in the first 2 years after removal

Guidelines for prevention and management

Existing UK guidelines (see the guidelines for the prevention and treatment of infections in patients with an absent or dysfunctional spleen, British Committee for Standards in Haematology) recommend prophylactic antibiotics during this period Some controversy exists however, with French and American guidelines not recommending antibiotic prophylaxis with penicillin due to the development of resistance Pneumococcal vaccination should be repeated every 5 years according to the guidelines because immunity is thought to be less persistent In patients who are penicillin allergic, erythromycin would be the default choice

22456 https://mypastest.pastest.com/Secure/TestMe/Browser/429893#Top 1/2 8/10/2016 MyPastest

Next Question

Previous Question Tag Question Feedback End Review

Difficulty: Difficult

Peer Responses

Session Progress

Responses Correct: 0

Responses Incorrect: 151

Responses Total: 151

Responses - % Correct: 0%

Blog (https://www.pastest.com/blog) About Pastest (https://www.pastest.com/about-us) Contact Us (https://www.pastest.com/contact-us) Help (https://www.pastest.com/help)

© Pastest 2016

https://mypastest.pastest.com/Secure/TestMe/Browser/429893#Top 2/2 8/10/2016 MyPastest

Back to Filters (/Secure/TestMe/Filter/429893/QA) Question 151 of 151

A 25-year-old well woman gives birth at full term to an otherwise well baby with unilateral microphthalmia. She recalls a rash during the first trimester of her pregnancy, but cannot remember any other details of the rash. Which one of the following options is the most likely causative agent?

A Syphilis

B Varicella zoster virus (VZV)

C Rubella

D Cytomegalovirus (CMV)

E Parvovirus B19

Explanation

Infections and pregnancy

Congenital Varicella zoster virus (VZV) infection following primary infection in the mother in the first 20 weeks of pregnancy is associated with about a 2% incidence of fetal abnormality The fetus develops episodes of shingles that affect development of the involved dermatome Congenital syphilis, CMV and rubella tend to cause symmetrical problems Parvovirus B19 is associated with miscarriage and fetal hydrops, but is not clearly associated with any developmental abnormality

3688

End Session

Previous Question Tag Question Feedback End Review

https://mypastest.pastest.com/Secure/TestMe/Browser/429893#Top 1/2 8/10/2016 MyPastest

Difficulty: Difficult

Peer Responses

Session Progress

Responses Correct: 0

Responses Incorrect: 151

Responses Total: 151

Responses - % Correct: 0%

Blog (https://www.pastest.com/blog) About Pastest (https://www.pastest.com/about-us) Contact Us (https://www.pastest.com/contact-us) Help (https://www.pastest.com/help)

© Pastest 2016

https://mypastest.pastest.com/Secure/TestMe/Browser/429893#Top 2/2